You are on page 1of 495

Page |1

Chapter 1
The Accounting Process

1. It is a systematic compilation of a group of accounts.


a. Chart of accounts c. Ledger
b. Trial balance d. Journal

2. It is a list of accounts and their balances.


a. Chart of accounts c. Ledger
b. Trial balance d. Journal

3. Which of the following criteria must be met before an event or item is recorded for accounting
purposes?
a. The event or item can be measured objectively in financial terms.
b. The event or item is relevant and reliable.
c. The event affects, or the item meets the definition of, a financial statement element.
d. All of these must be met.

4. An accounting record into which the essential facts and figures in connection with all
transactions are initially recorded is called the
a. ledger. c. trial balance.
b. account. d. none of these.

5. A trial balance may prove that debits and credits are equal, but
a. an amount could be entered in the wrong account.
b. a transaction could have been entered twice.
c. a transaction could have been omitted.
d. all of these.

6. When an item of expense is paid and recorded in advance, it is normally called a(n)
a. prepaid expense. c. estimated expense.
b. accrued expense. d. cash expense.

7. When an item of revenue or expense has been earned or incurred but not yet collected or paid, it
is normally called a(n) ____________ revenue or expense.
a. prepaid c. estimated
b. adjusted d. none of these

8. An unearned revenue can best be described as an amount


a. collected and currently matched with expenses.
b. collected but not currently matched with expenses.
c. not collected but currently matched with expenses.
d. not collected and not currently matched with expenses.
Page |2

9. Which of the following is a real (permanent) account?


a. Inventory
b. Sales
c. Accounts Receivable
d. Both Inventory and Accounts Receivable

10. Reversing entries are


1. normally prepared for prepaid, accrued, and estimated items.
2. necessary to achieve a proper matching of revenue and expense.
3. desirable to exercise consistency and establish standardized procedures.
a. 1 c. 3
b. 2 d. 1 and 2

11. Adjusting entries that may be reversed include


a. all accrued revenues.
b. all accrued expenses.
c. those that debit an asset or credit a liability.
d. all of these.

12. An entity’s unadjusted trial balance does not equal. The following information was determined:
• The debit posting for a sale on account was omitted. 5,000
• The balance of Prepaid assets was listed as a credit instead of debit 34,000
• The balance of Office expense was listed as Rent expense 16,000
• Accounts payable was listed as a debit instead of credit 4,000

How much is the difference between the total debits and total credits in the trial balance?
a. 65,000 b. 81,000 c. 30,000 d. 34,000

A
Solution:
Trial balance
Dr. Cr.
Corresponding credit
Debit to accounts of the debit to
receivable omitted 5,000 5,000 accounts receivable

Prepaid assets Corresponding credit


omitted and listed as of the debit to Prepaid
credit 34,000 34,000 assets

Prepaid assets listed


34,000 as a credit

Corresponding debit Accounts payable


of the credit to omitted and listed as
Accounts payable 4,000 4,000 debit

Accounts payable
listed as debit 4,000
Total Debits 8,000 73,000 Total Credits
Difference, excess of
65,000 total credits over
Page |3

total debits

13. Theta prepares its financial statements for the year to 30 April each year. The company pays rent
for its premises quarterly in advance on 1 January, 1 April, 1 July and 1 October each year. The
annual rent was ₱84,000 per year until 30 June 2000. It was increased from that date to ₱96,000
per year. What rent expense and end of year prepayment should be included in the financial
statements for the year ended 30 April 2001?
Expense Prepayment
a. 93,000 8,000
b. 93,000 16,000
c. 94,000 8,000
d. 94,000 16,000

D
Solution:
 Fiscal year period: May 1, 2000 to April 30, 2001
 Change in annual rent: June 30, 2000
 Rent expense:
o May 1, 2000 to June 30, 2000: 84,000 x 2/12 = 14,000
o July 1, 2000 to April 30, 2001: 96,000 x 10/12 = 80,000
o Total rent expense = (14,000 + 80,000) = 94,000

 Prepaid rent:
o Last payment date: April 1, 2001
o Amount paid: 96,000 ÷ 4 quarters = 24,000
o Unexpired portion as of April 30, 2001 = 24,000 x 2/3 = 16,000

14. On March 1, a company received ₱3,000 cash from a client as an advance for 12 months’ worth of
delivery services. The company initially recorded this receipt as a debit to cash and a credit to
delivery service revenue. The adjusting entry on December 31 would include a:
a. debit to delivery service revenue, ₱2,500.
b. credit to unearned delivery service revenue, ₱500.
c. credit to delivery service revenue, ₱500.
d. No adjusting entry was required because the delivery service was for a one-year period
exactly.

15. On August 1, a corporation received cash of ₱12,000 for one year's rent in advance and recorded
the transaction on that day as a credit to rent revenue. The December 31 adjusting entry is:
a. Rent revenue ₱5,000
Unearned rent revenue ₱5,000
b. Rent Revenue ₱7,000
Unearned rent revenue ₱7,000
c. Unearned rent revenue ₱5,000
Rent revenue ₱5,000
d. Unearned rent revenue ₱7,000
Page |4

Rent revenue ₱7,000

16. A corporation received cash of ₱24,000 on August 1 for one-year's rent in advance and recorded
the transaction on that day as a credit to unearned rent revenue for the full amount. The
December 31 adjusting entry is:
a. Rent revenue ₱10,000
Unearned rent revenue ₱10,000
b. Unearned rent revenue ₱24,000
Rent revenue ₱24,000
c. Rent revenue ₱14,000
Unearned rent revenue ₱14,000
d. Unearned rent revenue ₱10,000
Rent revenue ₱10,000

17. On July 1, a company paid a ₱600 premium for a three-year property insurance policy; insurance
expense was debited in full for the ₱600. The adjusting entry at the end of the year is:
a. Prepaid insurance ₱1,000
Insurance expense ₱1,000
b. Prepaid insurance ₱ 500
Insurance expense ₱ 500
c. Prepaid insurance ₱ 100
Insurance expense ₱ 100
d. Prepaid insurance ₱ 500
Insurance expense ₱ 500

18. A company paid its property taxes on April 1 for the period April 1, Year 1 to March 30, Year 2.
When the payment was made, the company debited property tax expense and credited cash for
₱12,000. The adjusting entry at the end of the Year 1 is:
a. prepaid property tax ₱9,000
property tax expense ₱9,000
b. prepaid property tax ₱3,000
property tax expense ₱3,000
c. property tax expense ₱3,000
prepaid tax expense ₱3,000
d. property tax expense ₱9,000
prepaid tax expense ₱9,000

19. On May 1, a company purchased a six-month subscription to an investment analysis service


publication. The ₱300 cash payment was debited to subscription expense at the time. The
adjusting entry on June 30, the end of the company's fiscal year, is:
a. subscription expense ₱100
subscription payable ₱100
b. prepaid subscriptions ₱100
subscriptions expense ₱100
c. prepaid subscription ₱200
Page |5

subscription expense ₱200


d. No adjusting entry was required because the subscription was for only six months not a full
year.

20. A sole proprietor took some goods costing ₱800 from inventory for his own use. The normal
selling price of the goods is ₱1,600. Which of the following journal entries would correctly record
this?
a. Drawings account 800
Inventory account 800
b. Drawings account 800
Purchases returns account 800
c. Sales account 1,600
Drawings account 1,600
d. None of these

“A wise man will hear and increase learning, and a man of understanding will attain
wise counsel.” (Proverbs 1:5)

- END -
Page |1

Chapter 2
Cash and Cash Equivalents

1. The amount reported as "Cash" on a company's statement of financial position normally should
exclude
a. postdated checks that are payable to the company.
b. cash in a payroll account.
c. undelivered checks written and signed by the company.
d. petty cash.

2. Which of the following would not be classified as cash?


a. Personal checks
b. Travelers' checks
c. Cashiers' checks
d. Postdated checks

3. What is the proper accounting treatment for a stale check?


a. Revert back to cash and accounts payable.
b. Revert back to cash and a credit to gain.
c. Ignored
d. Either a or b

4. Which of the following is not a basic characteristic of a system of cash control?


a. Use of a voucher system
b. Combined responsibility for handling and recording cash
c. Daily deposit of all cash received
d. Internal audits at irregular intervals

5. The principal purpose of a voucher system is to provide assurance that


a. all cash receipts are deposited intact in the bank.
b. all cash disbursements are approved before a check is issued.
c. all cash receipts are recorded in the accounting records.
d. all purchase invoices are supported by debit memoranda.

6. On January 1, 2002, Kyle Corporation established a petty cash fund of ₱400. On December 31,
2002, the petty cash fund was examined and found to have receipts and documents for
miscellaneous expenses amounting to ₱364. In addition, there was cash amounting to ₱44. What
entry would be required to record replenishment of the petty cash fund on December 31, 2002?

a. Petty Cash.................... 364


Cash Short and Over......... 8
Cash in bank 356
b. Miscellaneous Expense......... 364
Page |2

Cash Short and Over......... 8


Petty Cash.................. 356
c. Miscellaneous Expense......... 364
Cash Short and Over......... 8
Cash in bank 356
d. Miscellaneous Expense......... 356
Cash Short and Over......... 8
Cash in bank 364

7. On December 31, 2009, West Company had the following cash balances:

Cash in banks P1,800,000


Petty cash funds (all funds were reimbursed on 12/31/09) 50,000

Cash in banks includes P600,000 of compensating balances against short-term borrowing


arrangements at December 31, 2009. The compensating balances are not legally restricted as to
withdrawal by West. In the current assets section of West's December 31, 2009, balance sheet
(statement of financial position), what total amount should be reported as cash?
a. P1,200,000
b. P1,250,000
c. P1,800,000
d. P1,850,000

D (1,800,000 + 50,000) = 1,850,000

8. Trans Co. had the following balances at December 31, 2009:

Cash in checking account P 35,000


Cash in money market account 75,000
Treasury bill, purchased 11/1/2009, maturing 1/31/2010 350,000
Treasury bill, purchased 12/1/2009, maturing 3/31/2010 400,000

Tran’s policy is to treat as cash equivalents all highly liquid investments with a maturity of three
months or less when purchased. What amount should Trans report as cash and cash equivalents in
its December 31, 2009, balance sheet (statement of financial position)?
a. P110,000
b. P385,000
c. P460,000
d. P860,000

C (35,000 + 75,000 + 350,000) = 460,000

9. The cash balance of CAPSIZE OVERTURN Co. comprises the following:


Cash on hand 300,000
Cash in bank – savings – BPI 600,000
Page |3

Cash in bank – current – BPI (240,000)


Cash in bank – deposit in escrow – Metrobank 300,000
Cash in bank – current – Metrobank ( 60,000)
Cash in bank – current – BDO ( 90,000)
Total 810,000

Additional information:
• Cash on hand includes undeposited collections of P60,000.
• The cash in bank – savings maintained at BPI includes a P150,000 compensating balance which is
not restricted.

What amount of cash is reported in the financial statements?


a. 660,000
b. 810,000
c. 900,000
d. 960,000

A (300,000 + 600,000 – 240,000) = 660,000

10. As of December 31, 20x1, the petty cash fund of TUMULT COMMOTION Co. with a general
leger balance of P15,000 comprises the following:
Coins and currencies P 2,550
Petty cash vouchers:
Gasoline for delivery equipment P3,000
Medical supplies for employees 2,040 5,040
IOU’s:
Advances to employees 2,220
A sheet of paper with names of several employees
together with contribution to bereaved employee,
attached is a currency of 2,400
Checks:
Check drawn to the order of the petty cash custodian 3,000
Personal check drawn by the petty cash custodian 2,400

The entry to record the replenishment of the petty cash fund includes
a. A debit to cash short/overage account of P2,190 and a credit to cash on hand of P9,450.
b. A credit to cash short/overage account of P810 and a credit to cash of P12,450.
c. A debit to cash short/overage account of P810 and a credit to petty cash fund of P12,450.
d. A debit to cash short/overage account of P2,190 and a credit to cash in bank of P9,450.

D (2,550 + 5,040 + 2,220 + 3,000) = 12,810 per count – 15,000 accountability = (2,190) shortage

“There is a time for everything, and a season for every activity under the
heavens;” (Ecclesiastes 3:1)- END –
Page |1

Chapter 3
Bank Reconciliation

1. It is a report that is prepared for the purpose of bringing the balances of cash per records and per
bank statement into agreement.
a. Bank statement
b. Check Disbursement Voucher
c. Bank reconciliation
d. Bank deposit slip

2. These are deposits made but not yet credited by the bank to the depositor’s bank account.
a. Credit memos (CM)
b. Debit memos (DM)
c. Outstanding checks (OC)
d. Deposits in transit (DIT)

3. These are deductions made by the bank to the depositor’s bank account but not yet recorded by
the depositor.
a. Credit memos (CM)
b. Debit memos (DM)
c. Outstanding checks (OC)
d. Deposits in transit (DIT)

4. These are additions made by the bank to the depositor’s bank account but not yet recorded by
the depositor.
a. Credit memos (CM)
b. Debit memos (DM)
c. Outstanding checks (OC)
d. Deposits in transit (DIT)

5. These are checks drawn and released to payees but are not yet encashed with the bank.
a. Credit memos (CM)
b. Debit memos (DM)
c. Outstanding checks (OC)
d. Deposits in transit (DIT)

6. Which of the following is added to the cash balance per books when preparing a bank
reconciliation statement?
a. Credit memo
b. Debit memo
c. Outstanding check
d. Deposit in transit
Page |2

7. Which of the following is added to the cash balance per bank statement when preparing a bank
reconciliation statement?
a. Credit memo
b. Debit memo
c. Outstanding check
d. Deposit in transit

8. Which of the following represents a debit memo?


a. Collections made by the bank on behalf of the depositor.
b. Interest income earned by the deposit.
c. Loan proceeds directly credited or added by the bank to the depositor’s account.
d. Interest expense on a loan that is directly deducted from the depositor’s account.

9. Which of the following is not a debit memo?


a. Bank service charges
b. No sufficient funds checks (NSF)
c. Automatic debits representing payments of bills by the bank on behalf of the depositor
d. Direct deposits of customers to the depositor’s account

10. As an internal control, bank reconciliation statements are usually prepared


a. on a daily basis.
b. on a monthly basis.
c. annually every year-end.
d. whenever the accountant feels like it.

1. Entity A is preparing its November 30, 20x1 bank reconciliation statement. The following
information was determined:

• Cash balance per accounting books, Nov. 30, 20x1 ₱600,000


• Cash balance per bank statement, Nov. 30, 20x1 ₱860,000
• Credit memo ₱380,000
• Debit memo ₱ 60,000
• Deposits in transit ₱100,000
• Outstanding checks ₱ 40,000

Requirement: Prepare the bank reconciliation.

Bal. per books, end. ₱600,000 Bal. per bank, end. ₱860,000
Add: CM 380,000 Add: DIT 100,000
Less: DM (60,000) Less: OC (40,000)
Add/Less: Book errors - Add/Less: Bank errors -
Adjusted balance ₱920,000 Adjusted balance ₱920,000
Page |3

2. Entity A is preparing its February 28, 20x1 bank reconciliation statement. The following
information was determined:
• Cash balance per accounting books, Feb. 28, 20x1 ₱260,000
• Cash balance per bank statement, Feb. 28, 20x1 ₱205,000

When investigating the difference, the accountant determined the following:


a. A customer deposited ₱30,000 to Entity A’s bank account as payment for an account receivable.
This is not yet recorded in the books of accounts.
b. A ₱102,500 check deposited by Entity A during the month is not yet credited to Entity A’s
account.
c. A check drawn in the amount of ₱22,500 is not yet presented to the bank for payment.
d. The bank returned a check deposit amounting to ₱5,000 because of insufficiency in the funds of
the drawer. The check was received from a customer as payment for accounts receivable.

Requirements:
a. Prepare the bank reconciliation.
b. Prepare the adjusting (reconciling) entries.

Requirement (a): Bank reconciliation

Bal. per books, end. ₱260,000 Bal. per bank, end. ₱205,000
Add: CM 30,000 Add: DIT 102,500
Less: DM (5,000) Less: OC (22,500)
Add/Less: Book errors Add/Less: Bank errors
Adjusted balance ₱285,000 Adjusted balance ₱285,000

Requirement (b): Adjusting (Reconciling) entries

AJE (c) Cash 30,000


Accounts receivable 30,000
to record the collection of accounts receivable
AJE (d) Accounts receivable 5,000
Cash 5,000
to revert the NSF check back to accounts receivable
Page |4

1. Entity A is preparing its March 31, 20x1 bank reconciliation. The following information was
determined:
a. The cash balance per books is ₱280,000 while the cash balance per bank statement is ₱320,000.
b. Credit memo – ₱20,000
c. Debit memo – ₱15,000
d. Deposits in transit – ₱75,000
e. Outstanding checks – ₱25,000
f. The disbursements per books are overstated by ₱45,000.
g. The bank debits are understated by ₱40,000.

Requirement: Prepare the bank reconciliation.


SOLUTIONS

Bal. per books, end. 280,000 Bal. per bank, end. 320,000
Add: CM 20,000 Add: DIT 75,000
Less: DM (15,000) Less: OC (25,000)
Add/Less: Book errors: Add/Less: Bank errors:
Understatement 45,000 Overstatement (40,000)
Adjusted balance 330,000 Adjusted balance 330,000

2. Data concerning the cash records of Arones Company for the months of November and
December 20x1 are shown below:
November 30 December 31
Book balance
11,200 ?
Book debits 63,800
Book credits 56,400
Bank balance 30,000 40,800
Bank debits ?
Bank credits 54,600
Notes collected by bank 4,500 6,000
Bank service charge 40 200
NSF checks 1,760 2,800
Overstatement of check in payment
of salaries 3,800 2,400
Deposit in transit 12,000 22,500
Outstanding checks 19,500 35,700
Deposit of 123 Corporation erroneously
credited to ABC Co.’s account 4,800 3,600

Requirement: Prepare the proof of cash.


Page |5

Per books:
Nov. 30 Receipts Disbursements Dec. 31
Balance per books 11,200 63,800 56,400 18,600
Note collected by bank:
November 4,500 (4,500)
December 6,000 6,000
Bank service charges
November (40) (40)
December 200 (200)
NSF checks:
November (1,760) (1,760)
December 2,800 (2,800)
Book errors:
November 3,800 (3,800)
December (2,400) 2,400
Adjusted balances 17,700 61,500 55,200 24,000

Per bank:
Nov. 30 Receipts Disbursements Dec. 31
Balance per bank 30,000 54,600 43,800 40,800
Deposits in transit
November 12,000 (12,000)
December 22,500 22,500
Outstanding checks
November (19,500) (19,500)
December 35,700 (35,700)
Bank errors:
November (4,800) (4,800)
December (3,600) (3,600)
Adjusted balances 17,700 61,500 55,200 24,000
Page |1

Chapter 4

TRUE OR FALSE
1. Accounts receivable are to be reported at their net realizable value.

2. The direct write-off method for uncollectible accounts does not provide for the matching of
current revenues with related expenses.

3. The use of the direct write-off method is acceptable under generally accepted accounting
principles.

4. Doubtful accounts expense is normally reported as a deduction from sales in the income
statement.

5. The entry to write off an uncollectible account under the allowance method is a debit to Doubtful
Accounts Expense and a credit to Accounts Receivable.

6. The method of estimating uncollectible accounts expense based on the accounts receivable
balance emphasizes the determination of the net realizable value of the receivables.

7. When estimating collectability based on an analysis of the accounts receivable balance, any
existing balance in the allowance for doubtful accounts is ignored.

8. The aging method of estimating doubtful accounts is a variation of the percentage of ending
receivables method.

9. The "list" sales price less any trade discount is the invoice amount.

10. Sales discounts are normally reported as selling expenses.

“The heart of the discerning acquires knowledge, for the ears of the wise seek it
out.” (Proverbs 18:15)
- END –

ANSWERS
1. TRUE 6. TRUE
2. TRUE 7. FALSE
3. FALSE 8. TRUE
Page |2

4. FALSE 9. TRUE
5. FALSE 10. FALSE
Page |3

1. Which of the following is incorrect?


a. The operating cycle always is one year in duration.
b. The operating cycle sometimes is longer than one year in duration.
c. The operating cycle sometimes is shorter than one year in duration.
d. The operating cycle is a concept applicable both to manufacturing and retailing enterprises.

2. The category "trade receivables" includes


a. advances to officers and employees.
b. income tax refunds receivable.
c. claims against insurance companies for casualties sustained.
d. none of these.

3. Which of the following should be recorded in Accounts Receivable?


a. Receivables from officers
b. Receivables from subsidiaries
c. Dividends receivable
d. None of these

4. When the direct write-off method of recognizing bad debt expense is used, the entry to write off
a specific customer account would
a. increase net income.
b. have no effect on net income.
c. increase the accounts receivable balance and increase net income.
d. decrease the accounts receivable balance and decrease net income.

5. When comparing the allowance method of accounting for bad debts with the direct write-off
method, which of the following is true?
a. The direct write-off method is exact and also better illustrates the matching principle.
b. The allowance method is less exact but it better illustrates the matching principle.
c. The direct write-off method is theoretically superior.
d. The direct write-off method requires two separate entries to write off an uncollectible
account.

6. When the allowance method of recognizing bad debt expense is used, the entry to record the
write-off of a specific uncollectible account would decrease
a. allowance for doubtful accounts.
b. net income.
c. net realizable value of accounts receivable.
d. working capital.

7. When a specific customer's account is written off by a company using the allowance method, the
effect on net income and the net realizable value of the accounts receivable is
Net Realizable Value
Net Income of Accounts Receivable
Page |4

a. None None
b. Decrease Decrease
c. Increase Increase
d. Decrease None

8. When the allowance method of recognizing bad debt expense is used, the entries at the time of
collection of a small account previously written off would
a. increase net income.
b. increase the allowance for doubtful accounts.
c. decrease net income.
d. decrease the allowance for doubtful accounts.

9. A method of estimating bad debts that focuses on the balance sheet rather than the income
statement is the allowance method based on
a. direct write-off.
b. aging the trade receivable accounts.
c. credit sales.
d. specific accounts determined to be uncollectible.

10. The entry

Accounts Receivable xxx


Allowance for Uncollectible Accounts xxx

would be made when


a. a customer pays its account balance.
b. a customer defaults on its account.
c. a previously defaulted customer pays its outstanding balance.
d. estimated uncollectible receivables are too low.

“Do not be wise in your own eyes; fear the LORD and shun evil. “
(Proverbs 3:7)

- END –
Page |5

1. At January 1, 20x1, Judy Co. had a credit balance of ₱260,000 in its allowance for uncollectible
accounts. Based on past experience, 2% of Judy 's credit sales have been uncollectible. During
20x1, Judy wrote off ₱325,000 of uncollectible accounts. Credit sales for 20x1 were ₱9,000,000. In
its December 31, 20x1, balance sheet, what amount should Judy report as allowance for
uncollectible accounts?
a. 115,000
b. 180,000
c. 245,000
d. 440,000

A (260K + (2% x 9M) – 325K = 115K

2. On the December 31, 20x6, balance sheet of Esther Co., the current receivables consisted of the
following:

Trade accounts receivable 93,000


Allowance for uncollectible accounts (2,000)
Claim against shipper for goods lost in transit (November 20x6) 3,000
Selling price of unsold goods sent by Esther on consignment at
130% of cost (not included in Esther's ending inventory) 26,000
Security deposit on lease of warehouse used for storing some
inventories 30,000
Total 150,000

At December 31, 20x6, the correct total of Esther's current net receivables was
a. 94,000
b. 120,000
c. 124,000
d. 150,000

A (93,000 – 2,000 + 3,000) = 94,000

3. The following information is from the records of Prosser, Inc. for the year ended December 31,
2002.
Allowance for Doubtful Accounts, January 1, 2002 .. ₱ 6,000 (cr)
Sales, 2002 ....................................... 2,920,000
Sales Returns and Allowances, 2002 ................ 32,000

If the basis for estimating bad debts is 1% of net sales, the correct amount of doubtful accounts
expense for 2002 is
a. ₱22,800.
b. ₱23,200.
c. ₱28,880.
Page |6

d. ₱34,880.

C (2,920,000 – 32,000) x 1% = 28,880

4. An analysis and aging of the accounts receivable of Shriner Company at December 31 revealed
the following data:

Accounts Receivable .................................. ₱450,000


Allowance for Doubtful Accounts (before adjustment) .. 25,000 (cr)
Required ending balance of allowance ............... 32,000 (cr)

The net realizable value of the accounts receivable at December 31 should be


a. ₱450,000.
b. ₱443,000.
c. ₱425,000.
d. ₱418,000.

D (450,000 – 32,000) = 418,000

5. Maple Company provides for doubtful accounts expense at the rate of 3 percent of credit sales.
The following data are available for last year:

Allowance for Doubtful Accounts, January 1 ........ ₱ 54,000 (cr)


Accounts written off as uncollectible during the
year ............................................ 60,000
Collection of accounts written off in prior years .
(customer credit was re-established) .............. 15,000
Credit sales, year-ended December 31 .............. 3,000,000

The allowance for doubtful accounts balance at December 31, after adjusting entries, should be
a. ₱45,000.
b. ₱84,000.
c. ₱90,000.
d. ₱99,000.

D [54,000 – 60,000 + 15,000 + (3,000,000 x 3%)] = 99,000

6. Based on the aging of its accounts receivable at December 31, Pribob Company determined that
the net realizable value of the receivables at that date is ₱760,000. Additional information is as
follows:
Accounts Receivable at December 31 ................ ₱880,000
Page |7

Allowance for Doubtful Accounts at January 1 ...... 128,000 (cr)


Accounts written off as uncollectible during the
year ............................................ 88,000

Pribob's doubtful accounts expense for the year ended December (31 is
a. ₱80,000.
b. ₱96,000.
c. ₱120,000.
d. ₱160,000.

A
Allowance for doubtful accounts
128,000 beg.
Write-offs 88,000 80,000 Bad debts expense (squeeze)
- Recoveries
end. 120,000 a

a
(880,000 – 760,000) = 120,000

7. Based on its past collection experience, Ace Company provides for bad debts at the rate of 2% of
net credit sales. On January 1, 2002, the allowance for doubtful accounts credit balance was
₱10,000. During 2002, Ace wrote off ₱18,000 of uncollectible receivables and recovered ₱5,000 on
accounts written off in prior years. If net credit sales for 1999 totaled ₱1,000,000, the doubtful
accounts expense for 2002 should be
a. ₱17,000.
b. ₱20,000.
c. ₱23,000.
d. ₱35,000.

B (1,000,000 x 2%) = 20,000

8. Richards Company uses the allowance method of accounting for bad debts. The following
summary schedule was prepared from an aging of accounts receivable outstanding on
December 31 of the current year.

No. of Days Probability


Outstanding Amount of Collection
0-30 days ₱500,000 .98
31-60 days 200,000 .90
Over 60 days 100,000 .80

The following additional information is available for the current year:


Page |8

Net credit sales for the year .................. ₱4,000,000


Allowance for Doubtful Accounts:
Balance, January 1 ............................. 45,000 (cr)
Balance before adjustment, December 31 ......... 2,000 (dr)

If Richards determines bad debt expense using 1.5 percent of net credit sales, the net realizable value
of accounts receivable on the December 31 balance sheet will be
a. ₱738,000.
b. ₱740,000.
c. ₱742,000.
d. ₱750,000.

C
Allowance for doubtful
accounts
Dec. 31 (unadjusted) 2,000
Write-offs - 60,000 Bad debts (4M x 1.5%)
- Recoveries
end. 58,000

(500,000 + 200,000 + 100,000) = 800,000 – 58,000 = 742,000

9. Gekko, Inc. reported the following balances (after adjustment) at the end of 2002 and 2001.
12/31/2002 12/31/2001
Total accounts receivable ................. ₱105,000 ₱96,000
Net accounts receivable ................... 102,000 94,500

During 2002, Gekko wrote off customer accounts totaling ₱3,200 and collected ₱800 on accounts
written off in previous years. Gekko's doubtful accounts expense for the year ending December 31,
2002 is
a. ₱1,500.
b. ₱2,400.
c. ₱3,000.
d. ₱3,900.

D
Allowance for doubtful accounts
1,500 beg. (96K - 94.5K)
Write-offs 3,200 3,900 Bad debts (squeeze)
800 Recoveries
end. (105K - 102K) 3,000
Page |9

10. Gray Company had an accounts receivable balance of ₱50,000 on December 31, 2001, and
₱75,000 on December 31, 2002. The company wrote off ₱20,000 of accounts receivable during
2002, and collected ₱3,000 on an account written off in 2000. Sales for the year 2002 totaled
₱620,000. All sales were on account. The amount collected from customers on accounts
receivable during 2002, including recoveries, was
a. ₱575,000.
b. ₱578,000.
c. ₱600,000.
d. ₱595,000.

1. B
Accounts receivable
beg. 50,000
Sales on account 620,000 578,000 Collections, including recoveries
Recoveries 3,000 20,000 Write-offs
75,000 end.

“For the Lord gives wisdom; from his mouth come knowledge and understanding.”
(Proverbs 2:6)
- END -
Page |1

Chapter 5
Notes Receivable
1. Present value is
a. the value now of a future amount.
b. the amount that must be invested now to produce a known future value.
c. always smaller than the future value.
d. all of these.

2. Which of the following factors would show the largest value for an interest rate of 12% for six
periods?
a. Present value of 1
b. Present value of an ordinary annuity of 1
c. Present value of an annuity due of 1
d. Answer cannot be determined

3. What factor should you use if you want to determine the value now of a ₱1,000 payment due in
three years’ time?
a. Future value of 1
b. Present value of 1
c. Present value of an ordinary annuity of 1
d. Present value of an annuity due of 1

4. What factor should you use for a ₱1,000 note receivable that is collectible in five annual
installments of ₱200 starting one year hence?
a. Present value of 1
b. Present value of an ordinary annuity of 1
c. Present value of an annuity due of 1
d. Any of these

5. What factor should you use for a ₱2,000 note receivable that is collectible in full after five years?
a. Present value of 1
b. Present value of an ordinary annuity of 1
c. Present value of an annuity due of 1
d. Any of these

6. Which of the following results to the smallest value?


a. Present value of an annuity due of 1 @12%, n=5
b. Present value of an ordinary annuity of 1 @12%, n=5
c. Present value of 1 @12%, n=5
d. Present value of 1 @14%, n=5

7. A higher interest rate results to


a. increased amount of present value.
b. decreased amount of present value.
c. same amount of present value.
d. Answer cannot be determined due to insufficient data
Page |2

8. A shorter period results to


a. increased amount of present value.
b. decreased amount of present value.
c. same amount of present value.
d. shorter accountant.

9. The present value of 1 for a period of zero equals


a. 1.
b. 0.
c. Error!
d. Answer depends on the interest rate

10. Multiplying a lump sum future amount by a Present Value of 1 factor results to
a. Future amount.
b. Future value of 1.
c. Present value.
d. Present value of 1.

“Be kind and compassionate to one another, forgiving each other, just as in Christ
God forgave you.” (Ephesians 4:32)

- END -
Page |3

1. An entity sells goods either on cash basis or on 6-month installment basis. On January 1, 20x1,
goods with cash price of ₱50,000 were sold at an installment price of ₱75,000. Which of the
following statements is correct?
a. Net receivable of ₱75,000 is recognized on the date of sale.
b. Net receivable of ₱50,000 is recognized upon full payment of the total price.
c. The ₱20,000 difference between the cash price and installment price is recognized as interest
income on the date of sale.
d. Net receivable of ₱50,000 is recognized on the date of sale.

2. An entity sells goods for ₱150,000 to a customer who was granted a special credit period of 1
year. The entity normally sells the goods for ₱120,000 with a credit period of one month or with
a ₱10,000 discount for outright payment in cash. How much is the initial measurement of the
receivable?
a. 150,000
b. 120,000
c. 130,000
d. 110,000

Solution:
Normal selling price with credit period of one month 120,000
Discount for cash on delivery (10,000)
Cash price equivalent of the goods sold 110,000

Use the following information for the next two questions:


On January 1, 20x1, ABC Co. sold a transportation equipment with a historical cost of ₱1,000,000 and
accumulated depreciation of ₱300,000 in exchange for cash of ₱100,000 and a noninterest-bearing
note receivable of ₱800,000 due on January 1, 20x4. The prevailing rate of interest for this type of
note is 12%.

3. How much is the interest income in 20x1?


a. 68,331
b. 76,532
c. 85,714
d. 96,000
A
Solution:
Initial measurement: 800,000 x PV of 1 @12%, n=3 = 569,424

Subsequent measurement:
Date Interest income Unearned interest Present value
1/1/x1 230,576 569,424
12/31/x1 68,331 162,245 637,755
12/31/x2 76,531 85,714 714,286
12/31/x3 85,714 - 800,000

4. How much is the carrying amount of the receivable on December 31, 20x2?
a. 800,000
b. 569,424
c. 637,755
Page |4

d. 714,286

(See solution above)

Use the following information for the next three questions:


On January 1, 20x1, ABC Co. sold transportation equipment with a historical cost of ₱20,000,000 and
accumulated depreciation of ₱7,000,000 in exchange for cash of ₱500,000 and a noninterest-bearing
note receivable of ₱8,000,000 due in 4 equal annual installments starting on December 31, 20x1 and
every December 31 thereafter. The prevailing rate of interest for this type of note is 12%.

5. How much is the interest income in 20x1?


a. 728,946
b. 678,334
c. 728,964
d. 704,236

Solution:
Initial measurement: (8M ÷ 4) x PV ordinary annuity of 1 @12%, n=4 = 6,074,699

Subsequent measurement:
Interest
Date Collections Amortization Present value
income
1/1/20x1 6,074,699
12/31/20x1 2,000,000 728,964 1,271,036 4,803,663
12/31/20x2 2,000,000 576,440 1,423,560 3,380,102
12/31/20x3 2,000,000 405,612 1,594,388 1,785,714
12/31/20x4 2,000,000 214,286 1,785,714 0

6. How much is the current portion of the receivable on December 31, 20x1?
a. 1,271,036
b. 1,423,560
c. 3,380,102
d. 1,594,388

(See solution above)

7. How much is the carrying amount of the receivable on December 31, 20x2?
a. 4,803,663
b. 3,380,102
c. 6,074,699
d. 6,000,000

(See solution above)

Use the following information for the next three questions:


On January 1, 20x1, ABC Co. sold transportation equipment with a historical cost of ₱12,000,000 and
accumulated depreciation of ₱7,000,000 in exchange for cash of ₱100,000 and a noninterest-bearing
note receivable of ₱4,000,000 due in 4 equal annual installments starting on January 1, 20x1 and
every January 1 thereafter. The prevailing rate of interest for this type of note is 12%.
Page |5

8. How much is the interest income in 20x1?


a. 408,230
b. 278,334
c. 328,964
d. 288,220

(See solution below)

9. How much is the carrying amount of the receivable on December 31, 20x1?
a. 1,690,510
b. 892,857
c. 2,690,051
d. 1,594,388

C
Solutions:
Initial measurement: (4M ÷ 4) x PV annuity due of 1 @12%, n=4 = 3,401,831

Subsequent measurement:
Interest
Date Collections Amortization Present value
income
Jan. 1, 20x1 3,401,831
Jan. 1, 20x1 1,000,000 - 1,000,000 2,401,831
Jan. 1, 20x2 1,000,000 288,220 711,780 1,690,051
Jan. 1, 20x3 1,000,000 202,806 797,194 892,857
Jan. 1, 20x4 1,000,000 107,143 892,857 0

The carrying amount of the notes receivable as of December 31, 20x1 is determined as follows:
Carrying amount of notes receivable - Jan. 1, 20x2 1,690,051
Add back: Collection on Jan. 1, 20x2 1,000,000
Carrying amount of notes receivable - Dec. 31, 20x1 2,690,051

10. How much is the carrying amount of the receivable on January 1, 20x3?
a. 892,857
b. 3,380,102
c. 6,074,699
d. 6,000,000

(See solution above)

Use the following information for the next two questions:


On January 1, 20x1, ABC Co. sold machinery with historical cost of ₱3,000,000 and accumulated
depreciation of ₱900,000 in exchange for a 3-year, ₱2,100,000 noninterest-bearing note receivable due
in equal semi-annual payments every July 1 and December 31 starting on July 1, 20x1. The
prevailing rate of interest for this type of note is 10%.

11. How much is the interest income in 20x1?


a. 88,825
b. 177,649
c. 128,964
Page |6

d. 164,591

D
Solution:
Initial measurement: (2.1M ÷ 6) x PV ordinary annuity of 1 @5%, n=6 = 1,776,492

Subsequent measurement:
Interest
Date Collections Amortization Present value
income
Jan. 1, 20x1 1,776,492
July 1, 20x1 350,000 88,825 261,175 1,515,317
Dec. 31, 20x1 350,000 75,766 274,234 1,241,083
July 1, 20x2 350,000 62,054 287,946 953,137
Dec. 31, 20x2 350,000 47,657 302,343 650,794
July 1, 20x3 350,000 32,540 317,460 333,333
Dec. 31, 20x3 350,000 16,667 333,333 0

Interest income in 20x1 = (88,825 + 75,766) = 164,591

12. How much is the carrying amount of the receivable on December 31, 20x1?
a. 1,241,083
b. 982,378
c. 1,690,051
d. 1,594,388

(See solution above)

13. On January 1, 20x1, ABC Co. sold machinery costing ₱3,000,000 with accumulated depreciation
of ₱1,100,000 in exchange for a 3-year, ₱900,000 noninterest-bearing note receivable due as
follows:
Date Amount of installment
December 31, 20x1 400,000
December 31, 20x2 300,000
December 31, 20x3 200,000
Total 900,000

The prevailing rate of interest for this type of note is 10%. How much is the carrying amount of
the receivable on December 31, 20x1?
a. 467,354
b. 438,016
c. 376,345
d. 428,346
B
Solution:
Initial measurement:
PV of P1 @ 10%,
Date Collections Present value
n= 1 to 3
Dec. 31, 20x1 400,000 0.90909 363,636
Dec. 31, 20x2 300,000 0.82645 247,935
Dec. 31, 20x3 200,000 0.75131 150,262
Totals 900,000 761,833

Subsequent measurement:
Page |7

Date Collections Interest income Amortization Present value


Jan. 1, 20x1 761,833
Dec. 31, 20x1 400,000 76,183 323,817 438,016
Dec. 31, 20x2 300,000 43,802 256,198 181,818
Dec. 31, 20x3 200,000 18,182 181,818 0

Use the following information for the next two questions:


On January 1, 20x1, ABC Co. sold inventory costing ₱1,800,000 with a list price of ₱2,200,000 and a
cash price of ₱2,000,000 in exchange for a ₱2,400,000 noninterest-bearing note due on December 31,
20x3.

14. How much is the initial measurement of the receivable?


a. 1,800,000
b. 2,200,000
c. 2,000,000
d. 2,400,000

C – equal to cash price equivalent.

15. How much is the carrying amount of the receivable on December 31, 20x1?
a. 2,125,390
b. 2,135,341
c. 2,098,343
d. 2,000,000

A
Solution:
First trial: (at 10%)
Future cash flows x PV factor at x% = PV of note
 2,400,000 x PV of P1 @ 10%, n=3 = 2,000,000
 (2,400,000 x 0.751315) = 1,803,156 is not equal to 2,000,000
We need a substantially higher amount of present value. Therefore, we need to decrease substantially
the interest rate. Let’s try 6%.

Second trial: (at 6%)


Future cash flows x PV factor at x% = PV of note
 2,400,000 x PV factor at 6%, n=3 = 2,000,000
 (2,400,000 x 0.839619) = 2,015,086 is not equal to 2,000,000
We need a slightly lower amount of present value. Therefore, we need to increase slightly the interest
rate. Let’s try 7%.

Third trial: (at 7%)


Future cash flows x PV factor at x% = PV of note
 2,400,000 x PV factor at 7%, n=3 = 2,000,000
 (2,400,000 x 0.816298) = 1,959,115 is not equal to 2,000,000

In here, we need to perform interpolation. Looking at the values derived above, we can reasonably
expect that the effective interest rate is a rate between 6% and 7%.

To perform the interpolation, we will use the following formula:


x% - 6%
7% - 6%

Where: x% again is the effective interest rate.


Page |8

The formula was derived based on our expectation that the effective interest rate is somewhere between
6% and 7%.Notice that the lower rate appears in both the numerator and denominator of the formula
while x% appears in the numerator.

Let us substitute the amounts of present values computed earlier on the formula.
2,000,000 - 2,015,086 (15,086)
= = 0.2695
1,959,115 - 2,015,086 (55,970)

The amount computed is added to 6% to derive the effective interest rate. The effective interest rate is
6.2695% (6% + .2695%).

If other methods or tools were used, such as a financial calculator or spreadsheet application, the exact
rate is 6.265856927%.

The amortization table using 6.2695% as the effective interest rate is presented below.
Date Interest income Unearned interest Present value
Jan. 1, 20x1 400,000 2,000,000
Dec. 31, 20x1 125,390 274,610 2,125,390
Dec. 31, 20x2 133,251 141,359 2,258,641
Dec. 31, 20x3 141,606 -247 2,400,247

Notice that there is still a slight difference of ₱247. However, if this is deemed immaterial, we can regard
the computed rate as the effective interest rate.

“Not only so, but we also glory in our sufferings, because we know that suffering
produces perseverance;” (Romans 5:3)

-END-
Page |1

Chapter 6
Receivables – Additional Concepts

1. A 180-day, 12 percent interest-bearing note receivable is sold to a bank after being held for 45
days. The proceeds are calculated using a 15 percent interest rate. The note receivable has been
Discounted Pledged
a. Yes Yes
b. Yes No
c. No Yes
d. No No

2. A 90-day, 15 percent interest-bearing note receivable was immediately discounted at a bank at 12


percent. The proceeds received from the bank upon discounting would be the
a. maturity value less the discount at 15 percent.
b. maturity value less the discount at 12 percent.
c. face value less the discount at 15 percent.
d. face value less the discount at 12 percent.

3. The balance in Accounts Receivable is not reduced in recording which of the following types of
financing arrangements?
a. Assignment of specific accounts receivable
b. General assignment (pledge) of accounts receivable
c. Factoring of accounts receivable
d. Transfer of accounts receivable without recourse

4. When the accounts receivable of a company are sold outright to a company that normally buys
accounts receivable of other companies without recourse, the accounts receivable have been
a. transferred with recourse.
b. factored.
c. assigned.
d. pledged.

5. Which of the following is most likely not a condition before a transfer of receivables is accounted
for as a sale?
a. The transferred assets have been isolated from the transferor.
b. The transferor's obligation under the recourse provisions can be reasonably estimated.
c. The transferee has the right to pledge or exchange the transferred assets.
d. The transferor does not maintain effective control over the assets through an agreement to
repurchase the assets before their maturity.

6. Which of the following is true when accounts receivable are factored without recourse?
a. The transaction may be accounted for either as a secured borrowing or as a sale, depending
upon the substance of the transaction.
Page |2

b. The receivables are used as collateral for a promissory note issued to the factor by the owner
of the receivables.
c. The factor assumes the risk of collectability and absorbs any credit losses in collecting the
receivables.
d. The financing cost (interest expense) should be recognized ratably over the collection period
of the receivables.

7. Which of the following is a method of generating cash from accounts receivable?


Assignment Factoring
a. Yes No
b. Yes Yes
c. No Yes
d. No No

8. Which of the following may be used to compute for the net proceeds from the discounting of a
note receivable?
a. Maturity value multiplied by the Discount
b. Maturity value less the Discount
c. Face amount less the Discount
d. Maturity value multiplied by the Discount rate multiplied by the Discount period.

9. The entry to record a note receivable discounted with a bank most likely includes
a. Debiting cash equal to the maturity value of the note
b. Crediting note receivable equal to the maturity value of the note
c. Debiting cash equal to the face amount of the note
d. Crediting note receivable equal to the face amount of the note

10. Which of the following may result to the derecognition of a receivable?


a. The contractual rights to the cash flows from the receivable expire.
b. The receivable is transferred and the transferor retains control over the transferred receivable.
c. The receivable is impaired.
d. a and c.

“Jesus replied: ‘Love the Lord your God with all your heart and with all your soul
and with all your mind.’ This is the first and greatest commandment. And the
second is like it: ‘Love your neighbor as yourself.’” (Matthew 22:37-39)

-END-
Page |3

ANSWERS TO
1. B 6. C
2. B 7. B
3. B 8. B
4. B 9. D
5. B 10. D
Page |4

Use the following information for the next two questions:


Jason Co. assigned ₱1,000,000 of accounts receivable to Easy Finance Co. as security for a loan of
₱840,000. Easy charged a 2% commission on the amount of the loan; the interest rate on the note was
10%. During the first month, Jason collected ₱220,000 on the assigned accounts after deducting ₱760
of discounts. Jason accepted returns worth ₱2,700 and wrote off assigned accounts totaling ₱7,400.

1. The amount of cash Jason received from Easy at the time of the transfer was
a. ₱756,000. c. ₱823,200.
b. ₱820,000. d. ₱840,000.

C ₱840,000 – (₱840,000 x 2%) = ₱823,200.

2. Entries during the first month would include a


a. debit to Cash of ₱220,760.
b. debit to Bad Debt Expense of ₱7,400.
c. debit to Allowance for Doubtful Accounts of ₱7,400.
d. debit to Accounts Receivable of ₱230,860.

Use the following information for the next two questions:


On February 1, 2004, Norton Company factored receivables with a carrying amount of ₱500,000 to
Koch Company. Koch Company assessed a finance charge of 3% of the receivables and retains 5% of
the receivables. Relative to this transaction, you are to determine the amount of loss on sale to be
reported in the income statement of Norton Company for February.

3. Assume that Norton factors the receivables on a without recourse basis. The loss to be reported
is
a. ₱0. c. ₱25,000.
b. ₱15,000. d. ₱40,000.

B ₱500,000 × .03 = ₱15,000.

4. Assume that Norton factors the receivables on a with recourse basis. The recourse obligation has
a fair value of ₱2,500. The loss to be reported is
a. ₱15,000. c. ₱25,000.
b. ₱17,500. d. ₱42,500.

B (₱500,000 × .03) + ₱2,500 = ₱17,500.

5. On September 1, Riva Co. assigns specific receivables totaling ₱750,000 to Pacific Bank as
collateral on a ₱625,000, 12 percent note. Riva Co. will continue to collect the assigned accounts
receivable. Pacific also assesses a 2% service charge on the total accounts receivable assigned.
Riva Co. is to make monthly payments to Pacific with cash collected on assigned accounts
receivable. Collections of assigned accounts during September totaled ₱260,000 less cash
Page |5

discounts of ₱3,500. What were the proceeds from the assignment of Riva's accounts receivable
on September 1?
a. ₱610,000
b. ₱612,500
c. ₱625,000
d. ₱735,000

A [625,000 – (750,000 x 2%)] = 610,000

6. On September 1, Riva Co. assigns specific receivables totaling ₱750,000 to Pacific Bank as
collateral on a ₱625,000, 12 percent note. Riva Co. will continue to collect the assigned accounts
receivable. Pacific also assesses a 2 percent service charge on the total accounts receivable
assigned. Riva Co. is to make monthly payments to Pacific with cash collected on assigned
accounts receivable. Collections of assigned accounts during September totaled ₱260,000 less
cash discounts of ₱3,500. What amount is owed to Pacific by Riva Co. for September collections
plus accrued interest on the note to September 30?
a. ₱260,000
b. ₱262,750
c. ₱264,000
d. ₱266,250

B (260,000 – 3,500) + (625,000 x 12% x 1/12) = 262,750


1/12? - Sept. 1 – Sept. 30

7. Simpson Company held a ₱6,000, 3-month, 15 percent note. One month before maturity, it
discounted the note at 10 percent at a local bank. Approximately how much net income did
Simpson earn on the note?
a. ₱173
b. ₱52
c. ₱225
d. ₱60

A
MV = 6,000 + (6,000 x 15% x 3/12) = 6,225
D = 6,225 x 10% x 1/12 = 51.88
NP = 6,225 – 51.88 = 6,173.12
Net interest = 6,173.12 net proceeds less 6,000 face amount = 173.12
Page |6

8. If a 3-month non-interest-bearing note receivable of ₱10,000 is discounted at a bank at 10 percent,


how much cash is received?
a. ₱10
b. ₱1,010
c. ₱999
d. ₱9,750

D
MV = 10,000 + (10,000 x 0% x 3/12) = 10,000
D = 10,000 x 10% x 3/12 = 250
NP = 10,000 – 250 = 9,750

9. On January 1, Parent Company gave Kids, Inc. a ₱5,000, 2-month, 6 percent note in payment of
its account. One month later, Kids discounted the note at the bank at 8 percent. The cash that
Kids received from the bank was (rounded to the nearest dollar)
a. ₱4,960. c. ₱5,016.
b. ₱5,010. d. ₱5,022.

C
MV = 5,000 + (5,000 x 6% x 2/12) = 5,050
D = 5,050 x 8% x 1/12 = 33.67
NP = 5,050 – 33.67 = 5,016.33

10. On June 1, Clinton Corporation accepted a customer's ₱10,000, 9 percent, 3 month note. On July
1, the note was discounted at a bank at a rate of 12 percent. How much cash did Clinton receive
from the bank on the discounted note?
a. ₱9,800.00 c. ₱10,020.50
b. ₱9,942.50 d. ₱10,250.00

C
MV = 10,000 + (10,000 x 9% x 3/12) = 10,225
D = 10,225 x 12% x 2/12 = 204.50
NP = 10,225 – 204.50 = 10,020.50

“Bear in mind that our Lord’s patience means salvation, just as our dear
brother Paul also wrote you with the wisdom that God gave him.” (2 Peter 3:15)
- END -
Page |1

Chapter 7
Inventories
: TRUE OR FALSE

1. In a period of rising prices, the use of FIFO relates the current high costs of acquiring goods with
rising sales prices. As a result, FIFO tends to have a stabilizing effect on gross profit margins.

2. The gross method of accounting for purchase discounts is theoretically preferable to the net
method.

3. The gross method of accounting for purchase discounts reflects the fact that discounts not taken
are in effect credit-related expenditures incurred for failure to pay within the discount period.

4. The specific identification method is a highly objective approach to matching historical costs
with revenues.

5. The specific identification, as an inventory method, matches the flow of recorded costs to the
physical flow of goods.

6. With FIFO, inventories are reported on the balance sheet at or near their current value.

7. Unlike other inventory cost methods, the average cost approach provides the same unit cost for
items of equal utility.

8. FIFO provides income tax savings during periods of falling prices.

9. Inventories are measured at net realizable value (NRV).

10. Inventory write-downs and reversals of write-downs are always recognized in profit or loss.

“Put on the full armor of God, so that you can take your stand against the
devil’s schemes.” (Ephesians 6:11)
-END-
Page |2

ANSWERS

1. FALSE – Under FIFO, COGS is measured using the cost of the earliest purchases. Accordingly in
a period of rising prices, the FIFO method matches lower costs to rising sales prices.

2. FALSE
3. FALSE – net method
4. TRUE
5. TRUE
6. TRUE
7. TRUE
8. TRUE
9. FALSE – Lower of Cost and NRV
10. TRUE
Page |3

1. When using a perpetual inventory system,


a. no Purchases account is used.
b. a Cost of Goods Sold account is used.
c. two entries are required to record a sale.
d. all of these.

2. Goods in transit which are shipped f.o.b. shipping point should be


a. included in the inventory of the seller.
b. included in the inventory of the buyer.
c. included in the inventory of the shipping company.
d. none of these.

3. Which of the following items should be included in a company's inventory at the balance sheet
date?
a. Goods in transit which were purchased f.o.b. destination.
b. Goods received from another company for sale on consignment.
c. Goods sold to a customer which are being held for the customer to call for at his or her
convenience.
d. None of these.

Use the following information for the next two questions:


During 2004 Elway Corporation transferred inventory to Howell Corporation and agreed to
repurchase the merchandise early in 2005. Howell then used the inventory as collateral to borrow
from Norwalk Bank, remitting the proceeds to Elway. In 2005 when Elway repurchased the
inventory, Howell used the proceeds to repay its bank loan.

4. This transaction is known as a(n)


a. consignment.
b. installment sale.
c. assignment for the benefit of creditors.
d. product financing arrangement.

5. On whose books should the cost of the inventory appear at the December 31, 2004 balance sheet
date?
a. Elway Corporation
b. Howell Corporation
c. Norwalk Bank
d. Howell Corporation, with Elway making appropriate note disclosure of the transaction

6. Dane Co. received merchandise on consignment. As of March 31, Dane had recorded the
transaction as a purchase and included the goods in inventory. None of the consigned goods
have been sold during the period. The effect of this on its financial statements for March 31
would be
a. no effect.
b. net income was correct and current assets and current liabilities were overstated.
Page |4

c. net income, current assets, and current liabilities were overstated.


d. net income and current liabilities were overstated.

7. Eller Co. received merchandise on consignment. As of January 31, Eller included the goods in
inventory, but did not record the transaction. The effect of this on its financial statements for
January 31 would be
a. net income, current assets, and retained earnings were overstated.
b. net income was correct and current assets were understated.
c. net income and current assets were overstated and current liabilities were understated.
d. net income, current assets, and retained earnings were understated.

8. On June 15, 2004, Stilley Corporation accepted delivery of merchandise which it purchased on
account. As of June 30, Stilley had not recorded the transaction or included the merchandise in
its inventory. Stilley uses the periodic inventory system. The effect of the error on Stilley’s
balance sheet on June 30, 2004 would be
a. assets and stockholders' equity were overstated but liabilities were not affected.
b. stockholders' equity was the only item affected by the omission.
c. assets, liabilities, and stockholders' equity were understated.
d. none of these.

9. The use of a Discounts Lost account implies that the recorded cost of a purchased inventory item
is its
a. invoice price.
b. invoice price plus the purchase discount lost.
c. invoice price less the purchase discount taken.
d. invoice price less the purchase discount allowable whether taken or not.

10. In situations where there is a rapid turnover, an inventory method which produces a balance
sheet valuation similar to the first-in, first-out method is
a. average cost. c. joint cost.
b. base stock. d. prime cost.

“Be joyful in hope, patient in affliction, faithful in prayer.” (Romans 12:12)

- END -

Explanation to #9 – Stilley Corp.


Facts: The purchase on account is not recorded. The goods purchased are not included in the
physical count.
Page |5

Assumption: The cost of the unrecorded inventory is ₱10. ‘Net purchases’ is understated by
₱10 because the purchase
"should be" "made" transaction was not recorded.
Sales 200 200
BI 5 5
Ending inventory is understated by
Net purchases 100 90 ₱10 because the goods purchased
TGAS 105 95 were not included in the physical
EI (30) (20) count.
COGS 75 75
Gross profit 125 125
Profit is not affected.

Conclusion: Since profit is unaffected, equity is also unaffected. However:


1. Assets are understated because “Ending inventory” is understated.
2. Liabilities are understated because “Accounts payable” is understated.

Choices (a), (b) and (c) are all incorrect.

The answer is choice (d).


Page |6

1. The following information is available for Kerr Company for 2004:


Freight-in ₱ 60,000
Purchase returns 150,000
Selling expenses 300,000
Ending inventory 520,000

The cost of goods sold is equal to 300% of selling expenses. What is the cost of goods available for
sale?
a. ₱900,000. c. ₱1,330,000.
b. ₱1,480,000. d. ₱1,420,000.

D ₱520,000 + (3 × ₱300,000) = ₱1,420,000.

Use the following information for the next two questions:


Queen Co. records purchases at net amounts. On May 5 Queen purchased merchandise on account,
₱32,000, terms 2/10, n/30. Queen returned ₱2,000 of the May 5 purchase and received credit on
account. At May 31 the balance had not been paid.

2. The amount to be recorded as a purchase return is


a. ₱1,800. b. ₱2,040. c. ₱2,000. d. ₱1,960.

D ₱2,000 – (₱2,000 × .02) = ₱1,960

3. By how much should the account payable be adjusted on May 31?


a. ₱0. b. ₱680. c. ₱640. d. ₱600.

(₱32,000 – ₱2,000) × .02 = ₱600

Use the following information for the next two questions:


The following information was available from the inventory records of Moen Company for January:
Units Unit Cost Total Cost
Balance at January 1 3,000 ₱9.77 ₱29,310
Purchases:
January 6 2,000 10.30 20,600
January 26 2,700 10.71 28,917

Sales:
January 7 (2,500)
January 31 (3,200)
Balance at January 31 2,000
Page |7

4. Assuming that Moen does not maintain perpetual inventory records, what should be the
inventory at January 31, using the weighted-average inventory method, rounded to the nearest
peso?
a. ₱21,010. b. ₱20,474. c. ₱20,520. d. ₱20,720.

B (₱29,310 + ₱20,600 + ₱28,917) ÷ (3,000 + 2,000 + 2,700) = ₱10.237/unit


₱10.237 × 2,000 = ₱20,474.

5. Assuming that Moen maintains perpetual inventory records, what should be the inventory at
January 31, using the moving-average inventory method, rounded to the nearest peso?
a. ₱21,010. b. ₱20,474. c. ₱20,520. d. ₱20,720.

D Avg. on 1/6 ₱49,910 ÷ 5,000 = ₱9.982/unit


1/26 ₱53,872 ÷ 5,200 = ₱10.36/unit
₱10.36 × 2,000 = ₱20,720.

6. James Co. has the following data related to an item of inventory:


Inventory, March 1 200 units @ ₱4.20
Purchase, March 7 700 units @ ₱4.40
Purchase, March 16 140 units @ ₱4.50
Inventory, March 31 300 units

The value assigned to cost of goods sold if James uses FIFO is


a. ₱1,334. b. ₱1,280. c. ₱3,270. d. ₱3,216.

D 200 + 700 + 140 – 300 = 740 units


(200 × ₱4.20) + (540 × ₱4.40) = ₱3,216

Use the following information for the next two questions:


Transactions for the month of June were:
Purchases Sales
June 1 (balance) 1,200 @ ₱3.20 June 2 900 @ ₱5.50
3 3,300 @ 3.10 6 2,400 @ 5.50
7 1,800 @ 3.30 9 1,500 @ 5.50
15 2,700 @ 3.40 10 600 @ 6.00
22 750 @ 3.50 18 2,100 @ 6.00
25 450 @ 6.00
Page |8

7. Assuming that perpetual inventory records are kept in pesos, the ending inventory on a FIFO
basis is
a. ₱5,700. b. ₱5,760. c. ₱6,195. d. ₱6,300.

EI (in units) = 1,200 + 3,300 + 1,800 + 2,700 + 750 – 900 – 2,400 – 1,500 – 600 – 2,100 – 450 = 1,800

Units Unit cost Total cost


Ending inventory 1,800
From June 22 purchase (750) @3.50 2,625
balance 1,050
From June 15 purchase (1,050) @3.40 3,570
As allocated - 6,195

8. Assuming that perpetual inventory records are kept in units only, the ending inventory on an
average-cost basis, rounded to the nearest dollar, is
a. ₱5,940. b. ₱5,868. c. ₱5,910. d. ₱5,985.

Units Unit cost Total cost


June 1 (balance) 1,200 3.20 3,840
3 3,300 3.10 10,230
7 1,800 3.30 5,940
15 2,700 3.40 9,180
22 750 3.50 2,625
TGAS 9,750 31,815

TGAS (in units) = 1,200 + 3,300 + 1,800 + 2,700 + 750 = 9,750 units

Average cost = ₱31,815 ÷ 9,750 units = ₱3.26

Ending inventory (pesos) = 1,800 units x ₱3.26 = ₱5,868

9. The following information applied to Flynn, Inc. for 2004:


Merchandise purchased for resale ₱400,000
Freight-in 16,000
Page |9

Freight-out 10,000
Purchase returns 4,000

Flynn's 2004 inventoriable cost was


a. ₱400,000. b. ₱406,000. c. ₱412,000. d. ₱422,000.

C ₱400,000 + ₱16,000 – ₱4,000 = ₱412,000.

10. Tysen Retailers purchased merchandise with a list price of ₱90,000, subject to trade discounts of
20% and 10%, with no cash discounts allowable. Tysen should record the cost of this
merchandise as
a. ₱63,000. b. ₱64,800. c. ₱70,200. d. ₱90,000

B ₱90,000 × .8 × .9 = ₱64,800

“If you think education is expensive, try ignorance.” —Robert Orben


- END -
Page |1

Chapter 8
Inventory Estimation
1. On October 1, 20x1, the warehouse of ABC Co. and all inventories contained therein were
damaged by flood. Off-site back up of data base shows the following information:

Inventory, Jan. 1 10,000


Accounts payable, Jan. 1 3,000
Accounts payable, Sept. 30 2,000
Payments to suppliers 50,000
Freight-in 500
Purchase returns 500
Sales from Jan. to Sept. 80,000
Sales returns 5,000
Sales discounts 2,000
Gross profit rate based on sales 30%

Additional information:
Goods in transit as of October 1, 20x1 amounted to ₱1,000, cost of goods out on consignment is
₱1,200, and materials damaged by flood can be sold at a salvage value of ₱1,800. How much is the
inventory loss due to the flood?
a. 3,000 c. 4,400
b. 2,500 d. 4,900

A
Solution:
Accounts payable
3,000 Beginning balance
Payments to suppliers 50,000 49,000 Net purchases (squeeze)
Ending balance 2,000

The computed “Gross purchases” is extended to the “Inventory” T-account as follows:


Inventory
Beginning balance 10,000
Net purchases 49,000
Freight-in 500 52,500 Cost of goods sold *
7,000 End. bal. (squeeze)

*“Cost of goods sold” is computed as follows:


Gross sales 80,000
Sales returns (5,000)
Net sales 75,000
Multiply by: Cost ratio (100% - 20% GPR based on sales) 70%
Cost of goods sold 52,500
Page |2

Inventory, Sept. 30 (see T-account above) 7,000


Goods in transit (1,000)
Goods out on consignment (1,200)
Salvage value (1,800)
Inventory loss due to flood 3,000

2. On October 1, 20x1, the warehouse of ABC Co. and all inventories contained therein were razed
by fire. Off-site back up of data base shows the following information:
Inventory, Jan. 1 20,000
Net purchases 190,000
Net sales from Jan. to Sept. 240,000
Gross profit rate based on cost 25%

Twenty percent of the inventory contained in the warehouse has been salvaged from the fire while
half is partially damaged and can be sold as scrap at thirty percent of its cost. How much is the
inventory loss due to the fire?
a. 18,000 c. 9,000
b. 5,400 d. 11,700

D
Solution:
Inventory
Jan. 1 20,000
Net purchases 190,000 192,000 COGS (240K x 100/125)
18,000 Sept. 30 (squeeze)

Inventory, Sept. 30 18,000


Salvaged (20% x 18,000) (3,600)
Partially damaged (50% x 18,000 x 30%) (2,700)
Loss from fire 11,700
Page |3

Use the following information for the next two questions:


Presented below is information pertaining to ABC Co.:
Cost Retail
Inventory, January 1 21,750 35,000
Purchases 138,250 200,750
Freight-In 5,000 -
Purchase discounts 1,250 -
Purchase returns 13,000 21,500
Departmental Transfers-In (Debit) 2,500 3,750
Departmental Transfers-Out (Credit) 2,000 3,000
Markups 15,000
Markup cancellations 5,000
Markdowns 30,000
Markdown cancellations 7,500
Abnormal spoilage (theft and casualty loss) 12,500 17,500
Sales 109,500
Sales returns 6,250
Sales discounts 2,500
Employee discounts 1,250
Normal spoilage (shrinkage and breakages) 500

3. How much is the ending inventory under the Average cost method?
a. 60,750
b. 60,000
c. 61,050
d. 62,400
B
Solution:
Cost Retail
Inventory, January 1 21,750 35,000
Net purchases (a) 129,000 179,250
Departmental transfers-in (debit) 2,500 3,750
Departmental transfers-out (credit) (2,000) (3,000)
Net markups (15,000 – 5,000) 10,000
Net markdowns (30,000 – 7,500) (22,500)
Abnormal spoilage (theft and casualty loss) (12,500) (17,500)
Total goods available for sale 138,750 185,000
Net sales (b) (105,000)
Ending inventory at retail 80,000

(a)
Cost Retail
Purchases 138,250 200,750
Freight-In 5,000 -
Purchase discounts (1,250) -
Purchase returns (13,000) (21,500)
Net purchases 129,000 179,250
Page |4

The Average cost ratio is computed as follows:


Cost ratio Total goods avail. for sale at cost
= Total goods avail. for sale at sales price or
(Average cost method)
at retail
Average cost ratio = (138,750 ÷ 185,000) = 75%

(b) Net sales is computed as follows:


Sales 109,500
Sales returns (6,250)
Employee discounts 1,250
Normal spoilage 500
Net sales 105,000

The ending inventory at cost is estimated under the Average cost method as follows:
Ending inventory at retail (or at selling price) 80,000
Multiply by: Average cost ratio 75%
Ending inventory at cost 60,000

4. How much is the ending inventory under the FIFO cost method?
a. 60,750
b. 60,000
c. 61,050
d. 62,400

D
Solution:
Based on the solutions from the previous problem, the cost ratio under the FIFO cost method is computed
as follows:
(d) The FIFO cost ratio is computed as follows:
Cost ratio TGAS at cost less beg. inventory at cost
=
(FIFO cost method) TGAS at retail less beg. inventory at retail
FIFO cost ratio = [(138,750 – 21,750) ÷ (185,000 – 35,000)]
= 78%

The ending inventory at cost is estimated under the FIFO cost method as follows:
Ending inventory at retail 80,000
Multiply by: FIFO cost ratio 78%
Ending inventory at cost 62,400

“Blessed is the one who perseveres under trial because, having stood the test, that
person will receive the crown of life that the Lord has promised to those who love
him.” (James 1:12)
- END -
Page |1

Chapter 9
Investments
: TRUE OR FALSE
1. According to PFRS 9 Financial instruments, investments in stocks are initially recorded at cost and
all commissions, taxes, and other fees are expensed as incurred.

2. Unrealized holding gains and losses on investments in held for trading securities are recognized
in profit or loss.

3. Unrealized gains and losses on investments in equity securities measured at FVOCI are
recognized in the income statement.

4. A debit balance in the “Fair Adjustment – FVOCI” account implies a corresponding owners'
equity account with a credit balance of the same amount.

5. According to PFRS 9, the classification of financial assets for subsequent measurement purposes
is based on management's intentions.

6. The net reported balance in the “investment in equity securities – FVOCI” account is the original
cost plus a credit balance in the fair value adjustment account or minus a debit balance in the fair
value adjustment account.

7. When investments in held for trading securities are sold, the realized gain or loss is the
difference in the fair value since acquisition.

8. Unrealized holding gains on investments measured at fair value through other comprehensive
income are recognized as direct increases to owners' equity, rather than through the statement of
comprehensive income.

9. Increases in the fair value of held for trading securities and investments in equity securities
measured at FVOCI cause the related fair value adjustment account to decrease.

10. Investments in held for trading securities may be classified as current or long-term.

“Go ahead and be lazy; sleep on, but you will go hungry.” (Proverbs 19:15)

- END –

ANSWERS
1. FALSE 6. FALSE
Page |2

2. TRUE 7. FALSE
3. FALSE 8. FALSE
4. TRUE 9. FALSE
5. FALSE 10. FALSE
Page |3

1. Changes in fair value of this type of securities are accumulated as a separate component in the
stockholders' equity section of the balance sheet.
a. Financial assets measured at amortized cost
b. FVOCI securities
c. Held for trading securities
d. Designated financial assets

2. Which category includes only debt securities?


a. Financial assets measured at amortized cost
b. FVPL assets
c. Held for trading securities
d. FVOCI (election)

3. A correct valuation is
a. investment in equity securities at amortized cost.
b. held for trading securities at amortized cost.
c. debt securities, to be held until maturity to collect cash flows from principal and interests, at
fair value.
d. none of these.

4. Securities which could be classified as financial assets measured at amortized cost are
a. investment in stocks.
b. warrants.
c. municipal bonds.
d. treasury stock.

5. Which of the following is not correct regarding held for trading securities?
a. They are held to be sold in a short period of time.
b. Unrealized holding gains and losses are reported as part of profit or loss.
c. Any discount or premium is not amortized.
d. All of these are correct.

6. A debit balance in the “Fair Value Adjustment - FVOCI Securities” account at the end of a year
should be interpreted as
a. the net unrealized holding gain for that year.
b. the net realized holding gain for that year.
c. the net unrealized holding gain to date.
d. the net realized holding gain to date.

7. A debit balance in the “Fair Value Adjustment - Held for Trading Securities” account at the end
of a year should be interpreted as
a. the net realized holding gain to date.
b. the net unrealized holding gain to date.
c. the net realized holding gain for that year.
d. the net unrealized holding gain for that year.
Page |4

8. Unrealized holding gains or losses which are recognized in profit or loss are from securities
classified as
a. amortized cost.
b. FVOCI.
c. held for trading.
d. designated and held for trading.

9. An unrealized holding gain on a company's FVOCI securities should be reflected in the current
financial statements as
a. an extraordinary item shown as a direct increase to retained earnings.
b. a current gain resulting from holding securities.
c. a note or parenthetical disclosure only.
d. other comprehensive income and included in the equity section of the balance sheet.

10. Changes in fair value of an investment measured at fair value through other comprehensive
income
a. must be recognized in profit or loss.
b. must be recognized directly in equity.
c. may be recognized in profit or loss or directly in equity.
d. must be recognized in other comprehensive income and accumulated in a separate equity
account.

11. At initial recognition, an entity may make an irrevocable election to present in other
comprehensive income subsequent changes in the fair value of an investment in equity securities
within the scope of PFRS 9 that is not held for trading. In accounting for such financial
instruments, all of the following are true except
a. amounts presented in other comprehensive income are not be subsequently transferred to
profit or loss.
b. the entity may transfer any cumulative fair value gains or losses within equity.
c. dividends received on the investments are recognized in profit or loss.
d. cumulative fair value gains or losses are transferred to profit or loss when the financial asset
is derecognized.

12. An entity sells an investment that is measured at FVPL during the year. The realized gain or loss
on the sale is computed as
a. the difference between the sale price and the carrying amount of the investment as at the
date of sale.
b. the difference between the sale price and the original acquisition cost of the investment.
c. the difference between the net proceeds received from the sale and the carrying amount of
the investment as at the date of sale.
d. the difference between the net proceeds received from the sale and the carrying amount of
the investment as at the date of sale adjusted for any accumulated fair value gains or losses
recognized since the investment was acquired.
Page |5

13. For which type of investments would unrealized fair value gains and losses be accumulated in
an equity account?
a. Equity method securities
b. FVOCI securities
c. Held for Trading securities
d. Held-to-maturity securities

14. If the combined fair value of held for trading securities at the end of the year is less than the fair
value of the same portfolio of held for trading securities at the beginning of the year, the
difference should be accounted for by
a. reporting an unrealized loss in security investments in the stockholders' equity section of
the balance sheet.
b. reporting an unrealized loss in security investments in profit or loss.
c. a footnote to the financial statements.
d. a debit to Investment in Held for Trading Securities.

15. Information regarding Stone Co.’s portfolio of FVOCI securities is as follows:


Aggregate cost as of 12/31/03 170,000
Unrealized gains as of 12/31/03 4,000
Unrealized losses as of 12/31/03 26,000
Net realized gains during 2003 30,000

At December 31, 2002, Stone reported an unrealized loss of ₱1,500 in other comprehensive income to
reduce these securities to market. Under the accumulated other comprehensive income in
stockholders’ equity section of its December 31, 2003 balance sheet, what amount should Stone
report?
a. 26,000 c. 20,500
b. 22,000 d. 0

16. Caloy Co. bought 1,000 shares from Bayan Co. The shares have no active market, but an identical
or similar asset has an active market. The identical asset, however, has multiple markets. Caloy
determines that the identical asset has the following market values:
Market A Market B
Quoted price 500 600
Related transaction cost 25 150

How much is fair valuation of the investment?


a. 500,000 c. 450,000
b. 475,000 d. b or c

A
Solution:
Market A Market B
Quoted price 500 600
Related transaction cost (25) (150)
Page |6

Net selling price 475 450

The more advantageous market is Market A and the quoted price in this market is ₱500.

17. On January 1, 20x1, Allan Co. purchased ₱400,000 bonds for ₱392,000. The bonds mature on
January 1, 20x5 and pay 12% annual interest beginning January 1, 20x2. Transaction costs are
negligible. The bonds were classified as held for trading securities. On December 31, 20x1, the
bonds are selling at a yield rate of 10%. How much is the unrealized gain (loss) recognized on
December 31, 20x1?
a. 27,986
b. 31,298
c. 28,964
d. 33,359

D [(400,000 x PV of 1 @10%, n=4) + (400,000 x 12% x PV ordinary annuity of 1 @10%, n=4) =


425,359 – 392,000 = 33,359

The fair value of the bonds on Dec. 31, 20x1 is computed as follows:
Present
Future cash flows PV @10%, n=3 PV factors value
Principal 400,000 PV of P1 0.751315 300,526
Interest (400K x 12%) 48,000 PV of ordinary annuity 2.486852 119,369
Fair value as of December 31, 20x1 419,895

(419,895 – 392,000) = 27,895

18. On January 1, 20x1, Rizzi Co. purchased 12,000 shares of Andre, Inc. for ₱400,000. Commission
paid to broker amounted to ₱20,000. Management made an irrevocable choice to subsequently
measure the shares at fair value through other comprehensive income. On December 31, 20x1,
the shares were quoted at ₱40 per share. On January 3, 20x2, all of the shares were sold at ₱60
per share. Commission paid on the sale amounted to ₱24,000. How much is the unrealized gain
(loss) recognized in profit or loss on December 31, 20x1?
a. (60,000)
b. 60,000
c. (80,000)
d. 0

D – The investment is FVOCI. Any unrealized gain (loss) is recognized in OCI and not P/L.
Page |7

Use the following information for the next two questions:


Karen Co. purchased the following equity securities on January 1, 20x1 for a total amount of
₱360,000.
Cost
Alaska Co. preference shares ₱200,000
Valdez Co. ordinary shares 160,000
Totals ₱360,000

The shares did not qualify for recognition as held for trading. Accordingly, they were classified as
investment in equity securities measured at fair value through other comprehensive income.

On December 31, 20x1, the portfolio of Karen Co. comprised the following.
Fair value – 12/31/x1
Alaska Co. preference shares ₱240,000
Valdez Co. ordinary shares 60,000
Total ₱300,000

On December 31, 20x2, the portfolio of Karen Co. comprised the following:
Fair value – 12/31/x2
Alaska Co. preference shares ₱220,000
Valdez Co. ordinary shares 180,000
Total ₱400,000

On February 2, 20x3, all of the Alaska Co. preference shares were sold for ₱160,000 net of transaction
costs.

19. How much is the unrealized gain (loss) recognized in other comprehensive income on December
31, 20x1?
a. 60,000
b. (60,000)
c. 100,000
d. 0

B (300,000 – 360,000) = (60,000)

20. How much is the cumulative unrealized gain (loss) that is presented as a separate component in
equity as of December 31, 20x2?
a. 40,000
b. (40,000)
c. 100,000
d. 0

A (400,000 FV 12/31/x2 – 360,000 cost) = 40,000 unrealized gain


Page |8

“From the fruit of his mouth a man’s stomach is filled; with the harvest from his
lips he is satisfied.” (Proverbs 18:20)
- END –
Page |1

Chapter 10
Investments in Debt Securities
1. Securities classified as financial asset measured at amortized cost are reported at
a. acquisition cost.
b. acquisition cost plus amortization of a discount.
c. acquisition cost plus amortization of a premium.
d. fair value.

2. In accounting for investments in debt securities that are classified as held for trading securities,
a. a discount is reported separately.
b. a premium is reported separately.
c. any discount or premium is not amortized.
d. none of these.

3. According to PFRS 9 Financial Instruments, investments in debt securities that are classified at
amortized cost are initially measured at
a. cost including accrued interest.
b. maturity value.
c. cost including brokerage and other fees.
d. fair value plus brokerage and other fees.

4. Pippen Co. purchased ten-year, 10% bonds that pay interest semiannually. The bonds are sold to
yield 8%. One step in calculating the issue price of the bonds is to multiply the principal by the
table value for
a. 10 periods and 10% from the present value of 1 table.
b. 10 periods and 8% from the present value of 1 table.
c. 20 periods and 5% from the present value of 1 table.
d. 20 periods and 4% from the present value of 1 table.

5. Solo Co. purchased ₱300,000 bonds for ₱315,000. The securities are to be held until maturity to
collect the contractual cash flows. The entry to record the investment includes
a. a debit to Held-for-Trading Securities at ₱300,000.
b. a credit to Premium on Investments of ₱15,000.
c. a debit to Investment in bonds measured at amortized cost for ₱315,000.
d. none of these.

Use the following information for the next two questions:


On January 1, 20x1, Kevin Co. acquired 12%, P4,000,000 bonds for P4,198,948. The principal is due
on December 31, 20x3 but interest is made annually starting December 31, 20x1. The effective
interest rate on the bonds is 10%.

6. How much is the interest income recognized in 20x1?


a. 419,895 c. 407,273
b. 413,884 d. 480,000

7. How much is the carrying amount of the investment on December 31, 20x1?
Page |2

a. 4,198,948 c. 4,072,727
b. 4,138,843 d. 4,000,000

Solution:
Date Collections Interest income Amortization Present value
1/1/x1 4,198,948
12/31/x1 480,000 419,895 60,105 4,138,843

8. On April 1, 20x1, Ronald Ryan Co. acquired 12%, P4,000,000 bonds dated January 1, 20x1 at 98%
including interest. The bonds mature on December 31, 20x3 but pays annual interest at each
year-end. How much is the initial carrying amount of the investment?
a. 3,920,000 b. 3,800,000 c. 4,000,000 d. 4,120,000

B (4,000,000 x 98%) – (4,000,000 x 12% x 3/12) = 3,800,000


3/12? – January 1- March 30= 3 months equivalents

9. On January 1, 20x1, Mitch Co. acquired 12%, P4,000,000 bonds at 98. Commission paid to
brokers amounted to P204,000. Principal is due on December 31, 20x4 but interest payments are
made annually starting December 31, 20x1.

The adjusted effective interest rate on the investment is closest to


a. 12% b. 11% c. 10.2650% d. indeterminable

B
Solution:
Acquisition cost (4M x 98%) 3,920,000
Direct cost 204,000
Initial carrying amount 4,124,000

“Trial and error” approach:


Future cash flows x PV factor at x% = Present value
(4M x PV of P1 @ x%, n=4) + (4M x 12% x PV of an ordinary annuity of P1 @ x%, n=4) = 4,124,000

There is premium because the carrying amount is greater than the face amount. Therefore, the
effective interest rate must be lower than the nominal rate of 12%.

First trial: (using 11%)


Future cash flows x PV factor at x% = PV or initial carrying amount
 (4M x PV of P1 @ 11%, n=4) + (4M x 12% x PV of an ordinary annuity of P1 @ 11%, n=4) =
4,124,000
 (4M x 0.658731) + (480,000 x 3.102446) = 4,124,000
 (2,634,924 + 1,489,174) = 4,124,098 approximates 4,124,000 (a difference of only P98)

If the difference of P98 is judged immaterial, then 11% is deemed the effective interest rate.
Page |3

Use the following information for the next three questions:


On January 1, 20x1, ABC Co. acquired 10%, ₱1,000,000 bonds for ₱827,135. The bonds mature on
December 31, 20x3 and pay annual interest every December 31. ABC Co. incurred transaction costs
₱80,000 on the acquisition. The effective interest rate adjusted for the effect of the transaction costs is
14%.

The bonds are to be held under a “hold to collect and sell” business model. Information on fair
values is as follows:
December 31, 20x1…………………………….98
December 31, 20x2……………………………102
December 31, 20x3……………………………100

10. How much is the carrying amount of the investment on December 31, 20x1?
a. 935,134 b. 1,002,000 c. 980,000 d. 965,443

C = 1M x 98%

11. How much is the unrealized gain (loss) recognized in other comprehensive income on December
31, 20x1?
a. 45,866 b. (45,866) c. (37,899) d. 0

Amortization table
Interest
Date received Interest income Amortization Present value
1/1/x1 907,135
12/31/x1 100,000 126,999 26,999 934,134
12/31/x2 100,000 130,779 30,779 964,913
12/31/x3 100,000 135,088 35,088 1,000,000

 [(1M x 98%) – 934,134] = 45,866 Unrealized gain – OCI

12. How much is the interest income recognized in 20x2?


a. 126,999 c. 135,088
b. 130,779 d. 144,388

“Do nothing out of selfish ambition or vain conceit. Rather, in humility value
others above yourselves, not looking to your own interests but each of you to the
interests of the others. In your relationships with one another, have the same
mindset as Christ Jesus.” (Philippians 2:3-5)

- END –
Page |1

Chapter 11
Investments – Additional Concepts

1. When an investment in a debt security measured at amortized cost is transferred to held for
trading security, the carrying amount assigned to the held for trading security should be
a. its original cost.
b. its fair value at the date of the transfer.
c. the lower of its original cost or its fair value at the date of the transfer.
d. the higher of its original cost or its fair value at the date of the transfer.

2. When an investment in equity securities irrevocably elected on initial recognition to be


subsequently measured at FVOCI is transferred to held for trading because the company
anticipates selling the stock in the near future, the carrying amount assigned to the investment
upon entering it in the trading portfolio should be
a. its original cost.
b. its fair value at the date of the transfer.
c. the higher of its original cost or its fair value at the date of the transfer.
d. None of these

3. According to PFRS 9 Financial Instruments, investments in debt securities classified under the
amortized cost measurement category should be recorded on the date of acquisition at
a. lower of cost or market.
b. market value.
c. fair value plus brokerage fees and other costs incident to the purchase.
d. face value.

4. Which of the following is correct about the effective interest method of amortization?
a. The effective interest method applied to investments in debt securities is different from that
applied to bonds payable.
b. The amortization of a discount decreases from period to period.
c. The amortization of a premium decreases from period to period.
d. The effective interest method produces a constant rate of return on the book value (carrying
amount) of the investment from period to period.

5. A debt security is purchased at a discount. The entity wants to classify the investment as a
financial asset measured at FVOCI. The entry to record the amortization of the discount includes
a
a. debit to investment account.
b. debit to the discount account.
c. debit to Interest Revenue.
d. none of these.

6. On December 29, 20x1, an entity commits itself to purchase a financial asset for ₱10,000, which is
its fair value on commitment date (trade date). Transaction costs are immaterial. On December 31,
20x1 and on January 4, 20x2 (settlement date) the fair values of the asset are ₱12,000 and ₱15,000,
Page |2

respectively. If the entity uses the settlement date accounting and that the investment is
classified as held for trading, how much is the carrying amount of the investment in the
December 31, 20x1 statement of financial position?
a. 10,000 c. 15,000
b. 12,000 d. 0

D 0 - the entity uses the settlement date accounting

7. On January 1, 20x1, Dagul Co. acquired 10%, ₱4,000,000 bonds for ₱3,807,853. The principal is
due on January 1, 20x4 but interest is due annually starting December 31, 20x1. The yield rate on
the bonds is 12%. On July, 1 20x1, Dagul Co. changed its business model. It was ascertained that
the investment in bonds at amortized cost should be reclassified to held for trading securities on
reclassification date. The bonds were quoted at 102, 103 and 104 on July 1, 20x1, December 31,
20x1 and January 1, 20x2, respectively. How much is the gain (loss) on reclassification on
January 1, 20x2?
a. 243,676 c. 295,205
b. 255,205 d. 0
C
Solution:
Date Collections Interest income Amortization Present value
1/1/x1 3,807,853
12/31/x1 400,000 456,942 56 ,942 3,864,795
12/31/x2 400,000 463,775 63,775 3,928,571
12/31/x3 400,000 471,428 71,428 3,999,999

(4M x 104%) – 3,864,795 = 295,205

8. On March 31, 20x1, Budoy Co. received 10,000 stock rights from its investment in equity
securities to subscribe to new shares at ₱60 per share for every 4 rights held. Prior to issuance of
stock rights, the shares were selling at ₱80 per share. How much is the initial carrying amount of
the stock rights?
a. 20,000 c. 50,000
b. 40,000 d. cannot be determined

B Theoretical/Parity value = (80 – 60) / (4 + 1) = 4;


10,000 x 4 = 40,000

“Come to me, all you who are weary and burdened, and I will give you rest. 29
Take my yoke upon you and learn from me, for I am gentle and humble in heart,
and you will find rest for your souls.” (Matthew 11:28-29)

- END -
Page |1

Chapter 12
Other Long-Term Investments
1. Which of the following may be classified as “other long-term investments?”
a. Shares of stocks purchased as long-term investment.
b. Bonds acquired to be held as a long-term investment.
c. Treasury shares acquired at a deep discount and expected to be reissued at a future date that
exceeds 12 months from the reporting date.
d. Shares of stocks and bonds purchased using funds earmarked for the retirement of a bond
issuance.

2. Witt Corp. has outstanding at December 31, 2004, two long-term borrowings with annual
sinking fund requirements and maturities as follows:
Sinking fund requirements Maturities
2003 1,000,000 -
2004 1,500,000 2,000,000
2005 1,500,000 2,000,000
2006 2,000,000 2,500,000
2007 2,000,000 3,000,000
8,000,000 9,500,000

In the notes to its December 31, 2004 balance sheet, how should Witt report the above data?
a. No disclosure is required.
b. Only sinking fund payments totaling ₱8,000,000 for the next five years detailed by year need
be disclosed.
c. Only maturities totaling ₱9,500,000 for the next five years detailed by year need to be
disclosed.
d. The combined aggregate of ₱17,500,000 of maturities and sinking fund requirements detailed
by year should be disclosed.

3. On March 1, 2001, a company established a sinking fund in connection with an issue of bonds
due in 2013. At December 31, 2003, the independent trustee held cash in the sinking fund
account representing the annual deposits to the fund and the interest earned on those deposits.
How should the sinking fund be reported in the company’s balance sheet at December 31, 2003?
a. The cash in the sinking fund should appear as a current asset.
b. Only the accumulated deposits should appear as a noncurrent asset.
c. The entire balance in the sinking fund account should appear as a current asset.
d. The entire balance in the sinking fund account should appear as a noncurrent asset.

4. An issuer of bonds uses a sinking fund for the retirement of the bonds. Cash was transferred to
the sinking fund and subsequently used to purchase investments. The sinking fund
I. is increased by the income earned on the investments.
II. is not affected by the income earned on the investments.
Page |2

III. decreased when the investments are purchased.


a. I only. b. I and III. c. II and III. d. III only.

5. An increase in the cash surrender value is accounted for as


a. income recognized in profit or loss.
b. income recognized in other comprehensive income.
c. reduction to insurance expense.
d. none of these.

“Be strong and courageous. Do not fear of be in dread of them, for it is the Lord your
God who goes with you. He will not leave you or forsake you.” (Deuteronomy 31:6)

- END -
Page |1

Chapter 13
Basic Derivatives

1. Which of the following is not among the characteristics of a derivative?


a. it must have at least two or more notional amounts
b. its value changes in response to the change in an underlying
c. it requires no initial net investment or only a very minimal initial net investment
d. it is settled at a future date

2. Which of the following can be an underlying for a derivative?


a. temperature or climate c. interest or exchange rate
b. specified price d. all of these

3. Which of the following can be a notional amount for a derivative?


a. share price c. number of currency units
b. interest rate d. exchange rate

4. Entity X enters into a forward contract to sell 1,000,000 foreign currency units at a forward rate
of ₱0.50. At the reporting date and on settlement date, the current rates are ₱0.48 and ₱0.52,
respectively. Identify the notional amount and the underlying in the contract.

Notional amount Underlying


a. ₱0.50 1,000,000
b. 1,000,000 currency units Foreign currency
c. 1,000,000 currency units Forward rates
d. ₱0.50, ₱0.48 and ₱0.52 1,000,000

Use the following information for the next two questions:


ABC Co. expects the value of the won to increase in the next 30 days. Accordingly, on December 15,
20x1, ABC Co. enters into a 30-day forward contract to buy 10,000 wons at the forward rate of ₱1.24.
On December 31, 20x1, the forward rate was ₱1.27 and by January 15, 20x2, the spot rate moved to
₱1.30.

5. How much is the carrying amount of the derivative on December 31, 20x1?
a. 0 c. 300 liability
b. 300 asset d. 1,400 asset

6. How much is the net cash settlement on January 15, 20x2?


a. 300 receipt c. 600 receipt
b. 300 payment d. 600 payment
Page |2

 Dec. 15, 20x1 (Contract date)

Hedged item – None Forward contract (Derivative)


Dec. 15, 20x1
No entry

 Dec. 31, 20x1 (Reporting date)

The value of the derivative is computed as follows:

Purchase price under the forward contract (10,000 x 1.24) 12,400


Purchase price in the market (10,000 x 1.27) 12,700
Gain/ Derivative asset 300

Dec. 31, 20x1


Forward contract (asset).. 300
Gain on forward contract.. 300
[(1.27 forward rate – 1.24 forward rate) x
10K]

 Jan. 15, 20x2 (Settlement date)

Net cash settlement

Hedged item – None Forward contract (Derivative)


Jan. 15, 20x2 Jan. 15, 20x2
Cash [(1.30 – 1.24) x 10K]….. 600
Forward contract (asset)… 300
Gain on forward contract.... 300
[(1.30 – 1.27) x 10K]

Use the following information for the next two questions:


On December 1, 20x1, ABC Co. enters into a futures contract to sell 10,000 units of a commodity on
January 31, 20x2 for ₱100 per unit. The broker requires an initial margin deposit of ₱10,000. The
quoted prices per unit are as follows:

Dec. 1, 20x1 Dec. 31, 20x1 Jan. 31, 20x2


100 98 97

7. How much is the gain (loss) on the remeasurement of the derivative on December 31, 20x1?
a. 10,000 gain c. 20,000 loss
b. 20,000 gain d. 0

8. How much is the net cash settlement on January 31, 20x1?


a. 30,000 receipt c. 40,000 receipt
Page |3

b. 30,000 payment d. 40,000 payment

Hedged item – None Futures contract (Derivative)


Dec. 1, 20x1
Deposit with broker ……..10K
Cash………………………..10K

to record the initial margin deposit with


the broker

Hedged item – None Futures contract (Derivative)


Dec. 31, 20x1
Futures contract (asset)...20K
Gain on futures contract…..20K
[(100 - 98) x 10,000]

to record the value of the derivative


computed as the change in the underlying
multiplied by the notional amount.

Hedged item – None Futures contract (Derivative)


Jan. 31, 20x2
Cash ……………………… 40K
Deposit with broker…….....10K
Futures contract (asset)…. 20K
Gain on futures contract….10K
[(98 - 97) x 10,000]

to record the net cash settlement of the


futures contract.

9. Drive Co. acquires a call option on 1,000 units of a commodity at a strike price of ₱100 for ₱400
on March 1, 20x1. The call option is exercisable on July 1, 20x1. The movements in prices are
shown below:

Mar. 1, 20x1 June 30, 20x1


Spot prices ₱100 ₱120
Time value of option 400 100

How much is the net cash settlement on January 31, 20x1?


a. 20,000 receipt c. 20,100 receipt
b. 20,000 payment d. 20,100 payment
Hedged item – None Call option (Derivative)
Mar. 1, 20x1
Call option ……..…….. 400
Cash………..……………… 400

Hedged item – None Call option (Derivative)


June 30, 20x1
Call option ……..…….. 20,000
[(120 – 100) x 1,000]
Gain on call option………. 20,000

to record the increase in the fair value of


the call option due to the increase in
Page |4

intrinsic value.

June 30, 20x1


Loss on call option……….300
(400 – 100)
Call option……………………..300

to record the decrease in the fair value of


the call option due to the decrease in
time value.

Hedged item – None Call option (Derivative)


July 1, 20x1
Cash…………………20,000
[(120 – 100) x 1,000]
Loss on call option…....100
Call option ……..……..…..20,100
(400 + 20,000 – 300)

to record the net settlement of the call


option contract.

10. Tuba Co. enters into a “receive variable, pay fixed” interest swap on January 1, 20x1 for a
notional amount of ₱1,000,000. Under the terms of the contract, if the current rate increases
above 12% (i.e., the set rate), Tuba Co. shall receive the excess interest. If the current rate falls
below 12%, Tuba Co. shall pay the deficiency. Swap payment shall be made on December 31,
20x2. The current rates are as follows:
Jan. 1, 20x1……………………………12%
Jan. 1, 20x2……………………………15%

How much is the carrying amount of the derivative on December 31, 20x1?
a. 30,0000 liability c. 26,087 asset
b. 30,000 asset d. 26,087 liability

Solution:
 Jan. 1, 20x1

Hedged item – None Interest rate swap (Derivative)


Jan. 1, 20x1
No entry

 Dec. 31, 20x1

The net cash settlement on the swap is determined as follows:


20x1 20x2
Receive variable a 120,000 150,000
Pay 12% fixed 120,000 120,000
Net cash settlement - receipt - 30,000
Page |5

a
The interest rates used are the current rates as at the beginning of the year (i.e., 1M x 12% = 120,000) &
(1M x 15% = 150,000).

The net cash settlement in 20x2 is discounted to determine the fair value of the derivative on Dec. 31,
20x1:

30,000 x PV of 1 @ 15%, n=1 = 26,087 (asset)

Hedged item – None Interest rate swap (Derivative)


Dec. 31, 20x1
Interest rate swap…..26,087
Gain on int. rate swap…..26,087

to recognize the change in the fair value


of the interest rate swap

 Dec. 31, 20x2

Hedged item – None Interest rate swap (Derivative)


Dec. 31, 20x2
Cash…………………30,000
Interest rate swap……....26,087
Gain on int. rate swap…...3,913
to record the net cash settlement of the
interest rate swap

“Trust in the Lord with all your heart and lean not on your own understanding; in
all your ways acknowledge him, and he will make your paths straight.” (Proverbs 3:5-6)

- END -
Page |1

Chapter 14
Investments in Associates
1. PAS 28 generally applies when the level of ownership over another company is at what
percentage?
a. Less than 20%
b. 20%-30%
c. 20%-50%
d. More than 50%

2. When an investor uses fair value accounting to account for investments in common stock, cash
dividends received by the investor from the investee would normally be recorded as
a. a deduction from the investment account.
b. dividend revenue.
c. an addition to the investor's share of the investee's profit.
d. a deduction from the investor's share of the investee's profit.

3. Under the equity method in PAS 28, goodwill amortization


a. reduces the investment account.
b. increases the investment account.
c. reduces both investment income and the investment account.
d. is not recorded.

4. The equity method of accounting should be used when an investment


a. is composed of ordinary shares and it is the investor's intent to vote the ordinary shares.
b. ensures a source of supply such as raw materials.
c. enables the investor to exercise significant influence over the investee.
d. gives the investor voting control over the investee.

5. When an investor uses the equity method to account for investments in common stock, the
investment account will be increased when the investor recognizes
a. a proportionate share of the net income of the investee.
b. a cash dividend received from the investee.
c. periodic amortization of the goodwill related to the purchase.
d. depreciation related to the excess of fair value over carrying amount of the investee's
depreciable assets at the date of purchase by the investor.

6. When an investor uses the equity method, cash dividends received from the investee are recorded
as
a. an increase in the investment account.
b. a deduction from the investment account.
c. dividend revenue.
d. a deduction from the investor's share of the investee's profits.
Page |2

7. Dane, Inc. owns 35% of Marin Corporation. During the calendar year 2004, Marin had net
earnings of ₱300,000 and paid dividends of ₱30,000. Dane mistakenly recorded these transactions
using the fair value method rather than the equity method of accounting. Dane recognized
₱20,000 gain on the change in fair value of the investment during the year. What effect would this
have on the investment account, net income, and retained earnings, respectively?
a. Understate, overstate, overstate
b. Overstate, understate, understate
c. Overstate, overstate, overstate
d. Understate, understate, understate

8. Under the equity method of accounting for investments, an investor recognizes its share of the
earnings in the period in which the
a. investor sells the investment.
b. investee declares a dividend.
c. investee pays a dividend.
d. earnings are reported by the investee in its financial statements.

9. When a company holds between 20% and 50% of the outstanding stock of an investee, which of
the following statements applies?
a. The investor should always use the equity method to account for its investment.
b. The investor should use the equity method to account for its investment unless circumstances
indicate that it is unable to exercise "significant influence" over the investee.
c. The investor must use the fair value method unless it can clearly demonstrate the ability to
exercise "significant influence" over the investee.
d. The investor should always use the fair value method to account for its investment.

10. Byner Corporation accounts for its investment in the common stock of Yount Company
under the equity method. Byner Corporation should ordinarily record a cash dividend received
from Yount as
a. a reduction of the carrying value of the investment.
b. additional paid-in capital.
c. an addition to the carrying value of the investment.
d. dividend income.

“Do not be anxious about anything, but in everything by prayer and supplication
with thanksgiving let your requests be made known to God. And the peace of God,
which surpasses all understanding, will guard your hearts and your minds in
Christ Jesus.” (Philippians 4:6-7)

- END -
Page |3

Use the following information for the next nine questions:


The summarized balance sheets of Elston Company and Alley Company as of December 31, 2004 are
as follows:
Elston Company
Balance Sheet
December 31, 2004
Assets ₱800,000

Liabilities ₱100,000
Capital stock 400,000
Retained earnings 300,000
Total equities ₱800,000
Alley Company
Balance Sheet
December 31, 2004
Assets ₱600,000

Liabilities ₱150,000
Capital stock 370,000
Retained earnings 80,000
Total equities ₱600,000

1. If Elston Company acquired a 20% interest in Alley Company on December 31, 2004 for ₱130,000
and the fair value method of accounting for the investment were used, the amount of the debit to
Investment in Alley Company Stock would have been
a. 90,000.
b. 74,000.
c. 130,000.
d. 120,000.

2. If Elston Company acquired a 30% interest in Alley Company on December 31, 2004 for ₱150,000
and the equity method of accounting for the investment were used, the amount of the debit to
Investment in Alley Company Stock would have been
a. 190,000.
b. 150,000.
c. 120,000.
d. 135,000.

3. If Elston Company acquired a 20% interest in Alley Company on December 31, 2003 for ₱90,000
and during 2004 Alley Company had net income of ₱50,000 and paid a cash dividend of ₱20,000,
applying the fair value method would give a debit balance in the Investment in Alley Company
Stock account at the end of 2004 of
a. 74,000.
b. 90,000.
Page |4

c. 100,000.
d. None of these

The fair value on December 31, 2004 is not given

4. If Elston Company acquired a 30% interest in Alley Company on December 31, 2004 for ₱135,000
and during 2005 Alley Company had net income of ₱50,000 and paid a cash dividend of ₱20,000,
applying the equity method would give a debit balance in the Investment in Alley Company
Stock account at the end of 2005 of
a. 135,000.
b. 144,000.
c. 150,000.
d. 145,000.

B ₱135,000 + (₱50,000 × .3) – (₱20,000 × .3) = ₱144,000.

5. Elston Co. acquires 30% interest in Alley Company on December 31, 2004. The carrying amount
of Alley’s net assets on December 31, 2004 approximates its fair value. If the acquisition did not
result to any implied goodwill, how much is the acquisition cost of the investment?
a. 135,000.
b. 144,000.
c. 150,000.
d. 145,000.

A (370,000 + 80,000) = 450,000 net assets x 30% = 135,000

6. Elston Co. acquires 30% interest in Alley Company on December 31, 2004. The carrying amount
of Alley’s net assets on December 31, 2004 approximates its fair value. If the acquisition did not
result to any implied goodwill, how much is the acquisition cost of the investment?
a. 135,000.
b. 144,000.
c. 150,000.
d. 145,000.

A
Purchase price (squeeze)
Fair value of net assets acquired 135,000
(370,000 + 80,000) = 450,000 net assets x 30%] (135,000)
Goodwill 0

7. Elston Co. acquired 25% interest in Alley Company many years ago. The acquisition did not
result to any goodwill. At the time of acquisition, the carrying amount of Alley’s net assets
approximates its fair value. There have been no impairment losses on the investment. In
principle, the equity method would result to a carrying amount of the investment on December
31, 2004 of
a. 112,500.
Page |5

b. 135,000.
c. 144,000.
d. Cannot be determined; given information is insufficient

A (370,000 + 80,000) = 450,000 net assets x 25%] = 112,500

8. Elston Co. acquired 25% interest in Alley Company many years ago. The acquisition did not
result to any goodwill. At the time of acquisition, the carrying amount of Alley’s net assets
approximates its fair value. There have been no impairment losses on the investment. Alley
Company reported profit of ₱200,000 and declared dividends of ₱40,000 in 2004. Theoretically,
the carrying amount of the investment on December 31, 2003 would approximate which of the
following amounts?
a. 72,500.
b. 98,500
c. 112,500.
d. Cannot be determined; given information is insufficient

A
Investment in associate
12/31/2003 (squeeze) 72,500
Sh. in profit 50,000 10,000 Cash dividends
112,500* 12/31/2004

*(370,000 + 80,000) = 450,000 net assets x 25% = 112,500

9. Elston Co. acquired 25% interest in Alley Company many years ago. The acquisition did not
result to any goodwill. At the time of acquisition, the carrying amount of Alley’s net assets
approximates its fair value. There have been no impairment losses on the investment. The
carrying amount of the investment on January 1, 2004 is ₱98,500. Alley Company declared
dividends of ₱40,000 in 2004. If the proportionate share of Elston in the net assets of Alley at
December 31, 2004 reflects the carrying amount of Elston’s investment, how much would have
been Alley’s profit in 2004?
a. 200,000.
b. 24,000
c. 96,000.
d. Cannot be determined; given information is insufficient

Investment in associate
1/1/2004 98,500
Sh. In profit (squeeze) 24,000 10,000 Cash dividends
112,500* 12/31/2004

*(370,000 + 80,000) = 450,000 net assets x 25% = 112,500

24,000 ÷ 25% = 96,000


Page |6

10. Karter Company purchased 200 of the 1,000 outstanding shares of Flynn Company's common
stock for ₱180,000 on January 2, 2004. During 2004, Flynn Company declared dividends of ₱30,000
and reported earnings for the year of ₱120,000. If Karter Company uses the equity method of
accounting for its investment in Flynn Company, its Investment in Flynn Company account at
December 31, 2004 should be
a. ₱174,000.
b. ₱180,000.
c. ₱198,000.
d. ₱204,000.

C ₱180,000 + (₱120,000 × 20%) – (₱30,000 × 20%) = ₱198,000.


200 of the 100= 20%

“Peace I leave with you; my peace I give you. I do not give to you as the world gives. Do
NOT let your hearts be troubled and do NOT be afraid.” (John 14:27)

- END -
Page |1

Chapter 15
PPE Part 1

1. Property, plant, and equipment may properly include


a. deposits on machinery not yet received.
b. idle equipment classified as held for sale asset under PFRS 5.
c. land held for speculation, rather than for use in the entity’s normal business activities.
d. none of these.

2. Which of these is not a major characteristic of a PPE?


a. Possesses physical substance
b. Acquired for use in operations
c. Yields services over a number of years
d. All of these are major characteristics of a PPE.

3. PPE purchased on long-term credit contracts should be initially recognized at


a. the total amount of the future payments.
b. the future amount of the future payments.
c. the present value of the future payments.
d. none of these.

4. Nimbus Inc. purchased certain plant assets under a deferred payment contract. The agreement
was to pay ₱30,000 per year for ten years. The plant assets should be initially measured at
a. ₱300,000.
b. ₱300,000 plus imputed interest.
c. present value of ₱30,000 annuity for ten years at an imputed interest rate.
d. future value of ₱30,000 annuity for ten years at an imputed interest rate.

5. A company purchased land to be used as the site for the construction of a plant. Timber was cut
from the building site so that construction of the plant could begin. The proceeds from the sale
of the timber should be
a. classified as other income.
b. netted against the costs to clear the land and expensed as incurred.
c. deducted from the cost of the plant.
d. deducted from the cost of the land.

6. Cotton Hotel Corporation recently purchased Holiday Hotel and the land on which it is located
with the plan to tear down the Holiday Hotel and build a new luxury hotel on the site. The
demolition costs of the Holiday Hotel should be
a. depreciated over the period from acquisition to the date the hotel is scheduled to be torn
down.
b. written off as an extraordinary loss in the year the hotel is torn down.
c. capitalized as part of the cost of the land.
d. capitalized as part of the cost of the new hotel.

7. The cost of land typically includes the purchase price and all of the following costs except
Page |2

a. improvements, such as grading, filling, draining, and clearing.


b. survey costs.
c. cost of private driveways and parking lots.
d. assumption of any liens or mortgages on the property.

8. A donated plant asset for which the fair value has been determined, and for which incidental
costs were incurred in acceptance of the asset, should be recorded at an amount equal to its
a. incidental costs incurred.
b. fair value.
c. book value on books of donor and incidental costs incurred.
d. book value on books of donor.

9. The debit for a non-refundable sales tax properly levied and paid on the purchase of machinery
preferably would be a charge to
a. the machinery account.
b. a separate deferred charge account.
c. miscellaneous tax expense (which includes all taxes other than those on income).
d. accumulated depreciation--machinery.

10. A cost may be capitalized (capital expenditure) if


a. it clearly increases the useful life of an asset.
b. it increases the quantity of an asset.
c. it clearly increases the quality of an asset beyond its original state.
d. Any of these

“From the ends of the earth I call to you, I call as my heart grows faint; lead me to
the rock that is higher than I.” (Psalms 61:2)

- END -
Page |3

1. Merry Co. purchased a machine costing ₱125,000 for its manufacturing operations and paid
shipping costs of ₱20,000. Merry spent an additional ₱10,000 in testing and preparing the
machine for use. What amount should Merry record as cost of the machine?
a. 155,000 b. 145,000 c. 135,000 d. 125,000

A 125,000 + 20,000 + 10,000 = 155,000

2. Peterson, Inc. purchased a machine under a deferred payment contract on December 31, 20x1.
Under the terms of the contract, Peterson is required to make eight annual payments of ₱140,000
each beginning December 31, 20x2. The appropriate interest rate is 8%. The purchase price of the
machine is
a. 1,389,190 b. 1,120,000 c. 868,900 d. 804,520

D (140,000 x PV ordinary annuity @8%, n=8) = 804,529 answer choice is rounded-off

3. Marburg Manufacturing Company purchased a machine on January 2, 20x2. The invoice price of
the machine was ₱40,000, and the vendor offered a 2 percent discount for payment within ten
days. The following additional costs were incurred in connection with the machine:
Transportation-in 1,200
Installation cost 700
Testing costs prior to regular operation 550

If the invoice is paid within the discount period, Marburg should record the acquisition cost of the
machine at
a. 41,650 b. 41,100 c. 40,400 d. 39,200

A
Solution:
Purchase cost of machine net of discount (40K x 98%) 39,200
Transportation-in 1,200
Installation cost 700
Testing costs prior to regular operation 550
Total cost of machine 41,650

4. On July 1, 20x1, Town Company purchased for ₱540,000 a warehouse building and the land on
which it is located. The following data were available concerning the property:
Current appraised value Seller’s original cost
Land 200,000 140,000
Warehouse building 300,000 280,000
Totals 500,000 420,000

Town should record the land at


a. 140,000 b. 180,000 c. 200,000 d. 216,000

D [540,000 x (200,000 / 500,000)] = 216,000


Page |4

5. The Oscar Corporation acquired land, buildings, and equipment from a bankrupt company at a
lump-sum price of ₱180,000. At the time of acquisition, Oscar paid ₱12,000 to have the assets
appraised. The appraisal disclosed the following values:
Land 120,000
Buildings 80,000
Equipment 40,000

What cost should be assigned to the land, buildings, and equipment, respectively?
a. 64,000, 64,000, and 64,000
b. 90,000, 60,000, and 30,000
c. 96,000, 64,000, and 32,000
d. 120,000, 80,000, and 40,000

C
Solution:
Lump-sum purchase price 180,000
Appraisal fee 12,000
Total cost to be allocated 192,000

Fair values Allocation Costs


Land 120,000 (192K x 120/240) 96,000
Buildings 80,000 (192K x 80/240) 64,000
Equipment 40,000 (192K x 40/240) 32,000
240,000 192,000

6. On December 1, 20x1, Boyd Co. purchased a ₱400,000 tract of land for a factory site. Boyd razed
an old building on the property to make way for the construction of the new factory. Boyd sold
the materials it salvaged from the demolition. Boyd incurred additional costs and realized
salvage proceeds during December 20x1 as follows:

Demolition of old building ₱50,000


Legal fees for purchase contract and recording ownership 10,000
Title guarantee insurance 12,000
Proceeds from sale of salvaged materials 8,000

In its December 31, 20x1 statement of financial position, Boyd should report a balance in the land
account of
a. 464,000 b. 460,000 c. 442,000 d. 422,000

D 400,000 + 10,000 + 12,000 = 422,000

7. On February 12, Laker Company purchased a tract of land as a factory site for ₱175,000. An
existing building on the property was razed and construction was begun on a new factory
building in March of the same year. Additional data are available as follows:
Cost of razing old building 35,000
Title insurance and legal fees to purchase land 12,500
Page |5

Architect's fees 42,500


New building construction cost 875,000

The recorded cost of the completed factory building should be


a. 910,000 b. 917,500 c. 930,000 d. 952,500

D
Solution:
Cost of razing old building 35,000
Architect's fees 42,500
New building construction cost 875,000
Cost of factory building 952,500

8. Amble, Inc. exchanged a truck with a carrying amount of ₱12,000 and a fair value of ₱20,000 for
a truck and ₱5,000 cash. The fair value of the truck received was ₱15,000. At what amount
should Amble record the truck received in the exchange?
a. 7,000 b. 9,000 c. 12,000 d. 15,000

D (20,000 FV of asset given up less 5,000 cash received) = 15,000

9. On March 31, 20x1, Winn Company traded in an old machine having a carrying amount of
₱16,800, and paid a cash difference of ₱6,000 for a new machine having a total cash price of
₱20,500. On March 31, 20x1, what amount of loss should Winn recognize on this exchange?
a. 0 b. 2,300 c. 3,700 d. 6,000

B
Solution:
Date Machine – new (at fair value of asset received) 20,500
Loss on exchange
Machine – old (at carrying amt.) 2,300
Cash 16,800
6,000

10. On October 1, Takei, Inc. exchanged 8,000 shares of its ₱25 par value ordinary share for a parcel
of land to be used as site for a new plant. Takei's ordinary share had a fair value of ₱80 per share
on the exchange date. Takei received ₱36,000 from the sale of scrap when an existing building on
the site was razed. The land should be carried at
a. 200,000 b. 236,000 c. 604,000 d. 640,000

D (8,000 x 80) = 640,000 – the fair value of the securities issued is used because the fair value of the asset
received is not given. The demolition costs, net of scrap sale, are charged to the cost of the new building
constructed.

“He who obeys instructions guards his life, but he who is contemptuous of his ways
will die.” (Proverbs 19:16)
- END -
Page |1

Chapter 16
PPE Part 2
1. Depreciation of noncurrent operating assets is an accounting process for the purpose of
a. reporting declining asset values on the balance sheet.
b. allocating asset costs over the periods benefitted by use of the assets.
c. accounting for costs to reflect the change in general price levels.
d. setting aside funds to replace assets when their economic usefulness expires.

2. Which of the following principles best describes the conceptual rationale for the methods of
matching depreciation expense with revenues?
a. Partial recognition
b. Immediate recognition
c. Systematic and rational allocation
d. Associating cause and effect

3. The composite depreciation method


a. is applied to a group of homogeneous assets.
b. is an accelerated method of depreciation.
c. does not recognize gain or loss on the retirement of specific assets in the group.
d. excludes salvage value from the base of the depreciation calculation.

4. The sum-of-the-years'-digits method of depreciation is being used for a machine with a five-year
estimated useful life. What would be the fraction applied to the cost to be depreciated in the
fourth year?
a. 4/5
b. 2/5
c. 4/15
d. 2/15

5. In order to calculate the third year's depreciation on an asset using the sum-of- the-years'-digits
method, which of the following must be known about the asset?
a. Its acquisition cost
b. Its estimated residual value
c. Its estimated useful life
d. All the above must be known.

6. Which of the following statements is the assumption on which straight-line depreciation is


based?
a. The operating efficiency of the asset decreases in later years.
b. Service value declines as a function of time rather than use.
c. Service value declines as a function of obsolescence rather than time.
d. Physical wear and tear are more important than economic obsolescence.

7. A method that ignores residual value in calculating periodic depreciation expenses in the earlier
part of an asset’s useful life is the
a. productive-output method.
Page |2

b. group composite method.


c. sum-of-the-years'-digits method.
d. double-declining-balance method.

8. Which of the following depreciation methods applies a uniform depreciation rate each period to
an asset's carrying amount?
a. Straight-line
b. Units-of-production
c. Declining-balance
d. Sum-of-the-years'-digits

9. When does the cost of land affect an entity’s profit or loss?


a. As the asset is used through periodic charges for cost allocation
b. When the asset is revalued upwards
c. When the related revaluation is recognized on a piecemeal basis as the asset is used
d. When the asset is impaired or when it is sold above or below its carrying amount

10. Which of the following reasons provides the best theoretical support for accelerated
depreciation?
a. Assets are more efficient in early years and initially generate more revenue.
b. Expenses should be allocated in a manner that "smooths" earnings.
c. Repairs and maintenance costs will probably increase in later periods, so depreciation
should decline.
d. Accelerated depreciation provides easier replacement because of the time value of money.

11. When the estimate of an asset's useful life is changed,


a. depreciation expense for all past periods must be recalculated.
b. there is no change in the amount of depreciation expense recorded for future years.
c. only the depreciation expense in the remaining years is changed.
d. None of the above is true.

12. The sale of a depreciable asset resulting in a loss indicates that the proceeds from the sale were
a. less than the asset’s current market value.
b. greater than the asset’s cost.
c. greater than the asset’s carrying amount.
d. less than the asset’s carrying amount.

13. On January 1 Stockton Company acquired a machine with a four-year useful life. Stockton
estimates the residual value of the machine will be equal to ten percent of the acquisition cost.
The company is debating between using either the double-declining-balance method or the sum-
of-the-years'-digits method of depreciation. Comparing the depreciation expense for the first
two years computed using these methods, the depreciation expense for the double-declining-
balance method (compared to the sum-of-the-years'-digits method) will match which of the
patterns shown below?
First year Second year
a. Lower Lower
b. Lower Higher
c. Higher Lower
Page |3

d. Higher Higher

14. Which of the following utilizes the straight-line depreciation method?


Composite Depreciation Group Depreciation
a. Yes Yes
b. Yes No
c. No Yes
d. No No

15. A depreciable asset has an estimated 15 percent salvage value. At the end of its estimated useful
life, the accumulated depreciation would equal the original cost of the asset under which of the
following depreciation methods?
Productive-Output Sum-of-the-Years'-Digits Double-Declining-Bal.
a. Yes No No
b. No No No
c. No Yes No
d. Yes Yes Yes

“Pride goes before destruction, a haughty spirit before a fall.” (Proverbs 16:18)

- END -
Page |4

1. On January 1, 20x1 Buckle Co. purchased a machine that had a list price of ₱46,320. Buckle Co.
paid cash of ₱18,000 and executed a one-year non-interest-bearing note for the balance. The
going rate of interest was 18%. The machine has a 6-year life and no residual value. Depreciation
expense on the SYD basis at the end of 20x1 is:
a. 8,092
b. 12,000
c. 13,234
d. 14,690
B
Solution:
Cost =18,000 + [(46,320 – 18,000) x PV of 1 @18%, n=1] = 42,000
SYD denominator = 6 x [(6+1)/2] = 21
Depreciation = 42,000 x 6/21 = 12,000

The next four items are based on the following information:


Dirt Corporation schedule of depreciable assets at December 31, 20X7 was as follows:
Accum.
Asset Cost Depreciation Acquisition date Residual value
A 100,000 64,000 20X6 20,000
B 55,000 36,000 20X5 10,000
C 70,000 33,600 20X5 14,000

Dirt takes a full year’s depreciation expense in the year of an asset’s acquisition, and no depreciation
expense in the year of an asset’s disposition. The estimated useful life of each depreciable asset is 5
years.

2. Dirt depreciates asset A on the double-declining-balance method. How much depreciation


expense should Dirt record in 20X8 for asset A?
a. ₱32,000
b. ₱25,600
c. ₱14,400
d. ₱ 6,400

C DDB rate = 2/5 = 40%


(100,000 – 64,000) x 40% = 14,400

3. Dirt depreciates asset A on the double-declining-balance method. How much depreciation


expense should Dirt record in 20X9 for asset A?
a. ₱ 2,000
b. ₱ 5,600
c. ₱ 1,600
d. ₱ 8,640

C DDB rate = 2/5 = 40%

Year Carrying amt. Depreciation


(a) (b) = 40% x (a)
20x6 100,000 40,000
20x7 60,000 24,000
20x8 36,000 14,400
Page |5

20x9 21,600 8,640


20x10 12,960 5,184

(21,600 carrying amount - 20,000 residual value) = 1,600

4. Using the same depreciation method as used in 20X5, 20X6, and 20X7, how much depreciation
expense should Dirt record in 20X8 for asset B?
a. ₱ 6,000
b. ₱ 9,000
c. ₱11,000
d. ₱12,000

A
SYD = 5 x [(5 + 1) / 2] = 15
Depreciation, 20x8 = (55,000 – 10,000 residual value) x 2/15 = 6,000

5. Dirt depreciates asset C by the straight-line method. On June 30, 20X8, Dirt sold asset C for
₱28,000 cash. How much gain (loss) should Dirt record in 2008 on the disposal of asset C?
a. ₱ 2,800
b. ₱(2,800)
c. ₱(5,600)
d. ₱(8,400)

D 28,000 – (70,000 – 33,600)* = (8,400)


*”Dirt takes a full year’s depreciation expense in the year of an asset’s acquisition, and no depreciation expense in
the year of an asset’s disposition.”

6. Enter Sandman Co. purchased manufacturing equipment from Sad But True Co. on January 1,
20x8 at a total cost of ₱9,000,000. Enter Sandman uses the straight-line method of depreciation
and estimates that the equipment has a useful life of 10 years. On July 1, 20x8 and July 1, 20x9
Enter Sandman performed major regular inspections on the equipment costing ₱380,000 and
₱425,000, respectively. The costs of inspection satisfied the recognition criteria for capitalization.
How much is the carrying amount of the equipment on December 31, 2009?
a. 7,920,000
b. 7,875,000
c. 7,529,412
d. 7,600,000

D (9,000,000 x 8/10) + (425,000 latest inspection x 8/8.5) = 7,600,000

7. Tonyo Company uses the composite method of depreciation and has a composite rate of 25%.
During 20x1, it sold assets with an original cost of ₱100,000 (residual value of ₱20,000) for
₱80,000 and acquired ₱60,000 worth of new assets (residual value of ₱10,000). The original group
of assets had the following characteristics:
Total Cost ₱250,000
Total Residual Value 30,000

The above original group includes the assets sold in 20x8 but not the assets purchased in 20x8. How
much is the depreciation in 20x8?
Page |6

a. ₱62,500
b. ₱52,500
c. ₱47,500
d. ₱46,500
B
Adjusted total cost = 250,000 – 100,000 assets sold + 60,000 new assets acquired = 210,000

Depreciation rate = Annual depreciation / Total cost


25% = Annual depreciation / 210,000
Annual depreciation = 210,000 x 25% = 52,500

Use the following information for the next four items:


Light Company bought a machine for ₱300,000 on January 1, 20x8. The machine's useful life is 10
years and it is estimated to have a zero residual value and is depreciated using the straight-line
method.

The revalued amount of the machine is as follows:


December 31 Fair values of the machine
20x8 ₱ 360,000
20x9 335,000
2x10 320,000

The enacted tax rate was 30% for each year

8. The revaluation surplus in the equity section of Light Company’s December 31, 20x8 statement
of financial position is
a. 60,000
b. 90,000
c. 39,000
d. 63,000

D 360,000 – (300,000 x 9/10) = 90,000 x 70% = 63,000

9. The amount of depreciation expense to be recognized in 20x9 is


a. 32,500
b. 36,000
c. 40,000
d. 42,500

C 360,000 / 9= 40,000

10. The amount of revaluation surplus transferred to retained earnings in 20x9 is


a. 6,667
b. 7,000
c. 4,333
d. 10,000

B = 63,000 / 9 = 7,000
Page |7

11. The revaluation surplus in the equity section of Light Company's December 31, 2x10 statement
of financial position is
a. 77,000
b. 110,000
c. 123,443
d. 109,500

A 320,000 – (300,000 x 7/10) = 110,000 x 70% = 77,000

Use the following information for the next four items:


Information on Mix Co.’s equipment on June 30, 20x8 is shown below:

Equipment (at cost) ₱ 500,000


Accumulated depreciation 150,000
₱ 350,000

The equipment consists of two machines, Machine A and Machine B. Machine A has a cost of
₱300,000 and a carrying amount of ₱180,000. Machine B has a cost of ₱200,000 and a
carrying amount of ₱170,000. Both machines are measured using the cost model and depreciated on
a straight line basis over a ten-year period.

On December 31, 20x8, Mix Co. decided to change from the cost model to the revaluation model.
Information on this date follows:
Fair values Remaining useful life
Machine A ₱180,000 6 years
Machine B ₱155,000 5 years

On June 30, 20x9, Machine A and Machine B have fair values of ₱163,000 and ₱136,500, respectively,
and remaining useful lives of 5 years and 4 years, respectively. The tax rate is 30%.

12. How much is the depreciation expense for the fiscal year ended June 30, 20x9?
a. 59,900
b. 55,500
c. 50,000
d. 67,000

6/30/x8 to 12/31/x8
Machine A: (300K historical cost ÷ 10 yrs.) x 6/12 = 15,000
Machine B: (200K historical cost ÷ 10 yrs.) x 6/12 = 10,000

1/1/x9 to 6/30/x9
Machine A: (180K fair value ÷ 6 yrs.) x 6/12 = 15,000
Machine B: (155K fair value ÷ 5 yrs.) x 6/12 = 15,500

Total depreciation = (15K + 10K + 15K + 15.5K) = 55,500

13. How much is the revaluation surplus on December 31, 20x8?


a. 10,500
Page |8

b. (15,000)
c. (10,500)
d. 7,000

A
Machine A: (180K fair value – 165,000 carrying amount*) x 70% = 10,500
Machine B: (155K fair value – 160,000 carrying amount*) = (5,000) impairment loss

*The carrying amounts are computed as follows:


Machine A: (180K – depreciation 6/30/x8 to 12/31/x8 15,000) = 165,000
Machine A: (170K – depreciation 6/30/x8 to 12/31/x8 10,000) = 160,000

14. How much is the carrying amount of the equipment on June 30, 20x9?
a. 163,000
b. 335,000
c. 300,000
d. 299,500

D (163,000 + 136,500) = 299,500

15. Entity A has identified indications that its plant is impaired. The plant has a carrying amount of
₱56,000,000. An independent valuer determined the following:
• Replacement cost of the plant ₱90,000,000
• Actual life 15 years
• Effective life 25 years
• Remaining economic life 20 years

Entity A’s tax rate is 30%.

How much is the revaluation surplus, net of tax?


a. 16,000,000
b. 11,200,000
c. 18,250,000
d. 12,775,000

B
Replacement cost 90,000,000
Less: Depreciation (90M x 25(a)/45(b)) (50,000,000)
Fair value 40,000,000
Less: Carrying amount (56,000,000)
Revaluation surplus, gross of tax (16,000,000)
Multiply by: 70%
Revaluation surplus, net of tax 11,200,000

Effective life (Effective age)


(a)

Total economic life = Effective life + Remaining economic life


(b)

(10 + 30) = 40
Page |9

“Praise be to the God and Father of our Lord Jesus Christ, the Father of compassion
and the God of all comfort, who comforts us in all our troubles, so that we can
comfort those in any trouble with the comfort we ourselves have received from
God.” (2 Corinthians 1:3-4)

- END -
Page |1

Chapter 17
Depletion of Mineral Resources

1. Exploration and evaluation assets are initially measured at


a. cost.
b. revalued amount.
c. fair value.
d. a or b

2. Exploration and evaluation assets are exploration and evaluation expenditures recognized as
a. assets in accordance with the entity’s accounting policy.
b. expenses in accordance with applicable PFRSs.
c. assets in accordance with (a) above, subject to the limitations provided under PAS 8
Accounting Policies, Changes in Accounting Estimates and Errors.
d. any of these

Use the following information for the next two questions:


In 20x1, OBSTREPEROUS NOISY Mining Corp. acquired the right to use 1,000 acres of land to mine
for gold. The lease cost is ₱200,000,000, and the related exploration costs on the property amounted
to ₱40,000,000. It is the policy of OBSTREPEROUS Mining Corp. to capitalize all costs of exploration
and evaluation of mineral resources. Intangible development costs of drilling, tunnels, shafts, and
wells incurred before opening the mine amounted to ₱340,00,000. At the end of the mine’s economic
useful life, OBSTREPEROUS Mining Corp. is required by legislation to restore the site. Estimated
restoration costs have a fair value of ₱20,000,000. OBSTREPEROUS Mining Corp. estimates that the
mine will provide approximately 100,000,000 ounces of gold. Actual ounce of gold mined in 20x2
totaled 300,000 ounces.

3. How much is the depletion charge in 20x2?


a. 1,740,000 b. 1,800,000 c. 165,000 d. 150,000

B (200,000,000 + 40,000,000 + 340,000,000 + 20,000,000) x 300/100,000 = 1,800,000

4. Assuming that of the 300,000 ounces of gold extracted in 20x2, 280,000 ounces were sold and
20,000 ounces remain in inventory. How much depletion is recognized in the 20x2 (a) statement
of financial position and (b) statement of profit or loss and other comprehensive income,
respectively?
Statement of financial position Statement of profit or loss
a. 1,680,000 120,000
b. 116,000 1,624,000
c. 11,000 154,000
d. 120,000 1,680,000

D 1,800,000 x 20/300 = 120,000 inventory


1,800,000 x 280/300 = 1,680,000 COGS
Page |2

Use the following information for the next two questions:


In 20x1, BUCOLIC RURAL Co. acquired land for a total cost of ₱40,000,000 to be used to quarry
marble, limestone, and construction aggregates. Costs incurred to obtain legal right to explore the
property amounted to ₱8,000,000. Expenditures incurred in the exploration for and evaluation of
mineral resources before technical feasibility and commercial viability of extracting a mineral
resource are demonstrable totaled ₱12,000,000. Intangible development costs of drilling, tunnels,
shafts, and wells before the actual production totaled ₱20,000,000. BUCOLIC Co. estimates that total
recoverable reserves are 100,000,000 units. Furthermore, BUCOLIC Co. expects to sell the land for
₱4,800,000 after resource is depleted. However, no buyer will pay this price unless the mine is
drained, filled and leveled, a process that will cost ₱800,000. It is BUCOLIC’s policy to capitalize all
exploration costs.

Actual units quarried in 20x1 through 20x4 totaled 30,000,000 units. On January 1, 20x5, BUCOLIC
Co. estimated that the remaining recoverable reserves are only 25,000,000 units and after the
reserves are exhausted, the land will be sold for ₱3,200,000. Costs of disposal are estimated at
₱1,200,000. Actual units quarried in 20x5 totaled 6,000,000 units.

5. How much is the depletion charge in 20x5?


a. 13,284,000 b. 13,480,000 c. 13,280,000 d. 13,248,000

D
Solution:

Acquisition cost 40,000,000


Legal right 8,000,000
Exploration costs 12,000,000
Intangible development costs 20,000,000
Total cost of natural resource 80,000,000
Residual value (4.8M - .8M ) (4,000,000)
Depletion base – 20x1 76,000,000
Multiply by: units quarried over initial estimate 30M/100M
Accumulated depletion – Dec. 31, 20x4 22,800,000

Total cost of natural resource 80,000,000


Accumulated depletion – Dec. 31, 20x4 (22,800,000)
Carrying amount – Jan. 1, 20x5 57,200,000
Revised residual value (3.2M – 1.2M) (2,000,000)
Revised depletion base 55,200,000
Multiply by: units quarried over changed estimate 6M/25M
Depletion – 20x5 13,248,000

6. What is the carrying amount of the wasting asset on December 31, 20x5?
a. 43,852,000 b. 44,272,000 c. 42,720,00 d. 43,952,000

D
Solution:
Carrying amount – Jan. 1, 20x5 57,200,000
Depletion – 20x5 (13,248,000)
Carrying amount – Dec. 31, 20x5 43,952,000
Page |3

“A wise man will hear and increase learning, and a man of understanding will attain
wise counsel.” (Proverbs 1:5)

- END -
Page |1

Chapter 18
Government Grants
1. According to PAS 20, non-monetary grants are measured at
a. the fair value of the non-monetary asset.
b. nominal amount.
c. the amount of cash received or receivable.
d. a or b

2. According to PAS 20, government grants are presented in the financial statements using
a. a gross presentation. c. a or b
b. a net presentation. d. a functional presentation.

Use the following information for the next three questions:


On January 1, 20x1, Entity A received land with fair of ₱200,000 from the government conditioned
on the construction of a building on the lot. Entity A started immediately the construction and it was
completed on December 31, 20x1 for a total cost of ₱1,000,000. The building has an estimated useful
life of 10 years and zero residual value.

3. How much is the income from government grant in 20x1 and 20x2, respectively?
20x1 20x2
a. 0 200,000
b. 200,000 0
c. 0 20,000
d. 20,000 20,000

C 20x1 = 0, no depreciation yet is recognized from the building; therefore, no income yet is
recognized from the government grant (i.e., ‘matching’). The building starts to be depreciated in
20x2.
20x2 = 20,000 (200,000 ÷ 10 years)

4. How much is the carrying amount of the building on December 31, 20x2 under the following
presentations?
Gross presentation Net presentation
a. 1,000,000 800,000
b. 900,000 720,000
c. 800,000 640,000
d. 800,000 533,333

B Gross presentation: (1M x 9/10) = 900,000;


Net presentation: (1M – 200K) x 9/10 = 720,000

5. How much is the depreciation expense recognized in 20x3 under the following presentations?
Gross presentation Net presentation
a. 100,000 80,000
b. 100,000 100,000
c. 80,000 100,000
d. 80,000 80,000
Page |2

B Gross presentation: (1M ÷ 10) = 100,000;


Net presentation: (1M – 200K) ÷ 10 = 80,000

“A fool shows his annoyance at once, but a prudent man overlooks an insult.”
(Proverbs 12:16)
- END -
Page |1

Chapter 19
Borrowing Costs

1. According to PAS 23, borrowing costs are capitalized when


a. they relate directly to the acquisition, construction or production of a qualifying asset.
b. the entity chooses to capitalize them.
c. they are material and are expected to be incurred over more than one reporting period.
d. all of these

2. Which of the following is a qualifying asset?


a. Investment property measured at fair value
b. Building that is ready for its intended use upon purchase
c. Inventories that are routinely produced in large quantities on a continuous basis
d. An application software (intangible asset) that takes 3 years to develop

3. An entity starts the capitalization of borrowing costs to the cost of a qualifying asset when
a. expenditures for the asset are being incurred.
b. borrowing costs are being incurred.
c. activities necessary to prepare the asset for its intended use or sale are being undertaken.
d. all of the above conditions are met.

4. In which of the following instances is the capitalization of borrowing costs under PAS 23 would
most likely be suspended?
a. Construction is temporarily stopped for the curing of concrete.
b. Active development is stopped to give time for the engineers to reevaluate a design flaw.
c. The construction of a bridge is disrupted by troubled waters.
d. The construction of a building is discontinued because it is condemned by the government.
The resumption of development is uncertain.

Use the following information for the next two questions:


On January 1, 20x1, Entity A had the following general borrowings. A part of the proceeds was used
to finance the construction of a qualifying asset:
Principal
12% bank loan (1.5 years) ₱ 1,000,000
10% bank loan (3-year) 8,000,000

Expenditures made on the qualifying asset were as follows:


Jan. 1 ₱ 5,000,000
March 1 4,000,000
August 31 3,000,000
December 1 2,000,000

Construction was completed on December 31, 20x1.

5. How much borrowing costs are capitalized to the cost of the constructed qualifying asset?
Page |2

a. 1,045,000 c. 1,026,667
b. 971,111 d. 920,000

D Solution:
The average expenditure is computed as follows:
Month outstanding over
Date Expenditures Average expenditure
12 Months
(a) (b) (c)= (a) x (b)
Jan.1 5,000,000 12/12 5,000,000
Mar. 1 4,000,000 10/12 3,333,333
Aug. 31 3,000,000 4/12 1,000,000
Dec. 1 2,000,000 1/12 166,667
9,500,000

The capitalization rate is computed as follows:

Total interest expense on general borrowings


Capitalization rate =
Total general borrowings

Total interest expense on general borrowings


(1M x 12%) + (8M x 10%) 920,000
Divide by: Total general borrowings (1M + 8M) 9,000,000
Capitalization rate 10.22%

Capitalizable BC from formula = 9,500,000 x 10.22% = 971,111

971,111 vs. 920,000 actual borrowing costs = Capitalizable BC is 920,000, the lower amount

6. How much is the cost of the qualifying asset on initial recognition?


a. 13,010,000 c. 14,920,000
b. 15,045,000 d. 14,971,111

C (5M + 4M + 3M + 2M total expenditures on construction) + 920,000 capitalizable borrowing costs =


14,920,000

7. PAS 23 does not require which of the following disclosures?


a. The amount of borrowing costs capitalized during the period.
b. The capitalization rate used to determine the capitalizable borrowing costs.
c. Separate presentation of qualifying assets from other assets either on the face of the
statement of financial position or in the notes.
d. PAS 23 requires the disclosure of all of these information.
Page |3

“Trust in the Lord with all your heart and lean not on your own understanding; in
all your ways acknowledge him, and he will make your paths straight.” (Proverbs 3:5-6)

- END -
Page |4
Page |1

Chapter 20
Agriculture

1. Which of the following is not dealt with by PAS 41?


a. The accounting for biological assets.
b. The initial measurement of agricultural produce harvested from the entity’s biological assets.
c. The processing of agricultural produce after harvest.
d. The accounting treatment of government grants received in respect of biological assets.

2. Where there is a long aging or maturation process after harvest, the accounting for such
products should be dealt with by
a. PAS 41.
b. PAS 2, Inventories.
c. PAS 16, Property, Plant, and Equipment.
d. PAS 40, Investment Property.

3. Generally speaking, biological assets relating to agricultural activity should be measured using
a. Historical cost.
b. Historical cost less depreciation less impairment.
c. A fair value approach.
d. Net realizable value.

4. Entity A had a plantation forest that is likely to be harvested and sold in 30 years. The income
should be accounted for in the following way:
a. No income should be reported until first harvest and sale in 30 years.
b. Income should be measured annually and reported using a fair value approach that
recognizes and measures biological growth.
c. The eventual sale proceeds should be estimated and matched to the profit and loss account
over the 30-year period.
d. The plantation forest should be valued every 5 years and the increase in value should be
shown in the statement of recognized gains and losses.

5. Where the fair value of the biological asset cannot be determined reliably on initial recognition,
the biological asset should be measured at
a. Cost.
b. Cost less accumulated depreciation.
c. Cost less accumulated depreciation and accumulated impairment losses.
d. Net realizable value.

6. Which of the following costs are not included in costs to sell?


a. Commissions to brokers and dealers.
b. Levies by regulatory agencies.
c. Transfer taxes and duties.
Page |2

d. Transport and other costs necessary to get the assets to a market.

Use the following information for the next four questions:


The following information pertains to Madagascar Co.
Sheep 500,000 Wool 6,000
Rubber products 10,000 Thread 3,000
Trees in a timber plantation 95,000 Felled trees 8,000
Maize plants 40,000 Clothing 150,000
Lumber 62,000 Milk 9,000
Pigs 200,000 Carcass 7,000
Roasted peanuts 20,000 Sugar 67,000
Cotton plants 10,000 Harvested cotton 13,000
Peanut plants 5,000 Harvested peanuts 140,000
Sugarcane 25,000 Harvested cane 22,000
Tobacco plants 45,000 Picked leaves 3,000
Tea bushes 800,000 Oil palms 300,000
Dairy cattle 1,000,000 Picked grapes 2,000
Fruit trees 600,000 Picked fruit 10,000
Tea 43,000 Grape vines 2,000,000
Rubber trees 300,000 Harvested latex 10,000
Yarn 22,000 Cured tobacco 320,000
Carpet 33,000 Wine 500,000
Logs 45,000 Processed fruit 20,000
Wheat plants 60,000 Palm oil 50,000
Cheese 75,000 Bean plants 20,000
Sausages 88,000 Cured hams 92,000

7. How much is classified as biological assets that are accounted for under PAS 41 Agriculture?
a. 2,660,000 b. 2,000,000 c. 6,000,000 d. 2,250,000

8. How much is classified as property, plant and equipment that are accounted for under PAS 16
Property, Plant and Equipment?
a. 4,000,000 b. 4,860,000 c. 4,560,000 d. 3,650,000

9. How much is classified as agricultural produce?


a. 149,000 b. 248,000 c. 290,000 d. 230,000

10. How much is classified as inventory?


a. 1,480,000 b. 1,600,000 c. 1,540,000 d. 1,880,000

Light blue= biological asset


Green= PPE
Yellow= Agricultural Produce
Pink= Inventory

“The name of the Lord is a strong tower; the righteous run to it and are safe.”
(Proverbs 18:10)
Page |3

- END -

SOLUTIONS:

BA PPE AP INVTY
Sheep 500,000
Rubber products 10,000
Trees in a timber plantation 95,000
Maize plants 40,000
Lumber 62,000
Pigs 200,000
Roasted peanuts 20,000
Cotton plants 10,000
Peanut plants 5,000
Sugarcane 25,000
Tobacco plants 45,000
Tea bushes 800,000
Dairy cattle 1,000,000
Fruit trees 600,000
Tea 43,000
Rubber trees 300,000
Yarn 22,000
Carpet 33,000
Logs 45,000
Wheat plants 60,000
Cheese 75,000
Sausages 88,000
Wool 6,000
Thread 3,000
Felled trees 8,000
Clothing 150,000
Milk 9,000
Carcass 7,000
Sugar 67,000
Harvested cotton 13,000
Harvested peanuts 140,000
Harvested cane 22,000
Picked leaves 3,000
Oil palms 300,000
Picked grapes 2,000
Picked fruit 10,000
Grape vines 2,000,000
Harvested latex 10,000
Cured tobacco 320,000
Wine 500,000
Processed fruit 20,000
Palm oil 50,000
Bean plants 20,000
Cured hams 92,000
TOTALS 2,000,000 4,000,000 230,000 1,600,000
Page |1

Chapter 21
Investment Property
1. Which of the following would not be reported as investment property?
a. Property owned by the entity and leased out under one or more operating leases.
b. Property held by the entity to be leased out under one or more operating leases
c. Real estate held for an undetermined future use.
d. Property owned by the entity and leased out to another entity under a finance lease.

2. A property is classified as investment property if


a. it is leased out under a finance lease.
b. the owner-occupied portion of the property is significant.
c. the entity provides relatively insignificant ancillary services (e.g., security, janitorial services,
and the like) to the occupants of the property.
d. it is rented between a parent entity and a subsidiary and consolidated financial statements
are prepared for the group.

3. All of the following do not qualify as investment property, except


a. Machineries that are held for lease
b. Hotels or motels
c. Agricultural land purchased for appreciation purposes
d. Equipment purchased for an indeterminate purpose

4. How does the fair value model differ from the revaluation model?
a. Increases in carrying amount above a cost-based measure are recognized in equity
b. Changes in fair value are recognized in profit or loss
c. a and b
d. neither a nor b

5. Select the correct statement.


a. A leasing company should treat all its assets used in providing lease services as investment
property.
b. Investment properties that are to be disposed of without further development are treated as
investment property until they are derecognized.
c. All investment properties held for capital appreciation will be classified as held for sale in
the long run.
d. Investment properties being re-developed as investment properties on behalf of third parties
are investment properties.

6. Which of the following generally provides the best evidence of fair value of an investment
property?
a. Discounted cash flow projections based on reliable estimates of future cash flows.
b. Recent prices on less active markets with adjustments to reflect changes in economic
conditions.
c. Current prices for properties of a different nature or subject to different conditions.
d. Current prices on an active market for similar property in the same location and condition.

7. MODULATE Co. has the following assets.


Page |2

Vacant building to be leased out under operating lease 4,000,000


Building being constructed for TO ADJUST, Inc. 800,000
Building under construction to be used as office 1,600,000
Building under construction to be rented out under operating lease 400,000
Building rented out to MODULATE’s employees who pay rent at market rates 3,200,000
Office building awaiting disposal 200,000

How much is the total investment property?


a. 4,200,000 b. 4,400,000 c. 4,600,000 d. 7,600,000

B
Solution:
Vacant building to be leased out under operating lease 4,000,000
Building under construction to be rented out under
operating lease 400,000
Total investment property 4,400,000

8. On January 1, 20x1, NURTURE REAR Co. acquired a building with an estimated useful life of 10
years and residual value of ₱400,000 for a total cost of ₱4,000,000. The fair value of the building
on January 1, 20x1 is ₱4,800,000 while the fair value on December 31, 20x1 is ₱5,200,000.
NURTURE estimates that if the building is sold currently on December 31, 20x1, costs to sell
amount to ₱200,000. NURTURE uses the straight-line method in depreciating its PPE.
NURTURE uses the fair value model for its investment properties. The year-end adjusting entry
will include
a. 360,000 depreciation c. 200,000 unrealized gain
b. 400,000 unrealized gain d. 1,200,000 unrealized gain

D
Solution:
Dec. 31, Investment property (5.2M – 4M) 1,200,000
20x1 Unrealized gain from change in fair value 1,200,000

9. On December 31, 20x1, DECAPITATE BEHEAD Co. decided to lease out under operating lease
one of its buildings that was previously used as office space. The building has an original cost of
₱12,000,000 and accumulated depreciation of ₱8,000,000 as of January 1, 20x1. Annual
depreciation is ₱400,000. DECAPITATE Co. uses the fair value model for investment property.
The fair value of the building on December 31, 20x1 is ₱6,000,000. The entry to record the
transfer of the building to investment property includes a
a. credit to gain on reclassification for ₱2,000,000.
b. credit to revaluation surplus for ₱2,000,000.
c. debit to building for ₱12,000,000.
d. credit to revaluation surplus for ₱2,400,000.

Dec. 31, Investment property (fair value) 6,000,000


20x1 Accumulated depreciation – Bldg. (8M + 400K) 8,400,000
Building 12,000,000
Revaluation surplus (squeeze) 2,400,000
Page |3

10. PERIODIC REGULAR Co. acquired a building on January 1, 20x1 for a total cost of ₱24,000,000
and classified it as investment property. PERIODIC Co. uses the fair value model for its
investment property. On January 1, 20x5, when the carrying amount of the building is
₱16,000,000, the elevator in the building was replaced for a total cost of ₱3,200,000. It is
impracticable to determine the fair value of the replaced part. The fair value of the building on
December 31, 20x5 is ₱17,200,000. How much is the loss recognized during the year?
a. 3,200,000 b. 2,000,000 c. no loss d. indeterminable

B [17.2M fair value on Dec. 31, 20x5 – (16M carrying amount on Jan. 1, 20x5 + 3,200,000 cost of
replacement) = 2,000,000 loss

“Formal education will make you a living; self-education will make you a
fortune.” - Jim Rohn
- END -
Page |1

Chapter 22
Intangible Assets

1. Which of the following statements is correct?


a. Some intangible assets have physical substance.
b. Intangible assets are always classified as noncurrent assets even in cases where they are part
of a disposal group.
c. The process of recording the expiration of the economic benefits of an intangible asset is
called depletion.
d. Intangible assets can be obtained in one of two ways – external acquisition or internal
development.

2. Which of the following statements is true?


a. The capitalizable costs of patents are legal fees and other registration costs.
b. An identifiable tangible asset developed internally is never recognized in the accounts as an
asset.
c. Intangible assets usually have a residual value that must be considered in the amortization
of cost.
d. An intangible asset is usually amortized by a credit to an income account.

3. Intangible assets have all of the following characteristics, except:


a. their ownership confers rights, but no physical substance.
b. they have no physical substance.
c. they are relatively long-lived.
d. they provide benefits to current operations only.

4. Which of the following is not a consideration in determining the useful life of an intangible
asset?
a. Cost
b. Legal, regulatory, or contractual provisions
c. Provisions for renewal or extension
d. Expected actions of competitors

5. In accordance with the PFRSs, which of the following methods of amortization is normally not
recommended for intangible assets?
a. Units of production
b. Declining balance
c. Effective interest method
d. Straight line

6. A change in the amortization rate for an intangible asset should be accounted for
a. by retrospective restatement.
b. by retrospective application.
c. on a prospective basis.
Page |2

d. on a current basis.

7. Which of the following is an unidentifiable asset?


a. Goodwill
b. Private-to-private franchise
c. Deferred charges
d. Unearned rent

8. If a franchise becomes worthless before the end of its useful life, the carrying amount of the
franchise account should be charged as
a. franchise expense in the current period.
b. a prior period adjustment.
c. impairment loss.
d. amortization expense.

9. Which of the following costs related to computer software is capitalized to an intangible asset
account?
a. Costs of duplication and reproduction of software for sale
b. Development costs before technological feasibility is achieved
c. Coding and testing costs incurred to establish technological feasibility
d. Coding and testing costs incurred after technological feasibility is established but before the
product master is completed

10. Which of the following is a true statement concerning research and development (R&D) costs?
a. All R&D costs, without exception, must be charged to expense when incurred.
b. R&D costs can only be amortized over a life of 40 years or more.
c. Almost any treatment is acceptable for handling R&D costs.
d. Financial statements must disclose total R&D costs charged to expense in the period

11. A patent infringement suit may be either successful or unsuccessful. Which of the following
statements is correct?
a. If the lawsuit is successful, the cost of the lawsuit is expensed.
b. If the lawsuit is unsuccessful, the cost of the lawsuit is recognized as additional amortization
expense.
c. If the lawsuit is unsuccessful, the cost of the lawsuit is written-off from the carrying amount
of the related patent.
d. If the lawsuit is unsuccessful, the carrying amount of the related patent is amortized over its
remaining economic life.

12. An entity that incurs costs in defending a patent in an infringement suit should
a. expense the costs of all suits in the period in which they are incurred.
b. capitalize only the costs of unsuccessful suits.
c. capitalize only the costs of successful suits.
d. capitalize the cost of all suits regardless of the outcome.
Page |3

13. A purchased patent has a remaining legal life of 15 years. It should be


a. expensed in the year of the acquisition.
b. amortized over 15 years regardless of its useful life.
c. amortized over its useful life if less than 15 years.
d. not amortized.

14. The cost of a franchise is classified in the statement of financial position as a(n)
a. operational asset.
b. deferred charge.
c. intangible asset.
d. current asset.

15. Silverchair Airlines purchased airline gate rights from Tomorrow International Airport for
₱2,000,000. The rights have a legal life of five years; however, Silverchair can extend the rights
for another ten years over an indefinite number of extensions at a nominal cost. Silverchair
intends and has the ability to make the extensions. Other owners of similar rights have made the
right extensions in the past. Over what period of time should Silverchair amortize the gate
rights?
a. 5 years.
b. 15 years
c. 40 years.
d. The rights should not be amortized.

“I have never let my schooling interfere with my education.” – Mark Twain

- END -
Page |4

1. Sponge Co. incurred research and development costs in 20x1 as follows:


• Equipment acquired for use in various research and development projects 975,000
• Depreciation on the above equipment 135,000
• Materials used 200,000
• Compensation costs of personnel 500,000
• Outside consulting fees 150,000
• Indirect costs appropriately allocated 250,000

The total research and development costs charge in Sponge's 20x1 statement of profit or loss should
be:
a. 850,000
b. 1,085,000
c. 1,235,000
d. 1,825,000

C
Solution:
Depreciation on the above equipment 135,000
Materials used 200,000
Compensation costs of personnel 500,000
Outside consulting fees 150,000
Indirect costs appropriately allocated 250,000
Total R&D expense 1,235,000

2. Sanitarium Co. has the following assets, stated at cost, as of December 31, 20x1:
Macintosh laptops 600,000
EPSON printers 10,000
FUJI XEROX photocopying machines 250,000
Windows Operating Software 25,000

All of the listed assets above have an original estimated useful life of 5 years and that their
remaining useful life on December 31, 20x1 is 4 years. Sanitarium uses the straight line method for
its items of property, plant and equipment and intangible assets with finite useful life. How much
would be shown as property, plant and equipment in Sanitarium’s December 31, 20x1 statement of
financial position?
a. 880,000
b. 688,000
c. 708,000
d. 2,816,000
C
Solution:
Macintosh laptops 600,000
EPSON printers 10,000
FUJI XEROX photocopying machines 250,000
Windows Operating Software 25,000
Total PPE – gross 885,000
Multiply by: 4/5
Total PPE – net 708,000
Page |5

3. On January 1, 20x1, Devjo Co. bought a franchise from McDo Co. The franchise requires an
initial fee of ₱15,000,000 payable as follows: ₱5,000,000 down payment on contract date and the
balance in five equal annual installments starting one year from contract date plus 4% interest on
the outstanding principal balance. The current market rate as of contract date is 12%. How much
is the initial measurement of the intangible asset?
a. 14,887,922
b. 15,234,090
c. 13,765,340
d. 13,139,702
D
Solution:
The cash flows on the installment payments are computed as follows:
Date Principal Interest Total
1/1/x2 2,000,000 (10M x 4%) 400,000 2,400,000
1/1/x3 2,000,000 (8M x 4%) 320,000 2,320,000
1/1/x4 2,000,000 (6M x 4%) 240,000 2,240,000
1/1/x5 2,000,000 (4M x 4%) 160,000 2,160,000
1/1/x6 2,000,000 (2M x 4%) 80,000 2,080,000
Totals 10,000,000 1,200,000 11,200,000

The present value of the cash flows is computed as follows:


PV of 1 PV
Present value
Date Cash flows @12% factors
1/1x1 5,000,000 n=0 1 5,000,000
1/1/x2 2,400,000 n=1 0.8928571 2,142,857
1/1/x3 2,320,000 n=2 0.7971939 1,849,490
1/1/x4 2,240,000 n=3 0.7117802 1,594,388
1/1/x5 2,160,000 n=4 0.6355181 1,372,719
1/1/x6 2,080,000 n=5 0.5674269 1,180,248
Totals 13,139,702

4. Joshua Bank (a NON-VAT registered entity) purchased an intangible asset from Tristan Co. for
$590,000 when the exchange rate was ₱1: $.0204. A 5% discount was available on the purchase.
Non-transferrable taxes paid amounted to 20% of the invoice price, net of the discount. A 12%
VAT was also paid based on the amount that is gross of the non-transferrable taxes. Installation
and testing costs amounted to ₱40,000. How much will be recorded as intangible asset?
a. 36,976,059
b. 33,010,598
c. 33,050,589
d. 36,967,059

D [($590,000 ÷ $.0204) x 95% x 120% x 112%] + 40,000 = 36,967,059

The 12% VAT is capitalized because the entity is a NON-VAT registered entity.
Page |6

5. Chair Leader Co. purchased a patent on January 1, 20x3 for ₱120,000. The patent had a
remaining useful life of 10 years at that date. In January of 20x4, Chair incurred ₱54,000 in
acquiring another patent that clearly extended the existing patent’s life to 12/31/15. What amount
of amortization expense would Chair record in 20x4?
a. 12,000
b. 13,500
c. 14,500
d. 18,000

B [(120,000 x 9/10) + 54,000] ÷ 12 = 13,500

6. During 20x1, King Co. incurred the following costs:


• Testing in search for process alternatives 720,000
• Costs of testing prototype and design modifications 500,000
• Modification of the formulation of a process 1,220,000
• Research and development services performed by Queen Corp. for King 650,000

In King's 20x1 statement of profit or loss, research and development expense should be
a. 1,220,000.
b. 1,870,000.
c. 2,590,000.
d. 3,090,000.

D
Solution:
Testing in search for process alternatives 720,000
Costs of testing prototype and design modifications 500,000
Modification of the formulation of a process 1,220,000
Research and development services performed by Beck Corp. for Ming
650,000
Total R&D expense 3,090,000

7. In 20x4, Edwards Corporation incurred research and development costs as follows:


Materials and equipment 150,000
Personnel 180,000
Indirect costs 225,000
555,000

These costs relate to a product that will be marketed in 20x5. It is estimated that these costs will
be recouped by December 31, 20x7. The equipment has no alternative future use. What is the
amount of research and development costs that should be expensed in 20x4?
a. 0.
b. 330,000.
c. 405,000.
d. 555,000.

D (150,000 + 180,000 + 225,000) = 555,000


Page |7

8. Tyson Co. incurred the following costs during 20x1:


• Modification to the formulation of a chemical product 360,000
• Trouble-shooting in connection with breakdowns during commercial
production 450,000
• Costs of testing prototype and design modifications 600,000
• Seasonal or other periodic design changes to existing products 555,000
• Laboratory research aimed at discovery of new technology 675,000

In its statement of profit or loss for the year ended December 31, 20x1, Tyson should report
research and development expense of
a. 1,635,000.
b. 2,085,000.
c. 1,275,000.
d. 1,035,000.

A
Solution:
Modification to the formulation of a chemical product 360,000
Costs of testing prototype and design modifications 600,000
Laboratory research aimed at discovery of new technology 675,000
Total R&D expense 1,635,000

9. On June 30, 20x4, Dax, Inc. exchanged 6,000 shares of Trane Corp. ₱30 par value ordinary shares
for a patent owned by Gore Co. The Trane stock was acquired in 20x4 at a cost of ₱160,000. At
the exchange date, Trane ordinary shares had a fair value of ₱45 per share, and the patent had a
carrying amount of ₱320,000 in Gore's books. Dax should record the patent at
a. 160,000.
b. 180,000.
c. 270,000.
d. 320,000.

C (6,000 x 45) = 270,000

10. Ely Co. bought a patent from Backo Corp. on January 1, 20x4, for ₱180,000. An independent
consultant retained by Ely estimated that the remaining useful life is 30 years. Its unamortized
cost on Backo's accounting records was ₱90,000; the patent had been amortized for 5 years by
Backo. How much should be amortized for the year ended December 31, 20x4?
a. 0.
b. 3,000.
c. 6,000.
d. 12,000.

D (180,000 ÷ 15 years) = 12,000

“There is no greater education than one that is self-driven.” - Neil deGrasse Tyson
- END -
Page |1

Chapter 23
Impairment of Assets

1. Under PAS 36 Impairment of Assets, which of the following statements best describes 'value in
use'?
a. The present value of estimated future cash flows expected to arise from the continuing use of
an asset and from its ultimate disposal.
b. The amount of cash or cash equivalents that could currently be obtained by selling an asset
in an orderly disposal.
c. The net amount which an entity expects to obtain for an asset at the end of its useful life.
d. The amount at which an asset could be exchanged between knowledgeable, willing parties
in an arm's length transaction.

2. Under PAS 36 Impairment of Assets, which of the following terms best describes the higher of an
asset's fair value less costs of disposal and its value in use?
a. Recoverable amount
b. Revalued amount
c. Depreciable amount
d. Carrying amount

3. Under PAS 36 Impairment of assets, which of the following statements best describes the term
'impairment loss'?
a. The removal of an asset from an entity's statement of financial position
b. The amount by which the carrying amount of an asset exceeds its recoverable amount
c. The systematic allocation of an asset's cost less residual value over its useful life
d. The amount by which the recoverable amount of an asset exceeds its carrying amount

4. According to PAS 36 Impairment of Assets, which of the following terms is defined as: "The
smallest identifiable group of assets that generates cash inflows that are largely independent of
the cash inflows from other assets"?
a. Non-current assets
b. A cash-operating unit
c. An operating segment
d. A cash-generating unit

5. PAS 36 Impairment of assets should be applied in accounting for the impairment of which of the
following types of asset?
a. Assets arising from construction contracts
b. Non-current assets held for sale
c. Investment properties measured at fair value
d. Non-current assets measured at cost

6. According to PAS 36 Impairment of assets, which of the following are relevant in determining a
non-current asset's 'value in use'?
Page |2

I. The expected future cash flows from the asset


II. The carrying amount of the asset
III. The future annual depreciation expense in respect of the asset
IV. The time value of money

a. I, II, III c. I, IV
b. II, III, IV d. I, II, IV

7. An entity is considering whether to apply an impairment test to an individual asset or to the


cash-generating unit to which that asset belongs. Are the following statements true or false,
according to PAS 36 Impairment of assets?

1) If the individual asset does not generate cash inflows that are largely independent of those
from other assets, then the cash-generating unit should be identified.
2) If the individual asset generates an insignificant proportion of the cash inflows of the entity
as a whole, then the cash-generating unit should not be identified.

Statement (1) Statement (2)


a. False False
b. False True
c. True False
d. True True

8. The Naylor Company has determined that it needs to recognize an impairment loss on each of
two non-current assets; plant and land. The relevant amounts are as follows:
Plant Land
Original cost ₱700,000 ₱1,400,000
Previous revaluations Nil ₱ 450,000
Existing carrying amount ₱700,000 ₱1,850,000
Impairment loss to be recognized in year ₱200,000 ₱ 300,000

According to PAS 36 Impairment of Assets, how should each of the impairment losses be recognized?
Plant Land
a. In profit or loss In profit or loss
b. In profit or loss In other comprehensive income
c. In other comprehensive income In profit or loss
d. In other comprehensive income In other comprehensive income

9. On 1 January 20X2 The Prosper Company acquired a non-current asset with an estimated useful
life of 8 years for ₱320,000. Non-current assets are accounted for under the cost model and
depreciation is charged by the straight-line method.

On 1 January 20X7 an impairment review identified an impairment loss of ₱10,000 and the
remaining useful life was revised to four years. Are the following statements true or false,
according to PAS 36 Impairment of Assets?
Page |3

1) Future depreciation expenses should be measured by reference to the carrying amount after
deducting the impairment loss.
2) Future depreciation expenses should be measured by reference to the new estimate of the
remaining useful life.

Statement (1) Statement (2)


a. False False
b. False True
c. True False
d. True True

10. In testing a cash generating unit (CGU) for impairment the, bottom-up test means that
a. goodwill can be allocated to the CGU and an impairment loss has occurred if the recoverable
amount of the CGU is less than the carrying amount, including the allocated goodwill.
b. goodwill can be allocated to the CGU’s and an impairment loss occurred if the recoverable
amount of the CGU is less than its carrying amount, excluding the allocated goodwill
c. goodwill can be allocated to the CGU and an impairment loss has occurred if the recoverable
amount of the CGU is more than the carrying amount, including the allocated goodwill.
d. goodwill can be allocated to the CGU and an impairment loss has occurred if the recoverable
amount of the CGU is more than its carrying amount, excluding the allocated goodwill.

“Again, I tell you that if two of you on earth agree about anything you ask for, it
will be done for you by my Father in heaven. For where two or three come together
in my name, there am I with them.”
(Matthew 18:19-20)

- END -
Page |4

1. On December 31, 20x1, MASSIVE HEAVY Co. identified that its building with a carrying
amount of ₱2,400,000 has been impaired. In estimating the recoverable amount, MASSIVE has
determined that the fair value less costs of disposal of the asset is ₱1,600,000.

In estimating the value in use, MASSIVE determined the following:


Future cash in Future cash out
Year flows flows
20x1 1,200,000 400,000
20x2 1,120,000 400,000
20x3 1,040,000 320,000

Additional information:
• Each year’s estimated future cash flows include ₱40,000 representing cash outflows from future
restructuring not yet committed and ₱20,000 representing cash outflows on planned
improvement and enhancement of the asset.
• Not included in the estimated future cash flows are costs of day-to-day servicing of the asset
amounting to ₱8,000 per year.
• The discount rate is 10%.

How much is the impairment loss?


a. 407,424 b. 456,773 c. 365,472 d. 412,365

A Solution:
The adjusted future cash outflows are computed as follows:
Future costs not yet
Unadjusted cash Costs of day-to-day Adjusted cash
Year committed & Costs of
outflows servicing out flows
improvement
(a) (b) = (40K + 20K) (c) (d) = (a) - (b) + (c)
20x1 400,000 60,000 8,000 348,000
20x2 400,000 60,000 8,000 348,000
20x3 320,000 60,000 8,000 268,000

The future net cash flows are computed as follows:


Year Cash in flows Adjusted cash out flows Net cash flows
(a) (b) (c) = (a) - (b)
20x1 1,200,000 348,000 852,000
20x2 1,120,000 348,000 772,000
20x3 1,040,000 268,000 772,000

The value in use is computed as follows:


Year Net cash flows PV of ₱1 factors Present value
20x1 852,000 PV of ₱1 @10%, n=1 0.909091 774,544
20x2 772,000 PV of ₱1 @10%, n=2 0.826446 638,016
20x3 772,000 PV of ₱1 @10%, n=3 0.751315 580,016
1,992,576

The recoverable amount is determined as follows:


Fair value less costs of disposal 1,600,000
Value in use 1,992,576
Recoverable amount (higher) 1,992,576
Page |5

Impairment loss is computed as follows:


Recoverable amount 1,992,576
Carrying amount ( 2,400,000)
Impairment loss ( 407,424)

2. On January 1, 20x1, RIGHTEOUS MORAL Co. acquired a piece of equipment for ₱2,000,000. The
equipment is depreciated using the straight line method over an estimated useful life of 10 years
and residual value of ₱200,000.

On January 1, 20x6, RIGHTEOUS Co. determined that the equipment is impaired. Fair value less
costs of disposal is ₱560,000. Projected future net cash flows from revenues produced by the
equipment is ₱200,000 annually. The revised estimated useful life is 4 years and the new estimated
residual value is ₱40,000. The appropriate discount rate is 10%. How much is the depreciation
expense in 20x6?
a. 156,732 b. 155,324 c. 155,132 d. 154,324

B
Solution:
The value in use is computed as follows:
PV factors @ 10%,
Future cash flows n=4 Present value
Annual cash flows from revenues 200,000 3.169865 633,972
Residual value 40,000 0.683013 27,320
661,292

New carrying amount on Jan. 1, 20x6


(Value in use - higher) ₱661,292
Revised residual value ( 40,000)
New depreciable amount 621,292
Divide by: Revised useful life 4
Depreciation expense – 20x6 ₱155,323

3. Information on LISTLESS WEAK Co.’s impaired building is shown below:


Carrying amount 3,200,000
Revaluation surplus 320,000
Fair value less costs of disposal 2,800,000
Value in use 2,720,000

How much is the impairment loss?


a. 80,000 b. 400,000 c. 320,000 d. 0

A
Solution:
Recoverable amount (FVLCS – higher) ₱2,800,000
Carrying amount ( 3,200,000)
Excess over carrying amount ( 400,000)
Offset to revaluation surplus 320,000
Excess charged as Impairment loss (₱ 80,000)
Page |6

4. INSUPERABLE UNSURPASSABLE Co. determined that its trademark is impaired.


INSUPERABLE cannot estimate reliably the trademark’s fair value less costs of disposal.
However, the following information has been determined:

Carrying amount ₱520,000


Annual future cash flows from the trademark 40,000
Discount rate 10%

How much is the impairment loss?


a. 0 b. 80,000 c. 120,000 d. 400,000

C
Solution:
The recoverable amount or value in use is determined as follows:
Annual future cash flows from the trademark 40,000
Divide by: Discount rate 10%
Present value of indefinite cash flows (value in use) 400,000

Recoverable amount (value in use) ₱400,000


Carrying amount ( 520,000)
Impairment loss ₱120,000

5. One of MIME IMMITATE Co.’s machines has been impaired. Repairs and maintenance costs on
the machine have been increasing over the past years making the machine a bottleneck in
MIME’s production. At year-end, management made a decision to sell the machine very soon.
Information on the machine is shown below:
Carrying amount ₱400,000
Fair value less costs of disposal 200,000
Value in use 240,000

How much is the impairment loss?


a. 0 b. 160,000 c. 40,000 d. 200,000

D ₱200,000 (200,000 – 400,000).

Use the following information for the next two questions:


INSTIGATE PROVOKE Co. determined that one of its cash-generating units is impaired.
Information on the assets of the CGU is shown below:

Assets Carrying amount


Inventory 800,000
Investment property (at cost model) 1,600,000
Building 2,400,000
Goodwill 1,200,000
6,000,000
Page |7

It was estimated that the value in use of the CGU is ₱3,600,000 and its fair value less costs of disposal
is ₱2,400,000.

6. How much is the impairment loss?


a. 4,200,000 b. 3,200,000 c. 2,400,000 d. 2,000,000

C
Solution:
Impairment loss is computed as follows:
Recoverable amount (value in use – higher) ₱3,600,000
Carrying amount (6,000,000)
Impairment loss (₱2,400,000)

7. How much is the carrying amount of the building after the impairment testing?
a. 1,680,000 b. 1,120,000 c. 1,860,000 d. 2,040,000

A
Solution:
Allocation of Impairment
Assets Carrying amount Fraction Loss
Inventory N/A N/A -
Investment property 1,600,000 1,600/4,000 480,000
Building 2,400,000 2,400/4,000 720,000
4,000,000 4,000/4,000 1,200,000

2,400,000 – 720,000 allocation of impairment loss = 1,680,000

8. One of the machines of SKEWER PIERCE Co. has suffered physical damage but is still working,
although not as well as before it was damaged. The machine does not generate independent cash
inflows. The smallest identifiable group of assets that includes the machine and generates cash
inflows that are largely independent of the cash inflows from other assets is the production line
to which the machine belongs. Information on the machine and the production line is shown
below:

Carrying amount of machine ₱800,000


Fair value less costs of disposal of machine 600,000
Carrying amount of production line 32,000,000
Recoverable amount of production line 36,000,000

The budgets/forecasts approved by management reflect no commitment of management to replace


the machine. How much is the impairment loss?
a. 4,000,000 b. 200,000 c. 3,800,000 d. 0

D 0 - The production line is not impaired (i.e., the recoverable amount of ₱36M is higher than the
carrying amount of ₱32M).
Page |8

Use the following information for the next two questions:


On January 1, 20x1, FALLACIOUS MISLEADING Co. acquired a building for ₱4,000,000. The asset
is depreciated using the straight line method over an estimated useful life of 10 years.

On January 1, 20x6, the building was estimated to have a recoverable amount of ₱1,600,000.
Consequently, impairment loss was recognized on that date. There was no change in the estimated
useful life.

On January 1, 20x9, the building was estimated to have a new recoverable amount of ₱2,400,000 and
a remaining useful life of 3 years. The building is measured under the revaluation model.

9. How much of the impairment reversal is recognized in profit or loss?


a. 160,000 b. 1,760,000 c. 1,600,000 d. 0

A
Solution:
The carrying amount of the building on January 1, 20x9 is computed as follows:
Recoverable amount on January 1, 20x6 ₱1,600,000
Depreciation from 20x6 to 20x8 (1,600,000 x 3/5) ( 960,000)
Carrying amount on January 1, 20x9 ₱ 640,000

The carrying amount of the building on January 1, 20x9 had no impairment loss been recognized
previously is computed as follows:
Historical cost ₱4,000,000
Depreciation from 20x1 to 20x8 (4M x 8/10) ( 3,200,000)
Carrying amount on January 1, 20x9 (assuming no IL) ₱ 800,000

Recoverable amount - Jan.


2.4M 1, 20x9

CA had no IL recognized previously -


.8M Jan. 1, 20x9

.64M CA - Jan. 1, 20x9

Impairment loss recognized in profit or loss = (.8M - .64M) = ₱160,000

10. How much of the impairment reversal is recognized in equity?


a. 160,000 b. 1,760,000 c. 1,600,000 d. 0

C Impairment loss recognized in other comprehensive income (equity) = (2.4M - .8M) = ₱1,600,000

“Praise be to the God and Father of our Lord Jesus Christ, the Father of compassion
and the God of all comfort, who comforts us in all our troubles, so that we can
comfort those in any trouble with the comfort we ourselves have received from
God.” (2 Corinthians 1:3-4)
- END -
Page |1

1. A company’s trial balance totals were:


Debit …………………….₱387,642
Credit ……………………₱379,511

A suspense account was opened for the difference. Which of the following errors would have the
effect of reducing the difference when corrected?
a. The petty cash balance of ₱500 has been omitted from the trial balance
b. ₱4,000 received for rent of part of the office has been correctly recorded in the cash book and
debited to Rent expense account
c. No entry has been made in the records for a cash sale of ₱2,500
d. ₱3,000 paid for repairs to plant has been debited to the plant asset account.

B – the correction is credit rent expense account (which will decrease the overstated total debits) and
a credit to rent income/ unearned rent (which will increase the understated total credits).

Choice A is incorrect. If the omitted PCF balance is corrected, the total debits will be increased;
thereby increasing the difference between the total debits and credits.
Choice C is incorrect. The correction, which is debit cash and credit sales, will not affect the total
debits and credits.
Choice D is incorrect. The correction, which is a reclassification from plant asset account – a debit
account, to repairs expanse – also a debit account, will not affect the difference between the total
debits and credits.

2. A company paid its property taxes on October 1 for the period October 1, year 1 to September
30, year 2. When the payment was made the company debited property taxes expense and
credited cash for ₱8,000. The adjusting entry at December 31, year 1 would include which of the
following:
a. debit prepaid property taxes, ₱6,000.
b. credit prepaid property taxes, ₱6,000.
c. credit property tax expense, ₱2,000.
d. debit property tax expense, ₱6,000.

3. If during an accounting period an expense item has been incurred and consumed but not yet
paid for or recorded, the end-of-period adjusting entry would involve
a. a liability account and an asset account.
b. an asset or contra asset account and an expense account.
c. a liability account and an expense account.
d. a receivable account and a revenue account.

4. A trial balance
a. proves that debits and credits are equal in the ledger.
b. supplies a listing of open accounts and their balances that are used in preparing financial
statements.
c. is normally prepared three times in the accounting cycle.
d. all of these.
Page |2

5. It is a formal record where transactions are initially recorded.


a. Journal entries c. Master file
b. Ledger d. Journal

6. It is the basic storage of information in accounting.


a. Journal entry c. Debit or Credit
b. T-account d. Account

7. Which of the following is a recordable event or item?


a. Changes in managerial policy
b. The value of human resources
c. Changes in personnel
d. None of these

8. Errors revealed by a trial balance are


a. those errors resulting from transposition but not transplacement.
b. those errors resulting from either transposition or transplacement.
c. transplacement and transposition errors on both sides of a journal entry.
d. those errors which that have caused the total debits and total credits to be unequal.

9. Adjusting entries are necessary to


1. obtain a proper matching of revenue and expense.
2. achieve an accurate statement of assets and equities.
3. adjust assets and liabilities to their fair value.
a. 1 c. 3
b. 2 d. 1 and 2

10. Factors that shape an accounting information system include the


a. nature of the business c. volume of data to be handled.
b. size of the firm d. all of these.

11. An accrued revenue can best be described as an amount


a. collected and currently matched with expenses.
b. collected and not currently matched with expenses.
c. not collected and currently matched with expenses.
d. not collected and not currently matched with expenses.

12. At the end of the current year, the prepaid insurance account showed a debit the balance of
₱5,000; the balance at the beginning of the year was ₱6,000, and during the year the insurance
premiums paid amounted to ₱8,000. Assuming insurance premium payments are initially
entered in the prepaid insurance account ,the adjusting entry at the end of the year would
include:
a. debit prepaid insurance ₱9,000
b. credit prepaid insurance ₱1,000
c. debit insurance expense ₱7,000
Page |3

d. debit insurance expense ₱9,000

13. When an item of revenue is collected and recorded in advance, it is normally called a(n)
___________ revenue.
a. accrued c. unearned
b. prepaid d. cash

14. The information below was taken from the bank transfer schedule prepared during the audit of
Fox Co.’s financial statements for the year ended December 31, 2001. Assume all checks are
dated and issued on December 30, 2001.
Bank Accounts Disbursement date Receipt date
Check no. From To Per books Per bank Per books Per bank
101 National Federal Dec. 30 Jan. 4 Dec. 30 Jan. 3
202 County State Jan. 3 Jan. 2 Dec. 30 Dec. 31
303 Federal American Dec. 31 Jan. 3 Jan. 2 Jan. 2
404 State Republic Jan. 2 Jan. 2 Jan. 2 Dec. 31

Which of the following checks might indicate kiting?


a. #101 and #303.
b. #202 and #404
c. #101 and #404
d. #202 and #303

15. Trask Corporation's checkbook balance on December 31, 2001 was ₱8,000. In addition, Trask
held the following items in its safe on December 31:

Check payable to Trask Corporation, dated January 2, 2002, not


included in December 31 checkbook balance ₱2,000
Check payable to Trask Corporation, deposited December 20,
and included in December 31 checkbook balance, but
returned by bank on December 30, stamped "NSF." The
check was redeposited Jan. 2, 2002, and cleared Jan. 7 400
Post-dated checks 150
Check drawn on Trask Corporation's account, payable to a
vendor, dated and recorded December 31, but not mailed until
1,000
January 15, 2002

The proper amount to be shown as cash on Trask's balance sheet at December 31, 2001, is
a. ₱7,600.
b. ₱8,000.
c. ₱8,600.
d. ₱9,750.

C (8,000 – 400 + 1,000) = 8,600

16. Bank statements provide information about all of the following except
Page |4

a. checks cleared during the period.


b. NSF checks.
c. bank charges for the period.
d. errors made by the company.

17. Which of the following items would be added to the book balance on a bank reconciliation?
a. Outstanding checks
b. A check written for ₱63 entered as ₱36 in the accounting records
c. Interest paid by the bank
d. Deposits in transit

18. In preparing a bank reconciliation, interest paid by the bank on the account is
a. added to the bank balance.
b. subtracted from the bank balance.
c. added to the book balance.
d. subtracted from the book balance.

19. In preparing a monthly bank reconciliation, which of the following items would be added to the
balance reported on the bank statement to arrive at the correct cash balance?
a. Outstanding checks
b. Bank service charge
c. Deposits in transit
d. A customer's note collected by the bank on behalf of the depositor

20. Bank reconciliations are normally prepared on a monthly basis to identify adjustments needed in
the depositor's records and to identify bank errors. Adjustments should be recorded for
a. bank errors, outstanding checks, and deposits in transit.
b. all items except bank errors, outstanding checks, and deposits in transit.
c. book errors, bank errors, deposits in transit, and outstanding checks.
d. outstanding checks and deposits in transit.

21. In preparing its bank reconciliation for the month of February, James Company has made
available the following information:
Balance per bank statement, February 28 ₱18,025
Deposit in transit, February 28 3,125
Outstanding checks, February 28 2,875
Check erroneously deducted by bank from James' 125
account, February 10
Bank service charges for February 25

What is the corrected cash balance at February 28?


a. ₱18,125
b. ₱18,150
c. ₱18,275
d. ₱18,400
Page |5

Solution: (18,025 + 3,125 – 2,875 + 125) = 18,400

Use the following information for the next three questions:


The accounting records and bank statement of Entity A show the following information:

SUBSIDIARY LEDGER
CASH IN BANK - BPI CURRENT ACCOUNT
Date Description Debit Credit Balance
6/1 Bal. forwarded 881,000
6/11 Check #1113 130,800 750,200
6/15 Check #1114 220,000 530,200
6/16 Deposit 295,800 826,000
6/22 Deposit 670,000 1,496,000
6/24 Check #1115 80,000 1,416,000
6/28 Check #1116 380,000 1,036,000
6/29 Deposit 160,000 1,196,000

METROBANK
BANK STATEMENT - ENTITY A

Description Debit Credit Balance


Date
6/1 Bal. forwarded 881,000
6/10 Deposit 350,000 1,231,000
6/15 Payment 2,000 1,229,000
6/15 Check #1114 220,000 1,009,000
6/16 Deposit 295,800 1,304,800
6/20 Payment 50,000 1,254,800
6/22 Deposit 670,000 1,924,800
6/24 Check #1115 80,000 1,844,800
6/26 Check #1113 130,800 1,714,000
6/28 Deposit 410,000 2,124,000

Additional information:
• The payments of ₱2,000 and ₱50,000 shown on the bank statement pertain to the cost of
checkbook requested from the bank and the monthly amortization of a bank loan, respectively.
The loan payment includes payment for interest of ₱8,000.
• Deposits shown on the bank statement but not on the cash ledger represent collections of
accounts receivable.

22. How much is the deposit in transit?


a. 160,000
b. 102,000
c. 52,000
Page |6

d. 380,000

23. How much is the credit memo?


a. 52,000
b. 160,000
c. 760,000
d. 380,000

24. How much is the adjusted cash balance?


a. 1,940,000
b. 1,760,000
c. 1,380,000
d. 1,904,000

Solutions to #s 22 to 24:

SUBSIDIARY LEDGER
CASH IN BANK - BPI CURRENT ACCOUNT
Date Description Debit Credit Balance
6/1 Bal. forwarded 881,000
6/11 Check #1113 130,800 750,200
6/15 Check #1114 220,000 530,200
6/16 Deposit 295,800 826,000
6/22 Deposit 670,000 1,496,000
6/24 Check #1115 80,000 1,416,000
6/28 Check #1116 380,000 1,036,000
6/29 Deposit 160,000 1,196,000

METROBANK
BANK STATEMENT - ENTITY A

Description Debit Credit Balance


Date
6/1 Bal. forwarded 881,000
6/10 Deposit 350,000 1,231,000
6/15 Payment 2,000 1,229,000
6/15 Check #1114 220,000 1,009,000
6/16 Deposit 295,800 1,304,800
6/20 Payment 50,000 1,254,800
6/22 Deposit 670,000 1,924,800
6/24 Check #1115 80,000 1,844,800
6/26 Check #1113 130,800 1,714,000
6/28 Deposit 410,000 2,124,000
Page |7

Bank reconciling items:


• Deposit in transit: ₱160,000 deposit on 6/29.
• Outstanding check: ₱380,000 Check #1116.

Book reconciling items:


• Credit memos: ₱350,000 deposit on 6/10 + ₱410,000 deposit on 6/28 = ₱760,000.
• Debit memos: ₱2,000 payment on 6/15 + ₱50,000 payment on 6/20 = ₱52,000.

Bal. per books, end. 1,196,000 Bal. per bank, end. 2,124,000
Add: CM 760,000 Add: DIT 160,000
Less: DM (52,000) Less: OC (380,000)
Add/Less: Book errors - Add/Less: Bank errors -
Adjusted balance 1,904,000 Adjusted balance 1,904,000

Use the following information for the next three questions:


Information on ABC Co. is shown below:
30-Jul Aug. 31
Book balance 132,200 180,000
Book debits 60,000
Book credits ?
Bank balance 100,600 169,000
Bank debits 20,600
Bank credits ?
Notes collected by bank 10,000 35,000
Debit memos 7,800 8,900
Understatement of book receipts - 2,800
Deposit in transit 45,000 43,800
Outstanding checks 11,200 3,900

25. How much is the adjusted receipts in August?


a. 78,800
b. 80,700
c. 88,700
d. 87,800

26. How much is the adjusted disbursements in August?


a. 13,300
b. 17,800
c. 16,200
d. 14,300

27. How much is the adjusted balance of cash on August?


a. 136,200
b. 134,400
Page |8

c. 132,600
d. 208,900

Solutions to #s 25 to 27:
30-Jul Receipts Disbursements 31-Aug
Per books 132,200 60,000 12,200 180,000
ADD: CM
July 10,000 (10,000)
August 35,000 35,000
LESS: DM
July (7,800) (7,800)
August 8,900 (8,900)
Book errors:
August 2,800 2,800
Adjusted bal. 134,400 87,800 13,300 208,900

30-Jul Receipts Disbursements 31-Aug


Per bank 100,600 89,000 20,600 169,000
ADD: DIT
July 45,000 (45,000)
August 43,800 43,800
LESS: OC
July (11,200) (11,200)
August 3,900 (3,900)

Adjusted bal. 134,400 87,800 13,300 208,900

28. Under the allowance method of recognizing bad debts on trade accounts receivable, the effect of
writing off an account to an entity's working capital is
a. increase
b. decrease
c. either a or b depending on the current level of the entity's working capital
d. no effect

29. JG Company had an accounts receivable balance of ₱40,000 on December 31, 2001, and ₱65,000
on December 31, 2002. The company wrote off ₱10,000 of accounts receivable during 2002, and
collected ₱2,000 on an account written off in 2000. Sales for the year 2002 totaled ₱520,000. All
sales were on account. The amount collected from customers on accounts receivable during 2002
was
a. ₱487,000. c. ₱510,000.
b. ₱485,000. d. ₱495,000.

Solution:
Accounts receivable
beg. 40,000
Sales on account 520,000 487,000 Collections, including recoveries
Recoveries 2,000 10,000 Write-offs
Page |9

65,000 end.

30. RGI Company had an accounts receivable balance of ₱45,000 on December 31, 2001, and ₱60,000
on December 31, 2002. The company wrote off ₱12,000 of accounts receivable during 2002, and
collected ₱2,500 on an account written off in 2000. Sales for the year 2002 totaled ₱550,000. All
sales were on account. The total collections from customers in 2002 were
a. ₱535,000. c. ₱538,000.
b. ₱523,000. d. ₱525,500.

Solution:
Accounts receivable
beg. 45,000
Sales on account 550,000 525,500 Collections, including recoveries
Recoveries 2,500 12,000 Write-offs
60,000 end.

31. At the close of its first year of operations, December 31, 2004, Linn Company had accounts
receivable of ₱490,000, after deducting the related allowance for doubtful accounts. During 2004,
the company had charges to bad debt expense of ₱90,000 and wrote off, as uncollectible,
accounts receivable of ₱40,000. What should the company report on its balance sheet at
December 31, 2004 as accounts receivable before the allowance for doubtful accounts?
a. ₱620,000 c. ₱440,000
b. ₱540,000 d. ₱360,000

Solution:
Allowance for doubtful accounts
- beg.
Write-offs 40,000 90,000 Bad debts expense
- Recoveries
end. 50,000

490,000 + 50,000 = 540,000

32. Before year-end adjusting entries, Bass Company's account balances at December 31, 2004 for
accounts receivable and the related allowance for uncollectible accounts were ₱700,000 and
₱45,000, respectively. An aging of accounts receivable indicated that ₱62,500 of the December 31
receivables are expected to be uncollectible. The net realizable value of accounts receivable after
adjustment is
a. ₱682,500. c. ₱592,500.
b. ₱637,500. d. ₱655,000.

Solution: (700,000 – 62,500) = 637,500


P a g e | 10

33. During the year, Jantz Company made an entry to write off a ₱4,000 uncollectible account.
Before this entry was made, the balance in accounts receivable was ₱80,000 and the balance in
the allowance account was ₱4,500. The net realizable value of accounts receivable after the write-
off entry was
a. ₱80,000. c. ₱71,500.
b. ₱79,500. d. ₱75,500.

Solution: [(80,000 – 4,000) - (4,500 – 4,000)] = (76,000 – 500) = 75,500

34. The following information is available for Reagan Company:


Allowance for doubtful accounts at December 31, 2003 ₱ 8,000
Credit sales during 2004 400,000
Accounts receivable deemed worthless and written
off during 2004 9,000

It has been determined that an allowance for doubtful accounts of ₱9,500 is needed at December 31,
2004. What amount should Reagan record as "bad debt expense" for the year ended December 31,
2004?
a. ₱8,500 c. ₱10,500
b. ₱9,500 d. ₱17,500

Solution: (9,500 + 9,000 – 8,000) = 10,500

Use the following information for the next two questions:


A trial balance before adjustments included the following:
Debit Credit
Sales ₱425,000
Sales returns and allowance ₱14,000
Accounts receivable 53,000
Allowance for doubtful accounts 760

35. If the estimate of uncollectibles is made by taking 1% of net sales, the amount of the adjustment
is
a. ₱3,350. c. ₱4,250.
b. ₱4,110. d. ₱4,870.

Solution: (425,000 – 14,000) * 1% = 4,110

36. If the estimate of uncollectibles is made by taking 10% of gross account receivables, the amount
of the adjustment is
a. ₱4,540. c. ₱5,224.
b. ₱5,300. d. ₱6,060.

Solution: (53,000 x 10%) – 760 = 4,540

37. For the month of December, the records of Balin Corporation show the following information:
P a g e | 11

Cash received on accounts receivable ₱ 70,000


Cash sales 60,000
Accounts Receivable, December 1 160,000
Accounts Receivable, December 31 148,000
Accounts Receivable written off as uncollectible 2,000

The corporation uses the direct write-off method in accounting for uncollectible accounts receivable.

What are the gross sales for the month of December?


a. ₱118,000 c. ₱130,000
b. ₱120,000 d. ₱144,000

Solution: (148,000 + 70,000 + 60,000 + 2,000 – 160,000) = 120,000

38. An analysis and aging of accounts receivable of the Lucille Company at December 31, 2002,
showed the following:

Accounts Receivable .................................. ₱840,000


Allowance for Doubtful Accounts
(before adjustment) ................................ 36,000 (cr)
Accounts estimated to be uncollectible ............... 76,800

Compute for the net realizable value of the accounts receivable of Lucille Company at December 31,
2002.
a. ₱804,000 c. ₱763,200
b. ₱799,200 d. ₱727,200

Solution: [840,000 – (36,000 + 76,800)] = 727,200

39. Spongebob Squarepants lent ₱2,000 to Squidward for one year at 10% interest, all due at
maturity. He insisted the terms of the transaction be formalized in promissory note. In this
situation
a. the maturity value of the note is ₱2,000.
b. Spongebob Squarepants is considered the maker of the note and records the note as an asset
in his accounting records.
c. Spongebob Squarepants is considered the maker of the note and records the note as a
liability in his accounting records.
d. Squidward is considered the maker of the note and records the note as a liability in his
accounting records.

40. Scott Company received a one-year non-interest-bearing note receivable. When the note
receivable was recorded, which of the following were debited or credited?
Interest Receivable Discount on Note Receivable
a. Yes Yes
b. Yes No
c. No Yes
d. No No
P a g e | 12

41. The periodic cash flows from a debt instrument may be computed by
a. multiplying the future cash flows from the note by an appropriate present value factor.
b. dividing the initial carrying amount by an appropriate present value factor.
c. adding together the periodic interest income and the amortization.
d. dividing the face amount by the life of the instrument.

42. On May 1, 2004 a company purchased a new machine which it does not have to pay for until
May 1, 2006. The total payment on May 1, 2006 will include both principal and interest.
Assuming interest at a 10% rate, the cost of the machine would be the total payment multiplied
by what time value of money factor?
a. Future value of annuity of 1
b. Future value of 1
c. Present value of annuity of 1
d. Present value of 1

43. What is the effective interest rate of a bond or other debt instrument measured at amortized
cost?
a. The stated coupon rate of the debt instrument.
b. The interest rate currently charged by the entity or by others for similar debt instruments
(i.e., similar remaining maturity, cash flow pattern, currency, credit risk, collateral, and
interest basis).
c. The interest rate that exactly discounts estimated future cash payments or receipts through
the expected life of the debt instrument or, when appropriate, a shorter period to the net
carrying amount of the instrument.
d. The basic, risk-free interest rate that is derived from observable government bond prices.

44. Which of the following is true regarding non-interest bearing note receivables?
a. they are always discounted to their present value on initial recognition
b. they include a specified principal amount but an unspecified interest amount
c. they include a specified principal and specified interest
d. they cause no interest income to be recognized over their term
e. they include an unspecified principal and an unspecified interest

45. A company received two one-year notes in payment for merchandise sold. One note has a face
amount of ₱6,000 and was interest-bearing at an annual rate of 18 percent. The other note has a
face amount of ₱7,080 and was non-interest-bearing (its implied interest rate was 18 percent)
a. The total amount of cash ultimately to be received will be more for the interest-bearing note.
b. Both notes will cause the same total interest to be recognized.
c. The amount of interest revenue which should be recognized is more for the interest-bearing
note.
d. The amount which should be credited to sales revenue is more for the noninterest-bearing
note
P a g e | 13

46. Gary Snail Inc., received a 3-year non-interest bearing trade note for ₱50,000 on January 1, 20x1.
The current interest rate at that time was 15% for similar notes. Gary Snail recorded the receipt
of the note as follows:

(Dr) Notes receivable – trade ₱50,000


(Cr) Sales ₱50,000

The effect of this accounting for the notes receivable Gary Snail’s profit for years 20x1, 20x2 and 20x3
and retained earnings at the end of 20x3, respectively, shall to
a. overstate, overstate, understate, no effect
b. overstate, understate, understate, no effect
c. overstate, understate, understate, understate
d. no effect on any of these

47. Which of the following statements regarding interest methods of allocations is not true?
a. The term “interest methods of allocation” refers both to the convention for periodic
reporting and to the several approaches to dealing with changes in estimated future cash
flows.
b. Interest methods of allocation are reporting conventions that use present value techniques in
the absence of a fresh-start measurement to compute changes in the carrying amount of an
asset or liability from one period to the next.
c. Interest methods of allocation are grounded in the notion of current cost.
d. Holding gains and losses are generally excluded from allocation systems.

48. Which of the following is not an objective of using present value in accounting measurements?
a. To capture the value of an asset or a liability in the context of a particular entity.
b. To estimate fair value.
c. To capture the economic difference between sets of future cash flows.
d. To capture the elements that taken together would comprise a market price if one existed.

49. On July 1, 2002, Cornell Corp. received a one-year note with a face value of ₱900,000 and a stated
interest rate of 15 percent in exchange for a machine with a fair value of ₱1,000,000. Compute the
effective interest rate for Cornell Corp.
a. 16.67 percent
b. 15.0 percent
c. 3.5 percent
d. 11.11 percent

Solution:
Using trial and error:
Cash flows PV of 1 @3.5%, n=1 Present value
900,000.00 0.96618357 869,565.22
135,000.00 0.96618357 130,434.78
1,000,000.00
P a g e | 14

50. A 10 percent, ₱3,000, 3-month note receivable discounted at 12 percent for 2 months will result in
net proceeds of
a. ₱3,075.00.
b. ₱3,013.50.
c. ₱3,000.00.
d. ₱3,005.25.

Solution:
MV = 3,000 + (3,000 x 10% x 3/12) = 3,075
D = 3,075 x 12% x 2/12 = 61.50
NP = 3,075 – 61.50 = 3,013.50

“Consider it pure joy, my brothers and sisters, whenever you face trials of
many kinds, because you know that the testing of your faith produces
perseverance.” (James 1:2-3)
- END -
Page |1

NAME: Date:
Professor: Section: Score:

INTERMEDIATE ACCOUNTING 1
SECOND GRADING EXAMINATION

1. Entity A needs guidance in accounting for its inventories. Entity A should refer to which of the
following?
a. PAS 1
b. PAS 2
c. PAS 7
d. PAS 8

2. The debit side of a trial balance totals ₱800 more than the credit side. Which one of the following
errors would fully account for the difference?
a. ₱400 paid for plant maintenance has been correctly entered in the cash book and credited to
the plant asset account.
b. Discount received ₱400 has been debited to discount allowed account.
c. A receipt of ₱800 for commission receivable has been omitted from the records.
d. The petty cash balance of ₱800 has been omitted from the trial balance.

Explanations:
Page |2

3. To reduce in accounting costs, a firm always expenses its routine operating expenditures
immediately and then makes an adjusting entry at the end of the year if needed. For example, it
received ₱1,200 for one year's rent from a tenant on August 1 and immediately recorded ₱1,200
of rent revenue. The rental period begins August 1.The adjusting entry required at December 31
would include
a. cr. unearned rent ₱700 c. cr. rent revenue ₱700
b. dr. rent revenue ₱500 d. dr. unearned rent ₱500

4. Transactions are posted to the


a. book of original entry c. log book
b. book of final entry d. facebook

5. Why are certain costs capitalized when incurred and then depreciated over subsequent
reporting periods?
a. To reduce the income tax liability
b. To aid management in cash-flow analysis
c. To reflect the consumption of economic benefits from the asset
d. To adhere to the accounting constraint of conservatism

6. Which of the following would not be a correct form for an adjusting entry?
a. A debit to a revenue and a credit to a liability
b. A debit to an expense and a credit to a liability
c. A debit to a liability and a credit to a revenue
d. A debit to an asset and a credit to a liability

7. Which of the following is considered when depreciating an asset?


a. The cost of the asset
b. The useful life of the asset
c. The change in the fair value of the asset
d. Both a and b.

8. The information below is from the books of the Seminole Corporation on June 30:

Balance per bank statement ₱11,164


Deposits in transit 1,340
Bank charges not recorded 16
Note collected by bank and not recorded on books 1,120
Outstanding checks 1,100
NSF checks - not recorded on books nor redeposited 160

Assuming no errors were made, how much is the cash balance per books on June 30 before any
reconciliation adjustments?
a. 11,404
b. 10,980
c. 10,460
Page |3

d. 11,440

Solution:
Per books 10,460 (squeeze) Per bank, June 30 11,164 (start)

Credit memo 1,120 Deposits in transit 1,340

Debit memo Outstanding checks (1,100)


(16 + 160) (176)

Adjusted balance 11,404 Adjusted balance 11,404

9. Under the allowance method of recognizing bad debts on trade accounts receivable, the effect of
writing off an account to an entity's current ratio is
a. increase
b. decrease
c. increase if the entity's current ratio is higher than 1 prior to the write-off; decrease if the
entity's current ratio is lower than 1 prior to the write-off
d. no effect

10. On December 31, Central Savings & Loan discounted a 3-month, ₱70,000, non-interest-bearing
note dated October 31, at 12 percent. How much is the proceeds from the discounting?
a. 63,900
b. 48,550
c. 30,380
d. 69,300

Solution:
MV = 70,000 + (70,000 x 0% x 3/12) = 70,000
D = 70,000 x 12% x 1/12 = 700
NP = 70,000 – 700 = 69,300

11. Grant Company accepted a ₱400,000 face value, 6-month, 10 percent note dated May 15 from a
customer. On that same date Grant discounted the note at Eagle National Bank at a 12 percent
discount rate. How much cash should Grant receive from the bank on May 15?
a. ₱400,000
b. ₱396,000
c. ₱394,800
d. ₱387,200

Solution:
MV = 400,000 + (400,000 x 10% x 6/12) = 420,000
D = 420,000 x 12% x 6/12 = 25,200
NP = 420,000 – 25,200 = 394,800

12. Goods in transit that are shipped f.o.b. destination should be


a. included in the inventory of the seller.
b. included in the inventory of the buyer.
c. included in the inventory of the shipping company.
d. none of these.
Page |4

13. Cross Co. accepted delivery of merchandise which it purchased on account. As of December 31,
Cross had recorded the transaction, but did not include the merchandise in its inventory. The
effect of this on its financial statements for December 31 would be
a. net income, current assets, and retained earnings were understated.
b. net income was correct and current assets were understated.
c. net income was understated and current liabilities were overstated.
d. net income was overstated and current assets were understated.

14. All of the following costs should be expensed in the period they are incurred except for
a. manufacturing overhead costs for a product manufactured and sold in the same accounting
period.
b. costs which will not benefit any future period.
c. depreciation of idle manufacturing capacity resulting from an unexpected plant shutdown.
d. storage costs that are necessary in bringing the asset to its intended condition.

15. Which of the following cost flow formulas can be applied by an entity whose inventories that are
purchased last are sold first?
a. LIFO d. b or c
b. FIFO e. None of these
c. Weighted average cost

16. On June 1, 2004, Noll Corp. sold merchandise with a list price of ₱30,000 to Linn on account. Noll
allowed trade discounts of 30% and 20%. Credit terms were 2/15, n/40 and the sale was made
f.o.b. shipping point. Noll prepaid ₱600 of delivery costs for Linn as an accommodation. On June
12, 2004, Noll received from Linn a remittance in full payment amounting to
a. ₱16,464. b. ₱17,052. c. ₱17,064. d. ₱16,794.

C ₱30,000 × .7 × .8 = ₱16,800
(₱16,800 × .98) + 600 = ₱17,064.

17. The following information was derived from the 2004 accounting records of Kelly Co.:
Kelly's Goods
Kelly's Central Warehouse Held by Consignees
Beginning inventory ₱260,000 ₱ 28,000
Purchases 950,000 140,000
Freight-in 20,000
Transportation to consignees 10,000
Freight-out 60,000 16,000
Ending inventory 290,000 40,000
Kelly's 2004 cost of sales was
a. ₱940,000.
b. ₱1,000,000.
c. ₱1,068,000.
d. ₱1,078,000.
Page |5

D ₱260,000 + ₱28,000 + ₱950,000 + ₱140,000 + ₱20,000 + ₱10,000 – ₱290,000 – ₱40,000 =


₱1,078,000.

18. Dial Corp.'s accounts payable at December 31, 2004 totaled ₱800,000 before any necessary year-
end adjustments relating to the following transactions:
• On December 27, 2004, Dial wrote and recorded checks to creditors totaling ₱350,000 causing an
overdraft of ₱100,000 in Dial's bank account at December 31, 2004. The checks were mailed out
on January 10, 2005.
• On December 28, 2004, Dial purchased and received goods for ₱200,000, terms 2/10, n/30. Dial
records purchases and accounts payable at net amounts. The invoice was recorded and paid
January 3, 2005.
• Goods shipped f.o.b. destination on December 20, 2004 from a vendor to Dial were received
January 2, 2005. The invoice cost was ₱65,000.

At December 31, 2004, what amount should Dial report as total accounts payable?
a. ₱1,411,000.
b. ₱1,346,000.
c. ₱1,050,000.
d. ₱1,000,000.

B ₱800,000 + ₱350,000 + ₱196,000 = ₱1,346,000.

19. The balance in Iwig Co.'s accounts payable account at December 31, 2004 was ₱400,000 before
any necessary year-end adjustments relating to the following:
• Goods were in transit to Iwig from a vendor on December 31, 2004. The invoice cost was ₱50,000.
The goods were shipped f.o.b. shipping point on December 29, 2004 and were received on
January 4, 2005.
• Goods shipped f.o.b. destination on December 21, 2004 from a vendor to Iwig were received on
January 6, 2005. The invoice cost was ₱25,000.
• On December 27, 2004, Iwig wrote and recorded checks to creditors totaling ₱30,000 that were
mailed on January 10, 2005.
In Iwig's December 31, 2004 balance sheet, the accounts payable should be
a. ₱430,000
b. ₱450,000.
c. ₱475,000.
d. ₱480,000.

D ₱400,000 + ₱50,000 + ₱30,000 = ₱480,000.

20. Gear Co.'s accounts payable balance at December 31, 2004 was ₱1,100,000 before considering the
following transactions:
• Goods were in transit from a vendor to Gear on December 31, 2004. The invoice price was
₱80,000, and the goods were shipped f.o.b. shipping point on December 29, 2004. The goods
were received on January 4, 2005.
Page |6

• Goods shipped to Gear, f.o.b. shipping point on December 20, 2004, from a vendor were lost in
transit. The invoice price was ₱50,000. On January 5, 2005, Gear filed a ₱50,000 claim against the
common carrier.

In its December 31, 2004 balance sheet, Gear should report accounts payable of
a. ₱1,230,000.
b. ₱1,180,000.
c. ₱1,150,000.
d. ₱1,100,000.

A ₱1,100,000 + ₱80,000 + ₱50,000 = ₱1,230,000.

21. Dark Co. recorded the following data pertaining to raw material X during January 2004:
Date Units Unit cost
1/1/04 On hand 3,200 ₱2.00
1/11/04 Issue 1,600
1/22/04 Purchase 4,000 ₱2.35

The moving-average unit cost of X inventory at January 31, 2004 is


a. ₱2.18.
b. ₱2.22.
c. ₱2.25.
d. ₱2.35.

C [(1,600 × ₱2.00) + (4,000 × ₱2.35)] ÷ 5,600 = ₱2.25.

22. Barlow Company's Accounts Payable balance at December 31, 2002, was ₱1,800,000 before
considering the following transactions:
• Goods were in transit from a vendor to Barlow on December 31, 2002. The invoice price was
₱100,000, and the goods were shipped FOB shipping point on December 29, 2002. The goods
were received on January 4, 2003.
• Goods shipped to Barlow FOB shipping point on December 20, 2002, from a vendor were lost in
transit. The invoice price was ₱50,000. On January 5, 2003, Barlow filed a ₱50,000 claim against
the common carrier.

In its December 31, 2002 balance sheet, Barlow should report Accounts Payable of
a. 1,950,000 b. 1,900,000 c. 1,850,000 d. 1,800,000

A
Solution:
Unadjusted bal. 1,800,000
Goods purchased - FOB Shipping pt. 100,000
Goods lost - FOB Shipping pt. 50,000
Adjusted bal. 1,950,000

23. The balance in Master Company's accounts payable account at December 31, 2002, was
₱1,100,000 before considering the following information:
Page |7

• Goods shipped FOB shipping point on December 20, 2002 from a vendor to Master were lost in
transit. The invoice cost of ₱20,000 was not recorded by Master. On January 6, 2003, Master filed
a ₱20,000 claim against the common carrier.
• On December 27, 2002, a vendor authorized Master to return, for full credit, goods shipped and
billed at ₱35,000 on December 2, 2002. The returned goods were shipped by Master on December
27, 2002. A ₱35,000 credit memo was received and recorded by Master on January 6, 2003.

What amount should Master report as accounts payable in its December 31, 2002, balance sheet?
a. 1,120,000 b. 1,115,000 c. 1,085,000 d. 1,065,000

C
Solution:
Unadjusted bal. 1,100,000
Goods purchased - FOB shipping pt. 20,000
Purchase returns (35,000)
Adjusted bal. 1,085,000

24. The balance in Stockwell Company's accounts payable account on December 31, 2002, was
₱1,225,000 before the following information was considered:
• Goods shipped FOB destination on December 21, 2002, from a vendor to Stockwell were lost
in transit. The invoice cost of ₱45,000 was not recorded by Stockwell. On December 28, 2002,
Stockwell notified the vendor of the lost shipment.
• Goods were in transit from a vendor to Stockwell on December 31, 2002. The invoice cost
was ₱60,000, and the goods were shipped FOB shipping point on December 28, 2002.
Stockwell received the goods on January 6, 2003.

What amount should Stockwell report as accounts payable in its December 31, 2002, balance sheet?
a. 1,330,000 b. 1,285,000 c. 1,270,000 d. 1,225,000

B
Solution:
Unadjusted bal. 1,225,000
Goods purchased - FOB dest. -
Goods purchased - FOB shipping pt. 60,000
Adjusted bal. 1,285,000

25. When using the periodic inventory system, which of the following generally would not be
separately accounted for in the computation of cost of goods sold?
a. Trade discounts applicable to purchases during the period
b. Cash (purchase) discounts taken during the period
c. Purchase returns and allowances of merchandise during the period
d. Cost of transportation-in for merchandise purchased during the period

26. Goods out on consignment are


a. included in the consignee's inventory.
Page |8

b. recorded in a Consignment Out account which is an inventory account.


c. recorded in a Consignment In account which is an inventory account.
d. all of these

27. Miller Company needs an estimate of its ending inventory balance. The following information is
available:
Cost Retail
Sales revenue ............................. ₱180,000
Beginning inventory ....................... ₱ 35,000 62,000
Net purchases ............................. 100,000 135,000
Gross margin percentage ................... 30% 70%?

Given this information, when using the gross margin estimation method, ending inventory is
approximately
a. ₱1,000.
b. ₱9,000.
c. ₱19,000.
d. ₱11,650.

B 35,000 + 100,000 - (180,000 x 70%) = 9,000

28. The following information is available for the Becca Company for the three months ended June
30 of this year:

Inventory, April 1 of this year ...................... ₱1,200,000


Purchases ............................................ 4,500,000
Freight-in ........................................... 300,000
Sales ................................................ 6,400,000

The gross margin was 25 percent of sales. What is the estimated inventory balance at June 30?
a. ₱880,000
b. ₱933,000
c. ₱1,200,000
d. ₱1,500,000

C (1,200,000 + 4,500,000 + 300,000) – (6,400,000 x 75%) = 1,200,000

29. Petersen Menswear, Inc. maintains a markup of 60 percent based on cost. The company's selling
and administrative expenses average 30 percent of sales. Annual sales were ₱1,440,000.
Petersen's cost of goods sold and operating profit for the year are
Cost of Goods Sold Operating Profit
a. ₱864,000 ₱144,000
b. ₱864,000 ₱432,000
c. ₱900,000 ₱108,000
d. ₱900,000 ₱432,000
Page |9

Solution:
% Amount
Sales 100% 1,440,000
(1,440,000 x 100/160) or
COGS (900,000) (1,440,000 x 62.5%*)
Expenses 30% (432,000) (1,440,000 x 30%)
Profit 108,000

*(60% ÷ 160%) = 37.5% GPR based on sales


(100% - 37.5%) = 62.5% cost ratio

30. On October 31, a flood at Payne Company's only warehouse caused severe damage to its entire
inventory. Based on recent history, Payne has a gross profit of 25 percent of net sales. The
following information is available from Payne's records for the ten months ended October 31:
Inventory, January 1 .................................. ₱ 520,000
Purchases ............................................. 4,120,000
Purchase returns ...................................... 60,000
Sales ................................................. 5,600,000
Sales discounts ....................................... 400,000

A physical inventory disclosed usable damaged goods which Payne estimates can be sold for
₱70,000. Using the gross profit method, the estimated cost of goods sold for the ten months ended
October 31 should be
a. ₱680,000.
b. ₱3,830,000.
c. ₱3,900,000.
d. ₱4,200,000.

Solution: C (5,600,000 – 400,000) x 75%] = 3,900,000

31. Davis Company's accounting records indicated the following information:

Inventory, 1/1/02 ..................................... 1,000,000


Purchases during 2002 ................................. 5,000,000
Sales during 2002 ..................................... 6,400,000

A physical inventory taken on December 31, 2002, revealed actual ending inventory at cost was
₱1,150,000. Davis' gross profit on sales has regularly been about 25 percent in recent years. The
company believes some inventory may have been stolen during the year. What is the estimated
amount of missing inventory at December 31, 2002?
a. ₱50,000
b. ₱200,000
c. ₱350,000
d. ₱450,000

Solution: A (1,000,000 + 5,000,000) – (6,400,000 x 75%) = 1,200,000 – 1,150,000 = 50,000


P a g e | 10

32. On June 19, 2002, a fire destroyed the entire uninsured merchandise inventory of the Allen
Merchandising Company. The following data are available:

Inventory, January 1 .................................. ₱ 80,000


Purchases, January 1 through June 19 .................. 560,000
Sales, January 1 through June 19 ...................... 776,000
Markup percentage on cost ............................. 25%

What is the approximate inventory loss as a result of the fire?


a. ₱19,200
b. ₱27,200
c. ₱34,000
d. ₱58,000

A (80,000 + 560,000) – (776,000 x 100%/125%) = 19,200

33. Product X sells for ₱12.00; selling expenses are ₱2.40; normal profit is ₱3.00. If the cost of
Commodity X is ₱7.80, the lower of cost and NRV is
a. ₱5.40.
b. ₱6.00.
c. ₱6.60.
d. ₱7.80.

D – the cost

34. The following information is available for Torino Corp. for its most recent year:

Net sales ............................................. ₱3,600,000


Freight-in ............................................ 90,000
Purchase discounts .................................... 50,000
Ending inventory ...................................... 240,000

The gross margin is 40 percent of net sales. What is the cost of goods available for sale?
a. ₱1,680,000
b. ₱1,920,000
c. ₱2,400,000
d. ₱2,440,000

C (3,600,000 x 60%) = 2,160,000 COGS + 240,000 EI = 2,400,000

35. Changes in fair value are recognized in profit or loss for which type of financial assets?
a. Financial assets measured at amortized cost
b. FVOCI securities
c. Held to maturity debt securities
P a g e | 11

d. Financial assets designated at FVPL

36. Which securities are purchased with the intent of selling them in the near future?
a. Financial assets measured at amortized cost
b. FVOCI securities
c. Held for trading securities
d. Held-for-sale securities

37. Which of the following is not a debt security?


a. Convertible bonds
b. Commercial paper
c. Loans receivable
d. All of these are debt securities.

38. An unrealized holding loss on a company's FVOCI securities should be reflected in the current
financial statements as
a. an extraordinary item shown as a direct reduction from retained earnings.
b. a current loss resulting from holding securities.
c. a note or parenthetical disclosure only.
d. other comprehensive income and deducted in the equity section of the balance sheet.

39. An entity has financial assets held under a business model with the objective of holding financial
assets in order to collect contractual cash flows. Prior to maturity date, the entity sells a
significant portion of the financial assets. Which of the following statements is correct?
a. The change in circumstance is a prior period error.
b. Under the “hold to collect” business model, the entity needs to hold financial assets until
their maturity dates. A significant sale of financial assets before their maturity date
evidences an inability to hold and collect cash flows. Therefore, the remaining financial
assets shall be reclassified to either FVPL or FVOCI.
c. The remaining financial assets within the “hold to collect” business model need not be
reclassified. However, the change in circumstance may be relevant in assessing the business
model for new financial assets that have been acquired or originated.
d. The entity shall change its business model because of the change in circumstance. The
remaining financial assets shall be reclassified after the entity changes the business model.

Use the following information for the next two questions:


On January 1, 20x1, Gina Co. acquired 10%, ₱4,000,000 bonds for ₱3,807,853. The principal is due on
January 1, 20x4 but interest is due annually. The yield rate on the bonds is 12%.

40. How much is the interest income recognized in 20x1?


a. 456,942 b. 463,776 c. 471,429 d. 400,000

A (See amortization table below)

41. How much is the carrying amount of the investment on December 31, 20x1?
a. 3,807,853 b. 3,864,796 c. 3,928,571 d. 4,000,000
P a g e | 12

B (See amortization table below)


Solution:
Date Collections Interest income Amortization Present value
1/1/x1 3,807,853
12/31/x1 400,000 456,942 56,942 3,864,795

42. On December 29, 20x1, an entity commits itself to purchase a financial asset for ₱10,000, which is
its fair value on commitment date (trade date). Transaction costs are immaterial. On December 31,
20x1 and on January 4, 20x2 (settlement date) the fair values of the asset are ₱12,000 and ₱15,000,
respectively. If the entity uses the trade date accounting and that the investment is classified as
held for trading, how much is the carrying amount of the investment in the December 31, 20x1
statement of financial position?
a. 10,000 b. 12,000 c. 15,000 d. 0

B – the fair value on Dec. 31, 20x1

43. Tuba Co. enters into a “receive variable, pay fixed” interest swap on January 1, 20x1 for a
notional amount of ₱1,000,000. Under the terms of the contract, if the current rate increases
above 12% (i.e., the set rate), Tuba Co. shall receive the excess interest. If the current rate falls
below 12%, Tuba Co. shall pay the deficiency. Swap payment shall be made on December 31,
20x2. The current rates are as follows:
Jan. 1, 20x1……………………………12%
Jan. 1, 20x2……………………………15%

How much is the net cash settlement on January 1, 20x2?


a. 30,0000 payment
b. 30,000 receipt
c. 26,087 payment
d. 26,087 receipt

B (Receive 15% - Pay 12%) = 3% net receipt x 1,000,000 = 30,000 receipt

44. During the year, Entity A acquired investment in the shares of stocks of Entity B for ₱1,200,000.
Entity A made an irrevocable choice to measure the investment at FVOCI. At the end of the
reporting period, the investment has a fair value of ₱1,100,000. The change in fair value is
expected to persist over a very long period of time, and thus permanent. Which of the following
statements is correct?
a. Entity A recognizes a gain of ₱100,000.
b. Entity A recognizes a loss of ₱100,000 in profit or loss.
c. Entity A recognizes a loss of ₱100,000 in other comprehensive income.
d. Entity A ignores the change in fair value.

45. According to PAS 28, significant influence is the investor’s participation in the financial and
operating policy decisions of the investee but not control of these decisions. Which of the
following may an investor be unable to exercise significant influence?
a. participation in policy making process
P a g e | 13

b. material intercompany transactions


c. majority ownership of the investee concentrated among a small group of shareholders who
operate the investee without regard to the views of the investor
d. technological dependency

46. Consider the following statements.


I. In applying the Equity Method of accounting for investments in associates, dividends
received from the investee are considered a return of capital and should be credited to
stockholders’ equity of the investor.
II. A subsidiary is an affiliate that is not controlled by an enterprise directly, or indirectly,
through one or more intermediaries.

State whether the foregoing statements are correct.


a. Only I is correct
b. Only II is correct
c. I and II are correct
d. Neither I nor II is correct

47. Which of the following may provide evidence of significant influence even if the percentage of
ownership interest is less than 20%?
I. Representation on the board of directors or equivalent governing body of the investee.
II. Participation in policy-making processes, including participation in decisions about
dividends or other distributions.
III. Material transactions between the investor and the investee
IV. Interchange of managerial personnel.
V. Provision of essential technical information.
a. I, II b. I, II, III c. I, II, IV d. any of these

48. Bach Co. acquired a 20% interest in C Major Co. on December 31, 20x3 for ₱630,000. During 20x4,
C Major reported profit of ₱400,000 and paid cash dividends of ₱100,000. At December 31, 20x4,
the balance in the investment account should be
a. ₱630,000.
b. ₱690,000.
c. ₱670,000.
d. ₱720,000.
B ₱630,000 + (₱400,000 × .2) – (₱100,000 × .2) = ₱690,000.

49. The following statements relate to the fair value and equity methods of accounting for a stock
investment.
I. whenever an investment in equity securities does not qualify for equity method, the investor
is required to use PFRS 9 in accounting for dividend income received from the investee.
II. the fair value method of accounting for a stock investment recognizes the legal fact that the
investor and investee are one economic unit.
III. an investor may still be able to exercise significant influence over an investee even if the
investment is less than 20% of the voting stock of the investee.
P a g e | 14

IV. no adjustment to the investment account is made when changing from the equity to the fair
value method, or vice versa.

State whether the foregoing statements are true.


a. all of the statements are true
b. only one statement is true
c. only two statements are true
d. three statements are true

C - Statements I and III.

50. On January 1, 2003, Point, Inc. purchased 10% of Iona Co.’s common stock. Point purchased
additional shares bringing its ownership up to 40% of Iona’s common stock outstanding on
August 1, 2003. During October 2003, Iona declared and paid a cash dividend on all of its
outstanding common stock. Using PFRSs, how much income from the Iona investment should
Point’s 2003 income statement report?
a. 10% of Iona’s income for January 1 to July 31, 2003, plus 40% of Iona’s income for August 1
to December 31, 2003.
b. 40% of Iona’s income for August 1 to December 31, 2003 only.
c. 40% of Iona’s 2003 income.
d. Amount equal to dividends received from Iona.

51. Giovan Caresa owns 10% of the common stock of Gorthon Co. throughout the year. The Gorthon
Co. has no preferred stock outstanding. Giovan’s stock gives him the right to
a. be paid 10% of the firm’s profits in cash each year
b. receive dividends equal to 10% of the par value each year
c. receive dividends equal to 10% of the total dividends paid by the corporation for the year to
common stockholders
d. keep the corporation from issuing any additional stock unless he is willing to buy 10% of the
newly issued shares

52. Stock dividends on common stock should be recorded at their fair value by the investor when
the related investment is accounted for under which of the standards?
PFRS 9 PAS 28
a. Yes Yes
b. Yes No
c. No Yes
d. No No

53. In its financial statements, Pare, Inc. uses the fair value accounting for its 15% ownership of Sabe
Co. At December 31, 2003, Pare has a receivable from Sabe. How should the receivable be
reported in Pare’s December 31, 2003 financial statements?
a. The total receivable should be reported separately.
b. The total receivable should be included as part of the investment in Sabe, without separate
disclosure.
P a g e | 15

c. 85% of the receivable should be reported separately, with the balance offset against Sabe’s
payable to Pare.
d. The total receivable should be offset against Sabe’s payable to Pare, without separate
disclosure.

54. In its financial statements, Pulham Corp. uses the equity method of accounting for its 30%
ownership of Angles Corp. At December 31, 2003, Pulham has a receivable from Angles. How
should the receivable be reported in Pulham’s 2003 financial statements?
a. None of the receivable should be reported, but the entire receivable should be offset against
Angles’ payable to Pulham.
b. 70% of the receivable should be separately reported, with the balance offset against 30% of
Angles’ payable to Pulham.
c. The total receivable should be disclosed separately.
d. The total receivable should be included as part of the investment in Angles, without separate
disclosure.

55. When the equity method is used to account for investments in common stock, which of the
following affects the investor’s reported investment income?
Equipment amortization Cash dividends
related to purchase from investee
a. Yes Yes
b. No Yes
c. No No
d. Yes No

56. Park Co. uses the equity method to account for its January 1, 2003 purchase of Tun Inc.’s
common stock. On January 1, 2003, the fair values of Tun’s FIFO inventory and land exceeded
their carrying amounts. How do these excesses of fair values over carrying amounts affect Park’s
reported equity in Tun’s 2003 earnings?
Inventory excess Land excess
a. Decrease Decrease
b. Decrease No effect
c. Increase Increase
d. Increase No effect
Explanation: Land is not depreciable. Thus, the excess fair value is not amortized.

57. Peel Co. received a cash dividend from a common stock investment. Should Peel report an
increase in the investment account if it has classified the stock as FVOCI or uses the equity
method of accounting?
FVOCI Equity
a. No No
b. Yes Yes
c. Yes No
d. No Yes
P a g e | 16

58. On January 1, 20x1, Entity A acquires 30% interest in Entity B for ₱600,000. Entity B reports
profit of ₱200,000 and declares dividends of ₱50,000 in 20x1. How much is the carrying amount
of the investment in associate on December 31, 20x1?
a. 600,000
b. 660,000
c. 645,000
d. 630,000

Solution:
Investment in associate
1/1/x1 600,000
Sh. in profit (200K x 30%) 60,000 15,000 Dividends (50K x 30%)
645,000 12/31/x1

59. According to PAS 28, the reporting dates of the investor and its associate should not differ by
more than
a. one month.
b. two months.
c. three months.
d. six months.

60. Which of the following statements are in accordance with PAS 28?
I. When the associate has cumulative preference shares, the investor computes its share in the
profit or loss of the investee after deducting the preferred dividends, only when such
dividends are declared.
II. When the associate has non-cumulative preference shares, the investor computes its share in
the profit or loss of the investee after deducting the preferred dividends, whether or not such
dividends are declared.
a. true, true
b. true, false
c. false, true
d. false, false

61. Adjustments to the carrying amount of the investment in associate may be necessary for changes
in the investor’s proportionate interest in the investee arising from changes in the investee’s
equity that have not been recognized in the investee’s profit or loss. Which of the following may
not necessitate an adjustment in the investment in associate account?
a. Changes in revaluation surplus of associate
b. Changes in valuation of the associate’s FVOCI securities
c. Changes in the actuarial gains and losses of the associate’s defined benefit retirement plan.
d. Changes in the Allowance for doubtful accounts of the associate

62. When the accounting policies used by the investor and the associate do not match
a. PAS 28 requires appropriate adjustments to the associate’s financial statements to conform
them to the investor’s accounting policies for reporting like transactions and other events in
similar circumstances.
P a g e | 17

b. PAS 28 does not require appropriate adjustments to the associate’s financial statements to
conform them to the investor’s accounting policies for reporting like transactions and other
events in similar circumstances when it was not practicable to use uniform accounting
policies
c. PAS 28 requires the entity to discontinue the use of the equity method
d. In no instance should the accounting policies used by the investor and the associate be
different.

63. When financial statements of an associate used in applying the equity method are prepared as at
the end of the reporting period that is different from that of the investor,
a. the difference must be no greater than three months
b. the difference must be no greater than twelve months
c. the difference must be compensated by an interim financial statement
d. no difference must exist

64. When an entity elects to prepare separate financial statements, it shall account for its investment
in associates
a. at cost c. using the equity method
b. in accordance with PFRS 9 d. any of these

65. Young Co. acquired a 60% interest in Tomlin Corp. on December 31, 2003 for ₱630,000. Young
accounts for the investment using the equity method. During 2004, Tomlin had net income of
₱400,000 and paid cash dividends of ₱100,000. At December 31, 2004, the balance in the
investment account should be
a. ₱630,000.
b. ₱870,000.
c. ₱810,000.
d. ₱930,000.

C ₱630,000 + (₱400,000 × .6) – (₱100,000 × .6) = ₱810,000.

Use the following information for the next three questions:


Kimm, Inc. acquired 30% of Carne Corp.'s voting stock on January 1, 2004 for ₱360,000. During 2004,
Carne earned ₱150,000 and paid dividends of ₱90,000. Kimm's 30% interest in Carne gives Kimm the
ability to exercise significant influence over Carne's operating and financial policies. During 2005,
Carne earned ₱180,000 and paid dividends of ₱60,000 on April 1 and ₱60,000 on October 1. On July
1, 2005, Kimm sold half of its stock in Carne for ₱237,000 cash.

66. What amount should Kimm include in its 2004 income statement as a result of the investment?
a. ₱150,000.
b. ₱90,000.
c. ₱45,000.
d. ₱27,000.

C ₱150,000 × 30% = ₱45,000.

67. The carrying amount of this investment in Kimm's December 31, 2004 balance sheet should be
P a g e | 18

a. ₱360,000.
b. ₱378,000.
c. ₱405,000.
d. ₱333,000.

B ₱360,000 + ₱45,000 – (₱90,000 × 30%) = ₱378,000.

68. What should be the gain on sale of this investment in Kimm's 2005 income statement?
a. ₱57,000.
b. ₱52,500.
c. ₱43,500.
d. ₱34,500.

C ₱378,000 – (₱60,000 × 30%) + (₱180,000 × 50% × 30%) = ₱387,000; ₱237,000 – (₱387,000 ÷ 2) = ₱43,500.

69. On January 1, 2004, Sloane Co. purchased 25% of Orr Corp.'s common stock; no goodwill
resulted from the purchase. Sloane appropriately carries this investment at equity and the
balance in Sloane’s investment account was ₱480,000 at December 31, 2004. Orr reported net
income of ₱300,000 for the year ended December 31, 2004, and paid dividends totaling ₱120,000
during 2004. How much did Sloane pay for its 25% interest in Orr?
a. ₱435,000.
b. ₱510,000.
c. ₱525,000.
d. ₱585,000.

A ₱480,000 – (₱300,000 × 25%) + (₱120,000 × 25%) = ₱435,000.

70. Which of the following values is unlikely to be used in fair value measurement?
a. Quoted price in a market.
b. The most recent market transaction price.
c. The present value of the expected net cash flows from the asset.
d. Level 4 inputs in accordance with PFRS 13.

71. A gain or loss arising on the initial recognition of a biological asset and from a change in the fair
value less cost to sell of a biological asset should be included in
a. The net profit or loss for the period.
b. The statement of recognized gains and losses.
c. A separate revaluation reserve.
d. A capital reserve within equity.

72. When agricultural produce is harvested, the harvest should be accounted for by using PAS 2
Inventories or another applicable standard. For the purposes of that Standard, cost at the date of
harvest is deemed to be
a. Its fair value less costs to sell at point of harvest.
b. The historical cost of the harvest.
c. The historical cost less accumulated impairment losses.
d. Market value.
P a g e | 19

73. Contract prices are not necessarily relevant in determining fair value and the fair value of a
biological asset or agricultural produce is not adjusted because of the existence of a contract.
a. True.
b. False.
c. Maybe.
d. I don’t know.

74. Land that is related to agricultural activity is valued


a. At fair value.
b. In accordance with PAS 16 Property, Plant, and Equipment or PAS 40 Investment Property.
c. At fair value in combination with the biological asset that is being grown on the land.
d. At the resale value separate from the biological asset that has been grown on the land.

75. An unconditional government grant related to a biological asset that has been measured at fair
value less cost to sell should be recognized as
a. Income when the grant becomes receivable.
b. A deferred credit when the grant becomes receivable.
c. Income when the grant application has been submitted.
d. A deferred credit when the grant has been approved.

76. If a government grant is conditional on certain events, then the grant should be recognized as
a. Income when the conditions attaching to the grant are met.
b. Income when the grant has been approved.
c. A deferred credit when the conditions attached to the government grant are met.
d. A deferred credit when the grant is approved.

77. Where there is a production cycle of more than one year, PAS 41 encourages separate disclosure
of the
a. Physical change only.
b. Price change only.
c. Total change in value.
d. Physical change and price change.

78. Which of the following information should be disclosed under PAS 41?
a. Separate disclosure of the gain or loss relating to biological assets and agricultural produce.
b. The aggregate gain or loss arising on the initial recognition of biological assets and
agricultural produce and the change in fair value less cost to sell of biological assets.
c. The total gain or loss from biological assets, agricultural produce, and from changes in fair
value less cost to sell of biological assets.
d. There is no requirement in the Standard to disclose separately any gains or losses.

79. Entity A’s assets have a carrying amount of ₱100,000 before year-end adjustments. The PFRSs
require these assets to be measured at fair value at each reporting date. Location is a
characteristic of the assets. Information at year-end is as follows:
P a g e | 20

Active Market #1 Active Market #2


Quoted price ₱130,000 Quoted price ₱135,000
Transport costs 10,000 Transport costs 12,000
Costs to sell 2,000 Costs to sell 3,000

If neither Active Market #1 nor Active Market #2 is the principal market, how much is the fair value?
a. 135,000 c. 120,000
b. 132,000 d. 123,000

Solution:
The “most advantageous market” is determined as follows:
Active Market #1 Active Market #2
Quoted price 130,000 135,000
Transport costs (10,000) (12,000)
Costs to sell (2,000) (3,000)
Net sale proceeds 118,000 120,000

The fair value is computed as follows:


135,000 price in active market #2 – 12,000 transport costs = 123,000

80. The equity method of accounting for investments is discussed under


a. PAS 28
b. PAS 29
c. PAS 21
d. PFRS 2

“We want each of you to show this same diligence to the very end, so
that what you hope for may be fully realized.” (Hebrews 6:11)

- END -
Page |1

NAME: Date:
Professor: Section: Score:

INTERMEDIATE ACCOUNTING 1
FINAL GRADING EXAMINATION

Chapter 1 - The Accounting Process


1. What is the net effect of the under mentioned errors on the trial balance of a firm?
I. Total of sales was taken as P58,726 instead of P58,762.
II. A discount of P52 allowed to Mr. X was not posted in the discount account.
III. Sale of old furniture of P130 was credited to Machinery account.
IV. A credit sale of P250 to Mr. Y was posted twice in his account.

a. Credit total of trial balance will be more than that of debit total by P234
b. Debit total of trial balance will be more than that of credit total by P234
c. Credit total of trial balance will be more than that of debit total by P104
d. Debit total of trial balance will be more than that of credit total by P264
e. Debit total of trial balance will be more than that of credit total by P286

Solution:
I. Erroneous credit to sales 58,762 58,726
II. Omission of sales discount 0 52
III. No effect - both accounts are debits
IV. Double posting to accounts receivable 250 250
250
59,262 59,028
Excess of total debits 234

2. The credit total of a trial balance exceeds the debit total by P350. In investigating the cause
of the difference, the following errors were determined: a credit to accounts receivable of
P550 was not posted; a P5,000 debit to be made to the Purchases account was debited to
Accounts payable instead; a P3,000 credit to be made to the Sales account was credited to
the Accounts receivable account instead; the Interest payable account balance of P4,500 was
included in the trial balance as P5,400. The correct balance of the trial balance is
a. 7,540 b. 8,550 c. 9,250 d. 7,450

Solution:
Excess of total credits 350
Correction for credit to A/R not posted (550)
Correction for debit to purchases not posted 5,000
Correction for erroneous debit to A/P 5,000
Page |2

Correction for credit to sales not posted 3,000


Correction for erroneous credit to A/R 3,000
Correction for overstatement in interest payable (900)
Adjusted balances 7,450 7,450

3. These are the means by which the information accumulated and processed in financial
accounting is periodically communicated to the users. They are the end products of the
accounting process.
a. Financial statements
b. Financial Reporting Standards
c. Notes to financial statements
d. All of these

4. It represents the steps or accounting procedures normally used by entities to record transactions
and prepare financial statements. It implements the accounting process.
a. Accounting cycle c. Accounting evolution
b. Accounting process d. Accounting information system

5. Which of the following is a nominal (temporary) account?


a. Unearned Revenue
b. Salaries Expense
c. Inventory
d. Retained Earnings

Chapter 2 - Cash and Cash Equivalents


6. The following were taken from the records of SML Co. as of December 31, 20x1:
Checks drawn but not yet issued to payees ₱120,000
Customers’ checks dated January 15, 20x2 35,000
Customers’ checks dated Dec. 31, 20x1 40,000
SML’s check dated Jan. 15, 20x2 already 16,000
mailed to payee
Cash on hand 130,000
Employees’ checks representing unclaimed 14,000
salaries, held by the treasurer
Petty cash fund (fully replenished) 20,000

How much of the items listed above will be included in SML’s Dec. 31, 20x1 cash?
a. 340,000
b. 260,000
c. 280,000
d. 320,000

Solution:
Checks drawn but not yet issued to payees 120,000
Customers’ checks dated Dec. 31, 20x1 40,000
Page |3

SML’s check dated Jan. 15, 20x2 already mailed to payee 16,000
Cash on hand 130,000
Employees’ checks representing unclaimed salaries,
14,000
held by the treasurer
Petty cash fund (fully replenished) 20,000
Total 340,000

7. The accountant for Baccah Inc. established a petty cash fund of ₱1,400. During September, the
fund was depleted by the following disbursements:

Shipping expense ................................... ₱740


Travel expense .......................................... 240
Postage expense ......................................... 230
Miscellaneous supplies .................................. 170

In addition to receipts for the above items, the petty cash box contained ₱8 in coins and an IOU of ₱8
from the secretary handling the fund. The company uses a cash over and short expense account, as
needed. The company decided to decrease the petty cash fund to ₱1,000.

How much is the cash (shortage) or overage?


a. (4)
b. 4
c. (12)
d. 12

740 + 240 + 230 + 170 + 8 + 8 = 1,396 per count – 1,400 accountability = 4 shortage

Chapter 3 - Bank Reconciliation


8. Yesterday, you wrote a ₱2M check and gave it to a supplier as payment for the goods you have
purchased. Today, you received your bank statement. You noticed that the ₱2M check is not
reflected in the statement. What should you do?
a. Call your friends and celebrate, telling them that you just saved ₱2M.
b. Call the supplier and demand him or her to go to the bank and present the check for
payment.
c. Post the incident on your Facebook page and wait for likes.
d. Treat the ₱2M check as outstanding check in your bank reconciliation for today.

9. Entity A is preparing its March 31, 20x1 bank reconciliation. The following information was
determined:
• The cash balance per books is ₱280,000 while the cash balance per bank statement is
₱320,000.
• Credit memo – ₱20,000
• Debit memo – ₱15,000
• Deposits in transit – ₱75,000
• Outstanding checks – ₱25,000
• The disbursements per books are overstated by ₱45,000.
Page |4

• The bank debits are understated by ₱40,000.

How much is the adjusted balance of cash?


a. 370,000
b. 330,000
c. 285,000
d. 380,000

Solution:
Bal. per books, end. 280,000 Bal. per bank, end. 320,000
Add: CM 20,000 Add: DIT 75,000
Less: DM (15,000) Less: OC (25,000)
Add/Less: Book errors: Add/Less: Bank errors:
Understatement 45,000 Overstatement (40,000)
Adjusted balance 330,000 Adjusted balance 330,000

Chapter 4 - Accounts Receivable


10. At 30 September 2000, Z Ltd had a provision for doubtful debts of P37,000. During the year
ended 30 September 2001 the company wrote off debts totaling P18,000, and at the end of the
year it is decided that the provision for doubtful debts should be P20,000. What should be
included in the income statement for bad and doubtful debts?
a. P35,000 debit
b. P1,000 debit
c. P38,000 debit
d. P1,000 credit
B (20,000 + 18,000 – 37,000) = 1,000 bad debts expense

11. Light Co.’s accounts receivable balances at the beginning and end of the period were ₱80,000
and ₱100,000, respectively. Write-offs and recoveries during the period amounted to ₱10,000 and
₱8,000, respectively. Collections of sales on account during the period totaled ₱120,000,
excluding the recoveries.

How much is the total credit sales during the period?


a. 130,000
b. 150,000
c. 170,000
d. 210,000

Accounts receivable
beg. 80,000
Credit sales 150,000 120,000 Collections, excluding recoveries
10,000 Write-off
100,000 end.
Page |5

Chapter 5 - Notes Receivable


12. On January 1, 20x1, ABC Co. received a 3-year, noninterest bearing note of ₱133,100 in exchange
for land with carrying amount of ₱100,000. The note is due on December 31, 20x3. The effective
interest rate is 10%. How much is the carrying amount of the note on December 31, 20x2?
a. 133,100
b. 121,000
c. 110,000
d. 100,000

Solution:
Initial measurement:
₱133,100 x PV of ₱1 @10%, n= 3 = ₱100,000

Date Interest income Unearned interest Present value


1/1/x1 33,100 100,000
12/31/x1 10,000 23,100 110,000
12/31/x2 11,000 12,100 121,000
12/31/x3 12,100 - 133,100

Chapter 6 - Receivables – Additional Concepts


13. Nicole Company transferred loan assets with carrying amount and fair value of P100,000 to
Tristan Co. for cash amounting to P100,000. The terms of the transfer include a provision that
any individual loan could be called back but the aggregate amount of loans that could be
repurchased could not exceed P10,000. How much asset would be derecognized in the following
transaction?
a. 100,000
b. 90,000
c. 110,000
d. 10,000

B there is continuing involvement for the P10,000.

14. On June 30, 2002, Simon Company discounted a customer's ₱180,000, 6 month, 10 percent note
receivable dated April 30, 2002. A discount rate of 12 percent was charged by the bank. Simon's
proceeds from this discounted note would be
a. ₱169,200.
b. ₱172,800.
c. ₱181,440.
d. ₱185,220.

C
MV = 180,000 + (180,000 x 10% x 6/12) = 189,000
D = 189,000 x 12% x 4/12 = 7,560
NP = 189,000 – 7,560 = 181,440
Page |6

Chapter 7 - Inventories
15. The use of a Purchase Discounts account implies that the recorded cost of a purchased inventory
item is its
a. invoice price.
b. invoice price plus any purchase discount lost.
c. invoice price less the purchase discount taken.
d. invoice price less the purchase discount allowable whether taken or not.

Use the following information for the next two questions:


During 2004, which was the first year of operations, Luther Company had merchandise purchases of
₱985,000 before cash discounts. All purchases were made on terms of 2/10, n/30. Three-fourths of
the items purchased were paid for within 10 days of purchase. All of the goods available had been
sold at year end.

16. Which of the following recording procedures would result in the highest cost of goods sold for
2004?
1. Recording purchases at gross amounts
2. Recording purchases at net amounts, with the amount of discounts not taken shown
under "other expenses" in the income statement
a. 1
b. 2
c. Either 1 or 2 will result in the same cost of goods sold.
d. Cannot be determined from the information provided.

17. Which of the following recording procedures would result in the highest net income for 2004?
1. Recording purchases at gross amounts
2. Recording purchases at net amounts, with the amount of discounts not taken shown
under "other expenses" in the income statement
a. 1
b. 2
c. Either 1 or 2 will result in the same net income.
d. Cannot be determined from the information provided.

18. Which of the following is correct?


a. Selling costs are product costs.
b. Manufacturing overhead costs are product costs.
c. Interest costs for routine inventories are product costs.
d. All of these.

19. Entity A acquires inventories and incurs the following costs:


Purchase price, gross of trade discount 100,000
Trade discount 20,000
Non-refundable purchase tax, not included in the purchase price above 5,000
Freight-in (Transportation costs) 15,000
Commission to broker 2,000
Page |7

Advertisement costs 10,000

How much is the cost of the inventories purchased?


a. 102,000
b. 122,000
c. 97,000
d. 100,000

Solution:
Purchase price, gross of trade discount 100,000
Trade discount (20,000)
Non-refundable purchase tax 5,000
Freight-in (Transportation costs) 15,000
Commission to broker 2,000
Total cost of inventories 102,000

Chapter 8 - Inventory Estimation


20. On October 1, 20x1, the warehouse of ABC Co. and all inventories contained therein were
damaged by flood. Off-site back up of data base shows the following information:

Inventory, Jan. 1 14,500


Accounts payable, Jan. 1 6,000
Accounts payable, Sept. 30 3,000
Payments to suppliers 50,000
Freight-in 5,000
Purchase returns and discounts 2,500
Sales from Jan. to Sept. 75,000
Sales returns 5,000
Sales discounts 2,000
Gross profit rate based on sales 20%

Additional information:
Goods in transit as of October 1, 20x1 amounted to ₱2,000, cost of goods out on consignment is
₱1,200, and materials damaged by flood can be sold at a salvage value of ₱500.

How much is the inventory loss due to the flood?


a. 6,800
b. 7,200
c. 7,800
d. 8,200

Solution:
Accounts payable
6,000 Beginning balance
47,000 Net purchases (squeeze)
Page |8

Payments to suppliers 50,000


Ending balance 3,000

*Cost of goods sold is computed as follows:

Gross sales 75,000


Sales returns (5,000)
Net sales 70,000
Multiply by: Cost ratio (100% - 20% GPR based on sales) 80%
Cost of goods sold 56,000

Inventory, Sept. 30 (see T-account above) 10,500


Goods in transit (2,000)
Goods out on consignment (1,200)
Salvage value (500)
Inventory loss due to flood 6,800

Chapter 9 - Investments

21. It is the amount at which a financial asset is measured at initial recognition minus principal
repayments, plus or minus the cumulative amortization using the effective interest method of
any difference between that initial amount and the maturity amount, and minus any reduction
(directly or through the use of an allowance account) for impairment or uncollectability.
a. fair value c. amortized cost
b. discounted cost d. liquidation value

Use the following information for the next two questions:


Karen Co. purchased the following equity securities on January 1, 20x1 for a total amount of
P360,000.
Cost
Alaska Co. preference shares P200,000
Valdez Co. ordinary shares 160,000
Totals P360,000

The shares did not qualify for recognition as held for trading, thus they were classified as
investment in equity securities measured at fair value through other comprehensive income.
Page |9

On December 31, 20x1, the portfolio of Karen Co. comprised the following.
Fair value – 12/31/x1
Alaska Co. preference shares P240,000
Valdez Co. ordinary shares 60,000
Total P300,000

On December 31, 20x2, the portfolio of Karen Co. comprised the following:
Fair value – 12/31/x2
Alaska Co. preference shares P220,000
Valdez Co. ordinary shares 180,000
Total P400,000

On February 2, 20x3, all of the Alaska Co. preference shares were sold for P160,000 net of transaction
costs.

22. How much is the unrealized gain (loss) recognized in other comprehensive income on December
31, 20x1?
a. 60,000
b. (60,000)
c. 100,000
d. 0

B (300,000 – 360,000) = (60,000)

23. How much is the unrealized gain (loss) accumulated in equity as of December 31, 20x2?
a. 40,000
b. (40,000)
c. 100,000
d. 0

A (400,000 FV 12/31/x2 – 360,000 cost) = 40,000 unrealized gain

Chapter 10 – Investments in Debt Securities

24. On January 1, 20x1, Mitch Co. acquired 12%, P4,000,000 bonds at 98. Commission paid to
brokers amounted to P204,000. Principal is due on December 31, 20x4 but interest payments are
made annually starting December 31, 20x1.

The adjusted effective interest rate on the investment is closest to


a. 12% b. 11% c. 10.2650% d. indeterminable

B
Solution:
Acquisition cost (4M x 98%) 3,920,000
P a g e | 10

Direct cost 204,000


Initial carrying amount 4,124,000

“Trial and error” approach:


Future cash flows x PV factor at x% = Present value
(4M x PV of P1 @ x%, n=4) + (4M x 12% x PV of an ordinary annuity of P1 @ x%, n=4) = 4,124,000

There is premium because the carrying amount is greater than the face amount. Therefore, the effective
interest rate must be lower than the nominal rate of 12%.

First trial: (using 11%)


Future cash flows x PV factor at x% = PV or initial carrying amount
 (4M x PV of P1 @ 11%, n=4) + (4M x 12% x PV of an ordinary annuity of P1 @ 11%, n=4) =
4,124,000
 (4M x 0.658731) + (480,000 x 3.102446) = 4,124,000
 (2,634,924 + 1,489,174) = 4,124,098 approximates 4,124,000 (a difference of only P98)

If the difference of P98 is judged immaterial, then 11% is deemed the effective interest rate.

Use the following information for the next three questions:


On January 1, 20x1, ABC Co. acquired 10%, ₱1,000,000 bonds for ₱827,135. The bonds mature on
December 31, 20x3 and pay annual interest every December 31. ABC Co. incurred transaction costs
₱80,000 on the acquisition. The effective interest rate adjusted for the effect of the transaction costs is
14%.

The bonds are to be held under a “hold to collect and sell” business model. Information on fair
values is as follows:
December 31, 20x1…………………………….98
December 31, 20x2……………………………102
December 31, 20x3……………………………100

25. How much is the carrying amount of the investment on December 31, 20x1?
a. 935,134 b. 1,002,000 c. 980,000 d. 965,443

C = 1M x 98%

26. How much is the unrealized gain (loss) recognized in other comprehensive income on December
31, 20x1?
a. 45,866 b. (45,866) c. (37,899) d. 0

A Solution:

Amortization table
Interest
Date received Interest income Amortization Present value
1/1/x1 907,135
12/31/x1 100,000 126,999 26,999 934,134
12/31/x2 100,000 130,779 30,779 964,913
12/31/x3 100,000 135,088 35,088 1,000,000

 [(1M x 98%) – 934,134] = 45,866 Unrealized gain – OCI


P a g e | 11

27. How much is the interest income recognized in 20x2?


a. 126,999 c. 135,088
b. 130,779 d. 144,388

B (See table above)

Chapter 11 - Other long-term investments


28. On January 1, 20x1, ABC Co. insures the life of its president for ₱1,000,000. ABC Co. is the
beneficiary. Annual insurance premium of ₱20,000 is payable at the beginning of each year.
Information on the cash surrender value from the insurance policy is shown below:

Policy year Cash surrender value


Dec. 31, 20x1 -
Dec. 31, 20x2 -
Dec. 31, 20x3 21,000
Dec. 31, 20x4 28,000
Dec. 31, 20x5 40,000

How much is the insurance expense for the year 20x3?


a. 1,000
b. 7,000
c. 13,000
d. 0

Solution: 20,000 annual premium – (21,000 / 3) = 13,000

Chapter 12 - Basic Derivatives


Use the following information for the next three questions:
On March 1, 20x1, ABC Co. sold inventory to a foreign company for FC 1,000,000 (FC means foreign
currency) when the spot exchange rate is FC 40: ₱1. The payment is due on April 1, 20x1.

ABC Co. is concerned about the possible fluctuation in exchange rates, so on this date, ABC Co.
entered into a forward contract to sell FC 1,000,000 for ₱25,000 to a broker. According to the terms of
the forward contract, if FC 1,000,000 is worth less than ₱25,000 on April 1, 20x1, ABC Co. shall
receive from the broker the difference; if it is worth more than ₱25,000, ABC Co. shall pay the broker
the difference.

29. If the exchange rate on April 1, 20x1 is FC35: ₱1, how much is the net cash settlement?
a. 3,571 receipt
b. 3,571 payment
c. 4,231 receipt
d. 4,231 payment

Solution:
Fixed selling price ₱25,000
P a g e | 12

Selling price at current spot rate (1M ÷ 35) 28,571


Excess – payment to broker (₱3,571)

30. If the exchange rate on April 1, 20x1 is FC50: ₱1, how much is the net cash settlement?
a. 5,000 payment
b. 5,000 receipt
c. 6,223 payment
d. 6,223 receipt

Solution:
Fixed selling price ₱25,000
Selling price at current spot rate (1M ÷ 50) 20,000
Deficiency - receipt from broker ₱ 5,000

31. If the exchange rate on March 31, 20x1 is FC45: ₱1, how much is the fair value of the interest rate
swap?
a. 3,000 asset
b. 3,000 liability
c. 2,778 asset
d. 2,778 liability

Solution:
Fixed selling price ₱25,000
Selling price at current spot rate (1M ÷ 45) 22,222
Fair value of forward contract – receivable (asset) ₱ 2,778

32. ABC Co. does printing jobs for various customers. On January 1, 20x1, ABC Co. forecasted the
purchase of 1,000 reams of paper in the next quarter. The expected purchase date is on April 15,
20x1.

ABC Co. expects that the price of paper will fluctuate because of the upcoming elections. Thus, on
January 1, 20x1, ABC Co. enters into a forward contract to purchase 1,000 reams of paper at a
forward rate of ₱600 per ream. If the market price on April 15, 20x1 is more than ₱600, ABC Co. shall
receive the difference from the broker. On the other hand, if the market price is less than ₱600, ABC
Co. shall pay the difference to the broker. The forward contract will be settled net on April 15, 20x1.
The discount rate is 10%.

If the price of paper is ₱700 per ream on March 31, 20x1, how much is the derivative asset (liability)
to be recognized in ABC Co.’s first quarter financial statements?
a. 100,000 asset
b. 100,000 liability
c. 98,772 asset
d. 98,772 liability

Solution:
P a g e | 13

Fixed purchase price (₱600 x 1,000) 600,000


Purchase price at current market price (₱700 x 1,000) 700,000
Derivative asset - receivable from broker 100,000

Chapter 13 - Investment in associates (PAS 28)


33. On January 1, 20x1, Entity A acquires 25% interest in Entity B for ₱800,000. Entity B reports
profit of ₱1,000,000 and declares dividends of ₱100,000 in 20x1. How much is the carrying
amount of the investment in associate on December 31, 20x1?
a. 800,000
b. 1,250,000
c. 1,000,000
d. 1,025,000

Solution:
Investment in associate
1/1/x1 800,000
Sh. in profit (1M x 25%) 250,000 25,000 Dividends (100K x 25%)
1,025,000 12/31/x1

Chapter 14 - Agriculture (PAS 41)


34. Which of the following is outside the scope of PAS 41?
a. dairy cattle used in the production of milk
b. chickens used in the production of meat
c. rice plants and other crops that produce agricultural products only once
d. mango trees and other plants that produce agricultural products repeatedly over a long
period of time

35. Which of the following is considered a biological asset?


a. Carcass c. Pig
b. Ham d. Piggy bank

36. Which of the following is considered an agricultural produce?


a. fruit cocktail c. picked or harvested fruit
b. fruit tree d. dried fruit

37. According to PAS 41, biological assets are measured as follows:


Initial measurement Subsequent measurement
a. fair value less costs to sell fair value less costs to sell
b. cost cost less accumulated depreciation
c. cost cost less accumulated depreciation and
impairment losses
d. fair value less costs to sell cost
P a g e | 14

Chapter 15 -PPE - Initial measurement


38. When a company purchases land with a building on it and immediately tears down the building
so that the land can be used for the construction of a plant, the costs incurred to tear down the
building should be
a. amortized over the estimated time period between the tearing down of the building and the
completion of the plant.
b. expensed as incurred.
c. added to the cost of the plant.
d. added to the cost of the land.

39. Lakepoint Company recently accepted a donation of land with a fair value of ₱200,000 from an
unrelated party. The entry that Lakepoint should use to record this land is:
a. Plant.............................. 200,000
Gain from Receipt of Donated Plant 200,000
b. Land.............................. 200,000
Gain from Receipt of Donated Land 200,000
c. Land.............................. 200,000
Unrealized Gain from Receipt of
Donated Land.................. 200,000
d. Land.............................. 200,000
Retained Earnings................ 200,000

40. Small tools and containers used repeatedly for more than a year are classified on the balance
sheet as
a. current assets.
b. fixed assets (PPE).
c. deferred charges.
d. investments.

41. Which of the following is a capital expenditure?


a. Payment of an account payable
b. Retirement of bonds payable
c. Payment of income taxes
d. None of these

Chapter 16 -PPE - Subsequent measurement


42. You are a business manager. During the period, you have authorized the acquisition of a
machine that will be used in your company’s manufacturing activities in the next 5 years. In
your selection of an appropriate accounting policy for the recognition and measurement of the
machine, which of the following reporting standards is most relevant?
a. PAS 1
b. PAS 2
c. PAS 16
P a g e | 15

d. PAS 32

43. You are the sole proprietor of Entity A. As a requisite to your business loan application, you
were required by the bank to submit audited financial statements. During the audit of your
financial statements, the auditor questioned the carrying amount of your land. The auditor
believes that the carrying amount is overstated and needs to be written down to its recoverable
amount. In your discussions with your auditor, the auditor would most likely refer to this
standard in her report?
a. PAS 36
b. PFRS 1
c. PAS 26
d. PAS 12

44. Entity A acquires equipment on January 1, 20x1. Information on costs is as follows:

Purchase price, gross of ₱10,000 trade discount 800,000


Non-refundable purchase taxes 20,000
Delivery and handling costs 40,000
Installation costs 30,000
Present value of decommissioning and restoration
costs 10,000

How much is the initial cost of the equipment?


a. 900,000 c. 870,000
b. 820,000 d. 890,000

D (800K – 10K + 20K + 40K + 30K + 10K) = 890,000

45. (Use the information in the immediately preceding problem.) Assume the equipment has a useful life
of 10 years and a residual value of ₱90,000. Entity A uses the straight line method of
depreciation. How much are the depreciation expense in 20x1 and the carrying amount of the
equipment on December 31, 20x2, respectively?
Depreciation expense Carrying amount – 12/31/x2
a. 80,000 810,000
b. 80,000 730,000
c. 80,000 640,000
d. 80,000 580,000

B (890,000 – 90,000) ÷ 10 = 80,000;


890,000 – (80,000 x 2 years) = 730,000

46. (Use the information in the immediately preceding two problems.) Assume the equipment has a useful
life of 10 years and a residual value of ₱90,000. Entity A uses the straight line method of
depreciation. On December 31, 20x2, Entity A revalues the equipment at a fair value of ₱820,000.
There is no change in the residual value and the remaining useful life of the asset. How much
P a g e | 16

are the revaluation surplus on December 31, 20x2 and revised depreciation expense in 20x3 and
in subsequent periods, respectively?
Revaluation surplus Revised annual depreciation
a. 83,000 90,250
b. 89,000 91,050
c. 90,000 91,250
d. 92,000 92,150

C 820,000 fair value – 730,000 carrying amount on 12/31/x2 = 90,000;


(820,000 fair value – 90,000 residual value) ÷ 8 yrs. = 91,250

47. Entity A sells a machine that is classified as PPE for ₱1,700,000. Entity A pays the broker a 10%
commission. Information on the machine is as follows:
Carrying amount ₱1,900,000
Revaluation surplus 400,000

How much is the gain (loss) from the sale?


a. (200,000) c. (30,000)
b. (370,000) d. 30,000

(1,700,000 x 90%) – 1,900,000 = (370,000)


The revaluation surplus is transferred directly to retained earnings. Hence, it does not affect the gain
or loss on the sale.

Chapter 17 -Depletion of Mineral Resources (PFRS 6)


48. According to PFRS 6, expenditures on exploration for and evaluation of mineral resources are
recognized as
a. assets.
b. expenses.
c. a or b depending on the entity’s accounting policy.
d. not accounted for

49. In 20x1, ABC Mining Corp. acquired the right to use 1,000 acres of land to mine for gold. The
lease cost is ₱50,000,000, and the related exploration costs on the property amounted to
₱10,000,000. It is the policy of ABC Mining Corp. to capitalize all costs of exploration and
evaluation of mineral resources. Intangible development costs for drilling, tunnels, shafts, and
wells incurred before opening the mine amounted to ₱85,000,000. At the end of the mine’s
economic useful life, ABC Mining Corp. is required by legislation to restore the site. Estimated
restoration costs have a fair value of ₱5,000,000. ABC Mining Corp. estimates that the mine will
provide approximately 100,000,000 ounces of gold. ABC extracted 300,000 ounces of gold in
20x2. How much is the depletion charge in 20x2?
a. 450,000
b. 480,000
c. 360,000
d. 460,000
P a g e | 17

Solution:
Acquisition cost 50,000,000
Exploration costs (capitalized per ABC’s accounting policy) 10,000,000
Intangible development costs 85,000,000
Restoration costs (fair value) 5,000,000
Total cost of natural resource 150,000,000

150,000,000 x 300,000/100,000,000 = 450,000

Chapter 18 -Government grants (PAS 20)


50. On January 1, 20x1, Entity A receives a financial aid from the government amounting to ₱1M as
compensation for losses it has incurred on a recent calamity. How much income from
government grant will Entity A recognize in 20x1?
a. 1,000,000 c. 53,334
b. 100,000 d. 0

51. Entity A receives land from the government conditioned that the land will only be used in Entity
A’s primary business activities and should never be sold. If in case, Entity A decides not to use
the land in its primary business activities, it shall return the land to the government. Which of
the following standards is least likely to be relevant in accounting for the land?
a. PAS 2
b. PAS 16
c. PAS 20
d. All of these are relevant

Chapter 19 - Borrowing costs (PAS 23)


52. Which of the following is a qualifying asset?
a. Biological asset measured at fair value less costs to sell
b. A multi-million dollar executive jet plane that is ready for its intended use upon purchase
c. A second-hand heavy machinery that takes 2 years to refurbish and customize for its
intended use
d. A long-term note receivable (financial asset)

53. On January 1, 20x1, Entity A obtained a 12%, ₱6,000,000 loan, specifically to finance the
construction of a building. The proceeds of the loan were temporarily invested and earned
interest income of ₱180,000. The construction was completed on December 31, 20x1. How much
borrowing costs are capitalized to the cost of the constructed building?
a. 540,000 c. 720,000
b. 480,000 d. 0

A (6M x 12%) – 180,000 = 540,000


P a g e | 18

54. Clay Company started construction of a new office building on January 1, 20x3, and moved into
the finished building on July 1, 20x4. Of the building’s ₱2,500,000 total cost, ₱2,000,000 was
incurred in 20x3 evenly throughout the year. Clay’s incremental borrowing rate was 12%
throughout 20x3, and the total amount of interest incurred by Clay during 20x3 was ₱102,000.
What amount should Clay report as capitalized interest at December 31, 20x3?
a. 102,000 b. 120,000 c. 150,000 d. 240,000

A (2,000,000 ÷ 2) = 1,000,000 average expenditures x 12% = 120,000;


The actual interest incurred of 102,000 is lower than the computed interest. Therefore, 102,000 is
capitalized.

55. According to PAS 23, borrowing costs that do not directly relate to the acquisition, construction
or production of a qualifying asset are
a. capitalized as cost of the qualifying asset.
b. expensed.
c. expensed, except when the borrowing costs relate to other assets.
d. any of these as a matter of accounting policy choice

56. On January 1, 20x1, Entity A started the construction of a qualifying asset. The qualifying asset is
financed through general borrowings. The average expenditures during the year amounted to
₱9,500,000. The capitalization rate is 11%. The actual borrowing costs incurred during the period
were ₱1,990,000. How much are the borrowing costs eligible for capitalization?
a. 1,990,000
b. 1,045,000
c. 1,090,000
d. 990,000

Solution:
Capitalizable BC from formula = 9,500,000 x 11% = 1,045,000
1,045,000 vs. 1,990,000 actual borrowing costs = Capitalizable BC is 1,045, 000

57. On January 1, 20x1, Entity A obtained a 12%, ₱6,000,000 loan, specifically to finance the
construction of a building. The proceeds of the loan were temporarily invested and earned
interest income of ₱180,000. The construction was completed on December 31, 20x1 for a total
construction cost of ₱7,000,000. How much is the historical cost of the newly constructed
building?
a. 7,540,000 c. 7,000,000
b. 7,480,000 d. 6,460,000

A (6M x 12%) – 180,000 = 540,000 + 7M = 7.54M

Use the following information for the next two questions:


On January 1, 20x1, Entity A had the following general borrowings. A part of the proceeds was used
to finance the construction of a qualifying asset:
Principal
P a g e | 19

12% bank loan (1.5 years) ₱ 1,000,000


10% bank loan (3-year) 8,000,000

Expenditures made on the qualifying asset were as follows:


Jan. 1 ₱ 5,000,000
March 1 4,000,000
August 31 3,000,000
December 1 2,000,000

Construction was completed on December 31, 20x1.

58. How much borrowing costs are capitalized to the cost of the constructed qualifying asset?
a. 1,045,000 c. 1,026,667
b. 971,111 d. 920,000

D Solution:
The average expenditure is computed as follows:
Month outstanding over
Date Expenditures Average expenditure
12 Months
(a) (b) (c)= (a) x (b)
Jan.1 5,000,000 12/12 5,000,000
Mar. 1 4,000,000 10/12 3,333,333
Aug. 31 3,000,000 4/12 1,000,000
Dec. 1 2,000,000 1/12 166,667
9,500,000

The capitalization rate is computed as follows:

Total interest expense on general borrowings


Capitalization rate =
Total general borrowings

Total interest expense on general borrowings


(1M x 12%) + (8M x 10%) 920,000
Divide by: Total general borrowings (1M + 8M) 9,000,000
Capitalization rate 10.22%

Capitalizable BC from formula = 9,500,000 x 10.22% = 970,900

970,900 vs. 920,000 actual borrowing costs = Capitalizable BC is 920,000, the lower amount

59. How much is the cost of the qualifying asset on initial recognition?
a. 13,010,000 c. 14,920,000
b. 15,045,000 d. 14,970,900

C (5M + 4M + 3M + 2M total expenditures on construction) + 920,000 capitalizable borrowing costs =


14,920,000
P a g e | 20

Chapter 20 - Investment property (PAS 40)


60. Counting Crow’s investment property has a carrying amount of ₱3,600,000 under the fair value
model, before adjustment. If the fair value at year-end is ₱3,000,000, how much should be the
gain or loss on transfer if Counting Crow would shift to cost model?
a. gain of ₱600,000 reported as other comprehensive income
b. loss of ₱600,000 reported as other loss in the income statement
c. loss of ₱600,000 reported in equity as decrease in revaluation surplus
d. zero

61. Which of the following is an investment property?


a. Property that is currently being redeveloped to be sold in the ordinary course of business
operations.
b. Property that is currently being developed for future use as owner-occupied.
c. Property that is leased out to another entity under a finance lease
d. Building rented out in an operating lease whereby the owner provides minimal services.

62. Entity A acquires a building for ₱1,000,000. The building is to be leased out under various
operating leases. The building has an estimated useful life of 10 years and zero residual value.
Entity A uses the cost model for its property, plant and equipment and the fair value model for
its investment property. At the end of Year 1, the building is assessed to have a fair value of
₱1,080,000. How much should Entity A recognize in profit or loss in relation to the building?
a. 80,000 gain on change in fair value
b. 100,000 depreciation
c. 180,000 gain on change in fair value
d. b and c

63. You are a member of the board of directors of ABC Co. Your company acquired a building to be
held solely for rentals. You are tasked in selecting an appropriate accounting policy for the
building. In this regard, you will most likely refer to which of the following standards?
a. PAS 17
b. PAS 39
c. PAS 40
d. PAS 41

Chapter 21 - Intangible assets (PAS 38)


64. Which of the following statements is incorrect?
a. An intangible asset acquired through the issuance of the entity’s own equity instrument
is generally valued at the fair value of the intangible asset.
b. The amortization of intangible assets involves an adjusting entry that should not be
reversed in the next accounting period.
c. Some costs of internally generating an intangible asset are treated as outright expenses.
d. All annual payments made by a franchisee to the franchisor should be capitalized as cost
of the franchise.
P a g e | 21

65. The amortization of intangible assets results primarily from the application of the
a. full-disclosure principle
b. revenue principle
c. cost principle
d. systematic and rational allocation concept

66. In compliance with the disclosure requirements of PAS 38, the amortization of an intangible
asset is recorded as a:
a. debit to retained earnings and a credit to a contra account.
b. debit to retained earnings and a credit to the intangible asset account.
c. debit to amortization expense and a credit to the intangible asset account
d. debit to amortization expense and a credit to an intangible asset contra account.

67. Which of the following confers exclusive right to conduct business in a particular territory
a. Franchise
b. Trademark
c. Patent
d. Copyright

68. The research and development expense of Soundgarden Co. includes which of the following
items?
1) Costs of advertising a newly invented product.
2) Billings received by Soundgarden from Black Hole Sun Co. for research activities
performed by Black Hole Sun Co. for Soundgarden.
3) The depreciation on a building used in various R&D projects
4) Billings sent by Soundgarden to Ugly Kid Joe Co. for research activities performed by
Soundgarden for Ugly Kid Joe Co.
5) Costs of materials, labor and overhead incurred in generating a patent. The patent was
granted to Soundgarden during the period.
6) Overhead costs properly allocated to R&D projects of Soundgarden during the period.
7) Training costs of Soundgarden’s employees who are directly involved in R&D projects.
8) The amortization of patents used in Soundgarden’s research activities.

a. 2, 3 & 6
b. 2, 3 & 8
c. 2, 3, 6 & 8
d. 2, 3, 5, 6 & 8

69. Which of the following should be expensed as incurred by the franchisee for a franchise
with an estimated useful life of ten years?
a. Legal fees paid to the franchisee's lawyers to obtain the franchise
b. Periodic payments to the franchisor based on the franchisee's revenues
c. Amount paid to the franchisor for the franchise
P a g e | 22

d. Payment to a company, other than the franchisor, for the company's franchise

70. Should the following fees associated with the registration of an internally developed patent
be capitalized?
Legal fees Registration fees
a. No No
b. No Yes
c. Yes No
d. Yes Yes

71. Which of the following assets typically are amortized?


Patents Trademarks
a. No No
b. Yes Yes
c. No Yes
d. Yes No

72. What is proper time or time period over which to match the cost of an intangible asset with
revenues if it is likely that the benefit of the asset will last for an indefinite period?
a. Forty years
b. Fifty years
c. Immediately
d. At such time as reduction in value can be quantitatively determined.

73. Which of the following statements concerning patents is correct?


a. Legal costs incurred to successfully defend an internally developed patent should be
capitalized and amortized over the patent’s remaining economic life.
b. Legal fees and other direct costs incurred in registering a patent should be capitalized
and amortized on a straight-line basis over a five-year period.
c. Research and development contract services purchased from others and used to develop
a patented manufacturing process should be capitalized and amortized over the patent’s
economic life.
d. Research and development costs incurred to develop a patented item should be
capitalized and amortized on a straight-line basis over seventeen years.
e. None of these

74. Intangible assets, other than goodwill, are accounted for under
a. PAS 38.
b. PFRS 8.
c. PAS 26.
d. PAS 20.

75. ABC Co. made expenditures for the following:


• Cost in activities aimed at obtaining new knowledge ₱10,000
P a g e | 23

• Marketing research to study consumer tastes 5,000


• Cost of developing and producing a prototype model 3,000
• Cost of testing the prototype model for safety and environmental friendliness 40,000
• Cost revising designs for flaws in the prototype model 15,000
• Salaries of employees, consultants, and technicians involved in R&D 20,000
• Cost of conference for the introduction of the newly developed product including fee of
a model hired as endorser 100,000
• Advertising to establish recognition of the newly developed product 30,000

How much is recognized as research and development expense?


a. 68,000
b. 72,000
c. 88,000
d. 94,000

Solution:

• Cost in activities aimed at obtaining new knowledge ₱10,000


• Cost of developing and producing a prototype model 3,000
• Cost of testing the prototype model for safety and
environmental friendliness 40,000
• Cost revising designs for flaws in the prototype model 15,000
• Salaries of employees, consultants, and technicians
involved in R&D 20,000
Total research and development expense ₱88,000

76. ABC Co. made expenditures for the following:


• Cost incurred on search for alternatives for materials, devices, products, processes,
systems or services ₱10,000
• Cost of final selection of possible alternatives for a new process 8,000
• Trouble-shooting during commercial production 5,000
• Periodic or routine design changes to existing products 3,000
• Modification of design for a specific customer 40,000
• Payments made to XYZ, Inc. for R&D performed by XYZ for ABC 15,000
• Cost of R&D performed by ABC for Alpha Corp. 20,000

How much is recognized as research and development expense?


a. 33,000
b. 42,000
c. 52,000
d. 53,000

Solution:
• Cost incurred on search for alternatives for materials, devices, products,
processes, systems or services ₱10,000
• Cost of final selection of possible alternatives for a new process 8,000
P a g e | 24

• Payments made to XYZ, Inc. for R&D performed by XYZ for ABC 15,000
Total research and development expense ₱33,000

Chapter 22 - Impairment of assets (PAS 36)


77. During the audit of Entity A, a construction aggregates mining company, the auditor pointed
out that the valuation of Entity A’s plant asset may be overstated. The auditor called the board
of directors’ attention to the fact that the plant asset has operated only 55% of its normal
operating hours during the year. The reason for this is that there have been frequent breakdowns
in the plant during the period. When presenting his/her audit findings to the management, the
auditor will most likely refer to this Standard.
a. PAS 2
b. PFRS 6
c. PAS 36
d. PAS 40

78. Which of the following analysis on asset impairment is most likely to have been made by a CPA?
(where: RA = recoverable amount; FVLCD = fair value less costs of disposal; VIU = value in use;
CA = carrying amount; IL = impairment loss; > = greater than; < = less than)
a. if “FVLCD > CA,” then, “IL = 0”
b. if “FVLCD < VIU,” then, IL = > 0”
c. if “FVLCD > VIU,” then, “RA = FVLCD,” now, if “CA > RA,” then “IL = RA – CA”
d. if “FVLCD > VIU,” then, “RA = VIU,” now, if “CA < RA,” then “IL = RA – CA”

Use the following information for the next two questions:


On December 31, 20x1, Entity A determines that its building is impaired. Entity A gathers the
following information:

Building 2,000,000
Accumulated depreciation 600,000
Fair value less costs of disposal (FVLCD) 900,000
Value in use (VIU) 1,080,000

79. After the impairment, the building is assessed to have a remaining useful life of six years and no
residual value. How much is the impairment loss?
a. 320,000 c. 500,000
b. 180,000 d. 270,000

A Solution:

Recoverable amount (higher of FVLCD and VIN) 1,080,000


Less: Carrying amount (2,000,000 – 600,000) (1,400,000)
Impairment loss (320,000)

80. On December 31, 20x2, Entity A determines an indication that the impairment loss recognized in
the prior period may no longer exist. The revised recoverable amount of the building on
P a g e | 25

December 31, 20x2 is ₱1,280,000. If no impairment loss had been recognized in the prior period,
the carrying amount of the building on December 31, 20x2 would have been ₱1,200,000. How
much is the gain on reversal of impairment on December 31, 20x2?
a. 314,351 c. 303,315
b. 312,156 d. 300,000

D Solution:
C.A had no imp. loss been recognized in prior pd. 1,200,000*
C.A. at date of reversal (1,080,000 x 5/6) 900,000
Gain on reversal of impairment loss (profit or loss) 300,000

*Lower than new recoverable amount.

Bonus Question
81. Who invented the calculator?
a. Jose Rizal
b. Rodrigo Duterte
c. Taylor Swift
d. Blaise Pascal
P a g e | 26

“Do not be deceived: God cannot be mocked. A man reaps what he sows.”
- Galatians 6:7

- END -
Page |1

Chapter 1
Current Liabilities
:
1. A debtor firm’s 12/31/05 balance sheet is to be published 3/1/06. An obligation with a due date of
3/4/11 is also due on demand by the creditor. At 12/31/05, there is no indication that the creditor
intends to call in the debt. The obligation is a current liability.

2. Deposits taken from customers by public utilities should always be reported as current liabilities
by the utility.

3. Since a dividend is generally paid within a month or so, it usually is classified as current.

4. All liabilities must be due within 12 months of the current balance sheet to be classified as
current liabilities.

5. A current liability may be classified as a long-term liability if the entity has the intention to
refinance it after the balance sheet date.

6. Trade notes payable are normally presented as current liabilities.

7. Unearned revenue is considered a financial liability.

8. Financial liabilities are initially measured at fair value plus direct costs, except for financial
liabilities that are classified as financial liabilities measured at fair value through profit or loss,
whose transaction costs are expensed immediately.

9. Non-financial liabilities are initially measured at the best estimate of the amounts needed to
settle those obligations or the measurement basis required by other applicable standard.

10. The fact that a liability is used to fund trading activities does not in itself make that liability one
that is held for trading.

“If you want to earn more, learn more” – Anonymous


- END -
Page |2

ANSWERS
1. TRUE 6. TRUE
2. FALSE 7. FALSE
3. TRUE 8. FALSE
4. FALSE 9. TRUE
5. FALSE 10. TRUE
Page |3

Use the following information for the next two questions:


Eliot Corporation’s liabilities at December 31, 2008 were as follows:

Accounts payable and accrued interest P 2,000,000


5-year 10% Notes payable – due December 31, 2011 5,000,000

Part of the loan agreement is for Elliot to appropriate a fixed amount out of its accumulated profits
and losses annually until the amount of appropriation has equaled the face of the obligation. Failure
to comply with the loan agreement will make the loan payable on demand. As of December 31, 2008,
Elliot Corporation has yet to comply with the loan agreement.

1. In its December 31, 2008 balance sheet, Elliot should report current liabilities at
a. 2,000,000
b. 2,500,000
c. 5,000,000
d. 7,000,000

D 2M + 5M = 7M

2. Assuming the lender agreed on December 31, 2008 to provide a grace period of 12 months for
the entity to rectify the breach and assured Elliot Corporation that no demand of payment is to
be made within the grace period, what amount of current liabilities should Elliot Corporation
report in its December 31, 2008 balance sheet?
a. 2,000,000
b. 2,500,000
c. 5,000,000
d. 7,000,000

A 2M

3. Hudson Hotel collects 15% in city sales taxes on room rentals, in addition to a ₱2 per room, per
night, occupancy tax. Sales taxes for each month are due at the end of the following month, and
occupancy taxes are due 15 days after the end of each calendar quarter. On January 3, 20x1,
Hudson paid its November 20x0 sales taxes and its fourth quarter 20x0 occupancy taxes.
Additional information pertaining to Hudson's operations is:

20x0 Room rentals Room nights


October 100,000 1,100
November 110,000 1,200
December 150,000 1,800

What amounts should Hudson report as sales taxes payable and occupancy taxes payable in its
December 31, 20x0, balance sheet?
Page |4

Sales taxes Occupancy taxes Sales taxes Occupancy taxes


a. ₱39,000 ₱6,000 c. ₱54,000 ₱6,000
b. ₱39,000 ₱8,200 d. ₱54,000 ₱8,200

B
Solution:
20x0 Room rentals Room nights
October - 1,100
November 110,000 1,200
December 150,000 1,800
Total 260,000 4,100
Multiply by: Tax 15% 2
Total 20x0 unpaid taxes 39,000 8,200

Use the following information for the next two questions:


BUGS Appliance Company’s accountant has been reviewing the firm’s past television sales. For the
past years, BUGS has been offering a special service warranty on all televisions sold. With the
purchase of a television, the customer has the right to purchase a 3-year service contract for an extra
P600.

Information concerning past television and warranty contract sales is given below:

2007 2006
Television sales in units 550 460
Sales price per unit P5,000 P4,000
Number of service contracts sold 350 300
Expenses relating to television warranties 38,520 13,400

BUGS’ accountant has estimated from past records that the pattern of repairs has been 40% in the
year of sale, 36% first year after sale and 24% on 2nd year of sale. Sales of the contracts are made
evenly during the year.

4. What is the adjusted balance of the unearned service contract as of December 31, 2007?
a. 111,600
b. 168,600
c. 211,200
d. 243,600

D
2007 2006
Number of service contracts sold 350 300
Price per contract 600 600
Total 210,000 180,000
Divide by: (*sold 'evenly') 2 2
Page |5

Totals 105,000 90,000

From 2007: 2007 2008 2009 2010


Percentages earned in each period 40% 36% 24%
First 105,000 assumed to have been sold at the beg. of
the pd.: (105K x 40%; x 36%; x 24%) 42,000 37,800 25,200

Percentages earned in each period 0% 40% 36% 24%


Remaining 105,000 assumed to have been sold at the
end of the pd.: (105K x 0%; 40%; x 36%; x 24%) - 42,000 37,800 25,200

From 2006: 2006 2007 2008 2009 2010


Percentages earned in each period 40% 36% 24%
First 90,000 assumed to have been sold at the beg. of
the pd.: (105K x 40%; x 36%; x 24%) 36,000 32,400 21,600

Percentages earned in each period 0% 40% 36% 24%


Remaining 90,000 assumed to have been sold at the end
of the pd.: (105K x 0%; 40%; x 36%; x 24%) 36,000 32,400 21,600

Total earned portions (2006 & 2007 contracts) 36,000 110,400 133,800 84,600 25,200

Earned portion in:


2008 133,800
2009 84,600
2010 25,200
Total unearned portion as of Dec. 31, 2007 243,600

5. How much profit on service contract would be recognized in year 2007?


a. 42,000
b. 68,400
c. 71,880
d. 110,400

C
Earned portion in 2007 110,400
Expenses relating to television warranties (38,520)
Profit 71,880

“Pride goes before destruction, a haughty spirit before a fall.” (Proverbs 16:18)
- END -
Page |1

Chapter 2
Notes Payable
1. On August 1, 20x1, an entity acquired a new equipment that it does not have to pay for until
September 1, 20x5. The total payment on September 1, 20x5, will include both principal and
interest. The initial measurement of the note and the equipment is
a. payment for the principal multiplied by Present value of ₱1
b. payment for interest multiplied by Present value of ordinary annuity of ₱1
c. a plus b
d. total payment on the note multiplied by Present value of ₱1

2. Which of the following represents a liability?


a. The obligation to pay interest on a five-year note payable that was issued the last day of the
current year.
b. The obligation to pay for goods that a company expects to order from suppliers next year.
c. The obligation to provide goods that customers have ordered and paid for during the
current year.
d. The obligation to distribute share of a company's own common stock next year as a result of
a stock dividend declared near the end of the current year.

3. Interest expenses are incurred


a. only on interest-bearing liabilities
b. only on liabilities which are discounted to their present values
c. only on liabilities which are initially and subsequently measured at amortized cost
d. only due to passage of time

4. When interest expense is calculated using the effective-interest amortization method, interest
expense (assuming that interest is paid annually) always equals the
a. actual amount of interest paid.
b. carrying amount of the note multiplied by the stated interest rate.
c. carrying amount of the note multiplied by the effective interest rate.
d. maturity value of the note multiplied by the effective interest rate.

5. Loan origination fees are


a. added to the carrying amount of the loan payable and subsequently amortized using the
straight-line method.
b. recognized immediately as income
c. added to the carrying amount of the loan payable and subsequently amortized using the
effective interest method.
d. deducted from the carrying amount of the loan payable and subsequently amortized using
the effective interest method.

6. On March 1, 20X4, Fine Co. borrowed ₱10,000 and signed a two-year note bearing interest at 12%
per annum compounded annually. Interest is payable in full at maturity on February 28, 20X6.
What amount should Fine report as a liability for accrued interest at December 31, 20X5?
Page |2

a. 0 b. 1,000 c. 1,200 d. 2,320

D
Solution:
Interest expense in 20x4 (10,000 x 12% x 10/12) 1,000
Interest expense in 20x5 [(10,000 + 1,000) x 12%] 1,320
Interest payable (compounded) - 12/31/x5 2,320

7. On December 30, 20X6, Bart, Inc., purchased a machine from Fell Corp. in exchange for a
noninterest bearing note requiring eight payments of ₱20,000. The first payment was made on
December 30, 20X6, and the others are due annually on December 30. At the date of issuance, the
prevailing rate of interest for this type of note was 11%. On Bart's December 31, 20X6, balance
sheet, the note payable to Fell was
a. 94,240 b. 102,920 c. 104,620 d. 114,240

A
Solution:
Cash flow 20,000
PV of annuity due of 1 @11%, n=8 5.712
PV of note on Dec. 30, 20x6 114,240
Less: First installment on Dec. 31, 20x6 (20,000)
PV of note on Dec. 31, 20x6 94,240

The next two items are based on the following information:


House Publishers offered a contest in which the winner would receive ₱1,000,000, payable over 20
years. On December 31, 2000, House announced the winner of the contest and signed a note payable
to the winner for ₱1,000,000, payable in ₱50,000 installments every January 2. Also on December 31,
2000, House purchased an annuity for ₱418,250 to provide the ₱950,000 prize monies remaining after
the first ₱50,000 installment, which was paid on January 2, 2001.

8. In its December 31, 20x0, balance sheet, what amount should House report as note payable-
contest winner, net of current portion?
a. 368,250 b. 418,250 c. 900,000 d. 950,000

B 418,250 – the cash price equivalent of the annuity purchased.

9. In its 20x0 income statement, what should House report as contest prize expense?
a. 0 b. 418,250 c. 468,250 d. 1,000,000

C (418,250 + 50,000 first payment made immediately) = 468,250 total contest prize expense

10. On December 1, 20x5, Money Co. gave Home Co. a ₱200,000, 11% loan. Money paid proceeds of
₱194,000 after the deduction of a ₱6,000 nonrefundable loan origination fee. Principal and
interest are due in 60 monthly installments of ₱4,310, beginning January 1, 20x6. The repayments
yield an effective interest rate of 11% at a present value of ₱200,000 and 12.4% at a present value
of ₱194,000. What amount of income from this loan should Money report in its 20x5 income
statement?
a. 0 b. 1,833 c. 2,005 d. 7,833
Page |3

C (194,000 x 12.4% x 1/12) = 2,005

“He who obeys instructions guards his life, but he who is contemptuous of his ways
will die.” (Proverbs 19:16)

- END -
Page |1

Chapter 3
Bonds Payable & Other Concepts
1. The result on the year-end balance sheet of an issue of a 10-year term bond sold at face amount
four years ago with interest payable June 1 and December 1 each year, is a(an)
a. liability for accrued interest c. increase in deferred charges
b. addition to bonds payable d. contingent liability

2. Unamortized bond discount should be reported on the financial statements of the issuer as a
a. Direct deduction from the face amount of the bond
b. Direct deduction from the present value of the bond
c. Deferred charge
d. Part of the issue costs

3. Straight-line amortization of bond premium or discount:


a. can be used as an optional method of amortization in all situations.
b. provides the same total amount of interest expense and interest revenue as the effective
interest method over the life of the bonds.
c. provides the same amounts of interest expense and interest revenue each interest period as
the effective interest method.
d. is appropriate when the bond term is especially long.
e. is appropriate for deep discount bonds.

4. For a bond issue which sells for less than its face amount, the market rate of interest is
a. Dependent on the rate stated on the bond.
b. Equal to rate stated on the bond.
c. Less than rate stated on the bond.
d. Higher than rate stated on the bond.

5. The market price of a bond issued at a discount is the present value of its principal amount at the
market (effective) rate of interest
a. Less the present value of all future interest payments at the market (effective) rate of interest.
b. Less the present value of all future interest payments at the rate of interest stated on the
bond.
c. Plus the present value of all future interest payments at the market (effective) rate of interest.
d. Plus the present value of all future interest payments at the rate of interest stated on the
bond.

6. Which of the following is not a relevant consideration when evaluating whether to derecognize
a financial liability?
a. Whether the obligation has been discharged.
b. Whether the obligation has been canceled.
c. Whether the obligation has expired.
d. Whether substantially all the risks and rewards of the obligation have been transferred.
Page |2

7. What is the effective interest rate of a bond or other debt instrument measured at amortized
cost?
a. The stated coupon rate of the debt instrument.
b. The interest rate currently charged by the entity or by others for similar debt instruments
(i.e., similar remaining maturity, cash flow pattern, currency, credit risk, collateral, and
interest basis).
c. The interest rate that exactly discounts estimated future cash payments or receipts through
the expected life of the debt instrument or, when appropriate, a shorter period to the net
carrying amount of the instrument.
d. The basic, risk-free interest rate that is derived from observable government bond prices.

8. Which of the following statements is false?


a. Bonds carry no corporate ownership privileges.
b. A bond is a financial contract.
c. Bond prices remain fixed over time.
d. A bond issuer must pay periodic interest.

9. Most bonds:
a. are money market securities.
b. are floating-rate securities.
c. give bondholders a voice in the affairs of the corporation.
d. are interest-bearing obligations of governments or corporations.

10. In an “asset swap,” where a liability is settled through the transfer of noncash asset,
a. the gain or loss on settlement is computed as the difference between the carrying amount of
the liability extinguished and the fair value of the noncash asset transferred.
b. the gain or loss on settlement is computed as the difference between the carrying amount of
the liability extinguished and the carrying amount of the noncash asset transferred.
c. the gain or loss on settlement is computed as the difference between the carrying amount of
the liability extinguished and the more clearly determinable between the fair value of the
liability extinguished and the carrying amount of the noncash asset transferred.
d. no gain or loss is recognized

“There is a time for everything, and a season for every activity under the
heavens;” (Ecclesiastes 3:1)

- END –
Page |3

1. On January 2, 20x1, Nast Co. issued 8% bonds with a face amount of ₱1,000,000 that mature on
January 2, 20x7. The bonds were issued to yield 12%, resulting in a discount of ₱150,000. Nast
incorrectly used the straight-line method instead of the effective interest method to amortize the
discount. How is the carrying amount of the bonds affected by the error?

At Dec. 31, 20x1 At Jan. 2, 20x7 At Dec. 31, 20x1 At Jan. 2, 20x7
a. Overstated Understated c. Understated Overstated
b. Overstated No effect d. Understated No effect

B
Solution:
EFFECT ON DECEMBER 31, 20X1:
Using straight line method:
Discount on bonds - 1/2/x1 150,000
Divide by: Term 6
Annual amortization of discount 25,000

Discount on bonds - 1/2/x1 150,000


Amortization - 20x1 (25,000)
Discount on bonds - 12/31/x1 125,000

Face amount 1,000,000


Discount on bonds - 12/31/x1 (125,000)
Carrying amount - 12/31/x1 875,000

Using effective interest method:


Date Interest expense Payments Amortization Present Value
1/2/x1 850,000
12/31/x1 102,000 80,000 22,000 872,000

Carrying amounts - 12/31/x1:


Straight line (erroneous) 875,000
Effective interest method 872,000
Difference - overstatement (3,000)

EFFECT ON JANUARY 2, 20X7:


On January 2, 20x7, maturity date, there will be NO EFFECT of the error on the carrying amount of
the bonds because on this date, the discount would have been fully amortized under both the
straight line method and the effective interest method.
Page |4

2. On July 1, 2003, after recording interest and amortization, York Co. converted ₱1,000,000 of its
12% convertible bonds into 50,000 shares of ₱1 par value ordinary share. On the conversion date
the carrying amount of the bonds was ₱1,300,000, the fair value of the bonds was ₱1,400,000, and
York’s ordinary share was publicly trading at ₱30 per share. What amount of share premium
should York record as a result of the conversion?
a. 950,000 b. 1,250,000 c. 1,350,000 d. 1,500,000

B
Solution:
Carrying amount of bonds converted 1,300,000
Par value of shares issued (50,000 x 1) (50,000)
Share premium 1,250,000

3. On April 30, 20x5, Witt Corp. had outstanding 8%, ₱1,000,000 face amount, convertible bonds
maturing on April 30, 20x9. Interest is payable on April 30 and October 31. On April 30, 20x5, all
these bonds were converted into 40,000 shares of ₱20 par ordinary share. On the date of
conversion:
• Unamortized bond discount was ₱30,000.
• Each bond had a fair value of ₱1,080.
• Each share of stock had a fair value of ₱28.

What amount should Witt record as a loss on conversion of bonds?


a. 150,000 b. 110,000 c. 30,000 d. 0

D – No gain or loss is recognized when convertible bonds are converted into equity instrument.

4. Ray Corp. issued bonds with a face amount of ₱200,000. Each ₱1,000 bond contained detachable
stock warrants for 100 shares of Ray's common stock. Total proceeds from the issue amounted to
₱240,000. The fair value of each warrant was ₱2, and the fair value of the bonds without the
warrants was ₱196,000. The bonds were issued at a discount of
a. 0 b. 678 c. 4,000 d. 33,898
C
Solution:
Fair value of bonds without the warrants 196,000
Face amount of bonds 200,000
Discount on bonds (4,000)

5. On June 30, 20x9, King Co. had outstanding 9%, ₱5,000,000 face value bonds maturing on June
30, 2x14. Interest was payable semiannually every June 30 and December 31. On June 30, 20x9,
after amortization was recorded for the period, the unamortized bond premium and bond issue
costs were ₱30,000 and ₱50,000, respectively. On that date, King acquired all its outstanding
bonds on the open market at 98 and retired them. At June 30, 20x9, what amount should King
recognize as gain on redemption of bonds?
a. 20,000 b. 80,000 c. 120,000 d. 180,000
Page |5

B
Solution:
Redemption price (5M x 98%) 4,900,000
Less: Carrying amount of bonds:
Face amount 5,000,000
Unamortized premium 30,000
Unamortized issue costs (50,000) 4,980,000
Gain on retirement 80,000

6. On July 31, 20x0, Dome Co. issued ₱1,000,000 of 10%, 15-year bonds at par and used a portion of
the proceeds to call its 600 outstanding 11%, ₱1,000 face value bonds, due on July 31, 2x10, at
102. On that date, unamortized bond premium relating to the 11% bonds was ₱65,000. In its 20x0
income statement, what amount should Dome report as gain or loss, before income taxes, from
retirement of bonds?
a. 53,000 gain b. 0 c. (65,000) loss d. (77,000) loss

A
Solution:
Redemption price (600 x 1,000 x 102%) 612,000
Less: Carrying amount of bonds:
Face amount (600 x 1,000) 600,000
Unamortized premium 65,000 665,000
Gain on retirement 53,000

7. During 20x4 Peterson Company experienced financial difficulties and is likely to default on a
₱500,000, 15%, three-year note dated January 1, 20X2, payable to Forest National Bank. On
December 31, 20X4, the bank agreed to settle the note and unpaid interest of ₱75,000 for 20X4 for
₱50,000 cash and marketable securities having a carrying amount of ₱375,000. Peterson's
acquisition cost of the securities is ₱385,000. What amount should Peterson report as a gain from
the debt restructuring in its 20x4 income statement?
a. 65,000 b. 75,000 c. 140,000 d. 150,000

D
Solution:
Payment for the liability:
Cash 50,000
Carrying amount of investment securities 375,000 425,000
Carrying amount of liability settled:
Principal 500,000
Accrued interest 75,000 575,000
Gain on settlement 150,000

8. Casey Corporation entered into a troubled-debt restructuring agreement with First State Bank.
First State agreed to accept land with a carrying amount of ₱85,000 and a fair value of ₱120,000
Page |6

in exchange for a note with a carrying amount of ₱185,000. What amount should Casey report as
gain in its income statement?
a. 0 b. 35,000 c. 65,000 d. 100,000

D (185,000 carrying amt. of note - 85,000 carrying amt. of land) = 100,000 gain

9. Wood Corp., a debtor undergoing financial difficulties granted an equity interest to a creditor in
full settlement of a ₱28,000 debt owed to the creditor. At the date of this transaction, the equity
interest had a fair value of ₱25,000 and par value of ₱20,000. What amount should Wood
recognize as gain on restructuring of debt?
a. 0 b. 3,000 c. 5,000 d. 8,000

B (28,000 – 25,000) = 3,000

10. In 20X2, May Corp. acquired land by paying ₱75,000 down and signing a note with a maturity
value of ₱1,000,000. On the note’s due date, December 31, 20X7, May owed ₱40,000 of accrued
interest and ₱1,000,000 principal on the note. May was in financial difficulty and was unable to
make any payments. May and the bank agreed to amend the note as follows:
• The ₱40,000 of interest due on December 31, 20X7, was forgiven.
• The principal of the note was reduced from ₱1,000,000 to ₱950,000 and the maturity date
extended 1 year to December 31, 20X8.
• May would be required to make one interest payment totaling ₱30,000 on December 31,
20X8.
• The original effective interest rate is 10% while the current market rate on December 31, 20X7
is 12%.

As a result of the troubled debt restructuring, May should report a gain, before taxes, in its 20X7
income statement of
a. 0 b. 165,000 c. 60,000 d. 149,092

D
Solution:
The modification is analyzed as follows:
Old terms New terms
Principal 1,000,000 950,000
Accrued interest 40,000 30,000
Remaining term ('n') 1 year

The present value of the modified liability is computed as follows:


Future cash flows PV of 1 @10%, n=1 Present value
Principal 950,000 0.90909 863,636
Interest 30,000 0.90909 27,273
Present value of the modified liability 890,908

The difference between the old liability and the new liability is tested for substantiality.
Carrying amount of old liability
1,040,000
(1M principal + 40,000 accrued interest)
Present value of modified liability 890,908
Page |7

Difference 149,092

Difference 149,092
Divide by: Carrying amount of old liability 1,040,000
14.34%

The modification is considered substantial because the modification resulted to a present value of the
new obligation different by at least 10% of the present value (carrying amount) of old obligation.
Therefore, the old liability is extinguished and the difference of ₱149,092 is recognized as gain on
extinguishment.

“Blessed are the pure in heart, for they will see God.” (Matthew 5:8)
- END –
Page |8

Difference 149,092
Divide by: Carrying amount of old liability 1,040,000
14.34%

The modification is considered substantial because the modification resulted to a present value of the
new obligation different by at least 10% of the present value (carrying amount) of old obligation.
Therefore, the old liability is extinguished and the difference of ₱149,092 is recognized as gain on
extinguishment.
Page |1

Chapter 4
Provisions, Contingent Liabilities & Contingent Assets
1. In 20x1, an entity, a defendant, recognized a ₱100,000 provision for estimated losses from a
pending lawsuit. In 20x2, the entity wins the lawsuit and pays nothing. Which of the following
statements is correct in 20x2?
a. The entity shall restate its 20x1 financial statements to remove the provision.
b. The entity shall recognize a gain of ₱100,000 in 20x2.
c. The entity shall recognize a ₱100,000 deduction in its 20x2 expenses.
d. The entity shall only disclose the event in its 20x2 financial statements.

2. The board of directors of ABC Inc. decided on December 15, 20XX, to wind up international
operations in the Far East and move them to Australia. The decision was based on a detailed
formal plan of restructuring as required by PAS 37. This decision was conveyed to all workers
and management personnel at the headquarters in Europe. The cost of restructuring the
operations in the Far East as per this detailed plan was P2 million. How should ABC Inc. treat
this restructuring in its financial statements for the year-end December 31, 20XX?
a. Because ABC Inc. has not announced the restructuring to those affected by the decision and
thus has not raised an expectation that ABC Inc. will actually carry out the restructuring
(and as no constructive obligation has arisen), only disclose the restructuring decision and
the cost of restructuring of ₱2 million in footnotes to the financial statements.
b. Recognize a provision for restructuring since the board of directors has approved it and it
has been announced in the headquarters of ABC Inc. in Europe.
c. Mention the decision to restructure and the cost involved in the chairman’s statement in the
annual report since it a decision of the board of directors.
d. Because the restructuring has not commenced before year-end, based on prudence, wait
until next year and do nothing in this year’s financial statements.

3. Which of the following is the proper way to report a gain contingency?


a. As an accrued amount.
b. As deferred revenue.
c. As an account receivable with additional disclosure explaining the nature of the
contingency.
d. As a disclosure only.

4. Which of the following contingencies need not be disclosed in the financial statements or the
notes thereto?
a. Probable losses not reasonably estimable
b. Environmental liabilities that cannot be reasonably estimated
c. Guarantees of indebtedness of others
d. All of these must be disclosed.

5. To record an asset retirement obligation (ARO), the cost associated with the ARO is
a. expensed.
Page |2

b. included in the carrying amount of the related long-lived asset.


c. included in a separate account.
d. none of these.

6. In December 20x1, Mill Co. began including one coupon in each package of candy that it sells
and offering a toy in exchange for 50 centavos and five coupons. The toys cost Mill 80 centavos
each. Eventually 60% of the coupons will be redeemed. During December, Mill sold 110,000
packages of candy and no coupons were redeemed. In its December 31, 20x1, balance sheet,
what amount should Mill report as estimated liability for coupons?
a. 3,960
b. 10,560
c. 19,800
d. 52,800

A
Solution:
Liability for unredeemed coupon
Actual cost of
toys given out - 3,960 Premium expense *
3,960

The premium expense is computed as follows:


No. of packages sold 110,000
Multiply by: Estimate of redemption 60%
Total 66,000
Divide by: No. of coupons for each toy offering 5
Estimated no. of toys to be given out 13,200
Multiply by: Net cost per toy (.80 - .50) 0.30
Premium expense 3,960

7. In May 20x6, Caso Co. filed suit against Wayne, Inc. seeking ₱1,900,000 damages for patent
infringement. A court verdict in November 20x9 awarded Caso ₱1,5000,000 in damages, but
Wayne’s appeal is not expected to be decided before 2x10. Caso’s counsel believes it is probable
that Caso will be successful against Wayne for an estimated amount in the range between
₱800,000 and ₱1,100,000, with ₱1,000,000 considered the most likely amount. What amount
should Caso record as income from the lawsuit in the year ended December 31, 20x9?
a. 0
b. 800,000
c. 1,000,000
d. 1,500,000

A - A contingent gain that is probable is disclosed only.

8. During 20x0, Smith Co. filed suit against West, Inc. seeking damages for patent infringement. At
December 31, 20x0, Smith's legal counsel believed that it was probable that Smith would be
Page |3

successful against West for an estimated amount in the range of ₱75,000 to ₱150,000, with all
amounts in the range considered equally likely. In March 20x1, Smith was awarded ₱100,000 and
received full payment thereof. In its 20x0 financial statements, issued in February 20x1, how
should this award by reported?
a. As a receivable and revenue of ₱100,000.
b. As a receivable and deferred revenue of ₱100,000.
c. As a disclosure of a contingent gain of ₱100,000.
d. As a disclosure of a contingent gain of an undetermined amount in the range of ₱75,000 to
₱150,000.

D The event is still considered a contingent gain with only a range of estimated amounts as at December
31, 20x0 because the actual award occurred on March 20x1, after the financial statements has been
issued on February 20x1.

9. During January 20x9, Haze Corp. won a litigation award for ₱15,000 which was tripled to
₱45,000 to include punitive damages. The defendant, who is financially stable, has appealed only
the ₱30,000 punitive damages. Haze was awarded ₱50,000 in an unrelated suit it filed, which is
being appealed by the defendant. Counsel is unable to estimate the outcome of these appeals. In
its 20x9 financial statements, Haze should report what amount of pretax gain?
a. 15,000
b. 45,000
c. 50,000
d. 95,000

A
Solution:
Litigation award 45,000
Less: Amount appealed (no estimate of outcome) (30,000)
Total 15,000
Litigation award 50,000
Less: Amount appealed (no estimate of outcome) (50,000)
Total -
Gain 15,000

10. National Appliance Center sells washing machines that carry a three-year warranty against
manufacturer's defects. Based on company experience, warranty costs are estimated at ₱60 per
machine. During the year, National sold 48,000 washing machines and paid warranty costs of
₱340,000. In its income statement for the year ended December 31, National should report
warranty expense of
a. 680,000 b. 960,000 c. 2,200,000 d. 2,880,000

D (60 x 48,000) = 2,880,000

"So, if you think you are standing firm, be careful that you don’t fall! No
temptation has overtaken you except what is common to mankind. And God is
Page |4

faithful; he will not let you be tempted beyond what you can bear. But when
you are tempted, he will also provide a way out so that you can endure it."
(1 Corinthians 10:12-13)
- END -
Page |1

Chapter 5
Employee Benefits Part 1
1. It refers to a plan where plan assets, if any, are retained and managed by the employer.
a. Funded plan c. Unfunded plan
b. Non-contributory plan d. Delicate plan

2. These are pool of assets contributed by various unrelated employers to be used to pay retirement
benefits to participants without regard to the identity of the contributing employers.
a. Multi-employer plans c. Pooling of assets plan
b. State plans d. Secret plan

3. Multi-employer plans are treated as


a. Defined contribution plan c. Hybrid plan
b. Defined benefit plan d. a or b

4. These are established by legislation and are operated by a government agency which is not
subject to control or influence by the reporting entity.
a. State plans b. SSS c. GSIS d. Puro plan

5. State plans are


a. accounted for as defined contribution plan
b. accounted for as defined benefit plan
c. accounted for in the same way as multi-employer plans
d. accounted for only by the Commission on Audit

6. The accounting for defined contribution plan


a. is straightforward – actuarial computations are not required.
b. is complex – actuarial computations are required
c. is simple – not accounted for
d. is done only by CPAs

7. Under a defined contribution plan, the retirement benefits expense is


a. equal to an actuarially determined amount
b. equal to the agreed periodic contribution to the fund
c. equal to the contribution made during the period
d. zero, if no employee retired during the period

8. Employee benefits are all forms of consideration given by an entity in exchange for service
rendered by employees. Which of the following employee benefits is not within the scope of PAS
19?
a. Short-term d. Termination
b. Post-employment e. Share-based payments
c. Other long-term
Page |2

9. Which of the following employee benefits is not within the scope of PAS 19?
a. Semi-monthly salaries of employees
b. Employer’s share in SSS contributions
c. One sack rice allowance
d. Bonus in the form the entity’s shares

10. Accumulating compensated absences are those that


a. can be carried over to the next period if not fully used during the year of entitlement.
b. expire if not fully used during the year of entitlement.
c. can be carried over to the next period if not fully used during the year of entitlement and are
paid in cash when the employee leaves the company
d. are recognized only when actually taken by employees

"…Have I not commanded you? Be strong and courageous. Do not be frightened,


and do not be dismayed, for the Lord your God is with you wherever you go."
(Joshua 1:9)
Page |3

ANSWERS
1. C 6. A
2. A 7. B
3. D 8. E
4. A 9. D
5. C 10. A
Page |4

1. The last payday for a firm was December 27 on which it paid ₱40,000 to its employees, the
amount earned by employees through the pay period ending December 16. For the period
December 17 through December 31, the employees earned ₱12,000. The adjusting entry required
at December 31 would include:
a. cr. crash ₱12,000
b. dr. wages payable ₱12,000
c. dr. wages expense ₱12,000
d. dr. wages expense ₱40,000

2. Gavin Co. grants all employees two weeks of paid vacation for each full year of employment.
Unused vacation time can be accumulated and carried forward to succeeding years and will be
paid at the salaries in effect when vacations are taken or when employment is terminated. There
was no employee turnover in 20X6. Additional information relating to the year ended December
31, 20X6, is as follows:

Liability for accumulated vacations at 12/31/X5 ₱35,000


Pre-20X6 accrued vacations taken from 1/1/X6 to 9/30/X6
(the authorized period for vacations) 20,000
Vacations earned for work in 20X6 (adjusted to current rates) 30,000

Gavin granted a 10% salary increase to all employees on October 1, 20X6, its annual salary increase
date. For the year ended December 31, 20X6, Gavin should report vacation pay expense of
a. 45,000 b. 33,500 c. 31,500 d. 30,000

C
Solution:
Liability for accumulated vacations at 12/31/X5 35,000
Pre-20X6 accrued vacations taken from 1/1/X6 to 9/30/X6 (20,000)
Liability to be carried over to the next period 15,000
Multiply by: Increase in salary level in Oct. 20x6 10%
Additional liability due to the increase in salary level 1,500
Vacations earned in 20X6 (adjusted to current rates) 30,000
Vacation pay expense in 20x6 31,500

3. ANOMALOUS IRREGULAR Co. grants its employees twelve days paid vacation leave each
year. Per ANOMALOUS’s policy, employees are required to take vacation leave each year, but
not necessarily for their entire vacation leave entitlement. Vacation leaves not taken during a
year can be carried over indefinitely.

ADHERE has 500 employees with an average salary of ₱4,000 per day. The average annual pay
increase is 5%. During 20x1, total vacation leaves taken by employees were 5,400 days. Based on past
experience, 90% of unused vacation leave for a year are taken in the immediately following year.
Page |5

If unused vacation leaves vest, how much should ANOMALOUS accrue as liability for unused
vacation leave on December 31, 20x1?
a. 2,520,000 b. 25,200,000 c. 2,268,000 d. 0

A
Solution:
Total vacation leaves entitlement of employees in 20x1
6,000
(500 employees x 12 days each)
Vacation leaves taken in 20x1 (5,400)
Unused vacation leave carried over indefinitely 600
Multiply by: Expected pay rate in 20x2 (₱4,000 x 105%*) 4,200
Liability for unused vacation leaves 2,520,000

*100% + Average annual pay increase is 5%.

Use the following information for the next four questions:


ADHERE TO STICK Co. grants its managerial employees bonus in the form of profit sharing.
Information on operations in 20x1 is shown below:
Profit before tax ₱4,000,000
Bonus rate or percentage 10%
Income tax rate 30%

4. How much is the bonus “before bonus and before tax?”


a. 363,636 b. 280,000 c. 400,000 d. 288,660

C
Solution:
B = P x Br
B = 4,000,000 x 10%
B = 400,000

5. How much is the bonus “after bonus and before tax?”


a. 400,000 b. 363,636 c. 261,684 d. 245,798

B
Solution:
P
B = P -
1 + Br
4,000,000
B = 4,000,000 -
1 + 10%
B = 4,000,000 - 3,636,364
B = 363,636

6. How much is the bonus “before bonus and after tax?”


a. 363,636 b. 261,684 c. 245,798 d. 288,660
Page |6

D
Solution:
1 - Tr
B = P x
1/Br - Tr
1 - 30%
B = 4,000,000 x
1/10% - 30%
70%
B = 4,000,000 x
10 - 30%
70%
B = 4,000,000 x
9.7
B = 288,660

7. How much is the bonus “after bonus and after tax?”


a. 363,636 b. 261,682 c. 245,798 d. 288,660

B
Solution:
1 – Tr
B = P X
1/Br - Tr + 1
70%
B = 4,000,000 x
10 - 30% + 1
70%
B = 4,000,000 x
10.7
B = 261,682

8. ARTIFACT MAN MADE OBJECT Co. provides an incentive compensation plan under which its
president receives a bonus equal to 10% of ARTIFACT’s profit before tax but after deduction of
the bonus. ARTIFACT’s profit after tax and after bonus for the year is ₱2,545,456. Income tax rate
is 30%. How much is the bonus?
a. 245,798 b. 261,684 c. 363,636 d. 288,660

C
Solution:
Squeeze upwards
Profit before bonus and before tax 4,000,000
Bonus before tax but after bonus
(3,636,366 x 10%) (363,636)
Profit before tax but after bonus
(2,545,456 ÷ 70%) 3,636,366
Income tax (2,545,456 ÷ 70%) x 30% (1,090,909)
Profit after tax and after bonus 2,545,456 Start

Use the following information for the next two questions:


Page |7

AMNESTY PARDON Co. has a post-employment benefits plan that is considered as defined
contribution plan. According to the plan, AMNESTY agrees to contribute ₱800,000 annually to a
retirement fund for the benefit of its employees.

On December 31, 20x1, because of poor results of operations and insufficient working capital,
AMNESTY was only able to contribute ₱320,000 to the fund. On December 31, 20x2, because of a
profitable year, AMNESTY decided to contribute ₱1,800,000 to the retirement fund. On January 12,
20x3, an employee retired and was eligible to a ₱60,000 retirement benefits based on the operating
efficiency and investment earnings of the fund.

9. How much is the retirement benefits expense recognized in 20x2?


a. 800,000 b. 320,000 c. 1,800,000 d. 60,000

10. How much is the retirement benefits expense recognized in 20x3?


a. 800,000 b. 320,000 c. 1,800,000 d. 60,000

“A wise son heeds his father’s instruction, but a mocker does not respond
to rebukes.” - (Proverbs 13:1)
- END -
Page |1

Chapter 6
Employee Benefits Part 2
1. Which of the following components should be included in the calculation of net defined benefit
cost recognized for a period by an employer sponsoring a defined benefit pension plan?

Actual Return Amortization of


on Plan Assets, Unrecognized Prior Interest
If Any Service cost, If Any Cost

a. No No Yes
b. Yes No Yes
c. Yes Yes No
d. Yes Yes Yes

2. Which of the following concepts for postretirement benefit plans is comparable to the projected
unit credit method of pension plans?
a. Accrued benefit method pro-rated on service
b. Expected Postretirement Benefit Obligation (EPBO)
c. Actual return on plan assets
d. Expected return on plan assets

3. Which of the following statements is incorrect?


a. Minimum (corridor) amortization of net unrecognized gain or loss is allowed for
postretirement benefit plans.
b. Gains and losses on settlement of defined benefit retirement plans are recognized
immediately.
c. Actuarial gains and losses are recognized immediately.
d. Past service costs are recognized immediately.

4. The interest cost component of the net defined benefit cost is determined using
a. the settlement rate of interest.
b. the rate of return on high quality corporate bonds
c. both a and b.
d. neither a or b.

5. Financial reporting standards for pension currently in effect


a. allow both the accrued benefit and projected benefit methods.
b. allow only the accrued benefit method/ projected unit credit method.
c. allow only the projected benefit method.
d. do not allow either the accrued benefit or projected benefit methods.

6. Which of the following is not correct?


a. PAS 19 does not include any provisions for the recognition of an additional minimum
Page |2

liability.
b. PAS 19 does not allow for the recognition of a net pension asset equal to the computed
surplus in some circumstances.
c. PAS 19 requires the 10% corridor amount in calculating the amortization of deferred gains
and losses.
d. PAS 19 requires settlement gains and losses to be recognized immediately as part of
comprehensive income.

7. These are changes in the present value of the defined benefit obligation resulting from
experience adjustments and the effects of changes in actuarial assumptions.
a. Past service cost c. Settlement gains and losses
b. Actuarial gains and losses d. Interest cost

8. All of the following are demographic assumptions except:


a. future medical costs
b. mortality, both during and after employment
c. rates of employee turnover, disability and early retirement
d. claim rates under medical plans

9. According to PAS 19, which of the following is not a financial assumption?


a. the discount rate
b. future salary and benefit levels
c. the expected rate of return on plan assets
d. the proportion of plan members with dependents who will be eligible for benefits

10. According to PAS 19, the rate used to discount post-employment benefit obligations shall be
determined by reference to market yields at the end of the reporting period on
a. risk-free rate c. current bank rate
b. high quality corporate bonds d. effective interest rate

“Education is the passport to the future, for tomorrow belongs to those who prepare for
it today.” - Malcolm X
- END -
Page |3

1. The following information relates to the defined benefit pension plan of the McDonald
Company for the year ending December 31, 2002:

PV of defined benefit obligation, January 1 ₱4,600,000


PV of defined obligation, December 31 4,729,000
Fair value of plan assets, January 1 5,035,000
Fair value of plan assets, December 31 5,565,000
Interest income on plan assets 450,000
Actuarial loss 32,500
Employer contributions 425,000
Benefits paid to retirees 390,000
Discount rate 10%

The net amount of remeasurement of the net defined benefit liability (asset) included in the defined
benefit cost for 2002 would be
a. 77,500. b. 47,500. c. 32,500. d. 12,500.

D
Solution:

Remeasurements of the net defined benefit liability (asset):


(a) Actuarial (gain) loss 32,500
(b) Difference between interest income on plan assets
and return on plan assets (450,000 - 495,000) (45,000)
(c) Difference between the interest on the effect of the asset
ceiling and the change in the effect of the asset ceiling -
Defined benefit cost recognized in OCI 12,500

Fair value of plan assets


Jan. 1 5,035,000
Return on plan assets 495,000 390,000 Benefits paid
Contributions to the fund 425,000
5,565,000 Dec. 31

2. Flash Inc. has a defined benefit plan for its employees. The following information relates to this
plan:

Present value of defined benefit obligation, January 1, 2002 10,000,000


Fair value of plan assets, January 1, 2002 10,400,000
Service cost - 2002 800,000
Actual return on plan assets - 2002 900,000
Discount rate based on high quality corporate bonds 10%
Expected rate of return on assets 8%
Page |4

An actuarial loss of ₱20,000 was incurred during 2002. There was no unrecognized prior service cost
or unrecognized gains or losses. Flash's defined benefit cost for the year was
a. 880,000. b. 920,000. c. 640,000. d. 988,000.

C
Solution:

Service cost 800,000


Interest cost on the defined benefit obligation (10M x 10%) 1,000,000
Interest income on plan assets (10.4M x 10%) (1,040,000)
Actuarial (gains) and losses 20,000
Difference between interest income on plan assets and return
on plan assets (900,000 – 1,040,000 (140,000)
Defined benefit cost 640,000

3. Information on EQUANIMITY COMPOSURE Co.’s defined benefit plan is shown below:


 PV of defined benefit obligation, Jan. 1 ₱480,000
 PV of defined benefit obligation, Dec. 31 488,000
 Interest cost 10%
 Benefits paid to retirees 200,000
 Increase in present value of defined benefit obligation during the year due to
changes in actuarial assumptions 40,000

How much is the current service cost?


a. 120,000 b. 200,000 c. 160,000 d. 220,000

A
Solution:
PV of defined benefit obligation
480,000 Jan. 1
Benefits paid 200,000 120,000 Current service cost (squeeze)
48,000 Interest cost (480,000 x 10%)
40,000 Actuarial loss - increase in PV of PBO
Dec. 31 488,000

Use the following information for the next two questions:


PELLUCID CLEAR Co. agrees to provide lump-sum retirement benefits to employees equal to 6% of
final salary for each year of service. Information on an employee is shown below:
 Average annual salary level on January 1, 20x1 ₱12,000,000
 Average annual salary increase starting January 1, 20x2 and every year
thereafter. 3%
 Average service lives before entitlement to retirement benefits (January 1,
20x1 to December 31, 20x5) 5 years
 Discount rate per year 10%
Page |5

4. How much is the current service cost in 20x2?


a. 553,492 b. 669,724 c. 618,724 d. 608,840

D
Solution:
Final salary level (12M x 103% x 103% x 103% x 103%) 13,506,106
Multiply by: Percentage of benefit per year 6%
Benefit per year of service 810,366
Multiply by: No. of service years 5
Lump sum retirement benefit 4,051,832

(13,506,106 x 6%) = 810,366 benefit entitlement per year;


(810,366 x PV of 1 @10%, n=3) = 608,840 current service cost in 20x2 *(n=3 is from December 31, 20x2
to December 31, 20x5)

5. How much is the present value of the defined benefit obligation on December 31, 20x2?
a. 1,298,437 b. 1,217,680 c. 1,085,710 d. 1,908,117

B
Solution:
(13,506,106 x 6%) = 810,366 benefit entitlement per year;
(810,366 x 2 years passed x PV of 1 @10%, n=3) = 1,217,680

“And we know that in all things God works for the good of those who love him, who
have been called according to his purpose.” – (Romans 8:28)
- End
Page |1

Chapter 7
Leases Part 1
1. Entity A (customer) enters into a contract with Entity B (supplier) for the use of a data
processing equipment. According to the contract, Entity A shall operate the equipment only in
accordance with the standard operating procedures stated in the accompanying user’s manual.
In assessing the existence of a lease, does Entity A have the right to direct the use of the asset?
a. No, because the asset’s use is restricted.
b. Yes, because Entity A has the right to direct how and for what purpose the asset is used.
c. Yes, because the asset’s use is predetermined and Entity B is precluded from changing that
predetermined use.
d. Maybe yes, maybe no, but exactly I don’t know.

2. Which of the following is not one of the criteria when determining whether a contract is or
contains a lease?
a. Identified asset
b. Identified liability
c. Right to obtain substantially all of the economic benefits from use of an identified asset
throughout the period of use
d. Right to direct the use of the identified asset throughout the period of use

3. Which of the following statements is correct regarding the accounting for leases?
a. The lessor depreciates the leased asset under a finance lease.
b. The lessee depreciates the leased asset under a “short-term” or a “low-valued asset” lease.
c. When discounting lease payments the lessor and the lessee use the interest rate implicit in
the lease.
d. An entity can never be both a lessor and a lessee of a same leased asset.

4. According to PFRS 16, lease liabilities are presented in the lessee’s statement of financial position
a. separately from the other liabilities of the lessee.
b. together with other liabilities, with disclosure of the line items that include the lease
liabilities.
c. a or b
d. not presented in the lessee’s financial statements but only in the lessor’s financial statements

5. According to PFRS 16, right-of-use assets are presented in the lessee’s statement of financial
position
a. separately from the other assets of the lessee.
b. together with other assets as if they were owned, with disclosure of the line items that
include the right-of-use assets.
c. a or b
d. not presented in the lessee’s financial statements but only in the lessor’s financial statements
Page |2

6. On January 2, 20x9, Nori Mining Co. (lessee) entered into a 5-year lease for drilling equipment.
Nori recognized a lease liability of ₱240,000 at the commencement date. This amount includes
the ₱10,000 exercise price of a purchase option. At the end of the lease, Nori expects to exercise
the purchase option. Nori estimates that the equipment's fair value will be ₱20,000 at the end of
its 8-year life. Nori regularly uses straight-line depreciation on similar equipment. For the year
ended December 31, 20x9, what amount should Nori recognize as depreciation expense on the
leased asset?
a. 48,000 b. 46,000 c. 30,000 d. 27,500

D
Solution:
Cost 240,000
Residual value (fair value) (20,000)
Depreciable amount 220,000
Useful life 8
Depreciation expense 27,500

7. In the long-term liabilities section of its balance sheet at December 31, 20x9, Mene Co. reported a
lease liability of ₱75,000, net of current portion of ₱1,364. Payments of ₱9,000 were made on both
January 2, 2x10, and January 2, 2x11. Mene's incremental borrowing rate on the date of the lease
was 11% and the lessor's implicit rate, which was known to Mene, was 10%. In its December 31,
2x10, balance sheet, what amount should Mene report as lease liability, net of current portion?
a. 66,000 b. 73,500 c. 73,636 d. 74,250

Solution:
First step: Place the given information on the amortization table.
Date Payments Int. expense Amortization Present value
12/31/x9
1/1/x10 9,000 7,500 1,500 75,000

This is the lease obligation as of Dec.


31, 20x9, net of current portion.

8. Oak Co. leased equipment for its entire nine-year useful life, agreeing to pay ₱50,000 at the start
of the lease term on December 31, 20x8, and ₱50,000 annually on each December 31 for the next
eight years. The present value on December 31, 20x8, of the nine lease payments over the lease
term, using the rate implicit in the lease which Oak knows to be 10%, was ₱316,500. The
December 31, 20x8, present value of the lease payments using Oak's incremental borrowing rate
of 12% was ₱298,500. Oak made a timely second lease payment. What amount should Oak report
as lease liability in its December 31, 20x9, balance sheet?
a. 350,000 b. 243,150 c. 228,320 d. 0
B
Solution:
Date Payments Int. expense Amortization Present value
Page |3

12/31/x8 316,500
12/31/x8 50,000 - 50,000 266,500
12/31/x9 50,000 26,650 23,350 243,150

9. On January 2, 20x5, Marx Co. as lessee signed a five-year noncancelable equipment lease with
annual payments of ₱200,000 beginning December 31, 20x5. The five lease payments have a
present value of ₱758,000 at January 2, 20x5, based on interest of 10%. What amount should
Marx report as interest expense for the year ended December 31, 20x5?
a. 0 b. 48,000 c. 55,800 d. 75,800

D (758,000 x 10%) = 75,800

10. On January 1, 20x1, ABC Co. enters into a 4-year lease of office equipment. The rent in 20x1 is
₱10,000 and shall increase by 10% annually starting on January 1, 20x2. Rentals are payable at
the end of each year. ABC Co. pays the lessor a lease bonus of ₱5,000 on January 1, 20x1. ABC
Co. opts to use the practical expedient allowed under PFRS 16 for leases of low value assets.
How much is the lease expense in 20x1?
a. 10,000 b. 11,000 c. 11,603 d. 12,853

D Solution:

20x1 10,000
20x2 (10K x 110%) 11,000
20x3 (11K x 110%) 12,100
20x4 (12.1K x 110%) 13,310
Lease bonus 5,000
Total 51,410
Divide by: 4
Annual lease expense 12,853

"I have set the Lord continually before me. Because He is at my right hand, I will
not be shaken." – (Psalm 16:8)
Page |1

Chapter 8
Leases Part 2
1. Lessor Co. entered into two contract leases. Lease #1 transfers substantially all the risks and
rewards incidental to ownership of the leased asset. Lease #2 does not transfer substantially all
the risks and rewards incidental to ownership of the leased asset. How should Lessor Co.
classify the leases? (Lease #1); (Lease #2)
a. Finance, Operating c. Finance, Finance
b. Operating, Finance d. Operating, Operating

2. A lessor’s gross investment in a finance lease is computed as


a. lease payments plus unguaranteed residual value
b. present value of (a)
c. difference between (a) and (b)
d. sum of (a) and (b)

3. A lessor’s unearned interest income in a finance lease is computed as


a. lease payments plus unguaranteed residual value
b. present value of (a)
c. difference between (a) and (b)
d. sum of (a) and (b)

4. Which of the following does not correctly relate to the accounting for leases?
a. The underlying asset in a lease contract is recognized by the lessee in its financial statements.
b. The lessor recognizes a finance lease receivable equal to the net investment in a finance lease.
c. A manufacturer or dealer lessor recognizes gross profit or loss on commencement of a
finance lease in accordance with its policy for outright sales.
d. The lessor recognizes lease payments receivable from an operating lease as income in the
period earned.
e. The lessor continues to recognize an asset subject to a finance lease in its financial
statements.

5. Regarding the accounting for the residual value of a leased asset, which of the following
statements is incorrect?
a. A lessee accounts for a residual value only if it is guaranteed.
b. A lessor accounts for a residual value only if it is guaranteed.
c. A lessor accounts for a residual value whether guaranteed or not.
d. Both lessee and lessor will account for a residual value only if the leased asset reverts back to
the lessor.

6. Under operating leases, lessors


a. recognize rent income using a straight line basis, unless another method is more appropriate.
b. recognize interest income using the effective interest method.
Page |2

c. recognize different amounts of rent income each year depending on the contractual
payments
d. any of these

7. Security deposits that are refundable


a. are treated as unearned income by lessors under an operating lease.
b. are not discounted because they are normally of a short-term nature
c. are treated as receivable by lessees and as payable by lessors.
d. are discounted only by lessees but not by lessors

8. If the lessor recognizes rent income (lease income), then the lease must have been classified as
a. finance lease c. a or b
b. operating lease d. none of these

9. Which of the following statements is false regarding the accounting for leases?
a. The lessor may not use the straight line basis for recognizing lease income under an
operating lease if another systematic basis is more representative of the pattern in which
benefit from the use of the underlying asset is diminished.
b. The amount of lease income recognized each year under an operating lease is typically
constant even though the contractual payments increase every year by a certain amount
specified in the contract.
c. It is possible that the lessor does not depreciate the leased asset even if the lease is classified
as an operating lease.
d. Under an operating lease, the lessor capitalizes initial direct costs. These costs will increase
the lease income each year.

10. Which of the following is correct regarding the accounting for operating leases?
a. A lessor under an operating lease may classify the lease as either direct operating lease or
sales type operating lease.
b. A lessor includes a rent collected in advance as part of the cost of the leased asset.
c. A lessor includes initial direct costs incurred on the operating lease as part of the cost of the
leased asset to be recognized in profit or loss on the same basis as rent income is recognized.
d. A lessor includes initial direct costs incurred on the operating lease as part of the cost of the
leased asset to be recognized in profit or loss on the same basis as depreciation expense is
recognized.

"Come to me, all you who are weary and burdened, and I will give you rest." -
(Matthew 11:28-30)

- END -

Use the following information for the next five questions:


On January 1, 20x1, IMBROGLIO Co. leased equipment to COMPLICATION, Inc. Information on
the lease is shown below:
Page |3

Cost of equipment ₱ 1,200,000


Useful life of equipment 5 years
Lease term 4 years
Annual rent payable at the start of each year 400,000
Interest rate implicit in the lease 10%

Initial direct costs amounted to ₱80,000. The lease qualifies for sales type lease accounting.

1. How much is the gross investment in the lease on January 1, 20x1?


a. 2,000,000 b. 1,600,000 d. 1,200,000 d. 1,800,000

B (400,000 x 4 years) = 1,600,000

2. How much is the net investment in the lease on January 1, 20x1?


a. 1,200,000 b. 1,280,000 c. 1,394,740 d. 1,474,741

C (400,000 x PV annuity due @10%, n=4) = 1,394,741

3. How much is the total interest income (finance income) to be recognized by IMBROGLIO over
the lease term?
a. 205,260 b. 235,260 c. 125,259 d. 525,259

A (1,600,000 – 1,394,741) = 205,259

4. How much is the gross profit from the sale?


a. 114,740 b. 194,740 c. 125,259 d. 45,259

B (1,394,741 – 1,200,000 = 194,741

5. How much is the net profit from the sale?


a. 125,259 b. 45,259 c. 194,740 d. 114,740

D (194,741 – 80,000 initial direct costs) = 114,741

Use the following information for the next three questions:


On January 1, 20x1, YATAGHAN Financing Co. leased equipment to LONG KNIFE, Inc.
Information on the lease is shown below:

Cost of equipment ₱ 1,322,588


Useful life of equipment 5 years
Lease term 4 years
Annual rent payable at the end of each year 400,000
Interest rate implicit in the lease 10%
Residual value 80,000
Page |4

The equipment will revert back to YATAGHAN at the end of the lease term. The lease is classified as
direct financing lease.

6. Assuming the residual value is guaranteed, how much is the gross investment in the lease on
January 1, 20x1?
a. 1,600,000 b. 1,680,000 c. 1,520,000 d. 2,080,000

B (400,000 x 4) + 80,000 = 1,680,000

7. Assuming the residual value is unguaranteed, how much is the net investment in the lease?
a. 1,322,588 b. 1,267,948 c. 1,213,308 d. 1,345,981

A (400,000 x PV ordinary annuity @10%, n=4) + (80,000 X PV of 1 @10%, n=4) = (1,267,946 + 54,641) =
1,322,587

8. How much is the total interest income to be recognized by YATAGHAN over the lease term if
the residual value is unguaranteed and guaranteed, respectively?
Unguaranteed Guaranteed
a. 357,412 341,270
b. 341,270 357,412
c. 341,753 341,985
d. 357,412 357,412

D (1,680,000 - 1,322,587) = 357,413

9. Wall Co. leased office premises to Fox, Inc. for a five-year term beginning January 2, 20x9. Under
the terms of the operating lease, rent for the first year is ₱8,000 and rent for years 2 through 5 is
₱12,500 per annum. However, as an inducement to enter the lease, Wall granted Fox the first six
months of the lease rent-free. In its December 31, 20x9, income statement, what amount should
Wall report as rental income?
a. 12,000 b. 11,600 c. 10,800 d. 8,000

C
Solution:
Rent for the first year (8,000 x 6/12) 4,000
Rent for the subsequent years (12,500 x 4) 50,000
Total collection on rentals 54,000
Divide by: 5
Annual rent income 10,800

10. As an inducement to enter a lease, Arts, Inc., a lessor, grants Hompson Corp., a lessee, nine
months of free rent under a five-year operating lease. The lease is effective on July 1, 20x5, and
provides for monthly rental of ₱1,000 to begin April 1, 20x6. In Art's income statement for the
year ended June 30, 20x6, rent income should be reported as
a. 10,200 b. 9,000 c. 3,000 d. 2,550
Page |5

A
Solution:
Lease term in years 5
Multiply by: No. of months in a year 12
Lease term in months 60
Nine months free rent (9)
Total 51
Multiply by: Monthly rental 1,000
Total rental payments on the lease 51,000
Divide by: Lease term in years 5
Annual rent income (July 1 to June 30) 10,200

“Rejoice always, pray continually, give thanks in all circumstances; for this is God’s
will for you in Christ Jesus.” – (1 Thessalonians 5:16-18)

- END -
Page |1

Chapter 9
Income Taxes
1. All of the following can result in a temporary difference between pretax financial income and
taxable income except for
a. payment of premiums for life insurance.
b. depreciation expense.
c. provision for pending lawsuits.
d. product warranty costs.
(Adapted)

2. Which of the following items results in a temporary difference deductible amount for a given
year?
a. Premiums on officer's life insurance (company is beneficiary)
b. Recognition of unrealized gains on financial liabilities that are measured at fair value
through profit or loss.
c. Vacation pay accrual
d. Accelerated depreciation for tax purposes; straight-line for financial reporting purposes
(Adapted)

3. Which of the following temporary differences may result to a deferred tax liability?
a. Accrued warranty costs
b. Subscription revenue received in advance
c. Unrealized losses on held for trading securities
d. Depreciation
(Adapted)

4. When enacted tax rates change, the asset and liability method of interperiod tax allocation
recognizes the rate change as
a. a cumulative effect adjustment.
b. an adjustment to be netted against the current income tax expense.
c. a separate charge to the current year's net income.
d. a separate charge or benefit to income tax expense.
(Adapted)

5. Current financial reporting standards currently are moving toward the


a. no-deferral approach.
b. partial recognition approach.
c. comprehensive recognition approach.
d. discounted comprehensive recognition approach.
(Adapted)

6. If all temporary differences entering into the determination of pretax accounting income are
considered in the computation of deferred taxes and income tax expense, then
Page |2

a. the no-deferral approach is being applied.


b. the comprehensive recognition approach is being applied.
c. the partial recognition approach is being applied.
d. the net-of-tax method is being applied.
(Adapted)

7. If there is a change in the tax rate applicable in future periods, which of the following statements
is incorrect?
a. Current tax expense may be equal to taxable profit multiplied by the enacted tax rate(s)
applicable to the period(s) where the profit was earned.
b. Deferred tax asset or liability is computed based on the substantially enacted tax rate that is
applicable in the period where the deferred tax is expected to reverse.
c. Income tax expense is equal to accounting profit multiplied by the substantially enacted
future tax rate.
d. Deferred tax expense (benefit) is equal to the net change in deferred tax asset and deferred
tax liability during the year.

8. Which of the following situations would require interperiod income tax allocation procedures?
a. A temporary difference exists because the tax basis of capital equipment is less than its
reported amount in the financial statements.
b. Proceeds from an insurance policy on capital equipment lost in a fire exceed the book value
of the equipment.
c. Last period's ending inventory was understated causing both net income and income tax
expense to be understated.
d. Nontaxable interest payments are received on municipal bonds.
(Adapted)

9. The result of interperiod income tax allocation is that


a. wide fluctuations in a company's tax liability payments are eliminated.
b. tax expense shown in the income statement is equal to the deferred taxes shown on the
balance sheet.
c. tax liability shown in the balance sheet is equal to the deferred taxes shown on the previous
year's balance sheet plus the income tax expense shown on the income statement.
d. tax expense shown on the income statement is equal to income taxes payable for the current
year plus or minus the change in the deferred tax asset or liability balances for the year.
(Adapted)

10. Assuming no prior period adjustments, would the following allocations affect net income?
Interperiod Tax Allocation Intraperiod Income Tax Allocation
a. Yes Yes
b. Yes No
c. No Yes
d. No No
(Adapted)
Page |3

“The roots of education are bitter, but the fruit is sweet.” – Aristotle

- END -
Page |4

The next two items are based on the following:


Bee Corp. prepared the following reconciliation between book income and taxable income for the
year ended December 31, 20x0:
Pretax accounting income 500,000
Taxable income 300,000
Difference 200,000

Interest on municipal bonds 50,000


Lower depreciation per financial statements 150,000
Total differences 200,000

Bee's effective income tax rate for 20x0 is 30%. The depreciation difference will reverse equally over
the next three years at enacted tax rates as follows:
Years Tax rates
20x1 30%
20x2 25%
20x3 25%

1. In Bee's 20x0 income statement, the current portion of its provision for income taxes should be
a. 150,000 b. 125,000 c. 90,000 d. 75,000

C (300,000 taxable income x 30%) = 90,000

2. In Bee's 20x0 financial statements, the deferred portion of its provision for income taxes should
be
a. 60,000 b. 50,000 c. 45,000 d. 40,000

D
Solution:
Year Reversals* Tax rate Deferred tax
20x1 50,000 30% 15,000
20x2 50,000 25% 12,500
20x3 50,000 25% 12,500
40,000

*Lower depreciation per financial statements 150,000


Divide by: 3
Equal amounts of reversals 50,000

3. In its December 31, 20x0 balance sheet, Quinn Co. reported a deferred tax asset of ₱9,000 and no
deferred tax liability. For 20x1, Quinn reported pretax financial statement income of ₱300,000.
Temporary differences of ₱100,000 resulted in taxable income of ₱200,000 for 20x1. At December
31, 20x1, Quinn had cumulative taxable differences of ₱70,000. Quinn's effective income tax rate
Page |5

is 30%. In its December 31, 20x1, income statement, what should Quinn report as deferred
income tax expense?
a. 12,000 b. 21,000 c. 30,000 d. 60,000

C
Solution:
Decrease in DTA (the beginning balance) 9,000
Increase in DTL (70K TTD x 30%) 21,000
Deferred tax expense 30,000

4. On its December 31, 20x1, balance sheet, Shin Co. had income taxes payable of ₱13,000 and a
deferred tax asset of ₱20,000 before determining the need for a valuation account. Shin had
reported a deferred tax asset of ₱15,000 at December 31, 20x0. No estimated tax payments were
made during 20x1. At December 31, 20x1, Shin determined that it was more likely than not that
10% of the deferred tax asset would not be realized. In its 20x1 income statement, what amount
should Shin report as total income tax expense?
a. 8,000 b. 8,500 c. 10,000 d. 13,000
C
Solution:
DTA, Dec. 31, 20x1 before adjustment 20,000
Allowance (20,000 x 10%) (2,000)
DTA, Dec. 31, 20x1 after adjustment 18,000
DTA, Dec. 31, 20x0 15,000
Increase in DTA during 20x1 3,000

Income tax expense 10,000 (squeeze)


Add: Increase in DTA during 20x1 3,000
Current tax expense (equal to income tax payable) 13,000 (start)

5. Taft Corp. uses the equity method to account for its 25% investment in Flame, Inc. During 20x1,
Taft received dividends of ₱30,000 from Flame and recorded ₱180,000 as its equity in the
earnings of Flame. Additional information follows:
• All the undistributed earnings of Flame will be distributed as dividends in future periods.
• The dividends received from Flame are eligible for the 80% dividends received deduction.
• There are no other temporary differences.
• Enacted income tax rates are 30% for 20x1 and thereafter.

In its December 31, 20x1, balance sheet, what amount should Taft report for deferred income tax
liability?
a. 9,000 b. 10,800 c. 45,000 d. 54,000

A
Solution:
Share in associate’s profit 180,000
Page |6

Dividends received (30,000)


Share in undistributed earnings 150,000
Multiply by: Percentage subject to taxation (100% - 80%) 20%
Taxable temporary difference 30,000
Multiply by: Substantially enacted tax rate for future periods 30%
Deferred tax liability – year-end 9,000

6. Bishop Corporation began operations in 20x7 and had operating losses of ₱200,000 in 20x7 and
₱150,000 in 20x8. For the year ended December 31, 20x9, Bishop had pretax book income of
₱300,000. For the three-year period 20x7 to 20x9, assume an income tax rate of 40% and no
permanent or temporary differences between book and taxable income. In Bishop’s 20x9 income
statement, how much should be reported as total income tax expense?
a. 0 b. 40,000 c. 60,000 d. 120,000

D (300,000 pretax income x 40%) = 120,000. The reversal of deferred tax asset affects only the current
tax expense but not income tax expense

The next two items are based on the following:


Venus Corp.’s worksheet for calculating current and deferred income taxes for 20x2 follows:
20x2 20x3 20x4
Pretax income 1,400
Temporary differences:
Depreciation (800) (1,200) 2,000
Warranty costs 400 (100) (300)
Taxable income 1,000 (1,300) 1,700
Enacted rate 30% 30% 25%

Venus had no prior deferred tax balances. In its 20x2 income statement, what amount should Venus
report as:

7. Current income tax expense?


a. 420 b. 350 c. 300 d. 0

C (1,000 taxable income x 30%) = 300

8. Deferred income tax expense?


a. 350 b. 300 c. 120 d. 95

D
Solution:
Depreciation (FI>TI); TTD; DTL
(800 x 25% rate in 20x4, the yr. of reversal*) (200)
Warranty costs (FI<TI); DTD; DTA
(100 x 30% rate in 20x3) + (300 x 25% rate in 20x4) (30 105
Page |7

Deferred tax expense (95)


* Depreciation reverses in 20x4 because it is on this year that the ‘minus’ function becomes an
‘addition’.

9. Black Co., organized on January 2, 20x0, had pretax financial statement income of ₱500,000 and
taxable income of ₱800,000 for the year ended December 31, 20x0. The only temporary
differences are accrued product warranty costs, which Black expects to pay as follows:
20x1 ₱100,000
20x2 50,000
20x3 50,000
20x4 100,000

The enacted income tax rates are 25% for 20x0, 30% for 20x1 through 20x3, and 35% for 20x4. Black
believes that future years' operations will produce profits. In its December 31, 20x0, balance sheet,
what amount should Black report as deferred tax asset?
a. 50,000 b. 75,000 c. 90,000 d. 95,000

D
Solution:
Year of reversal Amounts Tax rate Deferred tax asset
20x1 100,000 30% 30,000
20x2 50,000 30% 15,000
20x3 50,000 30% 15,000
20x4 100,000 35% 35,000
95,000

10. Rom Corp. began business in 20x1 and reported taxable income of ₱50,000 on its 20x1 tax return.
Rom's enacted tax rate is 30% for 20x1 and future years. The following is a schedule of Rom's
December 31, 20x1, temporary differences in thousands of dollars:

12/31/x1 Future taxable (deductible) amounts


Carrying
amount over
(under) Tax
base 20x2 20x3 20x4 20x5
Equipment 10 (5) 5 5 5
Warranty liability (20) (10) (10)
Deferred compensation
liability (15) (5) (10)
Installment receivables 30 10 20
Totals 5 (5) (10) 25 (5)

What amount should Rom report as total deferred tax asset in its December 31, 20x1, balance sheet?
a. 0 b. 1,500 c. 4,500 d. 6,000
Page |8

A
Solution:

Concept: If the carrying amount (CA) of an asset exceeds its tax base (TB), the difference is a taxable
temporary difference which, if multiplied by the tax rate, results to a deferred tax liability.
“For an asset: CA > TB = difference is TTD; TTD x Tax rate = DTL”

Consequently:
“For a liability: CA < TB = = difference is TTD; TTD x Tax rate = DTL”

In all of the items in the problem, the carrying amounts of the assets (i.e., equipment and installment
receivables) as of December 31, 20x1 exceed their tax bases (CA>TB). Therefore, the differences are
taxable temporary differences which give rise to deferred tax liability and not deferred tax asset.

Also, the carrying amounts of the liabilities (i.e., warranty and deferred compensation) as of
December 31, 20x1 are less than their tax bases. Therefore, the differences are taxable temporary
differences which give rise to deferred tax liability and not deferred tax asset.

Conclusion: No deferred tax asset is recognized on December 31, 20x1.

"For God has not given us a spirit of fear and timidity, but of power, love, and self-
discipline." - (2 Timothy 1:7)

- END -
Page |1

Chapter 10
Shareholders’ Equity (Part 1)
1. The entry to record the reissuance of treasury shares above their original acquisition cost
includes
a. a credit to share premium
b. a debit to share premium
c. a debit to retained earnings
d. b and c

2. Ten thousand shares of ₱20 par value common stock were initially issued at ₱25 per share.
Subsequently, two thousand of these shares were purchased as treasury stock at ₱30 per share.
What is the effect of the purchase of the treasury stock on the amount reported in the balance
sheet for each of the following?
Share premium Retained earnings
a. No effect No effect
b. No effect Decrease
c. Decrease No effect
d. Decrease Decrease

3. The entry to record the retirement of shares at below their original acquisition cost includes
a. a debit to share premium arising from the original issuance
b. a debit to any share premium arising from treasury shares
c. a debit to retained earnings
d. all of these including (c) when (a) and (b) are insufficient to offset any difference between the
original issuance price and the retirement price.

4. In 20x1, Fogg, Inc., issued ₱10 par value ordinary share for ₱25 per share. No other share
transactions occurred until March 31, 20x1, when Fogg acquired some of the issued shares for
₱20 per share and retired them. Which of the following statements correctly states an effect of
this acquisition and retirement?
a. 20x1 profit is decreased.
b. 20x1 profit is increased.
c. Share premium is decreased.
d. Retained earnings is increased.

5. Redeemable preference shares are classified by the issuer as


a. financial liability
b. own equity, presented in shareholders’ equity
c. a or b
d. reduction of share capital in shareholders’ equity

6. In 20x0, Newt Corp. acquired 6,000 shares of its own ₱1 par value ordinary share at ₱18 per
share. In 20x1, Newt issued 3,000 of these shares at ₱25 per share. Newt uses the cost method to
account for its treasury stock transactions. What accounts and amounts should Newt credit in
20x1 to record the issuance of the 3,000 shares?
Page |2

Treasury sh. Sh. premium Retained earnings Ordinary sh.


a. ₱54,000 ₱21,000
b. ₱54,000 ₱21,000
c. ₱72,000 ₱3,000
d. ₱51,000 ₱21,000 ₱3,000
B
Solution:
Dec. Cash (3,000 x 25) 75,000
27, Treasury shares (3,000 x 18) 54,000
20x1 Share premium – Treasury shares 21,000

7. On December 1, 20x1, Line Corp. received a donation of 2,000 shares of its ₱5 par value ordinary
shares from a shareholder. On that date, the stock’s market value was ₱35 per share. The stock
was originally issued for ₱25 per share. By what amount would this donation cause total
stockholders’ equity to decrease?
a. 70,000 b. 50,000 c. 20,000 d. 0

Donation will not make the stockholders’ equity decrease

8. On July 1, 20x1, Vail Corp. issued rights to stockholders to subscribe to additional share of its
common stock. One right was issued for each share owned. A stockholder could purchase one
additional share for 10 rights plus ₱15 cash. The rights expired on September 30, 20x1. On July 1,
20x1, the market price of a share with the right attached was ₱40, while the market price of one
right alone was ₱2. Vail’s stockholders’ equity on June 30, 20x1, comprised the following:

Ordinary shares, ₱25 par value, 4,000 shares issued and outstanding…..₱100,000
Share premium…………………….……………………………………………..60,000
Retained earnings……………..…………………………………………………80,000

By what amount should Vail’s retained earnings decrease as a result of issuance of the stock rights
on July 1, 20x1?
a. 0 b. 5,000 c. 8,000 d. 10,000

9. On September 20x1, West Corp. made a dividend distribution of one right for each of its 120,000
shares of outstanding common stock. Each right was exercisable for the purchase of 1/100 of a
share of West's ₱50 variable rate preference share at an exercise price of ₱80 per share. On March
20, 20x3, none of the rights had been exercised, and West redeemed them by paying each
stockholder ₱0.10 per right. As a result of this redemption, West's stockholders' equity was
reduced by
a. 120 b. 2,400 c. 12,000 d. 36,000

C (120,000 rights x ₱0.10) = 12,000 debit to share premium

10. The following trial balance of Shaw Corp. at December 31, 20x1, has been adjusted except for
income tax expense.
Page |3

Dr. Cr.
Cash 675,000
Accounts receivable (net) 2,695,000
Inventory 2,185,000
Property, plant and equipment (net) 7,366,000
Accounts payable and accrued liabilities 1,801,000
Income tax payable 654,000
Deferred income tax liability 85,000
Ordinary shares 2,300,000
Share premium 3,680,000
Retained earnings, 1/1/x1 3,350,000
Net sales and other revenues 13,360,000
Costs and expenses 11,180,000
Income tax expense 1,129,000
Totals 25,230,000 25,230,000

Other financial data for the year ended December 31, 20x1:
• Included in accounts receivable is ₱1,000,000 due from a customer and payable in quarterly
installments of ₱125,000. The last payment is due December 30, 20x3. = Assets
• The balance in the deferred income tax liability account pertains to a temporary difference not
related to a balance sheet account that arose in a prior year, of which ₱15,000 is expected to be
paid in 20x2.
• During the year, estimated tax payments of ₱475,000 were charged to income tax expense. The
current and future tax rate on all types of income is 30%. In Shaw's December 31, 20x1, balance
sheet, = Liabilities

The working capital and the total shareholders’ equity as of December 31, 20x1 are
Working capital Total Shareholders’ Equity
a. 2,600,000 10,856,000
b. 2,881,000 10,856,000
c. 3,075,000 9,330,000
d. 3,075,000 10,856,000

D
Solutions:
Cash 675,000
Accounts receivable (net) [2.695M - 1M + (125K x 4)] 2,195,000
Inventory 2,185,000
Total current assets 5,055,000

Net sales and other revenues 13,360,000


Costs and expenses (11,180,000)
Profit before tax 2,180,000
Multiply by: Tax rate 30%
Income tax expense 654,000
Income tax payments during the year (475,000)
Page |4

Adjusted income tax payable 179,000

Accounts payable and accrued liabilities 1,801,000


Income tax payable 179,000
Total current liabilities 1,980,000

Working capital = Current assets – Current liabilities


Working capital = (5,055,000 – 1,980,000) = 3,075,000

Net sales and other revenues 13,360,000


Costs and expenses (11,180,000)
Profit before tax 2,180,000
Income tax expense (30% x 2,180,000) (654,000)
Profit after tax 1,526,000
Retained earnings, Jan. 1 3,350,000
Retained earnings, Dec. 31 4,876,000

Ordinary shares 2,300,000


Share premium 3,680,000
Retained earnings, Dec. 31, 20x1 4,876,000
Shareholders' Equity 10,856,000

"A cheerful heart is good medicine but a crushed spirit dries up the bones." - (Proverbs
17:22)

- END -
Page |5
Page |6

SOLUTIONS:
1. A
2. A
3. D
4. C
5. A

6. B
Solution:
Dec. 27, Cash (3,000 x 25) 75,000
20x1
Treasury shares (3,000 x 18) 54,000
Share premium – Treasury shares 21,000

7. D

8. A

9. C (120,000 rights x ₱0.10) = 12,000 debit to share premium

10. D
Solutions:
Cash 675,000
Accounts receivable (net) [2.695M - 1M + (125K x 4)] 2,195,000
Inventory 2,185,000
Total current assets 5,055,000

Net sales and other revenues 13,360,000


Costs and expenses (11,180,000)
Profit before tax 2,180,000
Multiply by: Tax rate 30%
Income tax expense 654,000
Income tax payments during the year (475,000)
Adjusted income tax payable 179,000

Accounts payable and accrued liabilities 1,801,000


Income tax payable 179,000
Total current liabilities 1,980,000

Working capital = Current assets – Current liabilities


Working capital = (5,055,000 – 1,980,000) = 3,075,000

Net sales and other revenues 13,360,000


Costs and expenses (11,180,000)
Profit before tax 2,180,000
Income tax expense (30% x 2,180,000) (654,000)
Profit after tax 1,526,000
Retained earnings, Jan. 1 3,350,000
Retained earnings, Dec. 31 4,876,000

Ordinary shares 2,300,000


Share premium 3,680,000
Retained earnings, Dec. 31, 20x1 4,876,000
Shareholders' Equity 10,856,000
Page |1

Chapter 11
Shareholders’ Equity (Part 2)
1. On settlement (distribution) date, any difference between the carrying amounts of the property
dividend payable and the non-cash asset distributed is
a. ignored
b. recognized in profit or loss
c. recognized directly in retained earnings
d. recognized but subject to a limit

2. Non-current assets declared as property dividends are


a. reclassified as “non-current assets held for distribution to owners” if the conditions under
PFRS 5 are met.
b. reclassified as current assets.
c. not reclassified but presented separately from the other assets.
d. not reclassified but disclosed only.

3. If shareholders are given a choice of receiving either property dividends or cash dividends, the
entity shall
a. estimate the dividend payable by considering both the fair value of each alternative and the
associated probability of shareholders selecting each alternative.
b. treat the dividends declared as if they are cash dividends.
c. treat the dividends declared as if they are property dividends.
d. not account for the dividends until their final settlements.

4. Which of the following may cause a change in the total shareholders’ equity?
a. “small” share dividends d. “large” share dividends
b. share splits e. none of these
c. recapitalization

5. Imagine you are a CPA. You are preparing the financial statements of your company for the year
ended December 31, 20x1. The board of directors declared dividends on February 1, 20x2. The
dividend declaration is not subject to further approval. The financial statements were authorized
for issue on April 1, 20x2. How should the dividends declared be accounted for in the 20x1
financial statements?
a. included in current liabilities c. disclosed only
b. included in noncurrent liabilities d. neither accrued nor disclosed

6. Ray Corp. declared a 5% stock dividend on its 10,000 issued and outstanding shares of ₱2 par
value common stock, which had a fair value of ₱5 per share before the stock dividend was
declared. This stock dividend was distributed 60 days after the declaration date. By what
amount did Ray’s current liabilities increase as a result of the stock dividend declaration?
a. 0 b. 500 c. 1,000 d. 2,500

A - Stock dividend payable is not a liability.


Page |2

7. Effective April 27, 20x1, the stockholders of Bennett Corporation approved a two-for-one split of
the company's common stock, and an increase in authorized common shares from 100,000 shares
(par value ₱20 per share) to 200,000 shares (par value ₱10 per share). Bennett's stockholders'
equity accounts immediately before issuance of the stock split shares were as follows:

Common stock, par value ₱20; 100,000 shares authorized;


50,000 shares outstanding ₱1,000,000
Share premium (₱3 per share on issuance of ordinary shares) 150,000
Retained earnings 1,350,000

What should be the balances in Bennett's additional paid-in capital and retained earnings accounts
immediately after the stock split is effected?
Share premium Retained earnings
a. ₱ 0 ₱ 500,000
b. ₱ 150,000 ₱ 350,000
c. ₱ 150,000 ₱1,350,000
d. ₱ 1,150,000 ₱ 350,000

C
Share splits do not affect total shareholders’ equity. The aggregate par value of outstanding shares
remains the same after a share split. The entry to record the share split is as follows:
Apr. Common stock (old) (50,000 sh. x ₱20) 1,000,000
27, Common stock (new) (100,000 sh. x ₱10) 1,000,000
20x1

8. On July 1, 1999, Bart Corporation has 200,000 shares of ₱10 par ordinary share outstanding and
the market price of the stock is ₱12 per share. On the same date, Bart declared a 1-for-2 reverse
stock split. The par of the stock was increased from ₱10 to ₱20 and one new ₱20 par share was
issued for each two ₱10 par shares outstanding. Immediately before the 1-for-2 reverse stock
split, Bart's share premium was ₱450,000. What should be the balance in Bart's share premium
account immediately after the reverse stock split is effected?
a. 0 b. 450,000 c. 650,000 d. 850,000

B Share premium is not affected by share splits.

9. The stockholders' equity section of Brown Co.'s December 31, 20x1, balance sheet consisted of
the following:

Ordinary shares, ₱30 par, 10,000 shares authorized and outstanding ₱300,000
Share premium 150,000
Retained earnings (deficit) (210,000)

On January 2, 20x2, Brown put into effect a stockholder-approved quasi-reorganization by reducing


the par value of the stock to ₱5 and eliminating the deficit against share premium. Immediately after
the quasi-reorganization, what amount should Brown report as share premium?
Page |3

a. (60,000) b. 150,000 c. 190,000 d. 400,000

Solution:
The entries to record the quasi-reorganization are as follows:
Jan. 2, Share capital [(₱30 – ₱5) x 10,000 sh.] 250,000
20x2
Share premium 250,000
to record the reduction of par value
Jan. 2, Share premium 210,000
20x2
Retained earnings
to wipe out the deficit 210,000

Share premium - Dec. 31, 20x1 150,000


Credit (see journal entries above) 250,000
Debit (see journal entries above) (210,000)
Share premium after quasi-reorganization 190,000

10. On January 2, 2000, the board of directors of Gimli Mining Corporation declared a cash dividend
of ₱1,200,000 to stockholders of record on January 18, 2000, and payable on February 10, 2000.
The dividend is permissible by law in Gimli's state of incorporation. Selected data from Gimli's
December 31, 1999, balance sheet follow:

Accumulated depletion ₱ 200,000


Capital stock 1,100,000
Additional paid-in capital 800,000
Retained earnings 500,000

The ₱1,200,000 dividend includes a liquidating dividend of


a. 800,000. b. 700,000. c. 600,000. d. 200,000.

B (1,200,000 dividends declared – 500,000 retained earnings) = 700,000

"Therefore do not worry about tomorrow, for tomorrow will worry about itself.
Each day has enough trouble of its own." - (Matthew 6:34)

- END -
Page |4

SOLUTIONS:

1. B
2. A
3. A
4. E
5. C

6. A - Stock dividend payable is not a liability.

7. C
Share splits do not affect total shareholders’ equity. The aggregate par value of outstanding shares
remains the same after a share split. The entry to record the share split is as follows:
Apr. 27, Common stock (old) (50,000 sh. x ₱20) 1,000,000
20x1
Common stock (new) (100,000 sh. x ₱10) 1,000,000

8. B Share premium is not affected by share splits.

9. C

Solution:
The entries to record the quasi-reorganization are as follows:
Jan. 2, Share capital [(₱30 – ₱5) x 10,000 sh.] 250,000
20x2
Share premium 250,000
to record the reduction of par value
Jan. 2, Share premium 210,000
20x2
Retained earnings
to wipe out the deficit 210,000

Share premium - Dec. 31, 20x1 150,000


Credit (see journal entries above) 250,000
Debit (see journal entries above) (210,000)
Share premium after quasi-reorganization 190,000

10. B (1,200,000 dividends declared – 500,000 retained earnings) = 700,000


Page |1

Chapter 12
Share-based Payments (Part 1)

Use the following information for the next two questions:


ABC Co. acquired equipment from XYZ, Inc. by issuing 1,000 of its ₱10 par value ordinary shares.
The equipment is carried in the books of XYZ, Inc. at ₱15,000. ABC Co.’s shares have a fair value ₱14
per share.

1. The fair value of the equipment is ₱12,000. What is the journal entry to record the acquisition of
the equipment?

Solution:

Date Equipment (fair value of asset received) 12,000


Share capital 10,000
Share premium 2,000

2. The fair value of the equipment cannot be determined reliably. What is the journal entry to
record the acquisition of the equipment?

Solution:

Date Equipment (1,000 sh. x ₱14) - FV of shares 14,000


Share capital 10,000
Share premium 4,000

3. On January 1, 20x1, ABC Co. contracted XYZ & Co., CPAs for an outsourced internal audit
engagement. ABC Co. has its own internal audit department which performs similar services to
those outsourced with XYZ. But due to lack of human resources and the immediate need of
management for the internal audit services, XYZ has been contracted.

On January 1, 20x1, XYZ & Co. agreed to receive 1,000 shares of ABC with par value per share of
₱100 in consideration for its services as there is no restriction for equity ownership for CPAs
providing internal audit services (unlike for financial audits).

The audit field work ended on March 1, 20x1 but the close-out meeting was held on March 10,
20x1. All of the services required under the contract have been substantially rendered as of
March 10, 20x1, with the exception of some follow-up procedures required under ISPPIA
(International Standards for the Professional Practice of Internal Auditing).

The fair values of the shares were ₱500 on January 1, 20x1, ₱600 on March 1, 20x1, and ₱620 on
March 10, 20x1 while the fair value of the services remained unchanged at ₱600,000 over the
engagement period.
Page |2

Requirement: Provide the journal entries on the following dates: Jan. 1, 20x1, March 1, 20x1 and
March 10, 20x1.

Solution:

Jan. 1, 20x1 No entry


March 1,
No entry
20x1
March 10, Professional fees (1,000 x ₱500*) 500,000
20x1 Share capital (1,000 x ₱100) 100,000
Share premium 400,000

* Services received from employees and others providing similar services are measured at the fair
value of the equity securities granted at grant date (i.e., January 1, 20x1).

Use the following information for the next three questions:


On January 1, 20x1, ABC Co. grants 1,000 share options to each of its 100 key employees conditional
upon each employee remaining in ABC’s employ over the next three years. ABC estimates that the
fair value of each share option is ₱15.

4. On the basis of a weighted average probability, ABC Co. estimates on January 1, 20x1 that 20 per
cent of employees will leave during the three-year period and therefore forfeit their rights to the
share options. Twenty (20) employees actually left the company during the three-year period.
Fifteen (15) employees left in 20x1 and the other five (5) left in 20x3.

Requirement: Provide the journal entries on the following dates: Jan. 1, 20x1, Dec. 31, 20x1, Dec.
31, 20x2, and Dec. 31, 20x3.

Solution:

Jan. 1,
Memo entry
20x1
Dec. 31, Salaries expense – share options 400,000
20x1 [(100 – 20) x 1,000 x ₱15 x 1/3]
Share premium – sh. options outstanding 400,000
Dec. 31, Salaries expense – share options 400,000
20x2 [(100 – 20) x 1,000 x ₱15 x 2/3] – 400,000
Share premium – sh. options outstanding 400,000
Dec. 31, Salaries expense – share options 400,000
20x3 [(100 – 20) x 1,000 x ₱15 x 3/3] – 800,000
Share premium – sh. options outstanding 400,000

5. On the basis of a weighted average probability, ABC Co. estimates on January 1, 20x1 that 20
employees will leave during the three-year period and therefore forfeit their rights to the share
options. During 20x1, 2 employees left. The entity revised its estimate of total employee
departures over the three-year period from 20 to 15 employees. During 20x2, additional 3
Page |3

employees left. The entity revised its estimate of total employee departures over the three-year
period from 15 to 12 employees. During 20x3, additional 5 employees left.

Requirement: Provide the journal entries on the following dates: Jan. 1, 20x1, Dec. 31, 20x1, Dec.
31, 20x2, and Dec. 31, 20x3.

Solution:
Jan. 1,
Memo entry
20x1
Dec. 31, Salaries expense – share options 425,000
20x1 [(100 – 15) x 1,000 x ₱15 x 1/3]
Share premium – sh. options outstanding 425,000
Dec. 31, Salaries expense – share options 455,000
20x2 [(100 – 12) x 1,000 x ₱15 x 2/3] – 425,000
Share premium – sh. options outstanding 455,000
Dec. 31, Salaries expense – share options 470,000
20x3 [(100 – 10) x 1,000 x ₱15 x 3/3] – 880,000
Share premium – sh. options outstanding 470,000

6. On the basis of a weighted average probability, ABC Co. estimates on January 1, 20x1 that 20
employees will leave during the three-year period and therefore forfeit their rights to the share
options. No employees have actually left the company during the three-year vesting period.

Requirement: Provide the journal entries on the following dates: Jan. 1, 20x1, Dec. 31, 20x1, Dec.
31, 20x2, and Dec. 31, 20x3.

Solution:
Jan. 1,
Memo entry
20x1
Dec. 31, Salaries expense – share options 400,000
20x1 [(100 – 20) x 1,000 x ₱15 x 1/3]
Share premium – sh. options outstanding 400,000
Dec. 31, Salaries expense – share options 400,000
20x2 [(100 – 20) x 1,000 x ₱15 x 1/3] – 400,000
Share premium – sh. options outstanding 400,000
Dec. 31, Salaries expense – share options 700,000
20x3 (100 x 1,000 x ₱15 x 3/3) – 800,000
Share premium – sh. options outstanding 700,000

“One of the teachers of the law came and heard them debating. Noticing that Jesus
had given them a good answer, he asked him, “Of all the commandments, which is
the most important?” “The most important one,” answered Jesus, “is this: ‘Hear, O
Israel: The Lord our God, the Lord is one. Love the Lord your God with all your
heart and with all your soul and with all your mind and with all your strength.’
Page |4

The second is this: ‘Love your neighbor as yourself.’ There is no commandment


greater than these.” – (Mark 12:28-31)

“Love is patient, love is kind. It does not envy, it does not boast, it is not proud. It
does not dishonor others, it is not self-seeking, it is not easily angered, it keeps no
record of wrongs. Love does not delight in evil but rejoices with the truth. 7 It
always protects, always trusts, always hopes, always perseveres.” – (1 Corinthians 13:4-8)

- END -
Page |5

1. An entity has granted share options to its employees. The total expense to the vesting date of
December 31, 20X6, has been calculated as ₱8 million. The entity has decided to settle the award
early, on December 31, 20X5. The expense charged in the income statement since the grant date
of January 1, 20X3, had been year to December 31, 20X3, ₱2 million, and year to December 31,
20X4, ₱2.1 million. The expense that would have been charged in the year to December 31, 20X5,
was ₱2.2 million. What would be the expense charged in the income statement for the year
December 31, 20X5?
a. 2.2 million. b. 8 million. c. 3.9 million. d. 2 million.

C (8M – 2M – 2.1M) = 3.9M

Use the following information for the next two questions:


On January 1, 20x1, Golf View Village Co. grants 1,000 share options to each of its 180 employees on
condition that the employees remain in Golf View’s employ until the end of 20x3. The exercise price
per share option is ₱20. The fair value per share option is ₱80.

On December 31, 20x1, Golf View modifies the share option grant by reducing the exercise price to
₱60. This resulted to an increase in the fair value per option before the modification of ₱100 to ₱120
after the modification.

2. What amount of compensation expense shall be recognized in 20x1?


a. 4,800,000 b. 3,600,000 c. 1,800,000 d. 1,200,000

A (1,000 x 180 x 80 x 1/3) = 4,800,000

3. What amount of compensation expense shall be recognized in 20x2?


a. 4,800,000 b. 6,600,000 c. 7,200,000 d. 9,600,000

B
Solution:
Original terms: (1,000 x 180 x 80 x 2/3) – 4.8M = 4,800,000;
Modification: (120 – 100) x 1,000 x 180 x ½ = 1,800,000
Total compensation expense, 20x2 = 4.8M + 1.8M = 6,600,000

Use the following information for the next two questions:


On January 1, 20x1, Creek Co. grants 1,000 share options to each of its 180 employees on condition
that the employees remain in Creek’s employ until the end of 20x3. The exercise price per share
option is ₱20. The fair value per share option is ₱80.

On December 31, 20x1, Creek Co. modifies the share option grant by extending the vesting period to
the end of 20x4.

4. What amount of compensation expense shall be recognized in 20x1?


a. 4,800,000 b. 3,600,000 c. 1,800,000 d. 1,200,000

A (1,000 x 180 x 80 x 1/3) = 4,800,000


Page |6

5. What amount of compensation expense shall be recognized in 20x2?


a. 4,800,000 b. 6,600,000 c. 2,400,000 d. 1,600,000

A (1,000 x 180 x 80 x 2/3) – 4,800,000 = 4,800,000

"The Lord is my light and my salvation, whom shall I fear? The Lord is the
stronghold of my life, of whom shall I be afraid?" – (Psalm 27:1)

- END -
Page |1

Chapter 13
Share-based Payments (Part 2)

1. Many shares and most share options are not traded in an active market. Therefore, it is often
difficult to arrive at a fair value of the equity instruments being issued. Which of the following
option valuation techniques should not be used as a measure of fair value in the first instance?
a. Black-Scholes model.
b. Binomial model.
c. Monte-Carlo model.
d. Intrinsic value.

2. Elizabeth, a public limited company, has granted 100 share appreciation rights to each of its
1,000 employees in January 20X4. The management feels that as of December 31, 20X4, 90% of
the awards will vest on December 31, 20X6. The fair value of each share appreciation right on
December 31, 20X4, is P10. What is the fair value of the liability to be recorded in the financial
statements for the year ended December 31, 20X4?
a. P300,000
b. P10 million
c. P100,000
d. P90,000

A 100 x 1,000 x 90% x 10 x 1/3 = 300,000

3. On January 1, 2008, ABC Company offered its chief executive officer, stock appreciation rights
with the following terms:
Predetermined price ₱100 per share
Number of shares 10,000 shares
Service period-3 years 2008, 2009 and 2010
Exercise date December 31, 2010

The stock appreciation rights are exercised on December 31, 2010. The quoted price of the ABC stock
is as follows: ₱118 on December 31, 2008, ₱112 on December 31, 2009 and ₱124 on December 31,
2010. ABC Company should record 2010 compensation expense at
a. 160,000
b. 60,000
c. 80,000
d. 20,000

A
2010: 10,000 x (124 – 100) x 3/3 = 240,000;
2009: 10,000 x (112 – 100) x 2/3 = 80,000;
(240,000 – 80,000) = 160,000
Page |2

4. On January 1, 2006, Pencil Company granted Mr. Mongol Staedtler Rotring, its president, 5,000
stock appreciation rights for future services to be rendered. The rights are exercisable and expire
three years thereafter beginning December 31, 2007. On exercise, Rotring is entitled to receive
cash for the excess of the market value of the stock on the exercise date over the market value on
the grant date. Rotring exercised all of the rights on December 31, 2007. The per share market
prices of Pencil’s stock were as follows:

January 1, 2006 ₱25


December 31, 2006 30
December 31, 2007 40

As a result of the stock rights, how much should Pencil recognize as compensation expense in 2007?
a. 62,500
b. 66,200
c. 68,500
d. 73,500

A (5,000 SARs x (40-25) x 2/2) – (5,000 SARs x (30-25) x ½)= 62,500

Use the following information for the next three questions:


On January 1, 20x1, PLUSH LUXORIOUS Co. granted 1,000 share appreciation rights (SARs) to
employees with the condition that the employees remain in service for the next three years.
Information on the SARs is shown below:
Date Number of SARs expected to vest Fair value of each SAR
Jan. 1. 20x1 1,000 40
Dec. 31, 20x1 900 48
Dec. 31, 20x2 800 60
Dec. 31, 20x3 750 64

All of the 750 SARs that vested were exercised on December 31, 20x3. The intrinsic value (which is
equal to the cash paid out) is equal to the fair value of the SARs of ₱64 on December 31, 20x3.

5. How much is the salaries expense recognized in 20x1?


a. 16,400 b. 14,400 c. 13,333 d. 0

B 900 x 48 x 1/3 = 14,400

6. How much is the accrued liability as of December 31, 20x2?


a. 19,000 b. 35,000 c. 32,000 d. 14,000

C 800 x 60 x 2/3 = 32,000

7. How much is the salaries expense recognized in 20x3?


a. 18,000 b. 12,400 c. 16,000 d. 0
Page |3

C 750 x 64 x 3/3 – 32,000 = 16,000

8. Doc, a public limited company, has purchased inventory of P100,000. The company has offered
the supplier a choice of settlement alternatives. The alternatives are either receiving 1,000 shares
of Doc six months after the purchase date (valued at P110,000 at the date of purchase) or
receiving a cash payment equal to the fair value of 800 shares as of December 31, 20X4
(estimated value P90,000 at the date of purchase). What should be the accounting entry at the
date of purchase of the inventory?
a. Inventory P90,000, liability P90,000.
b. Inventory P100,000, liability P100,000.
c. Inventory P100,000, liability P110,000, intangible asset P10,000.
d. Inventory P100,000, liability P90,000, equity P10,000.

Use the following information for the next two questions:


On January 1, 20x1, METTLE STRENGTH Co. issued share options to its employees. The fair value
of the share options on grant date is ₱2,000,000. The share options vest in three years. METTLE is
subject to a tax rate of 30% and is allowed a tax deduction for the intrinsic value of the share options.

9. If the intrinsic value of the share options on December 31, 20x1 is ₱1,600,000, how should
METTLE account for the tax effect of the share options?
a. recognize income tax benefit of ₱160,000 in profit or loss
b. recognize income tax benefit of ₱160,000 in equity
c. recognize income tax benefit of ₱133,336 in equity
d. recognize income tax benefit of ₱133,336 in profit or loss

A (1.6M tax deduction – 2M) = 0 excess tax deduction;


1.6M tax deduction x 1/3 x 30% = 160,000 recognized in P/L

10. If the intrinsic value of the share options on December 31, 20x1 is ₱2,400,000, how should
METTLE account for the tax effect of the share options?
a. recognize income tax benefit of ₱40,000 in profit or loss
b. recognize income tax benefit of ₱40,000 in equity
c. recognize income tax benefit of ₱166,667 in equity
d. recognize income tax benefit of ₱166,667 in profit or loss

B (2.4M tax deduction – 2M) = .4M excess tax deduction x 1/3 x 30% = 40,000 recognized in equity

“Therefore encourage one another and build each other up, just as in fact you are
doing.” – (1 Thessalonians 5:11)

- END -
Page |4
Page |1

Chapter 14
Book Value per Share

1. Which of the following is incorrect when computing for preference shareholders’ equity?
a. The liquidation value, or in the absence thereof, the aggregate par value, is allocated to the
preference shareholders’ equity.
b. If the preference shares are cumulative, all dividends in arrears are allocated.
c. If the preference shares are noncumulative, only the current year dividend is allocated.
d. If there are no dividends in arrears, only one-year dividends are allocated to the preference
shareholders’ equity.

2. The numerator used in the computation of book value per share is


a. market value of net assets
b. carrying amount of net assets
c. total outstanding shares
d. authorized capitalization

3. The denominator used in the computation of book value per share is


a. number of outstanding shares
b. weighted average outstanding shares
c. number of issued shares
d. number of authorized shares

4. Which of the following is excluded when computing for the number of shares outstanding?
a. treasurer shares c. issued shares
b. subscribed but unpaid shares d. treasury shares

5. Which of the following is incorrect when computing for book value per share when there are
fully participating preference shares?
a. The ordinary shareholders’ equity is allocated one year dividend in arrears.
b. The amount subject to participation is allocated to participating preference shares and
ordinary shares based on aggregate par values of outstanding shares.
c. Only participating preference shares and ordinary shares share on the amount subject to
allocation.
d. The ordinary shareholders’ equity is allocated the aggregate par value before participation
by participating preference shares.

6. Boe Corporation's stockholders' equity at December 31, 2008 was as follows:


6% noncumulative preference shares, ₱100 par (liquidation value ₱105 per share) 1,000,000
Ordinary shares, ₱100 par 3,000,000
Retained earnings 950,000

Preferred dividends have been paid up to December 31, 2008.

At December 31, 2008, Boe's book value per common share was
Page |2

a.131.70 b.130.00 c.129.70 d.128.00

B
Solution:
Total shareholders' equity 4,950,000
Preference shareholders' equity:
Liquidation value (10,000 shares x ₱105) 1,050,000
Dividends in arrears - (1,050,000)
Ordinary shareholders' equity 3,900,000

Ordinary shareholders' equity 3,900,000


Divide by: No. of ordinary shares outstanding 30,000
Book value per share (Ordinary shares) 130.00

7. Nova Corporation has an authorized capital of 10,000 shares of ₱100 par, 8% cumulative
preferred stock and 20,000 shares of ₱100 par common stock. The equity account balances at
December 31, 2008 are as follows:

Cumulative preferred stock 500,000


Common stock 1,100,000
Additional paid in capital 200,000
Retained earnings 260,000
Treasury stock, common-1,000 shares at cost (150,000)
Total shareholders' equity 1,910,000

Dividends on preferred stock are in arrears for 2007 and 2008. The book value of a share of common
stock at December 31, 2008 should be
a. 125 b. 191 c. 133 d. 141

C
Solution:
Total shareholders' equity 1,910,000
Preference shareholders' equity:
Aggregate par value 500,000
Dividends in arrears (500K x 8% x 2 yrs.) 80,000 (580,000)
Ordinary shareholders' equity 1,330,000
Ordinary shareholders' equity 1,330,000
Divide by: No. of ordinary sh. outstanding
10,000
(1.1M / 100par) - 1,000 treasury sh.
Book value per share (Ordinary shares) 133.00

8. Georgia, Inc. has an authorized capital of 1,000, ₱100 par, 8% cumulative preference shares and
100,000, ₱10 par, ordinary shares. The equity account balances at December 31, 20x1, are as
follows:

Cumulative preference share 50,000


Ordinary share 90,000
Share premium 9,000
Page |3

Retained earnings 13,000


Treasury shares, ordinary – 100 shares at cost (2,000)
Total 160,000

Dividends on preferred stock are in arrears for the year 20x1. The book value per ordinary share at
December 31, 20x1, should be
a. 11.78 b. 11.91 c. 12.22 d. 12.36

B
Solution:
Total shareholders' equity 160,000
Preference shareholders' equity:
Par value 50,000
Dividends in arrears (50,000 x 8%) 4,000 (54,000)
Ordinary shareholders' equity 106,000
Divide by: Ordinary shares outstanding* 8,900
Book value per share (Ordinary shares) 11.91

* (₱90,000 ÷ ₱10 par) – 100 treasury shares = 8,900 shares

9. Hoyt Corp.’s current balance sheet reports the following stockholders’ equity:

5% cumulative preference shares, ₱100 par value 250,000


Ordinary share, par value ₱3.50 per share 350,000
Share premium on ordinary shares 125,000
Retained earnings 300,000

Dividends in arrears on the preference share amount to ₱25,000. If Hoyt were to be liquidated, the
preference stockholders would receive par value plus a premium of ₱50,000. The book value per
ordinary share is
a. 7.75 b. 7.50 c. 7.25 d. 7.00

D
Solution:
Total shareholders' equity 1,025,000
Preference shareholders' equity:
Liquidation value (250K par + 50K premium) 300,000
Dividends in arrears 25,000 (325,000)
Ordinary shareholders' equity 700,000
Divide by: Ordinary shares outstanding (350K ÷ ₱3.50) 100,000
Book value per share (Ordinary shares) 7.00

10. Maga Corp.'s shareholders' equity at December 31, 20x1, comprised the following:
6% cumulative preference share, ₱100 par; liquidating value ₱110
per share; authorized, issued, and outstanding 50,000 shares 5,000,000
Ordinary share, ₱5 par; 1,000,000 shares authorized; issued
and outstanding 400,000 shares 2,000,000
Page |4

Retained earnings 1,000,000

Dividends on preferred stock have been paid through 20x0 but have not been declared for 20x1. At
December 31, 20x1, Maga's book value per ordinary share was
a. 5.50 b. 6.25 c. 6.75 d. 7.50

A
Solution:
Total shareholders' equity 8,000,000
Preference shareholders' equity:
Liquidation value (50,000 x ₱110) 5,500,000
Dividends in arrears (5,000,000 x 6%) 300,000 (5,800,000)
Ordinary shareholders' equity 2,200,000
Divide by: Ordinary shares outstanding (2,000,000 ÷ ₱5) 400,000
Book value per share (Ordinary shares) 5.50

“When you pass through the waters, I will be with you; and when you pass through
the rivers, they will not sweep over you. When you walk through the fire, you will
not be burned; the flames will not set you ablaze.” - (Isaiah 43:2)

- END -
Page |1

Chapter 15
Earnings Per Share

1. What is the correct treatment of a stock dividend issued in mid-year when computing the
weighted-average number of common shares outstanding for earnings per share purposes?
a. The stock dividend should be weighted by the length of time that the additional number of
shares are outstanding during the period.
b. The stock dividend should be included in the weighted-average number of common shares
outstanding only if the additional shares result in a decrease of 3 percent or more in earnings
per share.
c. The stock dividend should be weighted as if the additional shares were issued at the
beginning of the year.
d. The stock dividend should be ignored since no additional capital was received.

2. The EPS computation that is forward-looking and based on assumptions about future
transactions is
a. diluted EPS.
b. basic EPS.
c. continuing operations EPS.
d. extraordinary EPS.

3. When computing diluted earnings per share, stock options are


a. recognized only if they are dilutive.
b. recognized only if they are antidilutive.
c. recognized only if they were exercised.
d. ignored.

4. Of the following, select the incorrect statement concerning earnings per share.
a. During periods when all income is paid out as dividends, earnings per share and dividends
per share under a simple capital structure would be identical.
b. Under a simple capital structure, no adjustment to shares outstanding is necessary for a
stock split on the last day of the fiscal period.
c. During a period, changes in stock issued or reacquired by a company may affect earnings
per share.
d. During a loss period, the amount of loss attributed to each share of common stock should be
computed.

5. In applying the treasury stock method of computing diluted earnings per share, when is it
appropriate to use the average market price of common stock during the year as the assumed
repurchase price?
a. Always
b. Never
c. When the average market price is higher than the exercise price
d. When the average market price is lower than the exercise price
Page |2

6. On January 1, 20x6, Hage Corporation granted options to purchase 9,000 of its ordinary shares at
₱7 each. The market price was ₱10.50 per ordinary share on March 31, 20x6, and averaged ₱9 per
share during the quarter then ended. There was no change in the 50,000 shares of outstanding
common stock during the quarter ended March 31, 20x6. Profit for the quarter was ₱8,268. The
number of shares to be used in computing diluted earnings per share for the quarter is
a. 59,000 b. 50,000 c. 53,000 d. 52,000

D
Solution:
Option shares 9,000
Multiply by: Total exercise price 7
Proceeds from assumed exercise of options 63,000
Divide by: Average market price 9
Treasury shares assumed to have been purchased 7,000

Option shares 9,000


Treasury shares assumed to have been purchased (7,000)
Incremental shares 2,000
Add: Actual shares outstanding 50,000
Total weighted average shares outstanding 52,000

7. The 20x7 profit of Mack Co. was ₱100,000, and 100,000 shares of its common stock were
outstanding during the entire year. In addition, there were outstanding options to purchase
10,000 shares of common stock at ₱10 per share. These options were granted in 20x5 and none
had been exercised by December 31, 20x7. Market prices of Mack's common stock during 20x7
were:
January 1 ₱20 per share
December 31 ₱40 per share
Average price ₱25 per share

The amount which should be shown as Mack's diluted earnings per share for 20x7 is (rounded to the
nearest cent)
a. ₱100,000 ÷ 110,000 shares = ₱.91 c. ₱100,000 ÷ 106,000 shares = ₱.94
b. ₱100,000 ÷ 105,000 shares = ₱.95 d. ₱100,000 ÷ 107,500 shares = ₱.93

C
Solution:
Option shares 10,000
Multiply by: Total exercise price 10
Proceeds from assumed exercise of options 100,000
Divide by: Average market price 25
Treasury shares assumed to have been purchased 4,000

Option shares 10,000


Page |3

Treasury shares assumed to have been purchased (4,000)


Incremental shares 6,000
Add: Actual shares outstanding 100,000
Total weighted average shares outstanding 106,000

8. Jones Corp.'s capital structure was as follows: December 31


Outstanding shares of stock: 20x5 20x4
Ordinary 110,000 110,000
Convertible preference shares 10,000 10,000
8% convertible bonds 1,000,000 1,000,000

During 20x5, Jones paid dividends of ₱3.00 per share on its preference shares. The preference shares
are convertible into 20,000 ordinary shares. The 8% bonds are convertible into 30,000 ordinary
shares. Profit for 20x5 is ₱850,000. The income tax rate is 30%. The diluted earnings per share for
20x5 is
a. 5.48 b. 5.66 c. 5.81 d. 6.26

B
Solution:
The multiple potential ordinary shares are ranked in accordance with their dilutive effect as follows:
Incremental Incremental Incremental
Potential ordinary shares
earnings shares EPS Rank
a b c=a÷b
a. Convertible PS 30,000 20,000 1.50 1st
(₱3 x 10,000); (20,000)
b. Convertible bonds 56,000 30,000 1.87 2nd
(1,000,000 x 8% x 70%);
(30,000)

Diluted EPS is calculated by gradually considering the potential ordinary shares starting with the
first in rank as shown below:
Ordinary
Profit shares EPS
a b c=a÷b
Basic EPS from continuing
operations 820,000* 110,000 7.45
Convertible PS - (1st) 30,000 20,000
Diluted EPS #1 850,000 130,000 6.54 Dilutive
Convertible Bonds - (2nd) 56,000 30,000
Diluted EPS #2 906,000 160,000 5.66 Dilutive

* Numerator for basic EPS, net of preferred dividends: (850,000 – 30,000) = 820,000.

9. Information relating to the capital structure of the Galaxy Company is as follows:


Outstanding shares of: Dec. 31, 20x5 Dec. 31, 20x6
Page |4

Ordinary shares 90,000 90,000


Convertible preference shares 10,000 10,000
9% convertible bonds 1,000,000 1,000,000

During 20x6 Galaxy paid dividends of ₱6.00 per share on its preference shares. The preference share
is convertible into 10,000 ordinary shares. The 9% convertible bonds are convertible into 30,000
ordinary shares. The profit for the year ended December 31, 20x6, is ₱485,000. The income tax rate is
50%. What should be the diluted earnings per share for the year ended December 31, 20x6?
a. 3.79 b. 3.92 c. 4.08 d. 4.72
B
Solution:
The multiple potential ordinary shares are ranked in accordance with their dilutive effect as follows:
Incremental Incremental Incremental
Potential ordinary shares
earnings shares EPS Rank
a b c=a÷b
a. Convertible PS 60,000 10,000 6.00 2nd
(₱6.00 x 10,000); (10,000)
b. Convertible bonds 45,000 30,000 1.50 1st
(1,000,000 x 9% x 50%);
(30,000)

Diluted EPS is calculated by gradually considering the potential ordinary shares starting with the
first in rank as shown below:

Ordinary
Profit shares EPS
a b c=a÷b
Basic EPS from continuing
operations 425,000* 90,000 4.83
Convertible Bonds - (1st) 45,000 30,000
Diluted EPS #1 470,000 120,000 3.92 Dilutive
Convertible PS - (2nd) 60,000 10,000
Anti-
Diluted EPS #2 530,000 130,000 4.08 dilutive

* Numerator for basic EPS, net of preferred dividends: (485,000 – 60,000) = 425,000

10. Throughout 1998, J Co. had 10,000 ordinary shares outstanding. There was no potential dilution
of earnings per share except as follows:

In 20x7, J Co. agreed to issue 2,000 additional shares of its stock to the former stockholders of an
acquired company if the acquired company's earnings for any of the five years 20x8 through 2x12
exceeded ₱5,000.
Page |5

Results of operations for 20x8 were:


Profit of J Co. ₱10,000
Profit of acquired company 4,000
Consolidated profit ₱14,000

Diluted earnings per share for 1998 on a consolidated basis would be


a. ₱14,000 ÷ 10,000 = ₱1.40 c. ₱15,000 ÷ 10,000 = ₱1.50
b. ₱14,000 ÷ 12,000 = ₱1.17 d. ₱15,000 ÷ 12,000 = ₱1.25

A The contingent shares are ignored because the condition is not met.

“Have I not commanded you? Be strong and courageous. Do not be afraid; do not
be discouraged, for the LORD your God will be with you wherever you go.” – (Joshua
1:9)

- END -
Page |6

SOLUTIONS:
1. C
2. A
3. A
4. B
5. C

6. D
Solution:
Option shares 9,000
Multiply by: Total exercise price 7
Proceeds from assumed exercise of options 63,000
Divide by: Average market price 9
Treasury shares assumed to have been purchased 7,000

Option shares 9,000


Treasury shares assumed to have been purchased (7,000)
Incremental shares 2,000
Add: Actual shares outstanding 50,000
Total weighted average shares outstanding 52,000

7. C
Solution:
Option shares 10,000
Multiply by: Total exercise price 10
Proceeds from assumed exercise of options 100,000
Divide by: Average market price 25
Treasury shares assumed to have been purchased 4,000

Option shares 10,000


Treasury shares assumed to have been purchased (4,000)
Incremental shares 6,000
Add: Actual shares outstanding 100,000
Total weighted average shares outstanding 106,000

8. B
Solution:
The multiple potential ordinary shares are ranked in accordance with their dilutive effect as follows:
Incremental
Potential ordinary shares Incremental shares Incremental EPS Rank
earnings
a b c=a÷b
c. Convertible PS 30,000 20,000 1.50 1st
(₱3 x 10,000); (20,000)
d. Convertible bonds 56,000 30,000 1.87 2nd
(1,000,000 x 8% x 70%);
(30,000)

Diluted EPS is calculated by gradually considering the potential ordinary shares starting with the first in
rank as shown below:
Profit Ordinary shares EPS
a b c=a÷b
Basic EPS from continuing operations 820,000* 110,000 7.45
Convertible PS - (1st) 30,000 20,000
Diluted EPS #1 850,000 130,000 6.54 Dilutive
Convertible Bonds - (2nd) 56,000 30,000
Page |7

Diluted EPS #2 906,000 160,000 5.66 Dilutive

* Numerator for basic EPS, net of preferred dividends: (850,000 – 30,000) = 820,000.

9. B
Solution:
The multiple potential ordinary shares are ranked in accordance with their dilutive effect as follows:
Incremental
Potential ordinary shares Incremental shares Incremental EPS Rank
earnings
a b c=a÷b
c. Convertible PS 60,000 10,000 6.00 2nd
(₱6.00 x 10,000); (10,000)
d. Convertible bonds 45,000 30,000 1.50 1st
(1,000,000 x 9% x 50%);
(30,000)

Diluted EPS is calculated by gradually considering the potential ordinary shares starting with the first in
rank as shown below:

Profit Ordinary shares EPS


a b c=a÷b
Basic EPS from continuing operations 425,000* 90,000 4.83
Convertible Bonds - (1st) 45,000 30,000
Diluted EPS #1 470,000 120,000 3.92 Dilutive
Convertible PS - (2nd) 60,000 10,000
Diluted EPS #2 530,000 130,000 4.08 Anti-dilutive

* Numerator for basic EPS, net of preferred dividends: (485,000 – 60,000) = 425,000.

10. A The contingent shares are ignored because the condition is not met.
Page |1

1. The accounting standards used in the Philippines are adapted from the standards issued by the
a. Federal Accounting Standards Board (FASB).
b. International Accounting Standards Board (IASB).
c. Philippine Institute of Certified Public Accountants (PICPA).
d. Democratic People's Republic of Korea Accounting Standards Committee (DPKRASC).

2. The PFRSs consist of all of the following except


a. PFRSs.
b. PASs.
c. Interpretations.
d. Conceptual Framework.

3. The issuance of financial reporting standards in the Philippines is the responsibility of the
a. PICPA
b. FRSC
c. AASC
d. CPE Council

4. On November 1, 20x1, a company purchased a new machine that it does not have to pay for
until November 1, 20x3. The total payment on November 1, 20x3, will include both principal and
interest. Assuming interest at a 10% rate, the cost of the machine would be the total payment
multiplied by what time value of money concept?
a. PV of annuity of ₱1. c. FV of annuity of ₱1.
b. PV of ₱1. d. FV of ₱1.

5. Interest payment dates of a bond issue are March 1 and September 1, 20x1. The bond was issued
on June 1, 20x1. Interest expense for the year ended December 31, 20x1 would be for:
a. four (4) months c. seven (7) months
b. six (6) months d. ten (10) months

6. When a note payable is issued for property, goods, or services, the note is initially measured at
a. the fair value of the property, goods, or services.
b. the fair value of the note.
c. using an imputed interest rate to discount all future payments on the note.
d. choice (a) except when this is not determinable, in which case, whichever is the more clearly
determinable between (b) and (c).

7. When a note payable is exchanged for property, goods, or services, the stated interest rate is
presumed to be fair unless
a. no interest rate is stated.
b. the stated interest rate is unreasonable.
c. the stated face amount of the note is materially different from the current cash sales price for
similar items or from current market value of the note.
d. any of these.
Page |2

8. When debt is issued at a discount, interest expense over the term of the debt equals the cash
interest paid:
a. Minus discount. c. Plus discount.
b. Minus discount minus face amount. d. Plus discount plus face amount.

9. Which of the following statements is true?


a. A noninterest-bearing note sometimes is called a discounted note because the cash received
is more than the face amount of the note.
b. A debtor’s December 31, 20x1 statement of financial position is to be published on March 31,
20x2. An obligation with a due date of December 31, 20x6 is also due on demand by the
creditor. At December 31, 20x1, there is no indication that the creditor intends to call in the
debt. The obligation is a current liability.
c. The market rate of interest is the interest rate used to determine the amount of cash interest
that will be paid on the principal.
d. A debtor’s December 31, 20x1 statement of financial position is to be published on March 31,
20x2. An obligation due December 31, 20x6 has a due date which can be accelerated by the
creditor to the present date if the current ratio falls below 2:1. The current ratio on December
31, 20x1 is 2.2:1. The obligation is a current liability.

10. A short-term note payable may include all of the following except:
a. trade notes payable. c. unearned revenue.
b. nontrade notes payable. d. a current maturity of a long-term liability.

11. Interest expenses are


a. incurred only on interest-bearing obligations
b. incurred due to passage of time.
c. not incurred on redeemable preference shares issued
d. incurred only when the effective interest rate is stated in the instrument

12. Which of the following is not true about the discount on short-term notes payable?
a. The Discount on Notes Payable account has a debit balance.
b. The Discount on Notes Payable account should be reported as an asset on the balance sheet.
c. When there is a discount on a note payable, the effective interest rate is higher than the
stated discount rate.
d. All of these are true.

13. Which of the following statements is not correct?


a. The principal amount of a debt is the cash or cash equivalent amount borrowed.
b. When a noncash asset is acquired and the stated rate of interest is different from the current
market rate of interest, the cost of the asset is the present value of the future cash payments
discounted at the current market rate of interest rather than at the stated interest rate.
c. A company that receives cash in an amount less than the face amount of a noninterest-
bearing note payable should record the note at its discounted present value.
d. The carrying amount of a noninterest-bearing note payable due in lump sum will decrease as
time goes by.
Page |3

Use the following information for the next five questions:


On January 1, 20x1, ABRIDGE TO SHORTEN Company issued a 4-year, ₱1,000,000 noninterest
bearing note payable due in four equal annual installments. The effective interest rate is 12%.
ABRIDGE prepared the following pro-forma amortization table on an electronic spreadsheet:
A B C D E
1 Date Cash paid Interest expense Amortization Present value
2 Jan. 1, 20x1
3 Dec. 31, 20x1
4 Dec. 31, 20x2 250,000
5 Dec. 31, 20x3 Current 20x2 Non-current 20x2
6 Dec. 31, 20x4 0

14. The amount to be placed on cell E2 is


a. (1M ÷ 4 x PV of ordinary annuity of ₱1 @ 12%, n=4)
b. (1M x PV of ₱1 @12%, n=4)
c. (1M x PV of ordinary annuity of ₱1 @12%, n=4)
d. (1M x PV of ₱1 @12%, n=4) + (1M x 10% x PV of ordinary annuity of ₱1 @ 12%, n=4)

15. The amount to be placed on cell E6 is


a. (1M ÷ 4 x PV of ordinary annuity of ₱1 @ 12%, n=4) c. 1M
b. (1M x PV of ₱1 @12%, n=4) d. 0

16. Interest expense recognized in 20x2 is computed as


a. 12% x E3 c. C4 – D4
b. 12% x E4 d. 1M x 12%

17. The carrying amount of the note payable on December 31, 20x2 is equal to
a. E3 – D4 c. E4 – D4
b. E3 + D4 d. 1M

18. The value placed in cell B4 is equal to


a. 1M x 12% c. 1M – D3
b. 250,000 d. E4 – D5

19. The current portion of the note payable as of December 31, 20x2 is equal to
a. D4 c. D5
b. D3 d. E5

20. The noncurrent portion of the note payable as of December 31, 20x2 is equal to
a. E4 c. E3
b. D5 d. E5

Use the following information for the next nine questions:


On January 1, 20x1, HEARTEN ENCOURAGE CHEER Company issued a 4-year, ₱1,000,000,
noninterest-bearing note due on December 31, 20x4. The effective interest rate is 12%. HEARTEN
prepared the following pro-forma amortization table on an electronic spreadsheet:
Page |4

A B C D
Interest Present
Date Discount
1 expense value
2 Jan. 1, 20x1
3 Dec. 31, 20x1
4 Dec. 31, 20x2
5 Dec. 31, 20x3
6 Dec. 31, 20x4

21. The amount to be placed on cell B4 is


a. 10% x E3 c. ₱1M ÷ 4
b. 12% x D3 d. same with B3

22. The amount to be placed on cell D2 is computed as


a. (1M x PV of ₱1 @12%, n=4) + (1M x PV of ordinary annuity of ₱1 @ 12%, n=4)
b. (1M x PV of ₱1 @12%, n=4)
c. (1M x PV of ordinary annuity of ₱1 @12%, n=4)
d. (1M x PV of ₱1 @12%, n=4) + (1M x 10% x PV of ordinary annuity of ₱1 @ 12%, n=4)

23. Interest expense recognized in 20x3 is computed as


a. 12% x D3 c. C4 – D4
b. 12% x D4 d. 1M x 12%

24. The amount to be placed in cell C3 is computed as


a. C2 + B3 c. equal to C4
b. C2 – B3 d. I’m confused

25. The carrying amount of the note payable on December 31, 20x2 is equal to
a. D3 – B4 c. B4 + C4
b. D3 + B4 d. D3 + C4

26. The current portion of the note payable as of December 31, 20x2 is equal to
a. D4 c. D5
b. D3 d. none

27. The noncurrent portion of the note payable as of December 31, 20x2 is equal to
a. E4 c. E3
b. D5 d. none of these

28. The sum of cell C4 and cell D4 is


a. equal to D3 c. 1M
b. equal to D5 d. none of these

29. The value of cell D6 is


a. equal to D3 c. 1M
Page |5

b. equal to D5 d. zero

30. Which of the following statements about noninterest-bearing notes is false?


a. The face amount of a noninterest-bearing note may include both the principal and interest as
a single amount to be paid back at maturity date.
b. The principal amount of a noninterest-bearing note is its future cash flows discounted at its
effective interest rate.
c. The effective rate on a short-term noninterest-bearing note, with a specified term, cannot be
determined unless it is given on the face of the note.
d. Noninterest bearing is not a descriptive designation for this type of note because such notes
do bear interest.

31. Inter Company sells its products in reusable, expensive containers. The customer charged a
deposit for each container delivered and receives a refund for each container returned within
two years after the year of delivery. Inter accounts for the containers not returned within the
time limit as being retired by sale at the deposit amount. Information for 2006 is as follows:

Deposits for containers at December 31, 2005 from deliveries in:


2004 P 150,000
2005 430,000 P 580,000
Deposits for containers delivered in 2006 780,000
Deposits for containers returned in 2006 form deliveries in:
2004 P 90,000
2005 250,000
2006 286,000 626,000

What amount should Inter Company report as a liability for deposits on returnable containers at
December 31, 2006?
a. 494,000
b. 644,000
c. 674,000
d. 734,000

C (430,000 + 780,000 – 250,000 – 286,000) = 674,000

32. Impressed Company, a division of Philippine Realty Corporation maintains escrow accounts
and pays real estate taxes for Philippine’s mortgage customers. Escrow funds are kept in
interest-bearing accounts. Interest, less a 10% service fee, is credited to the mortgagee’s account
and used to reduce future escrow payments. Additional information follows:

Escrow accounts liability, January 1, 2008 P 900,000


Escrow payments received during 2008 1,500,000
Real estate taxes paid during 2008 1,900,000
Interest on escrow funds during 2008 90,000
Page |6

What amount should Impressed report as escrow accounts liability in its December 31, 2008 balance
sheet?
a. 491,000
b. 500,000
c. 581,000
d. 590,000

C 900,000 + 1,500,000 – 1,900,000 + (90,000 x 90%) = 581,000

33. Gallery Department Store sells gift certificates, redeemable for store merchandise that expires
one year after their issuance. Gallery has the following information pertaining to its gift
certificates sales and redemptions:

Unearned at December 31, 2005 P 600,000


2006 sales 2,000,000
2006 redemptions of prior-year sales 200,000
2006 redemptions of current-year sales 1,400,000

Gallery’s experience indicates that 10% of gift certificates sold will not be redeemed.

In its December 31, 2006 balance sheet, what amount should Gallery report as unearned revenue?
a. 400,000
b. 600,000
c. 800,000
d. 1,000,000

A 2,000,000 – 1,400,000 – (2,000,000 x 10%) = 400,000

34. Ivy Co. operates a retail store. All items are sold subject to a 6% state sales tax, which Ivy collects
and records as sales revenue. Ivy files quarterly sales tax returns when due, by the 20th day
following the end of the sales quarter. However, in accordance with state requirements, Ivy
remits sales tax collected by the 20th day of the month following any month such collections
exceed ₱500. Ivy takes these payments as credits on the quarterly sales tax return. The sales taxes
paid by Ivy are charged against sales revenue. Following is a monthly summary appearing in
Ivy's first quarter 2002 sales revenue account:
Debit Credit
January - 10,600
February 600 7,420
March - 8,480
600 26,500

In its March 31, 20x2, balance sheet, what amount should Ivy report as sales taxes payable
a. 600 b. 900 c. 1,500 d. 1,590

B
Solution:
Page |7

Total sales inclusive of sales tax (total credit) 26,500


Multiply by: 6%/106%
Total sales taxes collected 1,500
Remittance of sales tax in February (600)
Sales taxes payable 900

35. On January 1, 20x1 WRECK RUIN Co. acquired land by issuing a three-year, 12%, ₱4,000,000
note payable. Principal and interest are due on December 31, 20x3. How much is the interest
expense in 20x2?
a. 1,017,600 c. 537,600
b. 960,000 d. 764,213

C (4,000,000 x 112% x 12%) = 537,600

36. Karma Company sells televisions at an average price of P7,500 and also offers to each customer a
separate 3-year warranty contract for P750 that requires the company to perform periodic
services and to replace defective parts. During 2006, the company sold 300 televisions and 270
warranty contracts for cash. It estimates the 3-year warranty costs as P200 for parts and P400 for
labor and accounts for warranties separately. Assume sales occurred on December 31, 2008,
income is recognized on the warranties, and straight line recognition of warranty revenues
occurs.

What amount of current and non-current liability relative to warranty revenue would appear on the
December 31, 2009 balance sheet, respectively?
a. 0 and 202,500
b. 67,500 and 135,000
c. 135,000 and 67,500
d. 202,500 and 0
B (P750 x 270)= P202,500 / 3 = P67,500 to be earned in 2009 (sale occurred on December 31, 2008) P135,000
(P67,500 x 2) will be earned in 2010 and 2011.

37. ABC Co. is contemplating on issuing a 12%, 3-year, ₱1,000,000 bonds. Principal is due at
maturity but interest is due semi-annually every July 1 and December 31. ABC determines that
the current market rate on January 1, 20x1 is 14%. How much is the estimated issue price of the
bonds assuming ABC issues bonds on January 1, 20x1?
a. 666,342
b. 285,992
c. 952,334
d. 962,563

Solution: Issue price of bonds = Present value of future cash flows


Future cash flows PV @ 7%, n=6 PV factors Present value
Principal 1M PV of ₱1 0.666342 666,342
Interest 60K PV of ordinary annuity of ₱1 4.766540 285,992
Estimated issue price of the bonds on Jan. 1, 20x1 952,334

Use the following information for the next three questions:


Page |8

On January 1, 20x1, SCRAWNY SKINNY Co. issued 1,000, ₱4,000, 10%, 3-year bonds for ₱3,807,852.
Principal is due on December 31, 20x3 but interests are due annually every year-end. In addition,
SCRAWNY incurred bond issue costs of ₱179,316. The effective interest rate is 12% before adjustment
for bond issue costs and 14% after adjustment for bond issue costs.

38. How much is the carrying amount of the note on initial recognition?
a. 3,628,536 b. 4,000,000 c. 3,635,340 d. 3,754,309

A (3,807,852 – 179,316) = 3,628,536

39. How much is the interest expense in 20x1?


a. 435,424 b. 576,240 c. 507,995 d. 400,000

C Solution:
Interest
Date payments Interest expense Amortization Present value
Jan. 1, 20x1 3,628,536
Dec. 31, 20x1 400,000 507,995 107,995 3,736,531

40. How much is the carrying amount of the note on December 31, 20x1?
a. 3,401,832 b. 3,391,580 c. 3,288,776 d. 3,736 ,531

D (See amortization table above)

41. Entity A issues convertible bonds with face amount of ₱2,000,000 for ₱2,600,000. Each ₱1,000
bond is convertible into 10 shares with par value of ₱60 per share. On issuance date, the bonds
are selling at 102 without the conversion option. What is the value allocated to the equity
component on initial recognition?
a. 2,040,000
b. 540,000
c. 560,000
d. 460,000

Solution:
Issue price 2,600,000
Fair value of debt instrument without equity feature (2M x 102%) (2,040,000)
Equity component 560,000

42. On September 30, 20x1, ADMONISH WARN Co. issued new bonds with face amount of ₱10M
for a net issuance proceeds of ₱43,200,000. ADMONISH used the proceeds to retire an existing
10-year, 12%, ₱32,000,000 bonds issued five years earlier. The bonds have an unamortized
discount of ₱1,360,000 as of September 30, 20x1. ADMONISH reacquired the entire outstanding
bonds at a call premium of ₱1,600,000. Costs incurred that are directly attributable to the
retirement amounted to ₱200,000. ADMONISH has an income tax rate of 30%. How much is the
gain (loss) on the retirement of the bonds to be recognized in 20x1?
a. 3,160,000) b. (2,960,000) c. 2,960,000 d. (3,160,000)
Page |9

D
Solution:
Jan. 1, Bonds payable – old 32,000,000
20x1 3,160,000
Loss on extinguishment of bonds (squeeze)
Discount on bonds payable – old 1,360,000
33,800,000
Cash in bank
(32M + 1.6M call premium + 200K reacquisition costs)

43. On January 1, 20x1, POTENT POWERFUL Co. issued 5-year, 12%, ₱4,000,000 bonds for
₱4,303,264. Principal is due at maturity but interests are due annually. The effective interest rate
is 10%. On July 1, 20x3, POTENT called in the entire bonds and retired them at 102. The
retirement price includes payment for any accrued interest. How much is the gain (loss) on the
extinguishment of the bonds?
a. 328,897 b. (328,896) c. (118,948) d. 118,948

A
Solution:
Date Interest payments Interest expense Amortization Present value
Jan. 1, 20x1 4,303,264
Dec. 31, 20x1 480,000 430,328 49,672 4,253,592
Dec. 31, 20x2 480,000 425,360 54,640 4,198,948
July 1, 20x3 240,000 209,948 30,052 4,168,896

Carrying amount of bonds retired: (see table above) 4,168,896


Retirement price (Call price):
Retirement price including payment for
accrued interest (4M x 102%) 4,080,000
Accrued interest (4M x 12% x 6/12) (240,000) 3,840,000
Gain on extinguishment of bonds 328,896

44. On January 1, 20x1, TIPSY UNSTEADY Co. issued 10%, ₱12,000,000 bonds for ₱11,601,220.
Principal on the bonds matures in three equal annual installments. Interest is also due annually
at each year-end. The effective interest rate on the bonds is 12%. How much is the carrying
amount of the bonds on December 31, 20x1?
a. 7,844,635 b. 7,793,366 c. 7,683,343 d. 7,543,341

B
Solution:
Interest on outstanding
Date Principal payments principal balance Interest payments Total payments
Dec. 31, 20x1 4,000,000 12,000,000 x 10% 1,200,000 5,200,000
Dec. 31, 20x2 4,000,000 8,000,000 x 10% 800,000 4,800,000
Dec. 31, 20x3 4,000,000 4,000,000 x 10% 400,000 4,400,000

Date Total payments Interest expense Amortization Present value


P a g e | 10

Jan. 1, 20x1 11,601,220


Dec. 31, 20x1 5,200,000 1,392,148 3,807,852 7,793,368

45. Liabilities arise from either legal or constructive obligation. Which of the following is a source of
constructive obligation?
a. contract c. quasi-contract
b. law d. an established pattern of past practice

46. According to PAS 37, provisions are measured at


a. the entity’s best estimate of the settlement amount.
b. the expected value of the settlement amount.
c. the mid-point amount of a range of estimates.
d. any of these, whichever is most appropriate

47. According to PAS 37, a provision does not arise from


a. restructuring. c. product warranties.
b. future operating losses. d. constructive obligation.

48. According to PAS 37, a provision is


a. a present obligation that cannot be measured reliably.
b. a possible obligation that arises from past events.
c. a liability of uncertain timing or amount.
d. all of these

49. According to PAS 37, contingent liabilities are


a. recognized and disclosed.
b. always disclosed.
c. disclosed only, if their expected occurrence is probable.
d. not disclosed if their expected occurrence is remote.

50. Which of the following statements is correct?


a. A provision is recognized only when it represents a present obligation.
b. An event or transaction that meets both the “probable outflow of economic benefits” and
“reliable measurement” criteria is always recognized.
c. A contingent asset that is possible is ignored.
d. A contingent liability that is possible is ignored.

51. In 20x1, EXHAUSTIVE COMPLETE Co. received a court order requiring the cleanup of
environmental damages caused by one of EXHAUSTIVE’s factory. EXHAUSTIVE has no other
realistic alternative but to comply with the court order. Other entities have incurred around
₱60M for similar cleanup; however, EXHAUSTIVE’s best estimate of the cost of cleanup is ₱80M.
How much is the provision to be recognized?
a. 60M b. 80M c. 70M d. 0

B 80,000,000 – the best estimate


P a g e | 11

52. In 20x1, LUMINOUS SHINING Co. recalled a product due to a possible defect caused by a
malfunctioning factory equipment. The products recalled will be repaired free of charge.
LUMINOUS is uncertain whether all products recalled will have the possible defect. However,
the following estimate was made by LUMINOUS’s engineers and managerial accountants and
approved by the board of directors.
Repair cost Probability
80,000,000 5%
60,000,000 20%
40,000,000 35%
20,000,000 40%
100%

How much is the provision to be recognized?


a. 38M b. 50M c. 48M d. 32M

A (80M x 5%) + (60M x 20%) + (40M x 35%) + (20M x 40%) = 38M

53. In 20x1, a lawsuit was filed against WINSOME CAUSING PLEASURE Co. for patent
infringement. The plaintiff is claiming ₱400M in damages. WINSOME’s legal counsel believes
that it is probable that WINSOME will lose the lawsuit and pay damages of not less than ₱40M
but not more than ₱400M. The probability of any amount within the range is as likely as any
other amount also within the range. The plaintiff has offered to settle the lawsuit out of court for
₱360M but WINSOME did not agree to the settlement. How much is provision to be reported in
WINSOME’s year-end financial statements?
a. 360M b. 220M c. 400M d. 40M

B (400M + 40M) ÷ 2 = 220M

54. A manufacturer gives warranties at the time of sale to purchasers of its product. Under the terms
of the contract of sale, the manufacturer undertakes to make good, by repair or replacement,
manufacturing defects that become apparent within one year from the date of sale. On the basis
of experience, it is probable (i.e., more likely than not) that there will be some claims under the
warranties.

Sales of ₱40 million were made evenly throughout 20X1.

At December 31, 20x1 the expenditures for warranty repairs and replacements for the product sold
in 20x1 are expected to be made 50% in 20x1 and 50% in 20x2. Assume for simplicity that all the 20x2
outflows of economic benefits related to the warranty repairs and replacements take place on June
30, 20x2.

Experience indicates that 95% of products sold require no warranty repairs; 3% of products sold
require minor repairs costing 10% of the sale price; and 2% of products sold require major repairs or
replacement costing 90% of sale price. The entity has no reason to believe future warranty claims
will be different from its experience.
P a g e | 12

At December 31, 20x1, the appropriate discount factor for cash flows expected to occur on June 30,
20x2 is 0.95238. Furthermore, an appropriate risk adjustment factor to reflect the uncertainties in the
cash flow estimates is an increment of 6 per cent to the probability-weighted expected cash flows.

How much is the warranty provision at December 31, 20x1?


a. 424,000 b. 840,000 b. 800,000 d. 752,000

A
Solution:
The amount of the provision is estimated as follows:
Minor repairs (40M x 3% x 10%) 120,000
Major repairs (40M x 2% x 90%) 720,000
Total 840,000
Multiply by: Present value factor (given) 0.95238
Total 800,000
Multiply by: Risk adjustment (100% + 6%) 106%
Total 848,000
Multiply by: Amount to be settled in 20x2 50%
Warranty provision – Dec. 31, 20x1 424,000

Use the following information for the next two questions:


RISIBLE FUNNY Co. provides 3-year warranty for the products it sells. RISIBLE estimates that
warranty costs ₱400 per unit sold. As of January 1, 20x1, the liability for warranty has a balance of
₱800,000 for units sold in 20x0. During the year RISIBLE sold 5,000 units and actual warranty costs
incurred were ₱1,240,000.

55. How much is the warranty expense to be recognized in 20x1?


a. 2,000,000 b. 1,240,000 c. 3,240,000 d. 4,240,000

A (5,000 units sold x ₱400) = 2,000,000

56. How much is the balance of the warranty obligation as of December 31, 20x1?
a. 1,560,000 b. 2,000,000 c. 3,560,000 d. 2,800,000

A
Solution:
Estimated warranty liability
800,000 Jan. 1, 20x1 (given)
Actual warranty costs 1,240,000 2,000,000 Warranty expense
Dec. 31, 20x1 1,560,000

57. It is a type of retirement plan where the employer assures a definite amount of benefit to be
received by the employee. The risk that funds needed to pay the agreed benefits may be
insufficient is retained by the employer.
a. Defined contribution plan
b. Defined benefit plan
c. Leche plan
d. Plan vs. zombies
P a g e | 13

58. Entity A’s employees are entitled to six days paid sick leaves per year. Any unused sick leave is
converted to cash when the employee resigns or retires. The sick leave benefits are considered
a. vesting . c. non-accumulating.
b. non-vesting. d. monetizing.

59. Compensated absences that can be carried forward and used in future periods if not fully used
in the current period of entitlement are referred to as
a. contributory. c. accumulating.
b. non-contributory. d. vesting.

60. Under a profit-sharing plan, Entity A agrees to pay its employees 5% of its annual profit. The
bonus shall be divided among the employees currently employed as at year-end. Relevant
information follows:

Profit for the year ₱8,000,000


Employees at the beginning of the year 8
Average employees during the year 7
Employees at the end of the year 6

If you are one of the employees of Entity A, how much bonus do you expect to receive?
a. 66,667 c. 50,000
b. 57,143 d. 0

A (8M x 5%) ÷ 6 employees currently employed as at year-end = 66,667

61. Under this post-employment benefit plan, the retirement benefit cost is equal to the contribution
due for the period.
a. Defined contribution plan c. State plan
b. Defined benefit plan d. Multi-employer plan

62. WASTREL SPENDTHRIFT Co. pays salaries twice a month and does not pay salaries in
advance. Employees work five days a week and compensation are computed on these working
days. In December 20x1, WASTREL Co. paid the second semi-monthly salaries on December 26
which falls on a Friday. The next non-working holiday is on New Year’s Day. WASTREL has 100
employees who earn ₱4,000 per day. WASTREL’s cost accountant identified that 70% of salaries
incurred pertain to the production of goods. How much is the accrued salaries as of December
31, 20x1?
a. 360,000 b. 840,000 c. 1,600,000 d. 1,200,000

D
Solution:
Working days after last salary payment (Dec. 29, 30, and 31)* 3
Multiply by: Number of employees 100
Multiply by: Average pay per day 4,000
Accrued salaries – December 31, 20x1 1,200,000
*December 27 and 28 fall on weekend
P a g e | 14

63. An entity has 100 employees, who are each entitled to five (5) working days of paid sick leave
for each year. Unused sick leave may be carried forward for one calendar year. Sick leave is
taken first out of the current year’s entitlement and then out of any balance brought forward
from the previous year (a LIFO basis). At December 30, 20x1, the average unused entitlement is
two days per employee. The entity expects, based on past experience which is expected to
continue, that 92 employees will take no more than five days of paid sick leave in 20x2 and that
the remaining 8 employees will take an average of six and a half days each. The average salary
per day, per employee in 20x1 is ₱4,000 and it is not expected to change in 20x2. How much is
the accrued salaries as of December 31, 20x1?
a. 24,000 b. 48,000 c. 208,000 d. 0

B
Solution:
Total sick leave entitlement of employees in 20x2
(100 employees x 5 days each) 500
Sick leave expected to be taken in 20x2
(92 employees x 5 days each) (460)
Sick leave expected to be taken by the remaining 8
employees in 20x2 (8 x 6½ days each) (52)
Excess sick leave carried over from 20x1 (12)

12 x ₱4,000 = 48,000

64. On January 1, 20x1, PAGEANT SHOW Co. issued 10%, ₱12,000,000 bonds at a yield to maturity
interest of 18%. Principal and interest are due on December 31, 20x3. How much is the carrying
amount of the bonds on initial recognition?
a. 15,972,000 b. 9,721,052 c. 9,028,341 d. 9,183,273

B (12M x 110% x 110% x 110%) x PV of 1 @18%, n=3 = 9,721,052

65. On January 1, 20x1, VIGILANT WATCHFUL Co. issued its 10%, 3-year, ₱4,000,000 convertible
bonds for the face amount of ₱4,000,000. Each ₱4,000 bond is convertible into 8 shares with par
value of ₱400 per share. When the bonds were issued, they were selling at 98 without the
conversion option. VIGILANT incurred ₱200,000 transaction costs on the issue of the bonds.
How much is the equity component of the compound instrument?
a. 80,000 b. 200,000 c. 76,000 d. 123,489

C
Solution:
Issue price 4,000,000
Fair value of debt instrument without equity feature (4M x 98%) (3,920,000)
Equity component 80,000
P a g e | 15

Allocation of transaction
Component Allocated amounts from issue price Fraction costs
Debt component 3,920,000 3,920/4,000 196,000
Equity component 80,000 80/4,000 4,000
4,000,000 4,000/4,000 200,000

80,000 – 4,000 = 76,000

66. On January 1, 20x1, CRYSTALLINE TRANSPARENT Co. issued its 10%, 3-year, ₱4,000,000
convertible bonds at 105. Each ₱4,000 bond is convertible into 8 shares with par value per share
of ₱400. Principal is due on December 31, 20x3 but interests are due annually at each year-end.
When the bonds were issued, they were selling at a yield to maturity market rate of 12%without
the conversion option. On December 31, 20x2, all of the bonds were converted into equity.
Conversion costs incurred amounted to ₱80,000.

How much is the net increase in equity on December 31, 20x2 due to the conversion of the
bonds?
a. 3,392,148 +b. 3,234,998 c. 3,894,759 d. 3,848,571

D
Solution:
Future cash flows PV @ 12%, n=3 PV factors Present value
Principal 4M PV of ₱1 0.711780 2,847,120
Interest 400K PV of ordinary annuity of ₱1 2.401831 960,732
Fair value of debt instrument without conversion feature 3,807,852

The issue price is allocated to the liability and equity components as follows:
Issue price (4M x 105%) 4,200,000
Fair value of debt instrument without equity feature (3,807,852)
Equity component 392,148

The entry to record the issuance is as follows:


Jan. 1, Cash in bank (4M x 105%) 4,200,000
20x1 192,148
Disc. on bonds payable (4M – 3,807,852)
Bonds payable 4,000,000
Share premium – conversion feature 392,148

Amortization table:
Date Interest payment Interest expense Amortization Present value
Jan. 1, 20x1 3,807,852
P a g e | 16

Dec. 31, 20x1 400,000 456,942 56,942 3,864,794


Dec. 31, 20x2 400,000 463,775 63,775 3,928,569

The other pertinent entries prior to conversion are as follows:


Dec. 31, Interest expense 456,942
20x1
Discount on bonds payable 56,942
400,000
Cash in bank
Dec. 31, Interest expense 463,775
20x2
Discount on bonds payable 63,775
400,000
Cash in bank

The entries on to record the conversion are as follows:


Dec. 31, Bonds payable 4,000,000
20x2 71,431
Disc. on bonds payable (4M – 3,928,569)
Share capital a 3,200,000
728,569
Share premium (squeeze)
to record the conversion of bonds into equity
Dec. 31, Share premium 80,000
20x2 80,000
Cash in bank
to record the share issuance costs
Dec. 31, Share premium – conversion feature 392,148
20x2 392,148
Share premium
to reclassify the equity component of the compound instrument within equity
a
(4M face amount ÷ ₱4,000) x 8 shares x ₱400 par value) = 3,200,000

Net increase in equity = credit to share capital of 3,200,000 + credit to share premium of 728,569 –
debit to share premium of 80,000 = 3,848,569

Use the following information for the next three questions:


On January 1, 20x1, ELABORATE COMPLICATED Co. issued 3-year, 10%, ₱4,000,000 convertible
bonds for ₱4,400,000. Principal is due at maturity but interest is payable every year-end. The bonds
are convertible into 6,000 ordinary shares with par value of ₱400. At issuance date, the prevailing
market rate of interest for similar debt without conversion feature is 12%.

On December 31, 20x2, all the convertible bonds were retired for ₱4,000,000. The prevailing rate of
interest on a similar debt instrument as of December 31, 20x2 is 11% without the conversion feature.

67. How much is gain (loss) on the extinguishment of the bonds on December 31, 20x2?
a. 35,392 b. (35,392) c. 32,413 d. (32,413)

B
Solution:
The fair value of the bonds without the conversion feature is computed as follows:
Future cash flows PV @ 12%, n=3 PV factors Present value
Principal 4M PV of ₱1 0.711780 2,847,120
P a g e | 17

Interest 400K PV of ordinary annuity of ₱1 2.401831 960,732


Fair value of debt instrument without equity feature 3,807,852

The issue price is allocated to the liability and equity components as follows:
Issue price 4,400,000
Fair value of debt instrument without equity feature (3,807,852)
Equity component 592,148

The entry to record the issuance of the bonds is as follows:


Cash in bank
Jan. 1, 20x1 4,400,000
Discount on bonds payable 192,148
Bonds payable 4,000,000

Share premium – conversion feature 592,148

Amortization table:
Interest
Date payment Interest expense Amortization Present value
Jan. 1, 20x1 3,807,852
Dec. 31, 20x1 400,000 456,942 56,942 3,864,794
Dec. 31, 20x2 400,000 463,775 63,775 3,928,570

The other entries prior to retirement are as follows:


Dec. 31, 20x1 Interest expense 456,942
Discount on bonds payable 56,942
Cash in bank 400,000
Dec. 31, 20x2 Interest expense 463,775
Discount on bonds payable 63,775
Cash in bank 400,000

The fair value of the bonds without the conversion feature on retirement date (December 31, 20x2) is
computed as follows:
Future cash flows PV @ 11%, n=1 PV factors Present value
Principal 4M PV of ₱1 0.900901 3,603,604
Interest 400K PV of ordinary annuity of ₱1 0.900901 360,360
Fair value of debt instrument without equity feature 3,963,964

The retirement price is allocated to the liability and equity components as follows:
Total retirement price 4,000,000
Fair value of bonds without conversion feature – Dec. 31, 20x2 (3,963,964)
Equity component – debit to “share premium – conversion feature” 36,036

The gain or loss on extinguishment of bonds is computed as follows:


Carrying amount of bonds – Dec. 31, 20x2 3,928,570
P a g e | 18

Retirement price allocated to the bonds (3,963,964)


Loss on extinguishment of bonds (35,394)

68. How much is the net credit to “share premium” account on December 31, 20x2?
a. 556,110 b. 541,167 c. 514,571 d. 557,368

A
Solution:
The simple entries on December 31, 20x2 are as follows:
Dec. 31, Bonds payable 4,000,000
20x2
Loss on extinguishment of bonds (squeeze) 35,394
Discount on bonds (4M – 3,928,570) 71,430
Cash in bank (amt. allocated to the bonds) 3,963,964
to record the retirement of convertible bonds
Dec. 31, Share premium – conversion feature 36,036
20x2
Cash in bank (amount allocated to equity) 36,036
to record the allocation of retirement price to equity component
Dec. 31, Share prem. - conversion feature (592,148 - 36,036) 556,112
20x2
Share premium 556,112
to record forfeiture of conversion feature of retired convertible bonds

69. How much is the net increase (decrease) in equity due to the retirement of the bonds on
December 31, 20x2?
a. (36,036) b. 36,036 c. (592,148) d. 0

A
Solution:
Debits to “sh. prem.– conversion feature”
(36,036 + 556,112) (592,148)
Credit to share premium 556,112
Net decrease in equity (36,036)

70. The mother of accounting is


a. Mrs. Fra Luka Gaga
b. Accountant Mama
c. Mommy Showpao
d. None of these

“Consider it pure joy, my brothers and sisters, whenever you face trials of
many kinds, because you know that the testing of your faith produces
perseverance.” (James 1:2-3)
P a g e | 19

- END -
Page |1

1. PFRSs are adopted from the standards issued by the


a. IASC
b. IASCF
c. IASB
d. FASB

Use the following information for the next three questions:


On January 1, 20x1, KISMET FATE Co., purchased inventory with a list price of ₱4,400,000 and a
cash price of ₱4,000,000 by issuing a noninterest-bearing note of ₱4,800,000 due on December 31,
20x3.

2. How much is the carrying amount of the note on initial recognition?


a. 4,400,000 b. 4,000,000 c. 4,800,000 d. 3,786,309

B 4,000,000 – the cash price equivalent

3. How much is the interest expense in 20x1?


a. 400,000 b. 279,830 c. 250,780 d. none of these

C
Solution:
Trial and error approach
First trial: (at 10%)
Future cash flows x PV factor at x% = PV of note
 4,800,000 X PV of ₱1 @ 10%, n=3 = 4,000,000
 (4,800,000 x 0.751315) = 3,606,312 is not equal to 4,000,000
We need a substantially higher amount of present value. Therefore, we need to decrease substantially
the interest rate. Let’s try 6%.
Second trial: (at 6%)
Future cash flows x PV factor at x% = PV of note
 4,800,000 X PV factor at 6%, n=3 = 4,000,000
 (4,800,000 x 0.839619) = 4,030,171 is not equal to 4,000,000
We need a slightly lower amount of present value. Therefore, we need to increase slightly the interest
rate. Let’s try 7%.

Third trial: (at 7%)


Future cash flows x PV factor at x% = PV of note
 4,800,000 X PV factor at 7%, n=3 = 4,000,000
 (4,800,000 x 0.816298) = 3,918,230 is not equal to 4,000,000

In here, we need to perform interpolation. Looking at the values derived above, we can reasonably
expect that the effective interest rate is a rate between 6% and 7%.

To perform the interpolation, we will use the following formula:


x% - 6%
7% - 6%

Where: x% again is the effective interest rate.


Page |2

The formula is derived based on our expectation that the effective interest rate is somewhere between
6% and 7%. Notice that the lower rate appears in both the numerator and denominator of the formula
while x% appears in the numerator.

Let us substitute the amounts of present values computed earlier on the formula.
4,000,000 - 4,030,171 (30,171)
= = 0.2695
3,918,230 - 4,030,171 (111,941)

The amount computed is added to 6% to derive the effective interest rate. The effective interest rate is
6.2695% (6% + .2695%).

Interest expense in 20x1 = 6.2695% x 4,000,000 = 250,780

4. How much is the carrying amount of the note on December 31, 20x1?
a. 4,250,780 b. 4,279,830 c. 4,400,000 d. 4,000,000

A (4,000,000 x 106.2695%) = 4,250,780

5. On January 1, 20x1, ABC Co., acquired transportation equipment in exchange for cash of
₱100,000 and ₱1,000,000 noninterest-bearing note payable due in 4 equal annual installments
starting December 31, 20x1. The prevailing rate of interest for this type of note is 12%. How
much is the current portion of the note on December 31, 20x2?
a. 158,880
b. 177,945
c. 199,298
d. 223,214

Future cash flows – annual installments (₱1M ÷ 4) 250,000


Multiply by: PV of an ordinary annuity of ₱1 @12%, n=4 3.037349
Present value of note payable - Jan. 1, 20x1 759,337

Date Payments Interest expense Amortization Present value


Jan. 1, 20x1 759,337
Dec. 31, 20x1 250,000 91,120 158,880 600,458
Dec. 31, 20x2 250,000 72,055 177,945 422,513
Dec. 31, 20x3 250,000 50,702 199,298 223,214
Dec. 31, 20x4 250,000 26,786 223,214 0

6. Kemp Company must determine the December 31, 2005, year-end accruals for advertising and
rent expense. A P50,000 advertising bill was received January 7, 2006, comprising cost of P35,000
for advertisements in December 2006 issues, and P15,000 for advertisements in January 2006
issues of the newspaper.

A store lease, effective December 16, 2004, calls for fixed rent of P120,000 per month, payable one
month from the effective date and monthly thereafter. In addition, rent equal to 5% of net sales over
P6,000,000 per calendar year is payable on January 31 of the following year. Net sales for 2005 were
P9,000,000.
Page |3

In its December 31, 2005 balance sheet, Kemp should report accrued liabilities of
a. 260,000
b. 185,000
c. 210,000
d. 245,000
D 35+ 60 + 150

7. ABC Co. is contemplating on issuing a 12%, 3-year, ₱1,000,000 bonds. Principal is due at
maturity but interest is due semi-annually every July 1 and December 31. ABC determines that
the current market rate on January 1, 20x1 is 14%. ABC Co. plans to issue the bonds on
September 30, 20x1. How much is the estimated total proceeds from the issuance of the bonds on
September 30, 20x1?
a. 666,342
b. 962,563
c. 952,334
d. 992,563

Solution:

Interest Interest Present


Date payments expense Amortization value
Jan. 1, 20x1 952,335
July 1, 20x1 60,000 66,663 6,663 958,998
Sept. 30, 20x1 30,000 33,565 3,565 962,563

The estimated issue price pertaining to the bonds only on Sept. 30, 20x1 is ₱962,563.

The total proceeds is computed as follows:

Issue price pertaining to bonds only 962,563


Sold accrued interest (1M x 12% x 3/12) 30,000
Total issue price or cash proceeds 992,563

Only the accrued interest from July to Sept. is added to the issue price of the bonds because the last
interest payment date was on July 1.

8. On January 1, 20x1, SALIENT PROMINENT Co. issued 1,000, ₱4,000, 12%, 3-year bonds for
₱4,412,336. Principal is due on December 31, 20x3 but interests are due annually every year-end.
In addition, SALIENT incurred bond issue cost of ₱213,388.The effective interest rate before
adjustment for transaction costs is 8%. How much is the carrying amount of the note on
December 31, 20x1?
a. 4,019,832 b. 4,198,948 c. 4,288,776 d. 4,138 ,843

D
Solution:
The carrying amount of the bonds on initial recognition is computed as follows:
Issue price before transaction costs 4,412,336
Page |4

Transaction costs (Bond issue costs) (213,388)


Carrying amount - Jan. 1, 20x1 (net issue price) 4,198,948

Trial and error


First trial: (using 10%)
 (4M x PV of ₱1 @ 10%, n=3) + [(4M x 12%) x PV of an ordinary annuity of ₱1 @ 10%, n=3) = 4,198,948
 (4M x 0.751315) + (480,000 x 2.48685) = 4,198,948
 (3,005,260 + 1,193,688) = 4,198,948 is equal to 4,198,948

Since 10% exactly discounts the future cash flows to the initial carrying amount of the bonds, it shall be
regarded as the effective interest rate. No further interpolation is needed.

Interest
Date payments Interest expense Amortization Present value
Jan. 1, 20x1 4,198,948
Dec. 31, 20x1 480,000 419,895 60,105 4,138,843

Use the following information for the next two questions:


On January 1, 20x1, SPITEFUL MALICIOUS Co. issued 1,000, ₱4,000, 10%, 3-year bonds for
₱3,807,852. Principal is due on December 31, 20x3 but interests are due annually every year-end. The
effective interest rate is 12%. SPITEFUL Co. incorrectly used the straight line method instead of the
effective interest method to amortize the discount.

9. What is the effect of the error on the carrying amount of the bonds on December 31, 20x1? (over)
understated
a. 7,107 b. (7,107) c. 6,341 d. (6,341)

B
Solution:
Erroneous amortization of discount using straight line:
The erroneous straight-line amortization of the discount on bonds payable is computed as follows:
Face amount of bonds 4,000,000
Cash proceeds (3,807,852)
Discount on bonds payable - Jan. 1, 20x1 192,148
Divide by: Term of bonds (in years) 3
Annual amortization (straight line method) 64,049

Interest expense for 20x1 recognized under straight-line method:


Interest paid (4,000,000 x 10%) 400,000
Amortization of discount (see computation above) 64,049
Interest expense under straight-line method 464,049

Carrying amount of bonds on Dec. 31, 20x1 under straight-line method:


Carrying amount - Jan. 1, 20x1 3,807,852
Amortization of discount (see computation above) 64,049
Carrying amount - Dec. 31, 20x1 3,871,901

Amortization of discount under effective interest method:


Amortization table:
Interest Interest Present
Date payments expense Amortization value
Jan. 1, 20x1 3,807,852
Dec. 31, 20x1 400,000 456,942 56,942 3,864,794
Page |5

Effect on carrying amount of bonds as of Dec. 31, 20x1


Carrying amounts on Dec. 31, 20x1:
Straight-line 3,871,901
Effective interest rate 3,864,794
Difference - overstatement under straight-line 7,107

The carrying amount of the bonds on December 31, 20x1 under the straight line method is overstated by
₱7,107.

10. What is the effect of the error on the 20x1 profit? (over) understated
a. 7,107 b. (7,107) c. 6,341 d. (6,341)

A
Solution:
Effect on 20x1 profit
Interest expense in 20x1:
Straight-line (see computations above) 464,049
Effective interest rate (see computations above) 456,942
Difference - overstatement under straight-line 7,107

Since interest expense under straight-line is overstated, the profit under straight-line is understated by
₱7,107.

11. According to PAS 37, contingent liabilities are


a. recognized and disclosed.
b. always disclosed.
c. disclosed, only if their expected occurrence is remote.
d. not disclosed if their expected occurrence is remote.

12. Arrange the following steps in the accounting for defined benefit plans in the correct order.
I. Determine the components of the defined benefit cost to be recognized in P/L and OCI.
II. Determine the net defined benefit liability (asset)
III. Determine the deficit or surplus

a. I, III and II
b. III, II and I
c. II, III and I
d. I, II and III

13. Actuarial gains or losses result from the accounting for which of the following employee
benefits?
a. Short-term compensated absences
b. Post-employment defined contribution plans
c. Post-employment defined benefit plans
d. Profit sharing and bonus plans
Page |6

14. Which of the following factors is least likely to affect the amount of retirement benefits under a
defined benefit plan?
a. The age of the retiring employee.
b. The level of the employee’s compensation.
c. The employee’s length of service.
d. The amount of employer contributions to a fund.

15. Information on the defined benefit plan of Entity A as of December 31, 20x1 is as follows:
• Fair value of plan assets ₱ 800,000
• Present value of the defined benefit obligation ₱1,000,000

How much is (are) presented in Entity A’s December 31, 20x1 statement of financial position in
relation to its post-employment benefits plan?
a. ₱800,000 in noncurrent assets and ₱1M in noncurrent liabilities
b. ₱200,000 net defined benefit asset in noncurrent assets
c. ₱200,000 net defined benefit liability in noncurrent liabilities
d. ₱1M in noncurrent liabilities

16. The actuarial valuation report of Entity A’s post-employment benefit plan shows the following
information:

Service cost 300,000


Net interest on the net defined benefit liability (asset) 90,000
Remeasurements of the net defined benefit liability (20,000)
Total defined benefit cost 370,000

How much will be shown in profit or loss and in other comprehensive income?
Profit or loss Other comprehensive income
a. 370,000 0
b. 300,000 70,000
c. 390,000 (20,000)
d. 0 370,000

17. According to PAS 19, how are other long-term benefits accounted for?
a. similar to defined benefit plans.
b. similar to short-term employee benefits except that the cash flows are discounted.
c. similar to defined benefit plans except that all the components of the defined benefit cost is
recognized in other comprehensive income.
d. similar to defined benefit plans except that all the components of the defined benefit cost is
recognized in profit or loss.

18. Entity B, a trustee, undertakes to manage the retirement benefit fund of Entity A for the benefit
of Entity A’s employees. When reporting to Entity A regarding the status and performance of
the fund, Entity B would most likely apply which of the following standards?
a. PAS 19
b. PAS 24
Page |7

c. PAS 26
d. PFRS 6

Use the following information for the next two questions:


An entity is the defendant in a patent infringement lawsuit. The entity’s lawyers believe there is a
30% chance that the court will dismiss the case and the entity will incur no outflow of economic
benefits. However, if the court rules in favor of the claimant, the lawyers believe that there is a 20%
chance that the entity will be required to pay damages of ₱800,000 (the amount sought by the
claimant) and an 80% chance that the entity will be required to pay damages of ₱400,000 (the
amount that was recently awarded by the same judge in a similar case). Other outcomes are
unlikely.

The court is expected to rule in late December 20x2. There is no indication that the claimant will
settle out of court. A 7% risk adjustment factor to the probability-weighted expected cash flows is
considered appropriate to reflect the uncertainties in the cash flow estimates. An appropriate
discount rate is 10% per year.

19. How much is the provision for lawsuit at December 31, 20x1?
a. 436,360 b. 446,908 c. 326,836 d. 0

C
Solution:
At twenty per cent chance: (800K x 20%) 160,000
At eighty per cent chance: (400K x 80%) 320,000
Total 480,000
Multiply by: PV of P1 @10%, n=1 0.90909
Total 436,363
Multiply by: Risk adjustment (100% + 7%) 107%
Total 466,909
Multiply by: Probability of settlement (100% - 30%) 70%
Provision for lawsuit – Dec. 31, 20x1 326,836

20. Use the fact pattern above. However, in this question, the entity’s lawyers believe there is a 60
per cent chance that the court will dismiss the case and the entity will incur no outflow. How
much is the provision for lawsuit at December 31, 20x1?
a. 186,764 b. 446,908 c. 326,836 d. 0

D – the obligation is not probable, i.e., only 40% chance

Fact pattern:
On January 1, 20x1, Entity X (Customer) enters into a 4-year lease of equipment with Entity Y
(Supplier). The annual rent is ₱220,000, payable at the end of each year. The equipment has a
remaining useful life of 10 years. The interest rate implicit in the lease is 10% while the lessee’s
incremental borrowing rate is 12%. Entity X uses the straight-line method of depreciation. The
relevant present value factors are as follows:
- PV of an ordinary annuity of ₱1 @10%, n=4………… 3.16987
- PV of an ordinary annuity of ₱1 @12%, n=4………… 3.03735
Page |8

21. How much is the lease liability to be recognized by Entity X on initial recognition?
a. 702,345 c. 668,217
b. 697,371 d. 0

Solution:
Fixed payments 220,000
Multiply by: PV of an ordinary annuity of ₱1 @10%, n=4 3.16987
Lease liability 697,371

22. How much is the annual depreciation on the right-of-use asset?


a. 174,343 c. 167,054
b. 175,586 d. 0

Solution:
Cost of right-of-use asset 697,371
Divide by: Lease term (shorter) 4
Annual depreciation 174,343

Since the lease contract neither provides for the transfer of ownership to the lessee nor a ‘reasonably
certain’ purchase option, the asset is depreciated over the shorter of its useful life (10 yrs.) and the
lease term (4 yrs.).

23. Assume the lease qualifies for accounting as a lease of “low-value asset.” How much is the lease
liability to be recognized by Entity X on initial recognition?
a. 702,345 c. 668,217
b. 697,371 d. 0

24. How much is the lease (rent) expense in 20x1?


a. 220,000 c. 167,054
b. 174,343 d. 0

25. Assume the lease is a finance lease. How much is the net investment in the lease to be
recognized by Entity Y on initial recognition?
a. 702,345 c. 668,217
b. 697,371 d. 0

Solution:
Fixed payments 220,000
Multiply by: PV of an ordinary annuity of ₱1 @10%, n=4 3.16987
Net investment 697,371

26. Assume the lease is an operating lease. How much is the lease (rent) income in 20x1?
a. 220,000 c. 167,054
b. 174,343 d. 0
Page |9

27. As a result of differences between depreciation for financial reporting purposes and tax
purposes, the financial reporting basis of Noor Co.'s sole depreciable asset, acquired in 20x1,
exceeded its tax basis by ₱250,000 at December 31, 20x1. This difference will reverse in future
years. The enacted tax rate is 30% for 20x1, and 40% for future years. Noor has no other
temporary differences. In its December 31, 2001, balance sheet, how should Noor report the
deferred tax effect of this difference - Asset (Liability)?
a. ₱75,000 b. ₱100,000 c. (₱75,000) d. (₱100,000)

D
Solution:
Concept: If the carrying amount (CA) of an asset exceeds its tax base (TB), the difference is a taxable
temporary difference which, if multiplied by the tax rate, results to a deferred tax liability.
“For an asset: CA > TB = difference is TTD; TTD x Tax rate = DTL”

Excess of CA over TB of asset (Taxable temp. difference) 250,000


Multiply by: Enacted future tax rate 40%
Deferred tax liability 100,000

28. Provisions, contingent liabilities and contingent assets are accounted for using
a. PAS 37
b. PFRS 6
c. PAS 29
d. PAS 8

29. These are differences that do not have future tax consequences.
a. Permanent differences
b. Taxable differences
c. Temporary differences
d. Deductible differences

30. This type of difference will give rise to deferred tax asset.
a. Taxable temporary difference
b. Permanent difference
c. Deductible temporary difference
d. No difference

31. The tenant (as opposed to the landlord) in a lease contract is referred to as the
a. Lessor
b. Lessee
c. Leasee
d. Tenor

32. Which of the following is a characteristic of a finance lease?


a. The lease term is substantially less than the estimated economic life of the leased property.
b. The lease contains a bargain-purchase option.
P a g e | 10

c. The present value of the minimum lease payments at the beginning of the lease term is 75%
or more of the fair value of the property at the inception of the lease.
d. The lease obligation does not appear in the balance sheet of the lessee.

33. Leases are accounted for under


a. PAS 16
b. PFRS 14
c. PFRS 15
d. PFRS 16

34. In accounting for a defined benefit plan which is fully funded at the start of the year, any
difference between the defined benefit cost recognized and the contributions made to the fund
during the year should be reported as
a. An offset to the liability for past service costs.
b. Net defined benefit liability.
c. An operating expense in this period.
d. An accrued actuarial liability.

35. If not yet vested, past service cost (under the revised PAS 19)
a. is recognized immediately in profit or loss
a. is amortized over the vesting period which is at least 10 million years
b. prior period financial statements are restated
c. recognized as expense in the current and future periods until the end of the world or until
the moon turns blue, whichever comes earlier.

36. PARADIGM EXAMPLE Co. has a 10%, P4,000,000 loan payable as of December 31, 20x1 that is
maturing on July 1, 20x2. Interest on the loan is due every July 1 and December 31. On February
1, 20x2, PARADIGM Co. entered into a refinancing agreement with a bank to refinance the loan
on a long-term basis. Both parties are financially capable of honoring the agreement's provisions.
PARADIGM’s financial statements were authorized for issue on March 15, 20x2. How much is
presented as current liability in relation to the loan in PARADIGM’s 20x1 year-end financial
statements?
a. 4,000,000 b. 200,000 c. 4,200,000 d. 0

A – general rule

37. UNKEMPT UNTIDY Co. requires advance payments for custom-built guitar effects, gadgets,
and racks. The records of UNKEMPT Co. show the following:
• Unearned revenue, January 1, 20x1 P 4,000,000
• Advances received during 20x1 40,000,000
• Advances applied to orders shipped in 20x1 32,000,000
• Advances pertaining to orders cancelled in 20x1 1,200,000

How much is the current liability if the advance payments received are refundable?
a. 10,800,000 b. 13,200,000 c. 12,000,000 d. 0
P a g e | 11

(4M + 40M – 32M) = 12M

38. On January 1, 20x1, ABC Co. borrowed 10%, ₱1,000,000 loan from XYZ Bank. Principal is due on
January 1, 20x4 but interests are due annually starting January 1, 20x2. The bank charged ABC a
3% nonrefundable loan origination fee representing service fee. How much is the carrying
amount of the loan on initial recognition?
a. 1,000,000
b. 970,000
c. 930,000
d. 870,000

Principal amount 1,000,000


Origination fee (30,000)
Initial carrying amount of loan 970,000

39. If the current tax expense is greater than the income tax expense during the period, there must
be a
a. deferred tax benefit c. income tax payable
b. deferred tax expense d. prepaid income tax

40. An equipment cost P4,000. For tax purposes, depreciation of P2,400 has already been deducted
in the current and prior periods and the remaining cost will be deductible in future periods,
either as depreciation or through a deduction on disposal. Revenue generated by using the
equipment is taxable, any gain on disposal of the equipment will be taxable and any loss on
disposal will be deductible for tax purposes. How much is the tax base of the equipment?
a. 4,000 b. 2,400 c. 1,600 d. 0

41. Interest receivable has a carrying amount of P4,000. The related interest revenue will be taxed on
a cash basis. How much is the tax base of the asset?
a. 4,000 b. 2,400 c. 1,600 d. 0

42. A loan receivable has a carrying amount of P4,000. The repayment of the loan will have no tax
consequences. How much is the tax base of the asset?
a. 4,000 b. 2,400 c. 1,600 d. 0

43. Current liabilities include accrued expenses with a carrying amount of P4,000. The related
expense will be deducted for tax purposes on a cash basis. How much is the tax base of the
liability?
a. 4,000 b. 2,400 c. 1,600 d. 0

44. Current liabilities include accrued expenses with a carrying amount of P4,000. The related
expense has already been deducted for tax purposes. How much is the tax base of the liability?
a. 4,000 b. 2,400 c. 1,600 d. 0
P a g e | 12

Use the following information for the next six questions:


ABC Co. has pretax income of ₱100,000. The following information was gathered:
Loss on expropriation of property 35,000
Non-deductible premium on life insurance of key 6,000
employees
Interest income received on government securities
subjected to final tax (5,000)
Excess of accelerated depreciation used in taxation over
straight line depreciation used in financial reporting (10,000)
Warranty expense accrued for financial reporting
purposes but is tax deductible only when actually paid 15,000
Rent received in advance 8,000
Quarterly income tax payments (1st quarter to 3rd quarter) 20,000
Tax rate 30%
Beginning balance of taxable temporary difference 12,000
Beginning balance of deductible temporary difference 9,000

45. How much is the income tax expense?


a. 40,800
b. 42,600
c. 44,700
d. 46,200

Solution:

Multiply by
Description of items Description of items
Tax rate
Pretax income 100,000
Permanent differences:
Add: Non-deductible
expenses:
Loss on expropriation 35,000
Premium on life
insurance 6,000
Less: Non-taxable income
Non-taxable interest
income (5,000)
Accounting profit subject to tax 136,000 30% Income tax expense 40,800
Temporary differences:
Less:  Taxable temporary difference Less:  Deferred tax
(TTD) 'FI>TI': liability (DTL):
Excess depreciation (10,000) 30% (3,000)
Add:  Deductible temporary difference Add:  Deferred tax asset
(DTD) 'FI<TI' (DTA):
Warranty expense 15,000 30% 4,500
Rent received in advance 8,000 30% 2,400
Taxable profit 149,000 30% Current tax expense 44,700
P a g e | 13

46. How much is the current tax expense?


a. 40,800
b. 42,600
c. 44,700
d. 46,200

47. How much is the deferred tax expense (benefit)?


a. 4,900
b. (4,900)
c. 3,900
d. (3,900)

Deferred tax expense/benefit = Increase in DTL - Increase in DTA

Increase in DTL* (3,000)


Increase in DTA (4,500 + 2,400)* 6,900
Deferred tax benefit 3,900
*Amounts are taken from the solution above.

48. How much is the current tax payable?


a. 44,700
b. 42,600
c. 24,700
d. 22,600

Income tax payable is computed as follows:


Current tax expense (see table above) 44,700
Quarterly income taxes paid ( 20,000)
Income tax payable 24,700

49. How much is the deferred tax liability to be presented in the statement of financial position?
a. 3,600
b. 6,600
c. 3,000
d. 6,000

Deferred tax liability


3,600 beg. (₱12,000 TTD, beg. x 30%)
3,000 Increase (see solution above)
end. 6,600

50. How much is the deferred tax asset to be presented in the statement of financial position?
a. 9,000
b. 9,600
P a g e | 14

c. 10,200
d. 11,000

Deferred tax asset


beg. (₱9,000 DTD, beg. x 30%) 2,700
Increase (4,500 + 2,400) 6,900
9,600 end.

Use the following information for the next four questions:


ABC Co. has the following information from its comparative financial statements.
20x2 20x1
Trade account receivable from service revenues 1,500,000 1,200,000
Prepaid insurance 120,000 100,000
Building - net of accumulated depreciation 9,000,000 9,500,000
Estimated liability for warranty obligation 300,000 280,000

Additional information:
• ABC recognizes revenues from service fees as services are rendered but are taxed only when cash
is collected. Total collections in 20x2 amounted to ₱800,000.
• The prepaid insurance account pertains to the unexpired portion of life insurance premiums
taken on the life of key personnel. ABC is the irrevocable beneficiary of the insurance policy. Total
premiums paid in 20x2 were ₱50,000.
• The building was acquired on January 1, 20x1 and is depreciated over an estimated useful life of
20 years with no residual value. The straight line method of depreciation is used for financial
reporting while the double declining balance method is used for taxation.
• Warranty expense is recognized at the time goods are sold but are tax deductible only when
actually paid. Tax deductible warranty expense for 20x2 amounted to ₱40,000.
• Pretax income in 20x2 is ₱1,000,000. Income tax rate is 30%.

51. How much is the deferred tax asset as of December 31, 20x1?
a. 84,000
b. 96,000
c. 102,000
d. 114,000

Solutions:

Requirement (a) – DTL and DTA in statement of financial position in 20x1 and 20x2

 For an asset:
CA > TB = TTD or 'FI>TI' : TTD multiplied by tax rate results to DTL
P a g e | 15

20x1
Trade account receivable – December 31, 20x1:

Carrying amount – December 31, 20x1 1,200,000


Tax base* -
Taxable temporary difference 1,200,000
Multiply by tax rate 30%
Deferred tax liability – December 31, 20x1 360,000

* The tax base is zero because the receivable is taxable only when collected. (See previous discussion on tax bases)

Prepaid insurance – December 31, 20x1:


No temporary difference arises from the prepaid insurance because the premiums paid have no tax
consequence, i.e., not tax deductible.

Building – December 31, 20x1:


Carrying amount – December 31, 20x1 9,500,000
Tax base* (9,000,000)
Taxable temporary difference 500,000
Multiply by tax rate 30%
Deferred tax liability – December 31, 20x1 150,000

*The tax base of the building on Dec. 31, 20x1 is computed as follows:

Carrying amount – December 31, 20x1 9,500,000


Divide by: Unexpired life (straight line) a 19/20
Historical cost 10,000,000
Accumulated depreciation – Dec. 31, 20x1 (10M x 10%) b (1,000,000)
Tax base – December 31, 20x1 9,000,000

a The building is 1-year old as of December 31, 20x1 because the acquisition date is January 1, 20x1.
b Double declining rate is 10% (2 ÷ 20 years).

Estimated liability for warranty obligation – December 31, 20x1:


Carrying amount – December 31, 20x1 280,000
Tax base* -
Deductible temporary difference 280,000
Multiply by tax rate 30%
Deferred tax asset – December 31, 20x1 84,000

* The tax base is zero because the warranty is tax deductible only when paid. (See previous discussion on tax bases)

Deferred tax asset – December 31, 20x1 84,000

Deferred tax liability – December 31, 20x1 (360K + 150K) 510,000


P a g e | 16

52. How much is the deferred tax liability as of December 31, 20x1?
a. 360,000
b. 410,000
c. 510,000
d. 620,000

53. How much is the deferred tax asset as of December 31, 20x2?
a. 72,000
b. 86,000
c. 90,000
d. 110,000

Trade account receivable – December 31, 20x2:


Carrying amount – December 31, 20x2 1,500,000
Tax base -
Taxable temporary difference 1,500,000
Multiply by tax rate 30%
Deferred tax liability – December 31, 20x2 450,000

Building – December 31, 20x2:


Carrying amount – December 31, 20x2 9,000,000
Tax base* (8,100,000)
Taxable temporary difference 900,000
Multiply by tax rate 30%
Deferred tax liability – December 31, 20x2 270,000

*The tax base of the building on Dec. 31, 20x2 is computed as follows:

Historical cost (see previous computation) 10,000,000


Accumulated depreciation – Dec. 31, 20x2 [1M + (9M x 10%)] (1,900,000)
Tax base – December 31, 20x2 8,100,000

Estimated liability for warranty obligation – December 31, 20x1:


Carrying amount – December 31, 20x2 300,000
Tax base -
Deductible temporary difference 300,000
Multiply by tax rate 30%
Deferred tax asset – December 31, 20x2 90,000

Deferred tax asset – December 31, 20x2 90,000

Deferred tax liability – December 31, 20x2 (450K + 270K) 720,000

54. How much is the deferred tax liability as of December 31, 20x2?
a. 510,000
P a g e | 17

b. 680,000
c. 720,000
d. 810,000

55. How much is the income tax expense in 20x2?


a. 1,050,000
b. 350,000
c. 309,000
d. 105,000

Multiply by
Description of items Description of items
Tax rate
Pretax income 1,000,000
Permanent difference: a 30,000
Acctg. profit subj. to tax 1,030,000 30% ITE 309,000
Less:  TTD (700,000) 30%  DTL b (210,000)
Add:  DTD 20,000 30%  DTA 6,000
Taxable profit - 20x2 350,000 30% CTE 105,000

aThe permanent difference pertains to the insurance expense recognized in financial reporting but is
non-tax deductible. This is computed in the T-account below:

Prepaid insurance
Jan. 1, 20x2 100,000
Premiums paid 50,000 30,000 Insurance expense (squeeze)
120,000 Dec. 31, 20x2

b The amounts placed in the formula above are the changes in TTD, DTD, DTL and DTA,
respectively. The changes in DTL and DTA are computed as follows:

DTL - 20x2 720,000 DTA - 20x2 90,000


DTL - 20x1 (510,000) DTA - 20x1 (84,000)
Increase in DTL 210,000 Increase in DTA 6,000

56. How much is the current tax expense in 20x2?


a. 1,050,000
b. 350,000
c. 309,000
d. 105,000

Use the following information for the next two questions:


ABC Co. started its operations on January 1, 20x1. Information on temporary differences during the
first two years of operations is shown below:

Dec. 31, 20x2 Dec. 31, 20x1


Carrying Tax base Difference Carrying Tax base Difference
P a g e | 18

amount amount
Assets 100,000 90,000 10,000 120,000 100,000 20,000
Liabilities 50,000 43,000 7,000 60,000 45,000 15,000

Pretax incomes were ₱400,000 and ₱500,000 in 20x2 and 20x1, respectively. Income tax rate is 30%.

57. How much is the income tax expense in 20x2?


a. 150,000
b. 148,500
c. 120,000
d. 120,600

Solutions:

20x1
The 20x1 income tax expense and current tax expense are computed as:

Multiply by
Description of items Description of items
Tax rate
Pretax income 500,000
Permanent difference: -
Acctg. profit subj. to tax 500,000 30% ITE 150,000
Less:  TTD (20,000) 30% Less:  DTL (6,000)
Add:  DTD 15,000 30% Add:  DTA 4,500
Taxable profit - 20x1 495,000 30% CTE 148,500

20x2
The 20x2 income tax expense and current tax expense are computed as:

Multiply by
Description of items Description of items
Tax rate
Pretax income 400,000
Permanent difference: -
Acctg. profit subj. to tax 400,000 30% ITE 120,000
ADD:  TTD (10,000) 30% ADD:  DTL 3,000
LESS:  DTD 8,000 30% LESS:  DTA (2,400)
Taxable profit - 20x1 402,000 30% CTE 120,600

58. How much is the current tax expense in 20x2?


a. 150,000
b. 148,500
c. 120,000
d. 120,600
P a g e | 19

Use the following information for the next four questions:


On January 1, 20x1, ABC purchased machinery for ₱1,000,000. The equipment is depreciated using
the straight line method over an estimated useful life of 10 years with no residual value. On January
1, 20x3, the equipment was revalued at a fair value of ₱1,200,000 with no change in useful life. The
pretax income before deduction for depreciation expense in 20x3 is ₱1,000,000. Income tax rate is
30%.

59. How much is the deferred tax liability as of January 1, 20x3?


a. 400,000
b. 280,000
c. 120,000
d. 90,000

Solution:

Fair value 1,200,000


Carrying amount - January 1, 20x3 (1M x 8/10) (800,000)
Revaluation surplus before tax – Jan. 1, 20x3 400,000

The revaluation surplus before tax is allocated as follows:

Revaluation surplus after tax – Jan. 1, 20x3 (400K x 70%) 280,000


Deferred tax liability – Jan. 1, 20x3 (400K x 30%) 120,000

60. How much is the deferred tax liability as of December 31, 20x3?
a. 280,000
b. 245,000
c. 120,000
d. 105,000

Revaluation surplus – Dec. 31, 20x3 (280,000 x 7/8*) 245,000


Deferred tax liability – Dec. 31, 20x3 (120,000 x 7/8*) 105,000

*7 years remaining from the total 8-year life on previous revaluation date.

61. How much is the income tax expense in 20x3?


a. 850,000
b. 900,000
c. 270,000
d. 255,000

Multiply by
Description of items Description of items
Tax rate
Pretax income before depn. 1,000,000
P a g e | 20

Depreciation expense (150,000)


Pretax income after depn. 850,000
Permanent differences: -
Acctg. profit subj. to tax 850,000 30% ITE 255,000

Add: Excess of depreciation recognized


for financial reporting over taxation Add: Decrease in DTL (120K –
purposes (FI<TI) 50,000 30% 105K) 15,000
Taxable profit - 20x3 900,000 30% CTE 270,000

62. How much is the current tax expense in 20x3?


a. 850,000
b. 320,000
c. 270,000
d. 255,000

63. Entity A received a subscription for 2,000 shares at ₱18 per share on March 31, 20x1. Entity A’s
shares have a par value ₱5 per share. Entity A collected the subscription receivable on May 15,
20x1. Which of the following statements is correct?
a. Entity A should credit share premium for ₱13,000 on March 31, 20x1.
b. Entity A should credit share premium for ₱26,000 on March 31, 20x1.
c. Entity A should credit share premium for ₱13,000 on May 15, 20x1.
d. Entity A should credit share premium for ₱26,000 on May 15, 20x1.

2,000 sh. x (18 – 5) = 26,000 share premium recorded at the subscription date, not collection date

64. Entity A has the following share capital transactions during the year:
• Issued 10,000 shares with par value of ₱10 per share for a total consideration of ₱160,000.
• Received share subscriptions for 20,000 shares at a subscription price of ₱22 per share. Only
half of the subscriptions were collected by the end of the year.

How much is the total share premium arising from the share transactions above?
a. 60,000 c. 300,000
b. 320,000 d. 180,000

[160,000 – (10,000 x 10)] + [20,000 x (22 – 10)] = 300,000

65. Entity A was incorporated on January 1, 20x1 with an authorized capitalization is ₱1,000,000
divided into 100,000 shares with par value of ₱10 per share. The following were the share-related
transactions of Entity A during the year:
• Cash subscriptions of 30,000 shares at ₱12 per share.
• Subscriptions of 40,000 shares at ₱18 per share. Seventy-five percent of the subscription price
was collected during the year.
P a g e | 21

How much is the Entity A’s total shareholders’ equity after recording the transactions above?
a. 900,000 c. 540,000
b. 680,000 d. 360,000

Solution:
Share capital (30K x 10) 300,000
Subscribed shares (40K x 10) 400,000
Subscriptions receivable (40K x 18 x 25%) (180,000)
Share premium (30K x 2) + (40K x 8) 380,000
Total SHE 900,000

Short-cut: (30,000 x 12) + (40,000 x 18 x 75%) = 900,000

66. Entity A’s total shareholders’ equity was ₱900,000 before recording the following share
transactions:
• Received cash subscriptions for 10,000 shares with par value of ₱1 at ₱14 per share. Share
issuance costs amounted to ₱2,000.
• Received subscriptions for 20,000 shares at ₱20 per share. Twenty-five percent down
payment was collected on subscription date.
• Collected the remaining unpaid subscription price of 15,000 subscribed shares and issued the
related share certificates. Share issuance costs amounted to ₱3,000.

How much is the balance of Entity A’s total shareholders’ equity after recording the transactions
above? (Hint: Preparing journal entries makes this problem easier to solve.)
a. 1,490,000 c. 1,360,000
b. 1,510,000 d. 1,610,000

Solution:

• Date Cash (10,000 x ₱14) 140,000


Share capital (10,000 x ₱1) 10,000
Share premium 130,000
Date Share premium 2,000
Cash 2,000
• Date Cash (20,000 x ₱20 x 25%) 100,000
Subscriptions receivable (20K x ₱20 x 75%) 300,000
Subscribed share capital (20,000 x ₱1) 20,000
Share premium 380,000
• Date Cash (15,000 x ₱20 x 75%) 225,000
Subscriptions receivable 225,000
1. D Subscribed share capital (15,000 x ₱1) 15,000
ate
Share capital 15,000
2. D Share premium 3,000
ate
Cash 3,000

Total SHE before share transactions 900,000


P a g e | 22

Share capital (10,000 + 15,000) 25,000


Subscribed share capital (20,000 - 15,000) 5,000
Subscription receivable (300,000 – 225,000) (75,000)
Share premium (130,000 – 2,000 + 380,000 – 3,000) 505,000
Total SHE after share transactions 1,360,000

Short-cut: 900,000 + (10,000 x 14 – 2,000) + (20,000 x 20 x 25%) + (15,000 x 20 x 75% - 3,000) = 1,360,000

67. On February 26, 20x1, Entity A acquires 10,000 of its own shares for ₱3 per share. The shares
have a par value of ₱1 and were selling in the stock market at ₱4 per share on this date. To
record the reacquisition, Entity A should
a. debit Treasury shares account for ₱30,000.
b. credit Treasury shares account for ₱30,000.
c. debit Share premium account for ₱10,000.
d. credit Treasury shares account for ₱40,000.

10,000 x ₱3 cost = 30,000

68. Two years ago, Entity A reacquired 2,000 of its own shares with par value of ₱100 per share for
₱240,000. Today, Entity A reissues half of the treasury shares at ₱160 per share. The journal entry
to record the reissuance includes which of the following?
a. Credit to Retained earnings – unrestricted account for ₱240,000
b. Debit to Treasury shares account for ₱120,000
c. Credit to Share premium – treasury shares for ₱80,000
d. Credit to Share premium – treasury shares for ₱40,000

Solution:
Date Cash (1,000 x ₱160) 160,000
Treasury shares (240,000 x 1/2) 120,000
Share premium – treasury shares 40,000
Date Retained earnings – appropriated 120,000
Retained earnings – unrestricted 120,000

69. Entity A reacquires 10,000 of its own shares for ₱50. The shares have par value of ₱10 and were
originally issued at ₱15 per share. Subsequently, Entity A reissues the 10,000 shares at ₱48 per
share. The journal entry to record the reissuance involves which of the following?
a. Debit to Retained earnings for ₱20,000
b. Credit to Cash for ₱480,000
c. Debit to Share premium for ₱50,000
d. Debit to Treasury shares for ₱500,000

Solution:
Date Cash (10,00 x ₱48) 480,000
(a) Share premium – treasury shares -
P a g e | 23

(b) Retained earnings 20,000


Treasury shares (10,000 x ₱50) 500,000

70. Entity A reacquires 10,000 of its own shares for ₱50. The shares have par value of ₱10 and were
originally issued at ₱15 per share. Subsequently, Entity A reissues half of the reacquired shares
at ₱58 per share and retires the other half. The journal entry to record the retirement of the
shares includes which of the following? (Hint: Provide the entries for both the reissuance and the retirement.)
a. Debit to Retained earnings for ₱175,000
b. Credit to Share premium - retirement for ₱40,000
c. Debit to Share premium for ₱50,000
d. Debit to Retained earnings for ₱135,000

Solutions:

Date Cash (10,000 x ½ x ₱58) 290,000


Treasury shares (10,000 x ½ x ₱50) 250,000
Share premium – treasury shares 40,000

Date Share capital (5,000 x ₱10) 50,000


Share premium – original issuance (5K x ₱5) 25,000
(a) Share premium – treasury shares (see above) 40,000
(b) Retained earnings (balancing figure) 135,000
Treasury shares (5,000 x 50) 250,000

71. Entity A reacquires 1,000 of its own shares for ₱25 and immediately retires them. The shares
have par value of ₱10 and were originally issued at ₱30 per share. The journal entry to record the
retirement of the shares includes which of the following?
a. Debit to Retained earnings for ₱5,000
b. Credit to Treasury shares for ₱30,000
c. Credit to Share capital for ₱10,000
d. Credit to Share premium - retirement for ₱5,000

Solution:

Date Share capital (1,000 x ₱10) 10,000


Share premium – original issuance (1K x ₱20) 20,000
Cash (1,000 x 25) 25,000
Share premium – retirement 5,000

72. Entity A receives 20,000 shares with par value of ₱100 and fair value of ₱210 on November 2,
20x1. The shares have fair value of ₱220 per share on December 31, 20x1. How much additional
capital is recognized in Entity A’s December 31, 20x1 balance sheet as having resulted from the
receipt of the donated shares?
a. 2,000,000 c. 4,400,000
b. 4,200,000 d. 0
P a g e | 24

73. You and I are the accountants of A Corporation. Our company’s authorized capitalization is
₱100M divided into 100M shares with par value per share of ₱1. Which of the following
statements is correct?
a. If our company issues 10,000 shares for ₱5 each, we will recognize a share premium of
₱50,000.
b. Our company can issue shares at a subscription price that is below ₱1.
c. Our company can issue more than 100M shares without amending its articles of
incorporation.
d. If our company receives share subscription for 20,000 shares at ₱15 per share, we will most
likely recognize the related share premium on subscription date rather than on the collection
date.

Use the following information for the next two questions:


Information on ABC Co.’s operations during the year is shown below.
• Revenues are recognized for financial reporting at point of sale while revenues are taxed on cash
basis. Gross profit recognized for financial reporting amounted to ₱1,000,000 while taxable gross
profit is ₱800,000.
• Retirement benefit costs are deducted for financial reporting as services are rendered by
employees but are tax deductible only when actually paid to retiring employees. Current service
cost recognized during the year is ₱100,000 while benefits paid to retiring employees amounted to
₱150,000.
• Research costs amounting to ₱90,000 are expensed immediately during the year for financial
reporting. For taxation purposes, research costs are amortized over a three-year period.
Amortization of research cost deducted for taxation purposes is ₱30,000.
• Unrealized losses of ₱10,000 were recognized during the year in profit or loss on an investment in
held for trading equity securities. No equivalent adjustment was made for taxation purposes.
Any gain or loss on actual disposal of such securities is taxable (tax deductible).
• Payments during the year for fines, surcharges, and penalties arising from violation of law
amounted to ₱40,000.
• ABC reported pretax income of ₱100,000. Income tax rate is 30%.
• Any operating loss can be carried over to the next period. ABC expects to realize the economic
benefit of any operating loss carry forward.

74. How much is the deferred tax liability?


a. 75,000
b. 69,000
c. 82,000
d. 33,000

Solution:
Excess of gross profit recognized for financial reporting
over taxable gross profit (1M – 800,000) – ‘FI>TI’ 200,000
Excess of retirement benefits expense recognized for
taxation over retirement benefits expense recognized
for financial reporting (150,000 – 100,000) – ‘FI>TI’ 50,000
P a g e | 25

Taxable temporary differences 250,000


Multiply by tax rate 30%
Deferred tax liability 75,000

75. How much is the deferred tax asset?


a. 75,000
b. 69,000
c. 82,000
d. 33,000

Excess of research cost expensed for financial reporting


over tax deductible research expense (90K – 30K) – ‘FI<TI’ 60,000
Excess of loss recognized for financial reporting over tax
deductible loss – ‘FI<TI’ 10,000
Deductible temporary differences before operating loss
carry forward 70,000
Operating loss carry forward* 40,000
Deductible temporary differences – adjusted 110,000
Multiply by tax rate 30%
Deferred tax asset 33,000

*Operating loss carry forward is computed as follows:

Description of items
Pretax income 100,000
Add: Non-deductible losses on fines and surcharges 40,000
Accounting profit subject to tax 140,000
Less: Taxable temporary difference (TTD) 'FI>TI': (250,000)
Add: Deductible temporary difference before NOLCO (DTD) 'FI<TI' 70,000
Operating loss carry forward (40,000)

76. On December 31, 20x1, an entity has an asset of ₱4,000 for interest receivable that will be taxed
when the cash is received in 20x2. Tax is payable at 20% on the first ₱500,000 of taxable profit
earned and 30% on any remainder (i.e., excess above ₱500,000). In 20x1 the entity earned taxable
profit of ₱450,000. In 20x2 the entity expects to earn taxable profit of ₱550,000. How much is the
deferred tax liability on Dec. 31, 20x1?
a. 864.15
b. 748.19
c. 891.23
d. 836.40

Solution:
Tax on first ₱500,000 of taxable profit (500,000 x 20%) 100,000
Tax on excess taxable profit [(550,000 - 500,000) x 30%] 15,000
Total current tax expense in 20x2 115,000
P a g e | 26

Divide by: Expected taxable profit in 20x2 550,000


Average rate expected to apply on reversal date 20.91%

Taxable temporary difference 'CA > TB' (4,000 - 0) 4,000


Multiply by: Average rate 20.91%
Deferred tax liability – Dec. 31, 20x1 836.40

77. Wall Co. leased office premises to Fox, Inc. for a five-year term beginning January 2, 20x9. Under
the terms of the operating lease, rent for the first year is ₱8,000 and rent for years 2 through 5 is
₱12,500 per annum. However, as an inducement to enter the lease, Wall granted Fox the first six
months of the lease rent-free. In its December 31, 20x9, income statement, what amount should
Wall report as rental income?
a. 12,000 b. 11,600 c. 10,800 d. 8,000

C
Solution:
Rent for the first year (8,000 x 6/12) 4,000
Rent for the subsequent years (12,500 x 4) 50,000
Total collection on rentals 54,000
Divide by: 5
Annual rent income 10,800

78. As an inducement to enter a lease, Arts, Inc., a lessor, grants Hompson Corp., a lessee, nine
months of free rent under a five-year operating lease. The lease is effective on July 1, 20x5, and
provides for monthly rental of ₱1,000 to begin April 1, 20x6. In Art's income statement for the
year ended June 30, 20x6, rent income should be reported as
a. 10,200 b. 9,000 c. 3,000 d. 2,550

A
Solution:
Lease term in years 5
Multiply by: No. of months in a year 12
Lease term in months 60
Nine months free rent (9)
Total 51
Multiply by: Monthly rental 1,000
Total rental payments on the lease 51,000
Divide by: Lease term in years 5
Annual rent income (July 1 to June 30) 10,200

79. ABC Co. has the following information relating to its income tax on December 31, 20x1:
• Provision for probable loss on litigation of ₱300,000 is recognized for financial reporting. This
amount is tax deductible only when actually paid. ABC expects to pay for the accrued loss in
20x2.
P a g e | 27

• Revenue for financial reporting is recognized based on percentage of completion while revenue
for taxation purposes is recognized based on collections on progress billings. Total revenue
recognized for financial reporting is ₱1,000,000 while revenue recognized for taxation purposes is
₱800,000.
• Pretax income for the year is ₱1,000,000. Income tax rate for 20x1 is 30%. However, an enacted tax
law that will take effect starting January 1, 20x2 requires a tax rate of 32%.
• There are no temporary differences on January 1, 20x1.

How much is the income tax expense?


a. 320,000
b. 300,000
c. 298,000
d. 289,000

Solution:

Description of items Tax rates


Pretax income START 1M
Permanent differences -
Acctg. profit subj. to tax 1M N/A ITE 298K SQUEEZE
Less: Revenue (FI>TI) (200K) 32% DTL (64K)
Add: Provision (FI<TI) 300K 32% DTA 96K
Taxable profit 1.1M 30% CTE 330K

80. Who was the first Filipino Certified Public Accountant?


a. Lapu-lapu
b. Andres Bonifacio
c. Spongebob Squarepants
d. Don Vincente Fabella

“We want each of you to show this same diligence to the very end, so
that what you hope for may be fully realized.” (Hebrews 6:11)

- END -
Page |1

Chapter 23 – Current Liabilities


1. Alhambra Company offers three payment plans on its 12-month contracts. Information on the
three plans and the number of children enrolled in each plan for September 1, 2005 through
August 31, 2006 contract year follows:

Initial payment Monthly fees Number of


Plan per child per child children
#1 P500 P0 15
#2 200 30 12
#3 50 9
36

Alhambra received all initial payments on September 1, 2005, and P3,240 of monthly fees during the
period September 1 through December 31, 2005. In its December 31, 2005 balance sheet, what
amount should Alhambra report as deferred revenue?
a. 9,900
b. 3,300
c. 6,600
d. 4,380

C = initial payments (500 x 15) + 200 x 12)= 9,900 x 8/12 = 6,600

2. The following are taken from the records of ABC Co. as of year-end.

Accounts payable 2,000 SSS contributions payable 6,000


Utilities payable 7,000 Cash dividends payable 4,000
Accrued interest expense 6,000 Property dividends payable 7,000
Advances from customers 1,000 Share dividends payable 3,000
Unearned rent 9,000 Lease liability 35,000
Warranty obligations 5,000 Bonds payable 120,000
Income taxes payable 2,000 Discount on bonds payable (15,000)
Preference shares issued 10,000 Security deposit 2,000
Constructive obligation 11,000 Redeemable preferences
shares issued 14,000
Obligation to deliver a variable Unearned interest on
number of own shares worth a fixed receivables 3,000
amount of cash 10,000

How much is the total financial liabilities to be disclosed in the notes?


a. 172,000
b. 185,000
c. 192,000
d. 225,000
Page |2

Solution:

Accounts payable 2,000


Utilities payable 7,000
Accrued interest expense 6,000
Obligation to deliver a variable number of own shares
worth a fixed amount of cash 10,000
Cash dividends payable 4,000
Finance lease liability 35,000
Bonds payable 120,000
Discount on bonds payable (15,000)
Security deposit 2,000
Redeemable preference shares 14,000
Total financial liabilities 185,000

Chapter 24 – Notes Payable


3. Entity A purchases a TV set on a 6-month installment basis. The installment price is ₱120,000.
However, if the TV set is purchased outright in cash, the cash price would have been ₱100,000.
The payable will be initially recognized at
a. 100,000
b. 120,000
c. Present value of 120,000 discounted at the current market rate using 6 periods
d. None of these

4. Entity A purchases goods for ₱250,000 under a special credit period of 1 year. The seller
normally sells the goods for ₱220,000 with a credit period of one month or with a ₱5,000
discount for cash basis (i.e., outright payment in cash). The initial measurement of the payable is
a. 250,000
b. 220,000
c. 215,000
d. 200,000

Normal purchase price with credit period of one month 220,000


Discount for cash on delivery (5,000)
Cash price equivalent of the goods purchased 215,000

5. On January 1, 20x1, ABC Co. acquired transportation equipment in exchange for ₱100,000 cash
and ₱1,000,000, noninterest-bearing note payable due in 4 equal annual installments. The first
installment is due on January 1, 20x1. The succeeding installment payments are due every
December 31. The prevailing rate of interest for this type of note is 12%. How much is the
interest income in 20x1?
a. 120,000
b. 102,055
c. 72,055
d. 50,702
Page |3

Future cash flows – annual installments (₱1M ÷ 4) 250,000


Multiply by: PV of an annuity due of ₱1 @12%, n=4 3.401830
Present value of note payable - Jan. 1, 20x1 850,458

Amortization table: (Installment)


Interest
Date Payments expense Amortization Present value
Jan. 1, 20x1 850,458
Jan. 1, 20x1 250,000 - 250,000 600,458
Dec. 31, 20x1 250,000 72,055 177,945 422,513
Dec. 31, 20x2 250,000 50,702 199,298 223,214
Dec. 31, 20x3 250,000 26,786 223,214 0

6. On January 1, 20x1, ABC Co. acquired machinery by issuing a 3-year, ₱1,200,000 noninterest-
bearing note payable due as follows:
Date Amount of installment
December 31, 20x1 600,000
December 31, 20x2 400,000
December 31, 20x3 200,000
Total 1,200,000

The prevailing rate of interest for this type of note is 10%.

How much is the carrying amount of the note on December 31, 20x1?
a. 1,026,296
b. 867,312
c. 528,926
d. 489,762

Date Collections PV of ₱1 @ 10%, n= 1 to 3* Present value


Dec. 31, 20x1 600,000 0.90909 545,455
Dec. 31, 20x2 400,000 0.82645 330,579
Dec. 31, 20x3 200,000 0.75131 150,263
Totals 1,200,000 1,026,296
* PV of ₱1 @10%: n=1 is 0.90909; n=2 is 0.82645; and n=3 is 0.75131

Amortization table: (Installment)


Date Payments Interest expense Amortization Present value
Jan. 1, 20x1 1,026,296
Dec. 31, 20x1 600,000 102,630 497,370 528,926
Dec. 31, 20x2 400,000 52,893 347,107 181,818
Dec. 31, 20x3 200,000 18,182 181,818 0
Page |4

7. On January 1, 20x1, ABC Co. issued a ₱1,200,000 noninterest-bearing note due on December 31,
20x1 in exchange for inventory with a list price of ₱1,100,000 and a cash price of ₱1,000,000. How
much is the carrying amount of the note on December 31, 20x1?
a. 987,234
b. 1,000,000
c. 1,062,695
d. 1,129,321

First trial: (at 10%)


Future cash flows x PV factor at x% = PV of note
 1,200,000 X PV of ₱1 @ 10%, n=3 = 1,000,000
 (1,200,000 x 0.751315) = 901,578 is not equal to 1,000,000

We need a substantially higher amount of present value. Therefore, we need to decrease


substantially the interest rate. Let’s try 6%.

Second trial: (at 6%)


Future cash flows x PV factor at x% = PV of note
 1,200,000 X PV factor at 6%, n=3 = 1,000,000
 (1,200,000 x 0.839619) = 1,007,543 is not equal to 1,000,000

We need a slightly lower amount of present value. Therefore, we need to increase slightly the
interest rate. Let’s try 7%.

Third trial: (at 7%)


Future cash flows x PV factor at x% = PV of note
 1,200,000 X PV factor at 7%, n=3 = 1,000,000
 (1,200,000 x 0.816298) = 979,558 is not equal to 1,000,000

In here, we need to perform interpolation. Looking at the values derived above, we can
reasonably expect that the effective interest rate is a rate between 6% and 7%.

To perform the interpolation, we will use the following formula:


x% - 6%
7% - 6%

Where: x% again is the effective interest rate.

1,000,000 - 1,007,543 (7,543)


= = 0.2695
979,558 - 1,007,543 (27,985)

The amount computed is added to 6% to derive the effective interest rate. The effective
interest rate is 6.2695% (6% + .2695%).

Date Interest income Unearned interest Present value


Jan. 1, 20x1 200,000 1,000,000
Page |5

Dec. 31, 20x1 62,695 137,305 1,062,695


Dec. 31, 20x2 66,626 70,679 1,129,321
Dec. 31, 20x3 70,803 (124) 1,200,124

8. On January 1, 20x1, ABC Co. issued a 3-year, ₱1,000,000 noninterest-bearing note payable to
XYZ, Inc., a related party. The prevailing interest for similar type of obligation is 12%.The
proceeds received from the note is ₱1,000,000, equal to the face amount. How much is the “Day
1” difference? Gain (Loss)
a. 288,220
b. (288,220)
c. 222,880
d. (222,880)

Future cash flow 1,000,000


Multiply by: PV of ₱1, @12%, n=3 0.71178
Present value 711,780

The entry to record the note is as follows:


Jan. 1, Cash 1,000,000
20x1 288,220
Discount on note payable (1M – 711,780)
Note payable 1,000,000
Unrealized gain – “Day 1” Difference 288,220

9. On January 1, 20x1, ABC Co. issued a 3-year, 3%, ₱1,000,000 note payable in exchange for a
machine. Principal is due on January 1, 20x4 but interest is due annually every January 1. The
prevailing interest rate for this type of note is 12%. How much is the carrying amount of the note
on December 31, 20x1?
a. 783,835
b. 883,664
c. 847,895
d. 919,643

Future cash flows Present value factors @12%, n=3 Present value
Principal 1,000,000 0.71178 a 711,780
Annual interest (1M x 3%) 30,000 2.40183 b 72,055
Total 783,835
a (PV of ₱1 @12%, n=3)
b (PV of ordinary annuity of ₱1 @12%, n=3

Amortization table: (Installment)


Payments for
Date interests Interest expense Amortization Present value
Jan. 1, 20x1 783,835
Jan. 1, 20x2 30,000 94,060 64,060 847,895
Jan. 1, 20x3 30,000 101,747 71,747 919,643
Page |6

Jan. 1, 20x4 30,000 110,357 80,357 1,000,000

10. On December 1 a company borrowed ₱100,000 at 12% per year. The interest will be paid
quarterly, with the first payment due on March 1. What should the company report on its
income statement for December?
a. Interest expense of ₱12,000
b. Interest expense of ₱10,000
c. Interest expense of ₱1,000
d. Nothing

Solution: 100,000 x 12% x 1 month/12 = 1,000


11. On May 1, year 1, a company borrowed ₱3,000 cash and signed a 13 percent note payable due
April 30, year 3. Interest is paid each April 30. The accounting period ends December 31.the
adjusting entry at December 31,year 1 would include:
a. debit notes payable,₱390
b. credit interest payable ₱130
c. debit interest expense ₱390
d. credit interest payable ₱260

Chapter 25 – Bonds Payable & Other Concepts

12. Unamortized bond discount should be reported on the financial statements of the issuer as a
a. Direct deduction from the face amount of the bond
b. Direct deduction from the present value of the bond
c. Deferred charge
d. Part of the issue costs

13. For a bond issue which sells for less than its face amount, the market rate of interest is
a. Dependent on the rate stated on the bond.
b. Equal to rate stated on the bond.
c. Less than rate stated on the bond.
d. Higher than rate stated on the bond.

14. The market price of a bond issued at a discount is the present value of its principal amount at the
market (effective) rate of interest
a. Less the present value of all future interest payments at the market (effective) rate of interest.
b. Less the present value of all future interest payments at the rate of interest stated on the
bond.
c. Plus the present value of all future interest payments at the market (effective) rate of interest.
d. Plus the present value of all future interest payments at the rate of interest stated on the
bond.

15. Which of the following is not a relevant consideration when evaluating whether to derecognize
a financial liability?
a. Whether the obligation has been discharged.
Page |7

b. Whether the obligation has been canceled.


c. Whether the obligation has expired.
d. Whether substantially all the risks and rewards of the obligation have been transferred.

16. What is the effective interest rate of a bond or other debt instrument measured at amortized
cost?
a. The stated coupon rate of the debt instrument.
b. The interest rate currently charged by the entity or by others for similar debt instruments
(i.e., similar remaining maturity, cash flow pattern, currency, credit risk, collateral, and
interest basis).
c. The interest rate that exactly discounts estimated future cash payments or receipts through
the expected life of the debt instrument or, when appropriate, a shorter period to the net
carrying amount of the instrument.
d. The basic, risk-free interest rate that is derived from observable government bond prices.

17. Which of the following statements is false?


a. Bonds carry no corporate ownership privileges.
b. A bond is a financial contract.
c. Bond prices remain fixed over time.
d. A bond issuer must pay periodic interest.

18. Most bonds:


a. are money market securities.
b. are floating-rate securities.
c. give bondholders a voice in the affairs of the corporation.
d. are interest-bearing obligations of governments or corporations.

19. In an “asset swap,” where a liability is settled through the transfer of noncash asset,
a. the gain or loss on settlement is computed as the difference between the carrying amount of
the liability extinguished and the fair value of the noncash asset transferred.
b. the gain or loss on settlement is computed as the difference between the carrying amount of
the liability extinguished and the carrying amount of the noncash asset transferred.
c. the gain or loss on settlement is computed as the difference between the carrying amount of
the liability extinguished and the more clearly determinable between the fair value of the
liability extinguished and the carrying amount of the noncash asset transferred.
d. no gain or loss is recognized

20. Entity A issues convertible bonds with face amount of ₱2,000,000 for ₱2,600,000. Each ₱1,000
bond is convertible into 10 shares with par value of ₱60 per share. On issuance date, the bonds
are selling at 102 without the conversion option. What is value allocated to the equity
component on initial recognition?
a. 2,040,000
b. 540,000
c. 560,000
d. 460,000

Solution:
Page |8

Issue price 2,600,000


Fair value of debt instrument without equity feature (2M x 102%) (2,040,000)
Equity component 560,000

Chapter 26 – Provisions, Contingent liabilities and Contingent assets


21. An entity is the defendant in a patent infringement lawsuit. The entity’s lawyers believe there is
a 30% chance that the court will dismiss the case and the entity will incur no outflow of
economic benefits. However, if the court rules in favor of the claimant, the lawyers believe that
there is a 20% chance that the entity will be required to pay damages of ₱800,000 (the amount
sought by the claimant) and an 80% chance that the entity will be required to pay damages of
₱400,000 (the amount that was recently awarded by the same judge in a similar case). Other
outcomes are unlikely.

The court is expected to rule in late December 20x2. There is no indication that the claimant will
settle out of court. A 7% risk adjustment factor to the probability-weighted expected cash flows is
considered appropriate to reflect the uncertainties in the cash flow estimates. An appropriate
discount rate is 10% per year. How much is the provision for lawsuit at December 31, 20x1?
a. 436,360 b. 446,908 c. 326,836 d. 0

C
Solution:
At twenty per cent chance: (800K x 20%) 160,000
At eighty per cent chance: (400K x 80%) 320,000
Total 480,000
Multiply by: PV of P1 @10%, n=1 0.90909
Total 436,363
Multiply by: Risk adjustment (100% + 7%) 107%
Total 466,909
Multiply by: Probability of settlement (100% - 30%) 70%
Provision for lawsuit – Dec. 31, 20x1 326,836

22. A manufacturer gives warranties at the time of sale to purchasers of its product. Under the terms
of the contract of sale, the manufacturer undertakes to make good, by repair or replacement,
manufacturing defects that become apparent within one year from the date of sale. On the basis
of experience, it is probable (i.e., more likely than not) that there will be some claims under the
warranties.

Sales of ₱40 million were made evenly throughout 20X1.

At December 31, 20x1 the expenditures for warranty repairs and replacements for the product sold
in 20x1 are expected to be made 50% in 20x1 and 50% in 20x2. Assume for simplicity that all the 20x2
outflows of economic benefits related to the warranty repairs and replacements take place on June
30, 20x2.
Page |9

Experience indicates that 95% of products sold require no warranty repairs; 3% of products sold
require minor repairs costing 10% of the sale price; and 2% of products sold require major repairs or
replacement costing 90% of sale price. The entity has no reason to believe future warranty claims
will be different from its experience.

At December 31, 20x1, the appropriate discount factor for cash flows expected to occur on June 30,
20x2 is 0.95238. Furthermore, an appropriate risk adjustment factor to reflect the uncertainties in the
cash flow estimates is an increment of 6 per cent to the probability-weighted expected cash flows.

How much is the warranty provision at December 31, 20x1?


a. 424,000 b. 840,000 b. 800,000 d. 752,000

A
Solution:
The amount of the provision is estimated as follows:
Minor repairs (40M x 3% x 10%) 120,000
Major repairs (40M x 2% x 90%) 720,000
Total 840,000
Multiply by: Present value factor (given) 0.95238
Total 800,000
Multiply by: Risk adjustment (100% + 6%) 106%
Total 848,000
Multiply by: Amount to be settled in 20x2 50%
Warranty provision – Dec. 31, 20x1 424,000

23. As of December 31, 20x1, ROUSE AWAKEN Co. has adopted a detailed formal plan to close one
of its toys divisions and put up a new division to manufacture warfare weapons. The plan was
communicated through a public announcement and all of those affected by the closure were
informed. ROUSE estimates the following costs in relation to the closure of the division:

Termination benefits of employees terminated as a


result of the closure ₱4,000,000
Costs of retraining and relocating retained employees 8,000,000
Payment for unpaid purchases made by the division 16,000,000
New systems and distribution networks for the weapons
division 80,000,000
Marketing costs for the weapons to be manufactured by
the new division 24,000,000
Expected losses during the first year of operations of
the weapons division 80,000,000

How much is the provision to be recognized?


a. 4,000,000 b. 12,000,000 c. 84,000,000 d. 20,000,000

A 4,000,000 – termination benefits of employees terminated as a result of the closure.

Use the following information for the next two questions:


P a g e | 10

RISIBLE FUNNY Co. provides 3-year warranty for the products it sells. RISIBLE estimates that
warranty costs ₱400 per unit sold. As of January 1, 20x1, the liability for warranty has a balance of
₱800,000 for units sold in 20x0. During the year RISIBLE sold 5,000 units and actual warranty costs
incurred were ₱1,240,000.

24. How much is the warranty expense to be recognized in 20x1?


a. 2,000,000 b. 1,240,000 c. 3,240,000 d. 4,240,000

A (5,000 units sold x ₱400) = 2,000,000

25. How much is the balance of the warranty obligation as of December 31, 20x1?
a. 1,560,000 b. 2,000,000 c. 3,560,000 d. 2,800,000

A
Solution:
Estimated warranty liability
800,000 Jan. 1, 20x1 (given)
Actual warranty costs 1,240,000 2,000,000 Warranty expense
Dec. 31, 20x1 1,560,000

26. PROFUSE EXTRAVAGANT Co. launched a sales promotion in 20x1. For every ten empty packs
returned to PROFUSE plus ₱200, customers will receive a set of kitchen knives. PROFUSE
estimates that 40% of the packs sold will be redeemed. Information on transactions during the
year is as follows:
Units Amount
Sales 500,000 3B
Sets of kitchen knives purchased (₱800 per set) 300,000 240M
Number of packs redeemed 45,000

How much is the premium expense in 20x1?


a. 8,000,000 b. 12,000,000 c. 14,000,000 d. 16,000,000

B
Solution:
The premium expense is computed as follows:
Sales in units 500,000
Multiply by: Estimate of wrappers to be redeemed 40%
Estimated wrappers to be presented for redemption 200,000
Divide by: Required number of wrappers for redemption 10
Estimated number of premiums to be distributed 20,000
Multiply by: Net cost of premium
(₱800 purchase cost less ₱200 cash requirement from customer) 600
Premium expense 12,000,000

27. On January 1, 20x1, CONFOUND Co. guaranteed a ₱4,000,000 loan obtained by CONFUSE, Inc.
from a bank. On December 31, 20x1, CONFUSE defaulted on its loan and it became probable
that CONFOUND will be held liable to the bank for the ₱4,000,000 loan taken by CONFUSE.
How much is the provision to be recognized?
a. 4,000,000 b. 2,000,000 c. 1,000,000 d. 0
P a g e | 11

A 4,000,000

28. OBTUSE DULL Co. is involved in a tax dispute. OBTUSE has wrongfully paid taxes and is
claiming for refund of the taxes it has previously paid. As of December 31, 20x1, OBTUSE’s legal
counsel was very confident that OBTUSE will be able to recover the tax refund amounting to
₱40M in the coming year. The entry to recognize the probable receipt of the tax refund includes
a. a debit to receivable d. a and b
b. a credit to gain e. none of these
c. a debit to prepaid asset

E – the contingent asset is not recognized.

Chapter 27 – Employee Benefits (Part 1)


29. On December 1, 20x1, ABC Co. hired Juanita Perez to begin working on January 2, 20x2 at a
monthly salary of ₱4,000. ABC's balance sheet on December 31, 20x1 will show a liability of
a. ₱4,000.
b. ₱48,000.
c. ₱480,000.
d. No Liability

30. Entity A has 20 employees who are each entitled to one day paid vacation leave for each month
of service rendered. Unused vacation leaves are carried forward and can be used in future
periods if the current period’s entitlement is not used in full. Moreover, employees are entitled
to a cash payment for unused entitlement when they leave the entity. All the employees have
rendered service throughout the current year and have taken a total of 150 days of vacation
leaves. The average daily rate of the employees in the current period is ₱1,000. However, a 5%
increase in the rate is expected to take into effect in the following year. Based on Entity A’s past
experience, the average annual employee turnover rate is 20%. How much will Entity A accrue
at the end of the current year for unused entitlements?
a. 0 c. 90,000
b. 75,600 d. 94,500

[(20 employees x 1 day x 12 months) – 150 days] x ₱1,000 x 105% = 94,500. The 20% employee
turnover rate is irrelevant because the employee benefits are monetized.

31. Under its post-employment benefit plan, Entity A agrees to make annual contributions of
₱500,000 to a retirement fund. When an employee retires, he or she is entitled to a lump sum
payment and monthly pension payments to be determined based on the level of contributions
and the investment performance of the fund.

In 20x1, due to cash flow problems, Entity A was only able to contribute half of the agreed
contributions. In 20x2, Entity A contributed ₱900,000 to the fund. How much retirement benefit
expenses should Entity A recognize in 20x1 and 20x2, respectively?
20x1 20x2
P a g e | 12

a. 200,000 900,000
b. 200,000 500,000
c. 500,000 600,000
d. 500,000 500,000

32. Entity A has 20 employees who are each entitled to one day paid vacation leave for each month
of service rendered. Unused vacation leaves are carried forward and can be used in future
periods if the current period’s entitlement is not used in full. However, unutilized entitlements
are forfeited when employees leave the entity. All the employees have rendered service
throughout the current year and have taken a total of 150 days of vacation leaves. The average
daily rate of the employees in the current period is ₱1,000. However, a 5% increase in the rate is
expected to take into effect in the following year. Based on Entity A’s past experience, the
average annual employee turnover rate is 20%. How much will Entity A accrue at the end of the
current year for unused entitlements?
a. 0
b. 90,000
c. 75,600
d. 94,500

Solution: [(20 employees x 1 day x 12 months) – 150 days] x ₱1,000 x 105% x 80%* = 75,600
* The paid absences are non-vesting.

33. Under a profit-sharing plan, Entity A agrees to pay its employees 5% of its annual profit. The
bonus shall be divided among the employees currently employed as at year-end. Relevant
information follows:

Profit for the year ₱8,000,000


Employees at the beginning of the year 8
Average employees during the year 7
Employees at the end of the year 6

If you are an alumnus of Entity A, how much bonus do you expect to receive?
a. 66,667
b. 50,000
c. 57,143
d. 0

Explanation: Only those who are currently employed as at year-end are entitled to receive the bonus.

34. The computation of employee retirement benefits expense is addressed in this standard.
a. PAS 17
b. PFRS 7
c. PAS 19
d. PFRS 9
P a g e | 13

Chapter 28 – Employee Benefits (Part 2)

Use the following information for the next three questions:


Information on Entity A’s defined benefit plan is as follows:
PV of DBO – Jan. 1, 20x1 2,000,000
FVPA – Jan.1, 20x1 1,800,000
PV of DBO – Dec. 31, 20x1 2,900,000
FVPA, end. – Dec. 31, 20x1 2,600,000
Current service cost 400,000
Actuarial loss 200,000
Return on plan assets 120,000
Discount rate 10%

35. How much is the net defined benefit liability (asset) to be presented in Entity A’s December 31,
20x1 statement of financial position?
a. (300,000) c. (200,000)
b. 300,000 d. 200,000

B Solution: 2,900,000 PV of DBO – 2,600,000 FVPA = 300,000 deficit

36. How much is the component of the 20x1 defined benefit cost to be recognized in profit or loss?
a. 400,000 c.
b. 420,000 d.

B Solution: 400,000 service cost + 20,000 net interest (see computations below) = 420,000

37. How much is the component of the 20x1 defined benefit cost to be recognized in other
comprehensive income – (income)/ loss?
a. (140,000) c. 260,000
b. 140,000 d. (260,000)

C Solution:
Service cost:
(a) Current service cost 400,000
(b) Past service cost -
(c) Any (gain) or loss on settlement -
400,000
Net interest on the net defined benefit liability (asset):
(a) Interest cost on the DBO (2M, beg. x 10%) 200,000
(b) Interest income on plan assets (1.8M, beg. x 10%) (180,000)
(c) Interest on the effect of the asset ceiling -
20,000
Remeasurements of the net defined benefit liability (asset):
(a) Actuarial (gains) and losses 200,000
(b) Difference between interest income on plan assets and
return on plan assets (180,000 – 120,000) 60,000
(c) Difference between the interest on the effect of the asset -
P a g e | 14

ceiling and change in the effect of the asset ceiling


260,000
Total Defined Benefit Cost 680,000

38. You are the General Manager of Entity A. You have received the actuarial report for your
company’s defined benefit plan. The report shows the following information:
PV of DBO – Jan. 1, 20x1 1,500,000
FVPA – Jan. 1, 20x1 1,200,000
PV of DBO – Dec. 31, 20x1 1,800,000
FVPA, end. – Dec. 31, 20x1 1,310,000
Actuarial gain 100,000
Return on plan assets 110,000
Discount rate 5%

When reporting on your company’s year-end highlights of financial summary, which of the
following will you report to the Board of Directors (the ‘big bosses’)?
a. Your company’s net liability for retirement benefits has increased by ₱490,000.
b. Your company’s net liability for retirement benefits has decreased by ₱300,000.
c. Your company’s net liability for retirement benefits has increased by ₱190,000.
d. I will tell them nothing.

Solution:
Net defined benefit liability, beg. (1.5M – 1.2M) = 300,000
Net defined benefit liability, end. (1.8M – 1.310M) = 490,000
Increase = 190,000

Chapter 29 – Leases (Part 1)


39. Entity A (customer) enters into a contract with Entity B (supplier) for the use of a data
processing equipment. According to the contract, Entity A shall operate the equipment only in
accordance with the standard operating procedures stated in the accompanying user’s manual.
In assessing the existence of a lease, does Entity A have the right to direct the use of the asset?
a. No, because the asset’s use is restricted.
b. Yes, because Entity A has the right to direct how and for what purpose the asset is used.
c. Yes, because the asset’s use is predetermined and Entity B is precluded from changing that
predetermined use.
d. Maybe yes, maybe no, but exactly I don’t know.

40. Which of the following is not one of the criteria when determining whether a contract is or
contains a lease?
a. Identified asset
b. Identified liability
c. Right to obtain substantially all of the economic benefits from use of an identified asset
throughout the period of use
d. Right to direct the use of the identified asset throughout the period of use
P a g e | 15

41. Which of the following statements is correct regarding the accounting for leases?
a. The lessor depreciates the leased asset under a finance lease.
b. The lessee depreciates the leased asset under a “short-term” or a “low-valued asset” lease.
c. When discounting lease payments the lessor and the lessee use the interest rate implicit in
the lease.
d. An entity can never be both a lessor and a lessee of a same leased asset.

Choice (d) is incorrect. Under a sublease, the lessee is also the lessor of the same leased asset.

42. According to PFRS 16, lease liabilities are presented in the lessee’s statement of financial position
a. separately from the other liabilities of the lessee.
b. together with other liabilities, with disclosure of the line items that include the lease
liabilities.
c. a or b
d. not presented in the lessee’s financial statements but only in the lessor’s financial statements

43. According to PFRS 16, right-of-use assets are presented in the lessee’s statement of financial
position
a. separately from the other assets of the lessee.
b. together with other assets as if they were owned, with disclosure of the line items that
include the right-of-use assets.
c. a or b
d. not presented in the lessee’s financial statements but only in the lessor’s financial statements

44. On December 30, 20x5, Haber Co. leased a new machine from Gregg Corp. The following data
relate to the lease transaction at the inception of the lease:
Lease term 10 years
Annual rental payable at the end of each lease year ₱100,000
Useful life of machine 12 years
Implicit interest rate 10%

The lease has no renewal option, and the possession of the machine reverts to Gregg when the lease
terminates. At the inception of the lease, Haber should record a lease liability of
a. 0 b. 615,000 c. 630,000 d. 676,000

B
Solution:
Annual rent 100,000
PV of ordinary annuity of 1 @10%, n=10 6.15
PV of minimum lease payments 615,000

45. On January 2, 20x6, Ashe Company entered into a ten-year noncancellable lease requiring year-
end payments of ₱100,000. Ashe's incremental borrowing rate is 12% while the lessor's implicit
interest rate, known to Ashe, is 10%. Ownership of the property remains with the lessor at
expiration of the lease. There is no bargain purchase option. The leased property has an
P a g e | 16

estimated economic life of 12 years. What amount should Ashe capitalize for this leased
property on January 2, 20x6?
a. 1,000,000 b. 614,500 c. 565,000 d. 0

B (100,000 x PV of ordinary annuity of 1 @10%, n=10) = 614,500

46. Neal Corp. entered into a nine-year lease on a warehouse on December 31, 20x1. Lease payments
of ₱52,000, which includes payment for non-lease component of ₱2,000 (at stand-alone selling
price), are due annually, beginning on December 31, 20x1, and every December 31 thereafter.
Neal does not know the interest rate implicit in the lease; Neal's incremental borrowing rate is
9%. What amount should Neal report as lease liability at December 31, 20x1?
a. 280,000 b. 291,200 c. 450,000 d. 468,000

A
Solution:
Total consideration 52,000
Less: Payment for non-lease component (2,000)
Lease payments 50,000
Multiply by: PV of annuity due @9%, n=9 6.5348
Total 326,740
First payment due in advance (50,000)
Lease liability – Dec. 31, 20x1 276,740
*Answer choice is rounded-off

47. Robbins, Inc., leased a machine from Ready Leasing Co. The lease requires 10 annual payments
of ₱10,000 beginning immediately. The lease specifies an interest rate of 12% and a purchase
option of ₱10,000 at the end of the tenth year, even though the machine's estimated value on that
date is ₱20,000. It is reasonably certain that Robbins will exercise the purchase option. Robbins'
incremental borrowing rate is 14%. What amount should Robbins record the right-of-use asset at
the beginning of the lease term?
a. 62,160 b. 64,860 c. 66,500 d. 69,720
C
Solution:
Cash flows PV factors PV
Annual rent 10,000 PV annuity due @12%, n=10 6.3282 63,282
PO 10,000 PV of 1 @12%, n=10 0.3220 3,220
66,502

48. On January 1, 20x7, Babson, Inc., leased two automobiles for executive use. The lease requires
Babson to make five annual payments of ₱13,000 beginning January 1, 20x7. At the end of the
lease term, Babson guarantees the residual value of the automobiles will total ₱10,000. The
interest rate implicit in the lease is 9%. Babson's recorded lease liability on initial recognition is
a. 48,620 b. 44,070 c. 35,620 d. 31,070

A
Solution:
Cash flows PV factors PV
Annual rent 13,000 PV annuity due @9%, n=5 4.2397 55,116
Guaranteed RV 10,000 PV of 1 @9%, n=5 0.6499 6,499
Total 61,615
P a g e | 17

First payment due immediately (13,000)


Lease liability – initial recognition 48,615

49. On January 1, 20x1, ABC Co. enters into a 4-year lease of office equipment. The rent in 20x1 is
₱10,000 and shall increase by 10% annually starting on January 1, 20x2. Rentals are payable at
the end of each year. ABC Co. pays the lessor a lease bonus of ₱5,000 on January 1, 20x1. ABC
Co. opts to use the practical expedient allowed under PFRS 16 for leases of low value assets.
How much is the lease expense in 20x1?
a. 10,000 b. 11,000 c. 11,603 d. 12,853

D Solution:

20x1 10,000
20x2 (10K x 110%) 11,000
20x3 (11K x 110%) 12,100
20x4 (12.1K x 110%) 13,310
Lease bonus 5,000
Total 51,410
Divide by: 4
Annual lease expense 12,853

Chapter 30 – Leases (Part 2)


Use the following information for the next three questions:
On January 1, 20x1, Entity Y leases out an equipment to Entity X. Information on the lease is as
follows:
Lease term 3 years
Annual rent payable at the end of each year 100,000
Interest rate implicit in the lease 10%

The lease provides for the transfer of ownership of the equipment to the lessee at the end of the lease
term. The relevant present value factor is as follows:

50. How much is the gross investment on January 1, 20x1?


a. 500,000
b. 400,000
c. 300,000
d. 200,000

Fixed lease payments 100,000


Multiply by: Lease term 3
Total lease payments 300,000
Add: Unguaranteed residual value -
Gross investment in the lease 300,000

51. How much is the net investment on January 1, 20x1?


P a g e | 18

a. 238,664
b. 242,883
c. 248,685
d. 252,086

Fixed lease payments 100,000


Multiply by: PV of ordinary annuity of ₱1 @10%, n=3 2.48685
Net investment in the lease 248,685

52. How much is the unearned interest on January 1, 20x1?


a. 51,315
b. 52,313
c. 48,992
d. 47,652

Gross investment 300,000


Less: Net investment (248,685)
Unearned interest income 51,315

Chapter 31 – Income Taxes

53. If the current tax expense is less than the income tax expense during the period, there must be a
a. deferred tax benefit c. income tax payable
b. deferred tax expense d. prepaid income tax

54. Trade receivables have a carrying amount of P4,000. The related revenue has already been
included in taxable profit (tax loss). How much is the tax base of the asset?
a. 4,000 b. 2,400 c. 1,600 d. 0

55. Dividends receivable from a subsidiary have a carrying amount of P4,000. The dividends are not
taxable. How much is the tax base of the asset?
a. 4,000 b. 2,400 c. 1,600 d. 0

56. Current liabilities include accrued fines and penalties with a carrying amount of P4,000. Fines
and penalties are not deductible for tax purposes. How much is the tax base of the liability?
a. 4,000 b. 2,400 c. 1,600 d. 0

57. A loan payable has a carrying amount of P4,000. The repayment of the loan will have no tax
consequences. How much is the tax base of the liability?
a. 4,000 b. 2,400 c. 1,600 d. 0
P a g e | 19

Use the following information for the next four questions:


Taken from the records of ABC Co. as of December 31, 20x1 is the following information:
Carrying amount Tax base Difference
Computer software cost 500,000 - 500,000
Machinery 1,000,000 600,000 400,000
Accrued liability - health care 200,000 - 200,000

Additional information:
• Software development costs after technological feasibility was established were capitalized for
financial reporting. The costs were recognized as outright deductions for tax purposes.
• Straight line method is used in depreciating the machinery while sum-of-the-years’ digits method
is used for tax purposes.
• Health care benefits are accrued as incurred but are tax deductible only when cash is actually
paid.
• Pretax profit for 20x1 is ₱1,000,000. Income tax rate is 30%.
• There were no temporary differences as of January 1, 20x1.

58. How much is the deferred tax liability on December 31, 20x1?
a. 400,000
b. 900,000
c. 320,000
d. 270,000

Excess of carrying amount of software over its tax base 500,000


Excess of carrying amount of machinery over its tax base 400,000
Taxable temporary difference (TTD) 900,000
Multiply by: Tax rate 30%
Deferred tax liability – Dec. 31, 20x1 270,000

59. How much is the deferred tax asset on December 31, 20x1?
a. 270,000
b. 120,000
c. 90,000
d. 60,000

Excess of carrying amount of accrued liability over its tax base 200,000
Deductible temporary difference (DTD) 200,000
Multiply by: Tax rate 30%
Deferred tax asset – Dec. 31, 20x1 60,000

60. How much is the deferred tax expense/benefit in 20x1?


a. 210,000 expense
b. 210,000 benefit
c. 270,000 expense
d. 270,000 benefit
P a g e | 20

Increase in DTL* (270,000)


Increase in DTA* 60,000
Deferred tax expense (210,000)
*Amounts are taken from the table below.

61. How much is the current tax expense in 20x1?


a. 300,000
b. 160,000
c. 90,000
d. 64,000

Multiply by
Description of items Description of items
Tax rate
Pretax income 1,000,000
Permanent differences -
Acctg. profit subj. to tax 1,000,000 30% Income tax expense 300,000
Less:  TTD (900,000) 30% Less:  DTL (270,000)
Add:  DTD 200,000 30% Add:  DTA 60,000
Taxable profit 300,000 30% Current tax expense 90,000

62. ABC Co. is determining the amount of its pretax accounting income for the year by making
adjustment to taxable income from the company's year-end income tax return. The tax return
indicates taxable income of ₱100,000, on which a tax liability of ₱30,000 has been recognized
(₱100,000 x 30% = ₱30,000). Additional information is shown below:

Goodwill impairment loss not included as a deduction in


the tax return but may be deducted for financial
reporting. 35,000
Interest income on savings and time deposits with
private banks 6,000
Revenues from installment sales are recognized as
goods are sold but are taxed only when installment
payments are collected. 40,000
Excess of depreciation recognized for financial reporting
over depreciation recognized for taxation purposes due
to shorter depreciation period used for financial 10,000
reporting
Bad debt expense recognized using the allowance
method 15,000

How much is the pretax income?


a. 115,000
b. 100,000
c. 96,000
P a g e | 21

d. 86,000

Solution:

Pretax income (squeeze) 86,000


Add: Non-deductible expense:
Goodwill impairment loss not tax deductible 35,000
Less: Interest income subject to final tax (6,000)
Accounting profit subject to tax 115,000
Less:  Taxable temporary difference (TTD) 'FI>TI':
Revenues accrued but taxed on cash basis (40,000)
Add:  Deductible temporary difference (DTD) 'FI<TI'
Excess of depreciation 10,000
Bad debts recognized under allowance method 15,000
Taxable profit (start) 100,000

Chapter 32 – Shareholders’ Equity (Part 1)

63. Legal capital is the portion of contributed capital that cannot be distributed to the owners during
the lifetime of the corporation unless the corporation is dissolved and all of its liabilities are
settled first. For no-par value shares, legal capital is
a. the aggregate par value of shares issued and subscribed.
b. the total consideration received or receivable from shares issued or subscribed.
c. the aggregate stated value of shares issued and subscribed.
d. the aggregate market value of shares issued and subscribed.

64. Which of the following is not one of the basic shareholders rights?
a. The right to participate in earnings.
b. The right to maintain one's proportional interest in the corporation.
c. The right to participate in the proceeds of the sale of corporate assets upon liquidation of the
corporation.
d. The right to inspect the accounting records of the corporation.

65. On February 1, authorized ordinary share was sold on a subscription basis at a price in excess of
par value, and 20 percent of the subscription price was collected. On May 1, the remaining 80
percent of the subscription price was collected. Share premium would increase on
February 1 May 1
a. No Yes
b. No No
c. Yes No
d. Yes Yes

66. The entry to record the issuance of ordinary shares for fully paid share subscriptions is
a. a memorandum entry.
b. Dr. Common Stock Subscribed; Cr. Common Stock; Cr. Additional Paid-In Capital
P a g e | 22

c. Dr. Subscribed Share Capital; Cr. Subscriptions Receivable


d. Dr. Subscribed Share Capital; Cr. Share Capital

67. The issuance of shares of preferred stock to shareholders


a. increases preferred stock outstanding.
b. has no effect on preferred stock outstanding.
c. increases preferred stock authorized.
d. decreases preferred stock authorized.

68. Which of the following is an appropriate presentation of treasury stock?


a. As a marketable security
b. As a deduction at cost from total stockholders' equity
c. As a deduction at cost from total contingent liabilities
d. As a deduction at par from total stockholders' equity

69. Gains and losses on the purchase and resale of treasury stock may be reflected only in
a. share premium account.
b. share premium and retained earnings accounts.
c. income, paid-in capital, and retaining earnings accounts.
d. income and paid-in capital accounts.

70. The stockholders' equity section of Peter Corporation's balance sheet at December 31, 20X2, was
as follows:
Ordinary shares (₱10 par value, authorized 1,000,000
shares, issued and outstanding 900,000 shares) ₱ 9,000,000
Share premium 2,700,000
Retained earnings 1,300,000

On January 2, 20X3, Peter purchased and retired 100,000 shares of its stock for ₱1,800,000.
Immediately after retirement of these 100,000 shares, the balances in the share premium and retained
earnings accounts should be
Share premium Retained earnings
a. ₱ 900,000 ₱1,300,000
b. ₱1,400,000 ₱ 800,000
c. ₱1,900,000 ₱1,300,000
d. ₱2,400,000 ₱ 800,000

Solution:
Jan. 2, Share capital (100,000 x ₱10) 1,000,000
20x3 Sh. premium – orig. issuance (2.7M x 100K/900K) 300,000
Retained earnings 500,000
Cash 1,800,000

Share premium Retained earnings


Dec. 31, 20x2 2,700,000 1,300,000
Debit (300,000) (500,000)
P a g e | 23

Jan. 2, 20x3 2,400,000 800,000

71. Asp Co. was organized on January 2, 20x1, with 30,000 authorized shares of ₱10 par ordinary
shares. During 20x1 the corporation had the following capital transactions:

Jan. 5 Issued 20,000 shares at ₱15 per share.


July 14 Purchased 5,000 shares at ₱17 per share.
Dec. 27 Reissued the 5,000 shares held in treasury at ₱20 per share.

Asp used the cost method to record the purchase and reissuance of the treasury shares. In its
December 31, 20x1, balance sheet, what amount should Asp report as share premium in excess of
par?
a. 100,000
b. 125,000
c. 140,000
d. 115,000

Solution:
Jan. 5, Cash (20,000 x 15) 300,000
20x1
Ordinary share (20,000 x 10) 200,000
Share premium 100,000
July 14, Treasury shares (5,000 x 17) 85,000
20x1
Cash 85,000
Dec. 27, Cash (5,000 x 20) 100,000
20x1
Treasury shares (5,000 x 17) 85,000
Share premium – Treasury shares 15,000

100,000 + 15,000 = 115,000

72. In 20x0, Newt Corp. acquired 6,000 shares of its own ₱1 par value ordinary share at ₱18 per
share. In 20x1, Newt issued 3,000 of these shares at ₱25 per share. Newt uses the cost method to
account for its treasury stock transactions. What accounts and amounts should Newt credit in
20x1 to record the issuance of the 3,000 shares?
Treasury sh. Sh. premium Retained earnings Ordinary sh.
a. ₱54,000 ₱21,000
b. ₱54,000 ₱21,000
c. ₱72,000 ₱3,000
d. ₱51,000 ₱21,000 ₱3,000

Solution:
Dec. 27,Cash (3,000 x 25) 75,000
20x1
Treasury shares (3,000 x 18) 54,000
Share premium – Treasury shares 21,000

73. On December 1, 20x1, Line Corp. received a donation of 2,000 shares of its ₱5 par value ordinary
shares from a shareholder. On that date, the stock’s market value was ₱35 per share. The stock
P a g e | 24

was originally issued for ₱25 per share. By what amount would this donation cause total
stockholders’ equity to decrease?
a. 70,000
b. 50,000
c. 20,000
d. 0

Chapter 33 – Shareholders’ Equity (Part 2)


74. An enterprise has made all necessary adjusting entries and is now closing its accounts for the
period. Dividends of ₱30,000 were declared and distributed during the year. The entry to close
the dividends account would be
a. Retained earnings 30,000
Dividends 30,000
b. Dividends 30,000
Retained earnings 30,000
c. Income summary 30,000
Dividends 30,000
d. Dividends 30,000
Income summary 30,000

75. Nest Co. issued 100,000 shares of common stock (i.e., ordinary shares). Of these, 5,000 were held
as treasury stock at December 31, 20x1. During 20x2, transactions involving Nest's common
stock were as follows:
• May 3 - 1,000 shares of treasury stock were sold.
• August 6 - 10,000 shares of previously unissued stock were sold.
• November 18 - a 2-for-1 stock split took effect.

Laws in Nest's state of incorporation protect treasury stock from dilution. At December 31, 20x2,
how many shares of Nest's common stock were issued and outstanding?
Shares Issued Outstanding
a. 220,000 212,000
b. 220,000 216,000
c. 222,000 214,000
d. 222,000 218,000

Solution:
Issued Outstanding
Issued as of Dec. 31, 20x1 100,000 100,000
Treasury shares as of Dec. 31, 20x1 (5,000)
20x2 transactions:
May 3 - reissuance of treasury shares 1,000
Aug. 6 - issuance of new shares 10,000 10,000
Totals 110,000 106,000
Nov. 18 - 2-for-1 share split 2 2
Ending balances 220,000 212,000
P a g e | 25

76. At December 31, 20x0 and 20x1, Carr Corp. had outstanding 4,000 shares of ₱100 par value 6%
cumulative preferred stock and 20,000 shares of ₱10 par value common stock (i.e., ordinary
shares). At December 31, 20x0, dividends in arrears on the preferred stock were ₱12,000. Cash
dividends declared in 20x1 totaled ₱44,000. Of the ₱44,000, what amounts were payable on each
class of stock?
Preference shares Ordinary shares
a. ₱44,000 ₱ 0
b. ₱36,000 ₱ 8,000
c. ₱32,000 ₱12,000
d. ₱24,000 ₱20,000

Solution:
Total cash dividends declared 44,000
Dividends to preference sh. [(4,000 x 100 x 6%) + 12,000] (36,000)
Dividends to ordinary sh. 8,000

77. Arp Corp.’s outstanding capital stock at December 15, 20x1, consisted of the following:
• 30,000, 5% cumulative preference shares, par value ₱10 per share, fully participating as to
dividends. No dividends were in arrears.
• 200,000 ordinary shares, par value ₱1 per share.

On December 15, 20x1, Arp declared dividends of ₱100,000. What was the amount of dividends
payable to Arp’s ordinary stockholders?
a. 10,000
b. 34,000
c. 40,000
d. 47,500

Solution:
Total dividends declared 100,000
Allocation:
Basic allocation to preference shares: (30,000 x 10 x 5%) 15,000
Basic allocation to ordinary shares: (200,000 x 1 x 5%) 10,000
Excess subject to participation (100,000 – 15,000 – 10,000) 75,000
Participation of preference sh. (75,000 x 3/5) 45,000
Participation of ordinary sh. (75,000 x 2/5) 30,000
Balance -

Total dividends to ordinary shareholders = 10,000 + 30,000 = 40,000

78. The following stock dividends were declared and distributed by Sol Corp.:
Percentage of ordinary shares
outstanding at declaration date Fair value Par value
10 ₱15,000 ₱10,000
28 40,000 30,800
P a g e | 26

What aggregate amount should be debited to retained earnings for these stock dividends?
a. 40,800
b. 45,800
c. 50,000
d. 55,000

Solution:
10% ('small' dividend) - at fair value 15,000
28% ('large' dividend) - at par value 30,800
Total debit to retained earnings 45,800

79. Ray Corp. declared a 5% stock dividend on its 10,000 issued and outstanding shares of ₱2 par
value common stock, which had a fair value of ₱5 per share before the stock dividend was
declared. This stock dividend was distributed 60 days after the declaration date. By what
amount did Ray’s current liabilities increase as a result of the stock dividend declaration?
a. 0
b. 500
c. 1,000
d. 2,500

Explanation: Stock dividend payable is not a liability.

Chapter 34 – Share-based Payments (Part 1)


80. A share-based payment transaction is one in which an entity receives goods or services and pays
for them
a. by issuing its own equity instruments.
b. through cash, but the amount is based on the fair value of the entity’s equity instruments.
c. either a or b, as a choice given to either the entity or the supplier of the goods or services
d. any of these

81. Which of the following is excluded from the scope of PFRS 2?


a. Employee share option plans
b. Employee share appreciation rights
c. Purchase of goods from an unrelated party in exchange for an entity’s own shares of stocks
d. Transfer of equity instruments as consideration for a business combination

82. On February 1, 20x1, Entity A offered its employees share options subject to the offer being
ratified in the shareholders’ general meeting. The share option offer was approved in the
shareholders’ general meeting held on March 1, 20x1. Entity A issued the share options on April
1, 20x1. The fair value of the share options vary between these dates. For purposes of PFRS 2, the
share options should be valued at the fair value determined on
a. February 1, 20x1. c. April 1, 20x1.
b. March 1, 20x1. d. any of these
P a g e | 27

83. On January 1, 20x4, Entity A has granted 600 share options to each of its 100 employees. The
options vest in three years’ time. Each share option has a fair value of ₱100 on grant date.
Information on employee departure is as follows:
• January 1, 20x4: estimate of employees leaving the entity during the vesting period – 4%
• December 31, 20x4: revision of estimate of employees leaving to 5% before vesting date
• December 31, 20x5: revision of estimate of employees leaving to 6% before vesting date
• December 31, 20x6: actual employees leaving 5%

How much is the salaries expense in 20x5?


a. 2,000,000
b. 1,900,000
c. 1,860,000
d. 1,840,000

C
20x4: (600 x 100 x 100) x 95% x 1/3 = 1,900,000;
20x5: (600 x 100 x 100) x 94% x 2/3 = 3,760,000 - 1,900,000 = 1,860,000

Use the following information for the next two questions:


On January 1, 20x1, Entity A grants 100 share options to each of its 100 key employees conditional
upon each employee remaining in Entity A’s employ over the next 3 years. The fair value of each
share option is ₱15.

On the basis of a weighted average probability, Entity A estimates on January 1, 20x1 that about 20
employees (i.e., 20% or 20 out of the 100 employees) will leave during the three-year period and
therefore forfeit their rights to the share options.

During 20x1, 7 employees left. Entity A revises its estimate to a total of 25% employee departure
over the vesting period.

During 20x2, 9 employees left. Entity A revises its estimate to a total of 28% employee departure
over the vesting period.

During 20x3, 8 employees left. Therefore, the actual employee departure over the past three years is
24% [(7 + 9 + 8) ÷ 100].

84. How much is the salaries expense in 20x2?


a. 33,700
b. 37,500
c. 34,500
d. 42,000

Date Salaries expense


Jan. 1, 20x1 -
Dec. 31, 20x1 (10,000 (a) x 75%(b)) x ₱15 x 1/3 37,500
P a g e | 28

Dec. 31, 20x2 [(10,000 x 72%) x ₱15 x 2/3] - 37,500 34,500


Dec. 31, 20x3 [(10,000 x 76%) x ₱15 x 3/3] - 37,500 - 34,500 42,000

85. How much is the salaries expense in 20x3?


a. 33,700
b. 37,500
c. 34,500
d. 42,000

Chapter 35 – Share-based Payments (Part 2)


Use the following information for the next two questions:
On January 1, 20x1, Entity A grants 1,000 share appreciation rights (SARs) to employees with the
condition that the employees remain in service within the next 3 years. Information on the SARs is
shown below:

Date No. of SARs expected to vest Fair value of each SAR


Jan. 1. 20x1 1,000 10
Dec. 31, 20x1 900 12
Dec. 31, 20x2 800 15
Dec. 31, 20x3 750 16

86. How much is the salaries expense in 20x3?


a. 3,700
b. 4,500
c. 4,000
d. 4,400

Date Salaries expense


Jan. 1, 20x1 -
Dec. 31, 20x1 900 x ₱12 x 1/3) 3,600
Dec. 31, 20x2 (800 x ₱15 x 2/3) - 3,600 4,400
Dec. 31, 20x3 (750 x ₱16 x 3/3) – 3,600 – 4,400 4,000

87. How much is the accrued salaries payable on December 31, 20x2?
a. 4,400
b. 7,500
c. 8,000
d. 8,400

(3,600 + 4,400) = 8,000


P a g e | 29

Chapter 36 – Book Value per Share

Use the following information for the next three questions:


Caroline Co.’s equity structure at December 31, 20x1 is shown below:

10% Preference sh., ₱100 par (liquidation value ₱120 per share) 1,000,000
Ordinary shares, ₱100 par 3,000,000
Subscribed share capital - ordinary shares 100,000
Subscription receivable (60,000)
Retained earnings 900,000
Treasury shares (at cost) - 2,000 ordinary shares. (260,000)
Total shareholders' equity 4,680,000

88. The preference shares are cumulative. Dividends are in arrears for three years. How much is the
book value per ordinary share?
a. 150
b. 111.72
c. 112.37
d. 141.38

Total shareholders' equity, excldg.


4,740,000
subscription receivable (4.68M + 60K)
Preference shareholders' equity:
Liquidation value (10,000 shares x ₱120) 1,200,000
Dividends in arrears (1M x 10% x 3 yrs.) 300,000 (1,500,000)
Ordinary shareholders' equity 3,240,000

Preference shareholders' equity 1,500,000


Divide by: No. of preference shares outstanding 10,000
Book value per share (Preference shares) 150.00

Ordinary shareholders' equity 3,240,000


Divide by: No. of ordinary shares outstanding a 29,000
Book value per share (Ordinary shares) 111.72

a Ordinaryshares issued (3M / 100par) 30,000


Subscribed shares (100K / 100par) 1,000
Treasury shares (2,000)
Outstanding ordinary shares 29,000

89. The preference shares are noncumulative. Dividends are in arrears for three years. How much is
the book value per ordinary share?
a. 118.62
b. 112.62
c. 98.87
d. 122.39
P a g e | 30

Solution:

Total shareholders' equity, excldg.


4,740,000
subscription receivable (4.68M + 60K)
Preference shareholders' equity:
Liquidation value (10,000 shares x ₱120) 1,200,000
Dividends in arrears (1M x 10% x 1 yr.) 100,000 (1,300,000)
Ordinary shareholders' equity 3,440,000

Preference shareholders' equity 1,300,000


Divide by: No. of preference shares outstanding 10,000
Book value per share (Preference shares) 130.00

Ordinary shareholders' equity 3,440,000


Divide by: No. of ordinary shares outstanding 29,000
Book value per share (Ordinary shares) 118.62

90. The preference shares are cumulative. All dividends are paid up to end of the current year. How
much is the book value per ordinary share?
a. 120.00
b. 119.82
c. 118.62
d. 122.07

Solution:

Total shareholders' equity, excldg.


4,740,000
subscription receivable (4.68M + 60K)
Preference shareholders' equity:
Liquidation value (10,000 shares x ₱120) 1,200,000
Dividends in arrears - (1,200,000)
Ordinary shareholders' equity 3,540,000

Preference shareholders' equity 1,200,000


Divide by: No. of preference shares outstanding 10,000
Book value per share (Preference shares) 120.00

Ordinary shareholders' equity 3,540,000


Divide by: No. of ordinary shares outstanding a 29,000
Book value per share (Ordinary shares) 122.07

91. The shareholders' equity of ABC Construction, Inc. on December 31, 20x1 includes the following:

8% Preference shares, 20,000 shares, ₱100 par value 3,000,000


P a g e | 31

10% Preference shares, 10,000 shares, ₱300 par value 4,500,000


Ordinary shares, 50,000 shares, ₱100 par value 7,500,000
Share premium in excess of par 2,250,000
Retained earnings 3,350,000
Total shareholders' equity 20,600,000

The 8% stock is cumulative and fully participating. The 10% stock is noncumulative and fully
participating. Dividends have not yet been paid for 3 years.

How much is the book value per ordinary share?


a. 192.30
b. 200.30
c. 202.30
d. 205.30

Solution:

Total shareholders' equity 20,600,000


8% PS (aggregate par value) (3,000,000)
8% PS (dividends) - (3M x 8% x 3 yrs.) (720,000)
10% PS (aggregate par value) (4,500,000)
10% PS (dividends) - (4.5M x 10% x 1 yr.) (450,000)
Ordinary shares (aggregate par value) (7,500,000)
Ordinary shares (dividends) - (7.5M x 8% x 1 yr.) (600,000)
Amount for allocation 3,830,000
8% PS (3.83M x 3/15) (766,000)
10% PS (3.83M x 4.5/15) (1,149,000)
Ordinary shares (3.83M x 7.5/15) (1,915,000)
As allocated -

Ordinary shareholders' equity:


Aggregate par value 7,500,000
Dividends 600,000
Participation 1,915,000
Total 10,015,000
Divide by: Outstanding shares 50,000
Book value per ordinary share 200.30

Chapter 37 – Earnings per Share


92. The computation of earnings per share is addressed by
a. PAS 36
b. PFRS 3
c. PAS 33
d. PFRS 11

93. Entity A had 100,000, ₱10 par, 10% cumulative preference shares outstanding all throughout
20x1. Entity A reported profit after tax of ₱2,800,000 for the year ended December 31, 20x1.
The movements in the number of ordinary shares are as follows:
P a g e | 32

1/1/20x1 Ordinary shares outstanding 120,000


3/1/20x1 Shares issued for cash 42,000
9/30/20x1 Subscribed shares 20,000
11/1/20x1 Reacquisition of treasury shares (12,000)
Outstanding shares at the end of period 170,000

What is the basic earnings per share?


a. 18.92
b. 17.09
c. 18.07
d. 16.98

Solution:

Date No. of sh. Months outstanding Weighted average


(a) (b) (c) = (a) x (b)
1/1/20x1 120,000 12/12 120,000
3/1/20x1 42,000 10/12 35,000
9/30/20x1 20,000 3/12 5,000
11/1/20x1 (12,000) 2/12 (2,000)
158,000

Profit (Loss) less Preferred dividends


Basic EPS =
Weighted average number of outstanding ordinary shares

2,800,000 – (100,000 x ₱10 x 10%)


Basic EPS =
158,000

Basic EPS = ₱17.09

94. Entity A had the following instruments outstanding all throughout 20x1:

12% convertible bonds payable issued at face amount, each


₱1,000 bond is convertible into 30 ordinary shares ₱2,000,000
Ordinary shares, ₱10 par, 100,000 shares issued and
outstanding 1,000,000

Profit for the year is ₱800,000. Entity A’s income tax rate is 30%.

What is the diluted earnings per share in 20x1?


a. 6.28
b. 6.05
c. 6.15
d. 5.98
P a g e | 33

Solution:

Profit (Loss) plus After tax interest expense on convertible bonds

Diluted EPS = Weighted average number of outstanding ordinary shares plus Incremental
shares arising from the assumed conversion or exercise of dilutive potential
ordinary shares

800,000 + (2,000,000 x 12% x 70%*)


Diluted EPS =
100,000 + [(2,000,000 ÷ 1,000) x 30]
*70% = 1 – 30% tax rate
Diluted EPS = (968,000 ÷ 160,000) = 6.05

95. PAS 33 is intended to apply to which of the following?


a. Publicly-listed entities
b. Non-publicly listed entities
c. Financial institutions
d. All entities using the PFRSs

96. Which of the following does not result to a retrospective adjustment of prior-period EPS
information?
a. share dividends c. issuance of shares for cash
b. share split d. issuance of stock rights

97. Earnings per share is not required to be computed on


a. profit or loss from continuing operations.
b. results of discontinued operations.
c. profit or loss for the year.
d. other comprehensive income.

98. Entity A is computing for its basic earnings per share and has gathered the following
information:
Loss for the year (1,000,000)
Preferred dividends 50,000
Outstanding ordinary shares 100,000

There have been no changes in the number of outstanding ordinary shares during the period. What
is the basic earnings (loss) per share?
a. -10.50 c. -9.50
b. 10.50 d. 9.50

Solution: [(-1M) – 50K] ÷ 100,000 = -1,050,000 ÷ 100,000 = -10.5

99. Entity A had 200,000 ordinary shares outstanding all throughout 20x1. In 20x2, the following
share issuances occurred:
• On April 1, 20,000 shares were issued for cash.
• On September 30, a 10% bonus issue (share dividend) was declared.
P a g e | 34

• On November 1, a 2-for-1 share split was issued.

Entity A had the following profits: ₱2,200,000 in 20x2 and ₱1,800,000 in 20x1. What are the earnings
per share to be disclosed in Entity A’s 20x2 comparative financial statements?
20x2 20x1
a. 4.22 4.02
b. 4.37 4.07
c. 4.65 4.09
d. 4.78 4.12

Solution:
 The weighted average number of ordinary shares outstanding are adjusted retrospectively as
follows:

20x1 20x2
1/1 (200,000 x 110% x 2) 440,000 (200,000 x 110% x 2 x 12/12) 440,000
4/1 (20,000 x 110% x 2 x 9/12) 33,000
9/30 -
11/1 -
Weighted average 440,000 473,000

20x2 20x1
Profit after tax 2,200,000 1,800,000
Adjusted weighted ave. no. of outstanding sh. 473,000 440,000
Basic EPS 4.65 4.09

100. Entity A has 200,000 ordinary shares outstanding on January 1, 20x1. Entity A offers rights
issue to its existing shareholders that enable them to acquire 1 ordinary share at a subscription
price of ₱120 for every 5 rights held. The rights are exercised on May 1, 20x1. The market price of
one ordinary share immediately before exercise is ₱180. Entity A reported profit after tax of
₱2,900,000 in 20x1. What is the basic earnings per share in 20x1?
a. 12.58
b. 12.67
c. 11.92
d. 17.67

Solution:
Aggregate mkt. value of shares before exercise of rts.
(200,000 sh. x ₱180) 36,000,000
Add: Proceeds from exercise of rts. [(200,000 rts. ÷ 5) x ₱120] 4,800,000
Total 40,800,000
Divide by: Outstanding shares after exercise of rts.
[200,000 sh. before exercise + (200,000 rts. ÷ 5 rts. per sh.)] 240,000
Theoretical ex-rights fair value per share 170
P a g e | 35

Fair value of stocks immediately before the exercise of rights


Adjustment factor =
Theoretical ex-rights fair value per share

 The adjustment factor is 180/170.

Jan. 1: (200,000 x 180/170 x 4/12) 70,588


May. 1: (240,000 x 8/12) 160,000
Weighted average no. of outstanding ordinary shares 230,588

Profit for the year 2,900,000


Divide by: Weighted average no. of outstanding sh. 230,588
Basic earnings per share 12.58

“Do not be deceived: God cannot be mocked. A man reaps what he sows.”
- Galatians 6:7

- END -
Page |1

Chapter 1
Statement of Financial Position

1. Which of the following statements is correct?


a. PAS 1 Presentation of Financial Statements prescribes the basis for presentation of general and
special purpose financial statements to improve both inter-comparability and intra-
comparability.
b. Intra-comparability is also referred to as horizontal comparability while inter-comparability
is also referred to as vertical comparability.
c. Working capital is the net amount of a company’s relatively liquid resources. It is the excess
of total assets over total liabilities.
d. Equity is the residual interest in the net assets of an entity.

2. According to PAS 1, these are financial statements intended to serve the needs of users who do
not have the authority to demand financial reports tailored for their own needs.
a. General purpose financial statements
b. Common purpose financial statements
c. Regular financial statements
d. All-purpose financial statements

3. The assessment of an entity’s going concern shall cover a minimum period of


a. one year c. three years
b. three months d. any of these

4. In which of the following instances would a liability that would otherwise be presented as
current is presented as noncurrent?
a. The liability is payable on demand but the entity estimates that it is probable that the lender
will not demand payment within 12 months after the reporting period.
b. The liability is payable on demand but the lender promises the entity after the reporting
period that the lender will not demand payment in the next 12 months.
c. The entity enters into a refinancing agreement after the reporting period but before the
financial statements are authorized for issue.
d. The entity enters into a refinancing agreement and the refinancing agreement is completed
by the balance sheet date.

5. In a classified balance sheet, deferred tax assets/liabilities are presented as


a. non-current items if the deferred taxes are not expected to reverse within 12 months after the
reporting period
b. noncurrent items
c. current items
d. a or c

6. General purpose financial statements are those statements that cater to the
a. common and specific needs of a wide range of external and internal users.
b. common needs of a wide range of external and internal users.
c. common needs of a wide range of external users.
Page |2

d. specific needs of a wide range of external users.

7. In virtually all circumstances, a fair presentation is achieved by compliance with applicable


IFRSs. A fair presentation also requires an entity: (choose the incorrect statement)
a. to select and apply accounting policies in accordance with PAS 8 Accounting Policies,
Changes in Accounting Estimates and Errors. PAS 8 sets out a hierarchy of authoritative
guidance that management considers in the absence of a Standard or an Interpretation that
specifically applies to an item.
b. to present information, including accounting policies, in a manner that provides relevant,
reliable, comparable and understandable information.
c. to provide additional disclosures when compliance with the specific requirements in PFRSs
is insufficient to enable users to understand the impact of particular transactions, other
events and conditions on the entity’s financial position and financial performance.
d. to establish a system of internal control the responsibility for which is the entity’s
management. Furthermore, the entities financial statements should be audited by an
independent external party at least annually.

8. Each component of the financial statements shall be identified clearly. In addition, the following
information shall be displayed prominently, and repeated when it is necessary for a proper
understanding of the information presented:
I. The name of the reporting entity or other means of identification, and any change in that
information from the preceding balance sheet date;
II. Whether the financial statements cover the individual entity or a group of entities;
III. The balance sheet date or the period covered by the financial statements, whichever is
appropriate to that component of the financial statements;
IV. The presentation currency, as defined in PAS 21 The Effects of Changes in Foreign Exchange Rates
V. The level of rounding used in presenting amounts in the financial statements.

a. I, II, III c. I, II, IV, V


b. I, II, III, IV d. I, II, III, IV, V

9. When an entity’s balance sheet date changes and the annual financial statements are presented
for a period longer or shorter than one year, an entity shall disclose, in addition to the period
covered by the financial statements:
I. The reason for using a longer or shorter period
II. The fact that comparative amounts for the income statement, statement of changes in equity,
cash flow statement and related notes are not entirely comparable
III. The amounts charged to the beginning balance of the retained earnings, net of tax
IV. Pro-forma financial statements, as a supplemental information in the notes

a. I, II c. I, III, IV
b. I, III d. I, II, III, IV

10. All of the following statements are correct, except


a. The operating cycle of an entity is the time between the acquisition of assets for processing
and their realization in cash or cash equivalents.
b. When the entity’s normal operating cycle is not clearly identifiable, its duration is presumed
to be twelve months.
Page |3

c. Current assets include assets (such as inventories and trade receivables) that are sold,
consumed or realized as part of the normal operating cycle even when they are not expected
to be realized within twelve months after the balance sheet date.
d. Some liabilities are part of the working capital used in the entity’s normal operating cycle.
Such operating items are classified as current liabilities even if they are due to be settled
more than twelve months after the balance sheet date.
e. When an entity presents current and non-current assets and current and non-current
liabilities as separate classifications on the face of its balance sheet, it shall classify deferred
tax assets (liabilities) as current assets (liabilities) if the deferred tax assets (liabilities) are
expected to reverse within twelve months after the end of reporting period.

“Love is patient, love is kind. It does not envy, it does not boast, it is not proud. It does
not dishonor others, it is not self-seeking, it is not easily angered, it keeps no record of
wrongs. Love does not delight in evil but rejoices with the truth. It always protects,
always trusts, always hopes, always perseveres.” – (1 Corinthians 13:4-7)

- END -
Page |4

1. Below are the account balances prepared by the bookkeeper for SQUELCH TO SILENCE
Company as of December 31, 20x1:
Assets Liabilities
Cash 30,000 Accounts payable 40,000
Accounts receivable, net 88,000 Notes payable 200,000
Inventory 80,000
Prepaid income tax 16,000
Prepaid assets 10,000
Investment in subsidiary 20,000
Land held for sale 56,000
Property, plant and equipment 100,000
Totals 400,000 240,000

Additional information:
- Cash consists of the following:
Petty cash fund (unreplenished petty cash expenses, ₱3,000) 4,000
Cash in bank (20,000)
Payroll fund 28,000
Tax fund 14,000
Cash to be contributed to a sinking fund set up for the retirement
of bonds maturing on December 31, 20x3 4,000
Total Cash 30,000

- Checks amounting to ₱61,000 were written to suppliers and recorded on December 30, 20x1,
resulting to a bank overdraft of ₱20,000. The checks were mailed on January 5, 20x2.

- Accounts receivable consists of the following:


Accounts receivable 80,000
Allowance for uncollectability ( 10,000)
Credit balance in customers’ accounts ( 6,000)
Selling price of unsold goods sent on consignment to QUELL, Inc.
at 120% of cost and excluded from SQUELCH’s inventory 24,000
Accounts receivable, net 88,000

- The inventory includes cost of goods amounting to ₱20,000 that are expected to be sold beyond
12 months but within the ordinary course of business. Also, the inventory includes cost of
consigned goods received on consignment from Alpha-Numerix Co. amounting to ₱10,000.
- Prepaid income tax represents excess of payments for quarterly corporate income taxes during
20x1 over the actual annual corporate income tax as of December 31, 20x1.
- Prepaid assets includes a ₱4,000 security deposit on an operating lease which is expected to
expire on March 31, 20x3. The security deposit will be received on lease expiration.
- The land qualified for classification as “asset held for sale” under PFRS 5 Non-current Assets Held
for Sale and Discontinued Operations as of December 31, 20x1.
Page |5

- Accounts payable is net of ₱12,000 debit balance in suppliers’ accounts. Accounts payable
includes the cost of goods held on consignment from Alpha-Numerix Co. which were included
in inventory.
- The notes payable are dated July 1, 20x1 and are due on July 1, 20x4. The notes payable bears an
annual interest rate of 10%. Interest is payable annually.

How much is the adjusted working capital?


a. 334,000
b. 289,000
c. 264,000
d. 215,000

D Solution:

 The adjusted cash is computed as follows:


Cash – unadjusted 30,000
Unreplenished petty cash expenses ( 3,000)
Unreleased checks recorded as disbursement
resulting to overdraft 61,000
Contribution to sinking fund ( 4,000)
Adjusted cash balance 84,000

 The adjusted accounts receivable is computed as follows:


Accounts receivable 80,000
Allowance for uncollectibility (10,000)
Adjusted accounts receivable, net 70,000

 The adjusted inventory is computed as follows:


Inventory* 80,000
Cost of unsold goods sent out on consignment
excluded from inventory (24,000 ÷ 120%) 20,000
Cost of goods held on consignment (10,000)
Adjusted inventory 90,000
*The cost of inventory expected to be sold beyond 12 months but within the normal operating cycle is properly included as part of
cost of inventories presented as current assets.

 The adjusted prepaid assets are computed as follows:


Prepaid assets 10,000
Security deposit (to be presented as noncurrent) (4,000)
Adjusted prepaid assets 6,000

 The adjusted accounts payable is computed as follows:


Accounts payable (40,000 + 12,000 debit balance) 52,000
Unreleased checks recorded as disbursement
resulting to overdraft 31,000
Cost of goods held on consignment ( 10,000)
Adjusted accounts payable, net 103,000

 Accrued interest on the notes payable is computed as follows:


(P200,000 x 10% x 6/12) 10,000
The current assets and current liabilities are computed as follows:
Current assets Current liabilities
Cash 84,000 Accounts payable 103,000
Accounts receivable, net 70,000 Advances from customers 6,000
Page |6

Advances to suppliers 12,000 Interest payable 10,000


Inventory 90,000
Prepaid income tax 16,000
Prepaid assets 6,000
Land held for sale 56,000
Total current assets 334,000 Total current liabilities 119,000

The adjusted working capital is computed as follows:


Working capital = Current assets – Current liabilities
Working capital = P 334,000 – P 119,000
Working capital = P 215,000

2. The ledger of INFIRM SICK Co. as of December 31, 20x1 includes the following:

10% Note payable 80,000


12% Note payable 120,000
14% Mortgage note payable 60,000
Interest payable -

Additional information:
- INFIRM Co.’s financial statements were authorized for issue on April 15, 20x2.
- The 10% note payable is due on July 1, 20x2 and pays semi-annual interest every July 1 and
December 31. On January 28, 20x2, INFIRM Co. entered into a refinancing agreement with a bank
to refinance the entire note by issuing a long-term obligation.
- The 12% note payable is due on March 31, 20x2 and pays annual interest every March 31. On
January 31, 20x2, INFIRM Co. extended the maturity of the note to March 31, 20x3 under the
existing loan agreement. The extension of maturity date is at the option of INFIRM.
- The 14% mortgage note is due on December 31, 20x9. Per agreement with the creditor, INFIRM is to
pay quarterly interests on the note, failure to do so will render the note payable on demand.
INFIRM failed to pay the 3rd and 4th quarterly interests on the note during 20x1.

How much is the total current liabilities?


a. 119,000
b. 155,000
c. 172,000
d. 189,000

10% Note payable 80,000


Interest payable on the 12% note (120,000 x 12% x 9/12) 10,800
14% Mortgage note payable 60,000
Interest payable on the 14% note (60,000 x 14% x 6/12) 4,200
Current liabilities 155,000
Page |7

Use the following information for the next three questions:


The ledger of COLTISH UNDISCIPLINED Co. in 20x1 includes the following:
Jan. 1, 20x1 Dec. 31, 20x1
Current assets 1,200,000 ?
Noncurrent assets 4,000,000 ?
Current liabilities 900,000 1,000,000
Noncurrent liabilities ? 3,000,000

Additional information:
- COLTISH’s working capital as of December 31, 20x1 is twice as much as the working capital as
of January 1, 20x1.
- Total equity as of January 1, 20x1 is ₱1,700,000. Profit for the year is ₱2,400,000 while dividends
declared amounted to ₱1,000,000. There were no other changes in equity during the year.

3. How much is the total noncurrent liabilities as of January 1, 20x1?


a. 2,600,000
b. 2,800,000
c. 3,200,000
d. 3,400,000

Assets = Liabilities + Equity


(1,200,000 + 4,000,000) = (900,000 + Noncurrent liabilities) + 1,700,000
Noncurrent liabilities = 5,200,000 – 900,000 – 1,700,000
Noncurrent liabilities, Jan. 1, 20x1 = 2,600,000

4. How much is the total current assets as of December 31, 20x1?


a. 1,600,000
b. 800,000
c. 300,000
d. 2,200,000

Working capital = Current assets – Current liabilities


Working capital, Jan. 1, 20x1 = 1,200,000 – 900,000
Working capital, Jan. 1, 20x1 = 300,000

Working capital, Dec. 31, 20x1 = Working capital, Jan. 1, 20x1 times 2
Working capital, Dec. 31, 20x1 = 300,000 x 2 = 600,000
Working capital = Current assets – Current liabilities
600,000 = Current assets, Dec. 31, 20x1 – 1,000,000
Current assets, Dec. 31, 20x1 = 1,600,000

5. How much is the total noncurrent assets as of December 31, 20x1?


a. 4,500,000
b. 6,500,000
c. 5,800,000
d. 5,500,000
Equity
1,700,000 Jan. 1
Dividends 1,000,000 2,400,000 Profit for the year
Page |8

Dec. 31 3,100,000

Assets = Liabilities + Equity


(1,600,000 + Noncurrent assets) = (1,000,000 + 3,000,000) + 3,100,000
Noncurrent assets, Dec. 31, 20x1 = 4,000,000 + 3,100,000 – 1,600,000
Noncurrent assets, Dec. 31, 20x1 = 5,500,000

6. HARANGUE INFLATED SPEECH Co. had the following information for 20x1:
Accounts receivable turnover 10:1
Total assets turnover 2:1
Average receivables during the year ₱400,000
Total assets, January 1, 20x1 800,000

How much is the total assets as of December 31, 20x1?


a. 4,000,000
b. 3,800,000
c. 3,200,000
d. 2,800,000

C Solution:

Sales are computed as follows:


Net credit sales
Accounts receivable turnover =
Average accounts receivable

Net credit sales


10 =
400,000
Net credit sales = 4,000,000

Net credit sales


Total assets turnover =
Average total assets
Where:
Total assets, beg. + Total assets, end
Average total assets =
2
Net credit sales
Total assets turnover =
Average total assets
4,000,000
2 =
Average total assets
Average total assets = 4,000,000
2
Average total assets = 2,000,000

Total assets, Jan. 1 + Total assets, Dec. 31


Average total assets =
2
800,000 + Total assets, Dec. 31
2,000,000 =
2
Total assets, Dec. 31 = (2,000,000 x 2) - 800,000
Total assets, Dec. 31 = 3,200,000
Page |9

“A wise man will hear and increase learning, and a man of understanding will attain
wise counsel.” (Proverbs 1:5)

- END -
Page |1

Chapter 2
Statement of Comprehensive Income

1. The Statement of comprehensive income shows information on an entity’s financial position


during the period.

2. Revenue includes both income and gains.

3. The revenue earned by a merchandising business from its sales of goods is commonly referred to
as sales.

4. If income is greater than expenses, the difference is loss.

5. A statement of comprehensive income that shows expenses by their function is said to have been
prepared using the multi-step approach.

6. The revenue earned by a service business from rendering services is commonly referred to as
service fees.

7. Expenses decrease equity.

8. If total income is ₱100 while total expenses is ₱80, loss is ₱20.

9. The Statement of profit or loss and other comprehensive income is the same with the Income
statement.

10. A statement of comprehensive income that presents separately an entity’s cost of sales is said to
have been prepared using the single-step method.
Page |2

ANSWERS:
1. FALSE – financial performance
2. FALSE – income includes both revenue and gains
3. TRUE
4. FALSE
5. TRUE
6. TRUE
7. TRUE
8. FALSE – profit of ₱20
9. FALSE - different
10. FALSE – multi-step
Page |3

NAME: Date:
Professor: Section: Score:

: TRUE OR FALSE
1. The Statement of profit or loss and other comprehensive income is different from the income
statement.

2. The Statement of comprehensive income shows profit or loss only.

3. An entity can present an income statement alone in lieu of the statement of comprehensive
income.

4. The elements of the statement of comprehensive income are income and expenses.

5. Income increases equity.

6. The residual category of expenses under the function of expense method is “Distribution costs.”

7. Losses incurred on sales of noncurrent assets are presented under “Administrative expenses.”

8. Distribution costs (or Selling expenses) are costs attributable to selling activities.

9. The categories of expenses under the function of expense method include “Cost of sales,”
“Distribution costs,” and “Administrative expenses” only.

10. If profit is ₱100 while other comprehensive income is ₱80, total comprehensive income is ₱20.

“A man who asks is a fool for five minutes. A man who never asks is a fool for
life.”
- Chinese Proverb

- END –
Page |4

ANSWERS
1. TRUE
2. FALSE – profit or loss and other comprehensive income
3. FALSE – a statement showing other comprehensive income must also be presented
4. TRUE
5. TRUE
6. FALSE – Administrative expenses
7. FALSE – losses are included in the “Other expenses” category. If material, losses are presented
separately.
8. TRUE
9. FALSE – finance costs, income tax expense, and if applicable, other expenses are also categories
of expenses under the function of expense method
10. FALSE - ₱180 (100 + 80)
Page |5

Use the following information for the next two questions:


The following items were presented for the purpose of determining comprehensive income.

Profit for the year 2,000


Increase in revaluation surplus 1,000
Remeasurements of the net defined benefit liability (asset) - loss (200)
Net change in translation of foreign operation (400)
Dividends declared (100)
Stock rights 300

1. How much is the other comprehensive income?


a. 400
b. 600
c. 800
d. 2,000

Profit for the year 2,000

Other comprehensive income:

Revaluation gain 1,000


Remeasurements of the net defined benefit
liability (asset) (200)
Translation loss on foreign operation (400)
Total other comprehensive income (a) 400

Total comprehensive income (b) 2,400

2. How much is the total comprehensive income?


a. 1,800
b. 2,200
c. 2,400
d. 2,800
See solution above

Use the following information for the next two questions:


The records of Afternoon Sun Co. show the following information:

Interest expense ₱24,000


Cost of inventories sold 600,000
Insurance expense 100,000
Advertising expense 20,000
Freight-out 10,000
Freight-in 4,000
Loss on sale of equipment 2,000
Legal and other professional fees 12,000
Page |6

Rent expense (one-half occupied by sales department) 8,000


Sales commission expense 14,000
Doubtful accounts expense 16,000

3. How much is the total distribution (selling) costs?


a. 48,000
b. 56,000
c. 64,000
d. 108,000

Selling expenses Administrative expenses


Advertising expense P20K Insurance expense P100K
Freight-out 10 Legal and other professional fees 12
Rent expense (one half) 4 Rent expense (one half) 4
Sales commission expense 14 Doubtful accounts expense 16
Total selling expenses P48K Total administrative expenses P132K

4. How much is the total administrative expenses?


a. 24,000
b. 132,000
c. 226,000
d. 668,000
See solution above

5. The records of SOIREE EVENING PARTY Co. showed the following information:

Increase in accounts receivable 100,000


Collections on accounts 800,000
Cash sales 120,000
Increase in inventory 40,000
Freight-in 14,000
Freight-out 13,000
Decrease in accounts payable 60,000
Disbursements for purchases 480,000
Purchase discounts 4,000

How much is the gross profit for the year?


a. 662,000
b. 656,000
c. 648,000
d. 626,000
D

Sales on account are computed as follows:


Accounts receivable
A/R, beg. -
Sales on account (squeeze) 900,000 800,000 Collections on accounts
Page |7

100,000 A/R, end

Cost of sales is computed as follows:


Accounts payable
Purchase discounts 4,000 60,000 A/P, beg
Gross purchases
Disbursements for purchases 480,000 424,000 (squeeze)
A/P, end -

Inventory
Inventory, beg. -
Gross purchases 424,000 4,000 Purchase discounts
Freight in 14,000 394,000 Cost of sales (squeeze)
40,000 Inventory, end

Gross profit is computed as follows:


Cash sales 120,000
Credit sales 900,000
Total sales 1,020,000
Cost of sales (394,000)
Gross profit 626,000

6. The records of BRACKISH SALTY Co. showed the following information:


Accounts receivable, net, Jan. 1, 20x1 40,000
Accounts receivable, net, Dec. 31, 20x1 160,000
Accounts receivable turnover 4:1
Inventory, Jan. 1, 20x1 120,000
Inventory, Dec. 31, 20x1 60,000
Inventory turnover 3:1

How much is the gross profit for the year?


a. 120,000
b. 130,000
c. 132,000
d. 146,000

Simple Solution: ((40k+60k) /2 x 4)) – ((120k+60k) / 2 x 3 = 130,000

B Solution:

Net credit sales


Accounts receivable turnover =
Average accounts receivable

Where:
A/R, beg. + A/R, end.
Average accounts receivable =
2

Average accounts receivable = 40,000 + 160,000


Page |8

2
Average accounts receivable = 100,000

Net credit sales


Accounts receivable turnover =
Average accounts receivable

Net credit sales


4 =
100,000
Net credit sales = 400,000

Cost of sales
Inventory turnover =
Average inventory
Where:
Inventory, beg. + Inventory, end.
Average inventory =
2
Using the formulas given above, cost of sales is computed as follows:
120,000 + 60,000
Average inventory =
2
Average inventory = 90,000

Cost of sales
Inventory turnover =
Average inventory

Cost of sales
3 =
90,000
Cost of sales = 270,000

Gross profit is computed as follows:


Net credit sales 400,000
Cost of sales (270,000)
Gross profit 130,000
Page |9

7. The records of SURLY BAD TEMPERED Co. showed the following information:
Decrease in accounts payable 60,000
Disbursements for purchases 440,000
Increase in raw materials 100,000
Direct labor is 50% of raw materials used in production
Manufacturing overhead is 20% of prime costs
Increase in work-in-process inventory 40,000
Decrease in finished goods inventory 50,000

How much is the cost of goods sold?


a. 380,000
b. 464,000
c. 514,000
d. 546,000

Simple Solution:
440,000 - 60,000 - 100,000=280k
280,000 * 50%= 140k Accounts payable
420,000 * 20%= 84k

280,000 + 140,000 + 84,000


+ 50,000 – 40,000 = 514,000 60,000 A/P, beg.
Disbursements for purchases 440,000 380,000 Purchases (squeeze)
A/P, end -

Raw materials
inventory
RM Invty, beg. - Raw materials used in
Purchases 380,000 280,000 production (squeeze)
100,000 RM Invty, end.

Work-in-process inventory
WIP, beg. -
Cost of goods
RM used in production 280,000 manufactured
Direct labor (50% of RM) 140,000 464,000 (squeeze)
Production overhead* 84,000
40,000 WIP, end.

Total goods put into process 504,000 504,000

*Prime cost = Direct materials + Direct labor


Prime cost = 280,000 + 140,000 = 420,000
Production overhead = 20% x 420,000 = 84,000

Finished goods inventory


FG, beg. 50,000
Cost of goods sold
Cost of goods manufactured 464,000 514,000 (squeeze)
P a g e | 10

- FG, end

Total goods avail. for sale 514,000 514,000

8. PRENTICE A LEARNER Co. reported profit after tax of ₱210,000. PRENTICE’s income tax rate is
30%. Operating expenses for the year is 15% of sales and 25% of cost of sales. Other expenses were
10% of sales. How much is the total sales?
a. 1,800,000
b. 2,000,000
c. 2,200,000
d. 2,240,000

Simple Solution: (210,000 / (100-30%)) / 15 % = 2,000,000

Sales 100%
Cost of sales (15% / 25%) (60%)
Gross profit 40%
Operating expenses (15% of 100%) or (25% of 60%) (15%)
Other expenses (10% of 100%) (10%)
Profit before tax 15%

The profit after tax given in the problem is translated to profit before tax as shown below:
Profit after tax (given) 210,000
Divide by: (100% less 30% tax rate) 70%
Profit before tax 300,000

Sales (300,000 Profit before tax ÷ 15%) 2,000,000

9. The records of HACK TO CHOP Co. on December 31, 20x1 showed the following information:
Sales 2,000,000
Sales discounts 20,000
Cost of sales 800,000
Distribution costs 96,000
Administrative costs 240,000
Casualty loss on typhoon 40,000
Dividends received from investments in FVPL 24,000
Dividends received from investment in associate 48,000
Share in the profit of an associate 72,000
Dividends declared and paid 28,000
Interest expense 44,000
Unrealized gain on investments in FVPL 30,000
Unrealized gain on investments in FVOCI 38,000
Income tax expense 300,000
Loss on revaluation 26,000
Remeasurements of the net defined benefit liability (asset) - gain 22,000
Correction of understatement in depreciation in prior year 32,000
Translation adjustment of foreign operation - loss 8,000
P a g e | 11

How much is the profit for the year?


a. 886,000
b. 586,000
c. 612,000
d. 626,000

B Solution:

HACK TO CHOP Co.


Statement of profit or loss and other comprehensive income
For the year ended December 31, 20x1

Sales 2,000,000
Sales discounts (20,000)
Net sales 1,980,000
Cost of sales (800,000)
Gross profit 1,180,000
Distribution costs (96,000)
Administrative costs (240,000)
Dividends received from investments in FVPL 24,000
Share in the profit of an associate 72,000
Unrealized gain on investments in FVPL 30,000
Casualty loss on typhoon (40,000)
Interest expense (44,000)
Profit before tax 886,000
Income tax expense (300,000)
Profit for the year 586,000
Other comprehensive income:
Items that will not be reclassified subsequently to profit or loss:
Loss on revaluation (26,000)
Unrealized gain on investments in FVOCI 38,000
Remeasurements of defined benefit pension plans 22,000
34,000
Items that may be reclassified subsequently to profit or loss:
Loss on translation of foreign operation (8,000)
Other comprehensive income for the year 26,000
TOTAL COMPREHENSIVE INCOME FOR THE YEAR 612,000

10. WASHY PALE Co. has the following information on December 31, 20x1:
- Cost of sales is ₱260,000.
- Operating expenses are 13% of sales and 20% of cost of sales.
- Interest expense is 5% of sales.
- Income tax rate is 30%. There were no temporary differences during the year.

How much is the profit for the year?


a. 65,000
b. 140,000
c. 38,600
P a g e | 12

d. 47,600

D Solution:

Cost ratio is derived from the percentages of operating expenses over sales and cost of sales as follows:
Cost ratio = 13% / 20% = 65%

Amount
Sales 400,000 (260,000 COS ÷ 65%)
Cost of sales (260,000) (start)
Gross profit 140,000
Operating expenses (52,000) (400,000 x 13%) or (260,000 x 20%)
Interest expense (20,000) (400,000 x 5%)
Profit before tax 68,000
Income tax expense (20,400) (40,000 x 30%)
Profit after tax 47,600

“Do not be wise in your own eyes; fear the LORD and shun evil. “
(Proverbs 3:7)

- END –
Page |1

Chapter 3
Revenue from Contracts with Customers

1. ABC Co., a seller of concrete aggregates, enters into the following contracts:
i. A contract with Delta Co. to deliver goods. Payment is due one month after delivery.
ii. A contract with Echo Co. for the sale of 300 units of each of Products X and Y. The contract
states that the price of Product Y will be retrospectively reduced by 50% if Echo Co. makes a
cumulative purchase of at least 1,000 units of Product X within 6 months.
iii. A contract with Fafa Co. to deliver goods. At contract inception, Fafa Co. is broke. ABC Co.
expects that it can only collect 50% of the consideration.
iv. A contract with Gamma Co., an entity which is also engaged in the concrete aggregates
business, to exchange inventory to facilitate sales to customers in different geographical
areas of operations.

Identify the contracts to which PFRS 15 Revenue from Contract with Customers may not be applied.
a. Delta and Echo c. Fafa
b. Fafa and Gamma d. Gamma

2. Certain criteria must be met before a contract with a customer is accounted for under PFRS 15.
Which of the following precludes a contract from being accounted for under PFRS 15?
a. The consideration is collected in advanced
b. The contract is made orally
c. The contract does not result to a change in the risk, timing or amount of the entity’s future
cash flows.
d. The contract is neither oral nor written but rather implied by the entity’s business practices.

3. ABC Co. enters into a contract with XYZ, Inc. to deliver 2 apples, 3 mangoes, and 5 potatoes for a
total consideration of ₱100. In accounting for the contract, which of the following is probably not
true?
a. ABC Co. identifies three performance obligations in the contract.
b. ABC Co. allocates the ₱100 transaction price over the promises to deliver the apples,
mangoes and potatoes on the basis of relative stand-alone selling prices of those goods.
c. The allocation of the transaction price may result to the identification of a discount.
d. No revenue is recognized until all of the 2 apples, 3 mangoes and 5 potatoes are delivered
even though the 2 apples were delivered first before the mangoes and potatoes.

4. ABC Co., a manufacturer and dealer of printing machines, had the following transactions during
the period:
I. ABC Co. receives an order for the manufacture of a customized machine for a customer. The
customer pays half of the consideration at contract inception. The manufacturing lead time is
1 year. ABC Co. subcontracts a portion of the manufacturing to XYZ, Inc., another
manufacturer.
II. ABC Co. receives an order for a standard machine. Payment is due only after ABC Co. has
delivered and installed the machine. Additionally, the contract requires ABC Co. to perform
free maintenance services over a 3-month period after the machine is installed. ABC Co.
Page |2

completes the delivery and installation by the end of the reporting period; however, the
maintenance period is not yet over.
III. ABC Co. receives an order for 2 machines. The first machine is delivered at contract
inception but the second machine will be delivered after two months. Payment is due only
after both machines are delivered. By the end of the reporting period, the second machine is
not yet delivered and the consideration is not yet collected.

Identify the contracts to which PFRS 15 Revenue from Contract with Customers may be applied.
a. Contract 1 c. Contracts 1, 2 and 3
b. Contract 3 d. None of these

5. It is an agreement between two or more parties that creates enforceable rights and obligations.
a. obligation c. revenue
b. contract d. any of these

6. According to PFRS 15 Revenue from Contracts with Customers contracts with customers are
a. written c. implied
b. oral d. any of these

7. The best evidence for the stand-alone selling price of a good or service is
a. the list price of the good or service
b. the contractually stated price of the good or service
c. the observable price at which the good or service can be sold separately under similar
circumstances and to similar customers
d. the entity’s estimate of the stand-alone selling price

8. Revenue is recognized when (or as) the entity satisfies a performance obligation. According to
PFRS 15 Revenue from Contracts with Customers, revenue is measured at
a. the fair value of the consideration received or receivable
b. the transaction price
c. the stand-alone selling price of the good or services transferred
d. the amount of the transaction price allocated to the performance obligation satisfied.

9. During the period ABC Co. transfers goods to XYZ, Inc. Which of the following does not
indicate that the transaction is a consignment arrangement?
a. ABC Co. retains legal title over the goods until XYZ, Inc. sells them to third parties.
b. ABC Co. can require the return of any unsold goods within 60 days.
c. If XYZ, Inc. is not satisfied with the goods, XYZ, Inc. has the right to return them to ABC Co.
d. ABC Co. can require XYZ, Inc. to transfer the goods to 123 Corporation.
e. XYZ, Inc. is not obligated to remit any payment to ABC Co. unless XYZ, Inc. sells the goods

10. A non-refundable upfront fee that relates to administrative tasks to set up a contract is most
likely accounted for as
a. a prepayment and recognized as revenue only when the related goods or services are
transferred to the customer.
b. a contract asset that is presented separately from contract liability in the statement of
financial position
Page |3

c. as an outright expense
d. all of these

“As for everyone who comes to me and hears my words and puts them into
practice, I will show you what they are like. They are like a man building a house,
who dug down deep and laid the foundation on rock. When a flood came, the
torrent struck that house but could not shake it, because it was well built. But the
one who hears my words and does not put them into practice is like a man who
built a house on the ground without a foundation. The moment the torrent struck
that house, it collapsed and its destruction was complete.” (Luke 6:47-49)

- END -
Page |4

ANSWERS:
1. D 6. D
2. C 7. C
3. D 8. D
4. C 9. C
5. B 10. A
Page |5

1. On 1 July 20X7, The Pyretus Company, a manufacturer of office furniture, supplied goods to The
Natiso Company for ₱120,000 on condition that this amount was paid in full on 1 July 20X8.
Natiso had earlier rejected an alternative offer from Pyretus whereby they could have bought the
same goods by paying cash of ₱108,000 on 1 July 20X7. Under PFRS 15, how much relating to
this transaction should Pyretus recognize in profit or loss in respect of revenue and interest
income for the year ended 30 June 20X8?
Revenue Interest income
a. 108,000 12,000
b. 120,000 Nil
c. 108,000 Nil
d. 120,000 12,000
(Adapted)

Revenue = 108,000 cash selling price; Interest income = (120,000 – 108,000) = 12,000

2. On 1 July 20X7 The Otakamiro Company handed over to a client a new computer system. The
contract price for the supply of the system and after sales support for 12 months was ₱800,000.
Otakamiro estimates the cost of the after-sales support at ₱120,000 and it normally marks up
such costs by 50% when tendering for support contracts. Under PFRS 15, the revenue Otakamiro
should recognize in its financial year ended 31 December 20X7 is
a. 620,000 b. 800,000 c. 710,000 d. Nil
(Adapted)

Satisfaction of
Performance performance
obligations Allocation method Allocation of transaction price obligation Revenue
Expected cost plus a
After-sales support margin (120,000 x 150%) = 180,000 50%a 90,000

Computer system Residual (800,000 - 180,000) = 620,000 100% 620,000


710,000
a (6 mos. over 12 mos.)

3. On October 1, 20x3, Acme Fuel Co. sold 100,000 gallons of heating oil to Karn Co. at ₱3 per
gallon. Fifty thousand gallons were delivered on December 15, 20x3, and the remaining 50,000
gallons were delivered on January 15, 20x4. Payment terms were: 50% due on October 1, 20x3,
25% due on first delivery, and the remaining 25% due on second delivery. What amount of
revenue should Acme recognize from this sale during 20x3?
a. 75,000 b. 150,000 c. 225,000 d. 300,000
(AICPA)

(50,000 gallons delivered in 20x3 x ₱3 per gallon) = 150,000

4. In 20x2, Super Comics Corp. sold a comic strip to Fantasy, Inc. and will receive royalties of 20%
of future revenues associated with the comic strip. At December 31, 20x3, Super reported
royalties receivable of ₱75,000 from Fantasy. During 20x4, Super received royalty payments of
₱200,000. Fantasy reported revenues of ₱1,500,000 in 20x4 from the comic strip. In its 20x4
income statement, what amount should Super report as royalty revenue?
a. 125,000 b. 175,000 c. 200,000 d. 300,000
(AICPA)
Page |6

(1,500,000 x 20%) = 300,000

5. Lin Co., a distributor of machinery, bought a machine from the manufacturer in November 20x3
for ₱10,000. On December 30, 20x3, Lin sold this machine to Zee Hardware for ₱15,000, under
the following terms: 2% discount if paid within thirty days, 1% discount if paid after thirty days
but within sixty days, or payable in full within ninety days if not paid within the discount
periods. However, Zee had the right to return this machine to Lin if Zee was unable to resell the
machine before expiration of the ninety-day payment period, in which case Zee’s obligation to
Lin would be canceled. In Lin’s net sales for the year ended December 31, 20x3, how much
should be included for the sale of this machine to Zee?
a. 0 b. 14,700 c. 14,850 d. 15,000
(AICPA)

No revenue is recognized because the control over the machine is not transferred.

Use the following information for the next two questions:


DISCALCED BAREFOOTED NAKASAKASAKA Supermarket, Inc. awards customers loyalty
points for their purchases. A customer is entitled to one point for every ₱400 purchase. The points
accumulated may be redeemed for awards in the form of appliances, electronics, groceries and other
household items. DISCALCED estimates the stand-alone selling price of each point at ₱4.00. During
the period, DISCALCED made total sales of ₱40M to cardholders.

6. How much sales revenue is recognized?


a. 400,000 b. 40,000,000 c. 39,600,000 d. 0

₱40M – [(₱40M ÷ 400) x ₱4)] = 39,600,000

7. How much is the deferred revenue from loyalty points?


a. 400,000 b. 40,000,000 c. 39,600,000 d. 0

[(₱40M ÷ ₱400) x ₱4] = 400,000

8. Wren Corp.’s trademark was licensed to Mont Co. for royalties of 15% of sales of the
trademarked items. Royalties are payable semiannually on March 15 for sales in July through
December of the prior year, and on September 15 for sales in January through June of the same
year. Wren received the following royalties from Mont:
March 15 September 15
20x2 10,000 15,000
20x3 12,000 17,000

Mont estimated that sales of the trademarked items would total ₱60,000 for July through December
20x3. In Wren’s 20x3 income statement, the royalty revenue should be
a. 26,000 b. 29,000 c. 38,000 d. 41,000
(AICPA)

Royalty revenue for Jan. to June, 20x3


17,000
(received on Sept. 20x3)
Royalty revenue for July to Dec., 20x3 (60,000 x 15%) 9,000
Total royalty revenue 26,000
Page |7

9. Rill Co. owns a 20% royalty interest in an oil well. Rill receives royalty payments on January 31
for the oil sold between the previous June 1 and November 30, and on July 31 for oil sold
between December 1 and May 31. Production reports show the following oil sales:

June 1, 20x2 - November 30, 20x2 300,000


December 1, 20x2 - December 31, 20x2 50,000
December 1, 20x2 - May 31, 20x3 400,000
June 1, 20x3 - November 30, 20x3 325,000
December 1, 20x3 - December 31, 20x3 70,000

What amount should Rill report as royalty revenue for 20x3?


a. 140,000 b. 144,000 c. 149,000 d. 159,000
(AICPA)

Solution:
December 1, 20x2 - May 31, 20x3 400,000
December 1, 20x2 - December 31, 20x2 (50,000)
June 1, 20x3 - November 30, 20x3 325,000
December 1, 20x3 - December 31, 20x3 70,000
Oil sales in 20x3 745,000
Multiply by: 20%
Royalty revenue in 20x3 149,000

“Call upon me in the day of trouble; I will deliver you, and you shall glorify me.”
(Psalms 50:15)

- END –
Page |1

Chapter 4
Non-current Assets Held for Sale and Discontinued
Operations

1. If a noncurrent asset is sold after the end of the reporting period but before the financial
statements are authorized for issue, that asset is classified as current asset in the statement of
financial position.

2. PFRS 5 Non-current Assets Held for Sale and Discontinued Operations applies only to noncurrent
assets. Current assets are outside the scope of this standard.

3. According to PFRS 5, a non-current asset (or disposal group) is classified as held for sale if its
carrying amount will be recovered principally through continuing use rather than through a sale
transaction.

4. For purposes of applying the provisions of PFRS 5, an exchange transaction with commercial
substance may be treated as a sale.

5. Noncurrent assets classified as held for sale in accordance with PFRS 5 are measured at fair
value less costs to sell.

6. The assets and liabilities of a disposal group shall be offset and the net amount is presented as
either asset or liability.

7. Investment properties measured under the fair value model are within the scope of PFRS 5.

8. Property, plant and equipment measured under the revaluation model are within the scope of
PFRS 5.

9. “Highly probable” means more likely than not.

10. Meeting the criteria for held for sale classification after the balance sheet date but before the
financial statements are authorized for issue is a non-adjusting event after the reporting period.
Page |2

ANSWERS
1. FALSE 6. FALSE
2. TRUE 7. FALSE
3. FALSE 8. TRUE
4. TRUE 9. FALSE
5. FALSE 10. TRUE
Page |3

1. Non-current assets are presented as current items in the statement of financial position
a. only when they are expected to be sold within 12 months from the end of reporting period.
b. only if they are actually sold after the reporting period but before the date of authorization of
the financial statements for issue.
c. only when they qualify as held for sale assets under PFRS 5.
d. never presented as current items.

2. A noncurrent asset classified as held for sale in accordance with PFRS 5 has not been sold after a
year. The asset shall continue to be presented as held for sale under PFRS 5 if
a. the delay is due to events beyond the entity’s control
b. the entity remains committed to its plan to sell the asset
c. the noncurrent asset is actually sold after the reporting period but before the financial
statements were authorized for issue.
d. a and b

3. Which of the following statements is true regarding the accounting treatment of costs to sell
under PFRS 5?
a. Costs to sell are added to the fair value when determining the measurement basis for an
asset held for sale
b. Costs to sell are never discounted because held for sale assets should be sold within one year
c. Costs to sell are discounted if it is expected that the sale will be made beyond one year.
d. a and c

4. According to PFRS 5, gains and losses on remeasurement of assets held for sale are
a. recognized in profit or loss
b. recognized in other comprehensive income
c. recognized only for impairment losses
d. not recognized

5. Which of the following is included in profit from continuing operations?


a. extraordinary items c. other comprehensive income
b. discontinued operations d. income tax expense

Use the following information for the next two questions:


VISAGE APPEARANCE Co. is committed to a plan to sell its headquarters building and has
initiated actions to locate a buyer. As of this date, the building has a carrying amount of ₱5,000,000, a
fair value of ₱6,000,000 and estimated costs to sell of ₱200,000.

6. VISAGE Co. has an intention to transfer ownership of a building to a buyer after it vacates the
building. How should VISAGE Co. classify the headquarters building?
a. Included under property, plant and equipment at ₱5,000,000.
b. Included under property, plant and equipment at ₱5,800,000.
c. Classified as held for sale at ₱5,000,000
d. Classified as held for sale at ₱5,800,000

5,000,000 lower of carrying amount and fair value less costs sell
Page |4

7. VISAGE Co. will continue to use the building until the construction of a new headquarters is
completed. How should VISAGE Co. classify the headquarters building?
a. Included under property, plant and equipment at ₱5,000,000.
b. Included under property, plant and equipment at ₱5,800,000.
c. Classified as held for sale at ₱5,000,000
d. Classified as held for sale at ₱5,800,000

A – not available for immediate sale in its present condition

8. PERAMBULATE STROLL Co. is a commercial leasing and finance company. As of year-end,


PERAMBULATE holds equipment that is available either for sale or lease. PERAMBULATE is
not yet decided whether to sell or to lease the equipment. The equipment has a carrying amount
of ₱1,000,000, fair value of ₱1,200,000 and costs to sell of ₱50,000. How should PERAMBULATE
Co. classify the equipment?
a. Inventory, ₱1,000,000 c. Held for sale, ₱1,150,000
b. Investment property, ₱1,250,000 d. Held for sale, ₱1,000,000

A – Sale is not highly probable

9. In Baer Food Co.’s 20x3 single-step income statement, the section titled “Revenues” consisted of
the following:

Net sales revenue 187,000


Results from discontinued operations:
Loss from discontinued component Z including loss on disposal of ₱1,200 16,400
Less: Tax benefit 4,000 (12,400)
Interest revenue 10,200
Gain on sale of equipment 4,700
Cumulative change in 20x1 and 20x2 income due to change in depreciation
method (net of ₱750 tax effect) 1,500
Total revenues 191,000

In the revenues section of the 20x3 income statement, Baer Food should have reported total revenues
of
a. 197,200 b. 215,400 c. 203,700 d. 201,900

A
Solution:
Net sales revenue 187,000
Interest revenue 10,200
Adjusted total revenues 197,200

10. During 20x4, Lopez Corporation disposed of Pine Division, a major component of its business.
Lopez realized a gain of ₱500,000, net of taxes, on the sale of Pine's assets. Pine's operating
losses, net of taxes, were ₱600,000 in 2004. How should these facts be reported in Lopez's
income statement for 2004?
Total Amount to be Included in
Income from Results of
Page |5

Continuing Operations Discontinued Operations


a. 600,000 loss 500,000 gain
b. 100,000 loss 0
c. 0 100,000 loss
d. 500,000 gain 600,000 loss

C (600,000 loss – 500,000 gain) = 100,000 loss

“A fool shows his annoyance at once, but a prudent man overlooks an insult.” (Proverbs 12:16)
- END

SOLUTIONS

1. C
2. D
3. C
4. A
5. D
6. C 5,000,000 lower of carrying amount and fair value less costs sell
7. A – not available for immediate sale in its present condition
8. A – Sale is not highly probable

9. A
Solution:
Net sales revenue 187,000
Interest revenue 10,200
Adjusted total revenues 197,200

10. C (600,000 loss – 500,000 gain) = 100,000 loss


Page |1

Chapter 5
Statement of Changes in Equity

1. According to PAS 1, dividends declared by an entity are disclosed in the


a. Statement of financial position d. Notes
b. Statement of profit or loss and OCI e. Any of these
c. Statement of changes in equity f. c or d

2. Which of the following shows a correct effect on equity?


Transaction Effect on equity
a. Issuance of shares Decrease
b. Retirement of shares Increase
c. Profit Decrease
d. Loss Decrease

3. The amount of profit or loss appears in which of the following financial statements?
a. Statement of financial position
b. Statement of comprehensive income
c. Statement of changes in equity
d. b and c

4. The statement of changes in equity is dated


a. as of a point in time. c. after some time.
b. for a period of time. d. Not dated

5. The first line in the Statement of changes in owner’s equity is


a. Profit or loss c. Additional contributions
b. Beginning capital d. Drawings

“Be strong and courageous. Do not fear of be in dread of them, for it is the Lord your
God who goes with you. He will not leave you or forsake you.” (Deuteronomy 31:6)

- END -
Page |1

Chapter 6
Statement of Cash Flows

1. The net cash flow from operations must be reported using the direct method.

2. The direct method is generally favored by analysts and other users of cash flow statements.

3. Noncash items, such as depreciation expense, must be added to net income to arrive at net cash
provided by (used in) operating activities when the indirect method is used.

4. According to PAS 7 Statement of Cash Flows, an entity shall disclose the components of cash and
cash equivalents and shall present a reconciliation of the amounts in its statement of cash flows
with the equivalent items reported in the statement of financial position.

5. When using the indirect method, losses are subtracted from net income in computing cash flow
from operations.

6. The indirect method of reporting net cash flow from operations shows the operating cash
receipts and cash payments.

7. All transactions with investment securities, held for trading securities, FVOCI securities, and
amortized cost securities are reported in the investing section of the statement of cash flows.

8. The receipt of dividends should be classified as an operating activity only, while the payment of
dividends is to be classified as a financing activity only according to PAS 7.

9. When the indirect method is used, separate disclosure of interest paid and income taxes paid is
required.

10. Net cash provided by (used in) operating, financing, and investing activities are combined to
derive the amount of cash and cash equivalents at the end of the year.

“Be strong and courageous. Do not fear of be in dread of them, for it is the Lord
your God who goes with you. He will not leave you or forsake you.” (Deuteronomy 31:6)
- END -
Page |2

ANSWERS
1. FALSE 6. FALSE
2. TRUE 7. FALSE
3. TRUE 8. FALSE
4. TRUE 9. TRUE
5. FALSE 10. FALSE
Page |3

1. Stiggins Corporation had the following account balances for 2002:


31-Dec 1-Jan
Accounts Payable 67,200 58,200
Prepaid Rent Expense 24,600 37,200
Accounts Receivable (net) 84,000 66,600

Stiggins' 2002 profit is ₱450,000. What amount should Stiggins include as net cash provided by
operating activities in its 2002 statement of cash flows?
a. 436,200 b. 445,200 c. 453,600 d. 454,200

D
Solution:
Profit 450,000
Increase in accounts payable 9,000
Decrease in prepaid rent 12,600
Increase in accounts receivable, net (17,400)
Cash flow from operating activities 454,200

2. Chow Company's 2002 income statement reported cost of goods sold as ₱135,000. Additional
information is as follows:
31-Dec-02 31-Dec-01
Inventory 30,000 22,500
Accounts Payable 13,000 19,500

If Chow uses the direct method, what amount should Chow report as cash paid to suppliers in its
2002 statement of cash flows?
a. 121,000 b. 134,000 c. 136,000 d. 149,000

D
Solution:
Inventory
beg. 22,500
Net purchases (squeeze) 142,500 135,000 Cost of goods sold
30,000 end.

Accounts payable
19,500 beg.
Payments (squeeze) 149,000 142,500 Net purchases
end. 13,000

Use the following for the next three questions:


Frye Company uses the direct method to prepare its statement of cash flows. The company had the
following cash flows during 2002:
Cash receipts from the issuance of ordinary shares 400,000
Cash receipts from customers 200,000
Cash receipts from dividends on long-term investments 30,000
Cash receipts from repayment of loan made to
220,000
another entity
Cash payments for wages and other operating expenses 120,000
Cash payments for insurance 10,000
Page |4

Cash payments for dividends 20,000


Cash payments for taxes 40,000
Cash payment to purchase land 80,000

3. The net cash provided by (used in) operating activities is


a. 60,000 b. 40,000 c. 30,000 d. (20,000)
200,000
Cash receipts from customers
Cash receipts from dividends on long-term investments 30,000
Cash payments for wages and other operating
(120,000)
expenses
Cash payments for insurance (10,000)
Cash payments for taxes (40,000)
Cash flow from operating activities 60,000

4. The net cash provided by (used in) investing activities is


a. 220,000 b. 140,000 c. 60,000 d. (80,000)

Cash receipts from repayment of loan made to another 220,000


entity
Cash payment to purchase land (80,000)
Cash flow from investing activities 140,000

5. The net cash provided by (used in) all activities is


a. 580,000 b. 410,000 c. 380,000 d. (60,000)
400,000
Cash receipts from the issuance of ordinary shares
Cash payments for dividends (20,000)
Cash flow from financing activities 380,000
Net cash flows for the period 580,000

“Trust in the Lord with all your heart and lean not on your own understanding; in all
your ways acknowledge him, and he will make your paths straight.” (Proverbs 3:5-6)
- END -
Page |5

NAME: Date:
Professor: Section: Score:

1. The movements in the cash account of NONCHALANT COOL Co. during 20x2 are shown
below.
Cash
beg. 200
Sales 6,000 3,800 Purchases
Interest income 20 1,200 Operating expenses
Rent income 270 30 Interest expense
Dividend income 40 70 Income taxes
Held for trading securities 800 100 Investment in FVOCI
Sale of old building 520 1,100 Purchase of equipment
Collection of non-trade note 60 130 Loan granted to employee
Proceeds from loan with a bank 1,600 240 Payment of loan borrowed
Issuance of shares 970 200 Reacquisition of shares
90 Dividends
3,520 end.

Requirement: Prepare the statement of cash flows of NONCHALANT COOL Co. for the year ended
December 31, 20x2. (Use Option 1 in classifying cash flows from operating activities.)

2. IMPECCABLE FLAWLESS Co. had the following information during 20x2:

Accounts receivable, January 1, 20x2 1,200


Accounts receivable, December 31, 20x2 800
Sales on account and cash sales 16,000
Bad debts expense 400
Accounts payable, January 1, 20x2 700
Accounts payable, December 31, 20x2 400
Cost of sales 8,000
Increase in inventory 1,800
Operating expenses on accrual basis 2,440
Increase in accrued payables for operating expenses 820
Decrease in prepaid operating expenses 780
Property, plant, and equipment, January 1, 20x2 3,600
Property, plant, and equipment, December 31, 20x2 5,400

Additional information:
a. There were no write-offs of accounts receivable during the year.
b. Equipment with an accumulated depreciation of ₱400 was sold during the year for ₱240
resulting to a gain on sale of ₱30.

Requirements: Compute for the following:


(a) cash receipts from customers,
Page |6

(b) cash payments to suppliers,


(c) cash payments for operating expenses, and
(d) cash payments for acquisition of property, plant, and equipment.

3. DERISION OBJECT OF RIDICULE Co. has the following information as of December 31, 20x2:
Jan. 1 Dec. 31
Accounts receivable 8,000 10,000
Allowance for bad debts (200) (500)
Prepaid rent 1,920 1,600
Accounts payable 3,400 4,400

DERISION reported profit of ₱4,400 for the year, after depreciation expense of ₱100, gain on sale of
equipment of ₱120, and restructuring and other provisions of ₱200. None of the provisions
recognized during the period affected cash.

Requirement: Compute for the cash flow from operating activities using the indirect method.

4. TRAVAIL HARD LABOR Co. had the following information for 20x2:
a. Acquired 3-month treasury bills for ₱100,000.
b. Acquired equipment with a purchase price of ₱2,000,000 by paying 20% in cash and issuing
a note payable for the balance. There were no payments made on the note during the year.
c. Acquired land with fair value of ₱1,600,000 by issuing shares with aggregate par value of
₱1,200,000. The excess is credited to share premium.
d. Extended a ₱800,000 loan to a director.
e. Borrowed ₱640,000 from a bank. Used the cash proceeds as follows: ₱400,000 for additional
working capital and ₱240,000 to settle scrip dividends declared in 20x1.
f. Settled an outstanding note payable by issuing shares with aggregate par value of ₱400,000.
Share premium resulted from the transaction amounted to ₱140,000.

Requirements: Compute for the net cash flows from (used in) (a) investing activities and (b) financing
activities.

“Again, I tell you that if two of you on earth agree about anything you ask for, it
will be done for you by my Father in heaven. For where two or three come together
in my name, there am I with them.”
(Matthew 18:19-20)

- END -
Page |7

SOLUTIONS

1. Solution:
NONCHALANT COOL Company
Statement of cash flows
For the year ended December 31, 20x2

Cash flows from operating activities


Cash receipts from customers 6,000
Cash receipts for interest income 20
Cash receipts for rent income 270
Cash receipts for dividend income 40
Cash paid to suppliers (3,800)
Cash paid for operating expenses (1,200)
Cash generated from operations 1,330
Interest paid (30)
Income taxes paid (70)
Cash receipt from sale of held for trading securities 800
Net cash from operating activities 2,030

Cash flows from investing activities


Cash payment for acquisition of investment in FVOCI (100)
Cash receipt from sale of old building 520
Cash payment for acquisition of equipment (1,100)
Cash receipt from collection of loan granted 60
Cash payment for loan granted (130)
Net cash used in investing activities (750)

Cash flows from financing activities


Cash proceeds from loan borrowed 1,600
Cash payment for loan borrowed (240)
Cash proceeds from issuance of share capital 970
Cash payment for acquisition of treasury shares (200)
Cash payment for dividends (90)
Net cash from financing activities 2,040

Net increase in cash and cash equivalents 3,320


Cash and cash equivalents, beginning 200
Cash and cash equivalents, end 3,520

2. Solutions:
Requirement (a): Cash receipts from customers
Accounts receivable
Jan. 1, 20x2 1,200 - Write-offs
Collections of accounts receivables
Sales 16,000 16,400 (squeeze)
800 Dec. 31, 20x2
Page |8

Requirement (b): Cash payments to suppliers


Inventory
Jan. 1, 20x2 -
Net purchases (squeeze) 9,800 8,000 Cost of sales
1,800 Dec. 31, 20x2

Accounts payable
700 Jan. 1, 20x2
Payments for purchases on
account (squeeze) 10,100 9,800 Net purchases (accrual)
Dec. 31, 20x2 400

Requirement (c): Cash payments for operating expenses


Prepaid expense / Accrued expense
Prepaid expense, beginning 780 - Accrued expense, beg.
Cash paid for operating expenses
(squeeze) 840 2,440 Operating expense (accrual basis)
Accrued expense, end 820 - Prepaid expense, end

Requirement (d): Cash payments for acquisition of PPE


The entry for the sale of equipment is re-constructed as follows:
20x2 Cash on hand (given) 240
Accumulated depreciation (given) 400
Equipment (squeeze) 610
Gain on sale (given) 30
Property, plant and equipment
Jan. 1, 20x2 3,600
Historical cost of equipment sold (see
Acquisition of PPE (squeeze) 2,410 610 journal entry above)
5,400 Dec. 31, 20x2

3. Solution:
Cash flows from operating activities
Profit 4,400
Adjustments for:
Depreciation expense 100
Gain on sale of building (120)
Restructuring and other provisions 200
4,580
Increase in accounts receivable, net
[(5,000 – 250) – (4,000 – 100)] (1,700)
Decrease in prepaid rent 320
Increase in accounts payable 1,000
Net cash from operating activities 4,200

4. Solution:
Cash flows from investing activities
Cash payment for acquisition of equipment (2M x 20%) (400,000)
Cash proceeds of loan granted to officer (800,000)
Net cash used in investing activities (1,200,000)

Cash flows from financing activities


Page |9

Cash receipt from borrowings 640,000


Cash payment for dividends (240,000)
Net cash used in financing activities 400,000
Page |1

Chapter 7
Notes – Part 1

NAME: Date:
Professor: Section: Score:

ACCOUNTING POLICIES, CHANGES IN ESTIMATES & ERRORS

1. The primary distinction between a change in accounting estimate and the correction of an error is
the timing of availability of information; a change in estimate is based on new information not
previously available.

2. All changes in accounting policy are accounted for by retrospective application.

3. A company changes its depreciation method for machinery and equipment from sum-of-the-
years'-digits depreciation to the straight-line method. This is a change in accounting estimate.

4. A company reduces the lives of several patents from 17 to 10 years because of rapid technological
change. This is a change in accounting policy.

5. An analysis of the allowance for doubtful accounts showed the balance should be reduced by
₱27,500 due to recent changes in economic conditions. This is a change in accounting estimate.

6. A company changes from a non-acceptable to an acceptable accounting principle. This is a change


in accounting policy.

7. A company changes its depreciation method at the same time it recognizes a change in the
estimated useful life of the asset. This is a change in accounting policy.

8. The understatement of merchandise inventory is an example of an error that counterbalances after


two years.

9. Failure to record amortization is an example of a non-counter balancing error.

10. An understatement in ending inventory results to overstatement of profit during the period.

“Do not be anxious about anything, but in everything by prayer and supplication
with thanksgiving let your requests be made known to God. And the peace of God,
which surpasses all understanding, will guard your hearts and your minds in Christ
Jesus.” (Philippians 4:6-7)

- END -
Page |2

ANSWERS
1. TRUE 6. FALSE
2. FALSE 7. FALSE
3. TRUE 8. TRUE
4. FALSE 9. TRUE
5. TRUE 10. FALSE
Page |3

1. Conn Co. reported a retained earnings balance of ₱400,000 at December 31, 20X8. In August 20X9,
Conn determined that insurance premiums of ₱60,000 for the three-year period beginning January
1, 20X8, had been paid and fully expensed in 20X8. Conn has a 30% income tax rate. What amount
should Conn report as adjusted beginning retained earnings in its 20X9 statement of retained
earnings?
a. 420,000 b. 428,000 c. 440,000 d. 442,000

B 400,000 + [(60,000 x 2/3) x 70%] = 428,000

2. Foy Corp. failed to accrue warranty costs of ₱50,000 in its December 31, 20X4, financial statements.
In addition, a change from straight-line to accelerated depreciation made at the beginning of 20X5
resulted in a cumulative effect of ₱30,000 on Foy's retained earnings. Both the ₱50,000 and the
₱30,000 are net of related income taxes. What amount should Foy report as prior period
adjustment in 20X5?
a. 0 b. 30,000 c. 50,000 d. 80,000

C 50,000 – the correction for the prior period error. The change from straight-line to accelerated
depreciation is a change in accounting estimate that should be accounted for prospectively.

3. Loeb Corp. frequently borrows from the bank in order to maintain sufficient operating cash. The
following loans were at a 12% interest rate, with interest payable at maturity. Loeb repaid each
loan on its scheduled maturity date.
Date of loan Amount Maturity date Term of loan
11/1/x1 ₱ 5,000 10/31/x2 1 Year
2/1/x2 15,000 7/31/x2 6 Months
5/1/x2 8,000 1/31/x3 9 Months

Loeb records interest expense when the loans are repaid. As a result, interest expense of ₱1,500 was
recorded in 20x2. If no correction is made, by what amount would 20x2 interest expense be
understated?
a. 540 b. 620 c. 640 d. 720

A
Solution:
The correct interest expense in 20x2 is computed as follows:
(5,000 x 12% x 10/12) 500
(15,000 x 12% x 6/12) 900
(8,000 x 12% x 8/12) 640
Correct interest expense - 20x2 2,040

Correct interest expense - 20x2 2,040


Interest expense recognized 1,500
Understatement 540

The next three items are based on the following:


Declaration, Inc., is a calendar year corporation. Its financial statements for the years 20x2 and 20x1
contained errors as follows:
Page |4

20x2 20x1
Ending inventory ₱1,000 understated ₱3,000 overstated
Depreciation expense ₱800 understated ₱2,500 overstated

4. Assume that the proper correcting entries were made at December 31, 20x1. By how much will
20x2 income before income taxes be overstated or understated?
a. ₱200 understated. c. ₱2,700 understated.
b. ₱500 overstated. d. ₱3,200 understated.

A
Solution:
Ending inventory - 20x2 1,000
Depreciation expense - 20x2 (800)
Understatement 200

The counter-balancing error in 20x1 does not affect the 20x2 profit because proper correcting entries were made on
December 31, 20x1.

5. Assume that no correcting entries were made at December 31, 20x1. Ignoring income taxes, by
how much will retained earnings at December 31, 20x2, be overstated or understated?
a. ₱200 understated. c. ₱2,700 understated.
b. ₱500 overstated. d. ₱3,200 understated.

C
Solution:
Effect on profit (over) under statement:
Errors on: 20x1 20x2
Ending inventory - 20x1 (3,000) 3,000
Ending inventory - 20x2 1,000
Depreciation expense - 20x1 2,500
Depreciation expense - 20x2 (800)
(500) 3,200

Effect on profit - 20x1 (500)


Effect on profit - 20x2 3,200
Effect on R/E, 20x2 - understatement 2,700

6. Assume that no correcting entries were made at December 31, 20x1, or December 31, 20x2, and
that no additional errors occurred in 20x3. Ignoring income taxes, by how much will working
capital at December 31, 20x3, be overstated or understated?
a. ₱0. c. ₱1,000 understated.
b. ₱1,000 overstated. d. ₱1,700 understated.

A The working capital on December 31, 20x3 is not affected because all of the counter-balancing errors
would have counter-balanced already as of this point. Errors on depreciation do not affect working capital.
Page |5

7. Bren Co.'s beginning inventory at January 1, 20x3, was understated by ₱26,000, and its ending
inventory was overstated by ₱52,000. As a result, Bren's cost of goods sold for 20x3 was
a. Understated by ₱26,000. c. Understated by ₱78,000.
b. Overstated by ₱26,000. d. Overstated by ₱78,000.

C
Solution:
Effect on COGS -
(over)/understatement
Understatement in beg. Inventory 26,000
Overstatement in end. Inventory 52,000
Net effect on COGS –
understatement 78,000

The next three items are based on the following:


The bookkeeper of Latsch Company, which has an accounting year ending December 31, made the
following errors:
• A ₱1,000 collection from a customer was received on December 29, 20x0, but not recorded until
the date of its deposit in the bank, January 4, 20x1.
• A supplier's ₱1,600 invoice for inventory items received in December 20x0 was not recorded until
January 20x1. (Inventories at December 31, 20x0 and 20x1, were stated correctly, based on physical
count.)
• Depreciation for 20x0 was understated by ₱900. In September 20x0, a ₱200 invoice for office
supplies was charged to the Utilities Expense account. Office supplies are expensed as purchased.
• December 31, 20x0, sales on account of ₱3,000 were recorded in January 20x1.

Assume that no other errors have occurred and that no correcting entries have been made. Ignore
income taxes.

8. Profit for 20x0 was


a. Understated by ₱500. c. Overstated by ₱2,500.
b. Understated by ₱2,100. d. Neither understated nor overstated.

A
Solution:
Effect on profit (over)/understatement
Unrecorded collection -
Unrecorded purchases (1,600)
Understatement in depreciation (900)
Erroneous debit of office supplies
expense to utilities expense -
Unrecorded sales 3,000
Net effect on profit - understatement 500

9. Assume the same facts as above. Working capital at December 31, 20x0, was
a. Understated by ₱3,000. c. Understated by ₱1,400.
b. Understated by ₱500. d. Neither understated or overstated.

C
Solution:
Page |6

Effect on working capital (over)/understatement


Unrecorded collection -
Unrecorded purchases (1,600)
Understatement in depreciation -
Erroneous debit of office supplies
expense to utilities expense -
Unrecorded sales 3,000
Net effect on working capital - understatement 1,400

10. Assume the same facts as above. Total assets at December 31, 20x0, were
a. Overstated by ₱2,500. c. Understated by ₱2,500.
b. Overstated by ₱2,100. d. None of the above.

D
Solution:
Effect on total assets
(over)/understatement
Unrecorded collection -
Unrecorded purchases (Inventory is
properly stated) -
Understatement in depreciation (900)
Erroneous debit of office supplies
expense to utilities expense -
Unrecorded sales (Unrecorded
accounts receivable) 3,000
Net effect on total assets - understatement 2,100

“The name of the Lord is a strong tower; the righteous run to it and are safe.” (Proverbs
18:10)
- END -
Page |7

Instruction: State whether the following events are adjusting or non-adjusting events after the
reporting period. All events took place after the end of the reporting period but before the financial
statements are authorized for issue

1. The entity guarantees the debt of another entity.

2. Settlement of a contingent event at an amount below the provision recognized as at the end of the
reporting period.

3. Major acquisition of a competitor company.

4. The actual sale of noncurrent assets held for sale provides information on the asset’s fair value less
costs to sell as at the end of the reporting period.

5. The resolution of a contingent event which provides evidence of the entity’s obligation to pay
bonus to a key management personnel for the services he has rendered during the reporting
period.

6. The conditions under PFRS 5 Non-current Assets Held for Sale and Discontinued Operations for
classifying a noncurrent asset or disposal group as held for sale are met.

7. Change in income tax rate.

8. The discovery that the accumulated patent amortization is understated.

9. A board resolution to change the depreciation method used for buildings.

10. Adoption and announcing of a formal plant to close a business segment.

“For the Lord gives wisdom; from his mouth come knowledge and understanding.”
(Proverbs 2:6)

- END -
Page |8

ANSWERS
1. NON-ADJUSTING
2. ADJUSTING
3. NON-ADJUSTING
4. ADJUSTING
5. ADJUSTING
6. NON-ADJUSTING
7. NON-ADJUSTING
8. ADJUSTING
9. NON-ADJUSTING
10. NON-ADJUSTING
Page |9

1. BOWDLERIZE TO EDIT Company’s current reporting period ends on December 31, 20x1. The
following transactions occurred after the end of reporting period:
a. On January 5, 20x2, BOWDLERIZE declared ₱4,000,000 dividends.
b. On January 15, 20x2, BOWDLERIZE issued 1,000 shares with par value per share of ₱200 for
₱1,200 per share.
c. On January 20, 20x2, BOWDLERIZE installed an oil rig. Current legislation requires that the
oil rig be uninstalled at the end of its useful life and the site where it was installed be restored.
BOWDLERIZE estimates the present value of the decommissioning and restoration cost at
₱2,000,000.
d. On February 1, 20x2, a building with a carrying amount as of December 31, 20x1 of ₱1,000,000
was totally razed by fire.
e. On February 10, 20x2, BOWDLERIZE received notice of a litigation in relation to an accident
that happened on December 31, 20x1. BOWDLERIZE estimates a probable loss of ₱400,000.
f. On March 5, 20x2, BOWDLERIZE purchased a subsidiary for ₱20,000,000 in a business
combination accounted for using the acquisition method. Goodwill of ₱5,000,000 was
recognized on the business combination.
The financial statements were authorized for issue on March 1, 20x2.

Requirement: Compute for the total amount of adjusting events.

2. PANTALOON TROUSER Company’s current reporting period ends on December 31, 20x1. The
following transactions occurred after the end of reporting period:
a. On January 20, 20x2, a pending litigation was resolved requiring a settlement amount of
₱200,000. The 20x1 year-end financial statements included a provision for loss on litigation of
₱240,000.
b. Inventories costing ₱2,000,000 were recognized at their net realizable value of ₱1,800,000 in the
20x1 year-end financial statements. During January 20x2, the inventories were sold for
₱1,760,000. Actual selling costs amounted to ₱60,000.
c. The year-end accounts receivable include a ₱200,000 receivable from XYZ, Inc. No allowance
for doubtful accounts was recognized on this receivable as of December 31, 20x1. On February
3, 20x2, XYZ filed for bankruptcy. It was estimated that the receivable will not be collected.
d. The fair value of financial assets measured at fair value through profit or loss significantly
declined to ₱160,000 on February 28, 20x2. The financial assets are recognized in the 20x1 year-
end financial statements at ₱600,000 which is their fair value as of December 31, 20x1.
e. On March 5, 20x2, a case was resolved requiring a settlement amount of ₱400,000. The 20x1
year-end financial statements included a provision for loss on litigation of ₱300,000.
PANTALOON Company’s profit for the year ended December 31, 20x1 before consideration of the
above transactions is ₱4,400,000. The financial statements were authorized for issue on March 1, 20x2.
Requirement: Compute for the adjusted profit for the year. Provide journal entries.

“I press on toward the goal to win the prize for which God has called me heavenward in Christ
Jesus.” (Philippians 3:14)
- END –

SOLUTION

1. Solution:
P a g e | 10

Estimated liability on pending litigation 400,000


Total adjusting events 400,000

Adjusting events are those that provide evidence of conditions that existed at the end of the reporting
period. Those that are indicative of conditions that arose after the reporting period are non-adjusting
events.
Thus, the declaration of dividends, issuance of shares, and impairment of the building after
the reporting period are non-adjusting events. There is no present obligation for decommissioning and
restoration costs as of the end of reporting period because the oil rig was installed after the reporting
period.
The business combination is neither recognized nor disclosed in the December 31, 20x1
financial statements because the business combination is not an event after the reporting period, i.e., it
occurred after the financial statements were authorized for issue.

2. Solution:
Unadjusted profit, December 31, 20x1 4,400,000
Reduction in provision for loss on pending litigation
(240K – 200K) 40,000
Reduction in NRV of inventories [1.8M - (1.76M – 60K)] (100,000)
Impairment loss on receivables (200,000)
Adjusted profit, December 31, 20x1 4,140,000

Pertinent adjusting entries are:


Dec. 31, 20x1 Estimated liability on pending litigation 40,000
Probable loss on litigation 40,000
Dec. 31, 20x1 Cost of goods sold 100,000
Inventory 100,000
Dec. 31, 20x1 Impairment loss on receivables 200,000
Accounts receivable 200,000
Page |1

Chapter 8
Notes – Part 2

1. Which of the following statements is incorrect?


a. Related party transactions and outstanding balances with other entities in a group are
disclosed in an entity’s financial statements.
b. Intragroup related party transactions and outstanding balances are not eliminated in the
preparation of consolidated financial statements of the group.
c. Related party relationships are a normal feature of commerce and business.
d. A related party relationship could have an effect on the profit or loss and financial position of
an entity.
e. Knowledge of related party transactions, outstanding balances and relationships may affect
assessments of an entity’s operations by users of financial statements, including assessments of
the risks and opportunities facing the entity.

2. An entity’s ability to affect the financial and operating policies of an investee is through the
presence of
I. Control
II. Joint control
III. Significant influence
a. I only b. I or III c. Any of I, II, or III d. I, II and III

3. Which of the following statements is correct?


I. The profit or loss and financial position of an entity may be affected by a related party
relationship even if related party transactions do not occur.
II. The mere existence of the relationship may be sufficient to affect the transactions of the entity
with other parties.
a. True, true b. True, false c. False, false d. False, true

4. All of the parties enumerated are related to an entity , except


a. the entity is a subsidiary, an associate, or a venture in a joint venture.
b. the party is a member of the key management personnel of the entity or its parent.
c. the party is a close member of the family of an individual having control, significant influence,
or joint control over the entity or a member of the key management personnel of the entity or
its parent.
d. the party is a post-employment benefit plan for the benefit of employees of the entity, or of
any entity that is a related party of the entity.
e. two entities simply because they have a director or other member of key management
personnel in common

5. The following relates to the transactions of MISCELLANY MIXTURE Company during 20x1:
Directors' and officers' remuneration 4,000,000
Post-employment benefits of officers 400,000
Fringe benefits in the form of housing assistance to directors
and officers 10,000,000
Share options granted to officers 600,000
Page |2

Officers' expenses on travels, representation and


entertainment subject to liquidation and reimbursement 200,000
Loans to directors and officers 6,000,000
Sales to related entities 20,000,000

Requirements: Determine the amount of related party disclosures on MISCELLANY’s (a) separate
financial statements and (b) the group’s consolidated financial statements.

“So do not fear, for I am with you; do not be dismayed, for I am your God. I will
strengthen you and help you; I will uphold you with my righteous right hand.”
(Isaiah 41:10)

- END -
Page |3

ANSWERS
1. B
2. C
3. A
4. E

5. Solutions:
(a) Separate financial statements
Key management personnel compensation:
Directors' and officers' remuneration 4,000,000
Post-employment benefits of officers 400,000
Fringe benefits in the form of housing
assistance 10,000,000
to directors and officers
Share options granted to officers 600,000
15,000,000
Related party transactions and outstanding balances:
Loans to directors and officers 6,000,000
Sales to related entities 20,000,000
26,000,000
Total 20,500,000

Advances to officers for necessary expenses of the entity and subject to liquidation are not treated as
key management personnel compensation.

(b) Consolidated financial statements


Key management personnel compensation:
Directors' and officers' remuneration 4,000,000
Post-employment benefits of officers 400,000
Fringe benefits in the form of housing
assistance 10,000,000
to directors and officers
Share options granted to officers 600,000
15,000,000
Related party transactions and outstanding balances:
Loans to directors and officers 6,000,000
6,000,000
Total 10,500,000

Intercompany transactions and outstanding balances are eliminated in the consolidated financial
statements.
Page |4

1. MISCREANT UNBELIEVING Co. is preparing its year-end financial statements and has identified
the following operating segments:
Segments Revenues Profit (loss) Assets
A 2,000,000 400,000 28,000,000
B 2,400,000 280,000 36,000,000
C 540,000 (140,000) 24,000,000
D 480,000 (1,400,000) 2,000,000
E 580,000 100,000 2,800,000
Totals 6,000,000 (760,000) 92,800,000

Requirement: Identify the reportable segments.

2. LIMPID CLEAR Company engages in five diversified operations namely, operations A, B, C, D,


and E. Information on these segments are shown below:

Segments Revenues Profit (loss) Assets


A 1,600 400 20,000
B 1,600 200 4,000
C 100 20 2,000
D 300 40 4,000
E 400 140 14,000
Totals 4,000 800 44,000

Additional information:
a. For internal reporting purposes, segments A and B are considered as one operating segment.
b. Segment E is considered as an operating segment for internal decision making purposes.
c. Segments C and D have similar economic characteristics and share a majority of the aggregation
criteria.

Requirement: Identify the reportable segments.

3. TACITURN SILENT Co. is preparing its year-end financial statements and has identified the
following operating segments:
External Inter-segment Total
Segments revenues revenues revenues Profit Assets
A 2,400,000 1,200,000 3,600,000 1,400,000 24,000,000
B 800,000 200,000 1,000,000 800,000 14,000,000
C 500,000 - 500,000 200,000 2,000,000
D 400,000 400,000 160,000 1,600,000
E 300,000 300,000 140,000 1,400,000
F 200,000 200,000 100,000 1,000,000
Totals 4,600,000 1,400,000 6,000,000 2,800,000 44,000,000

Management believes that between segments C, D, E and F, segment C is most relevant to external
users of financial statements.
Page |5

Requirement: Identify the reportable segments.

4. FIDELITY LOYALTY Co. has the following information on its operating segments.
External Inter-segment Total
Segments revenues revenues revenues Profit Assets
A 2,400,000 1,200,000 3,600,000 1,400,000 24,000,000
B 800,000 200,000 1,000,000 800,000 14,000,000
C 500,000 - 500,000 200,000 2,000,000
D 400,000 - 400,000 160,000 1,600,000
E 300,000 - 300,000 140,000 1,400,000
F 200,000 - 200,000 100,000 1,000,000
Totals 4,600,000 1,400,000 6,000,000 2,800,000 44,000,000

Question: FIDELITY Co. shall provide disclosure for major customers if revenues from transactions
with a single external customer amount to how much?

“Let us therefore come boldly to the throne of grace, that we may obtain mercy and
find grace in time of need.”
(Hebrews 4:16)

- END –

SOLUTIONS
1. Answer: The reportable segments are segments A, B, C and D.
Solutions:
Revenue test
Page |6

The threshold under the revenue test is P600,000 or (P6,000,000 x 10%). Segments A and B are
reportable because each of their revenues is at least P600,000 or 10% of the total revenues.

Asset test
The threshold under the revenue test is P9,800,000 or (P92,800,000 x 10%). Segments A, B, and C are
reportable because each of their total assets is at least P9,800,000 or 10% of the total assets.

Profit or loss test


Segments Profit Loss
A 400,000
B 280,000
C (140,000)
D (1,400,000)
E 100,000
Totals 780,000 (1,540,000)

Based on the table above, the aggregate losses of P1,540,000 is higher than the aggregate profits.
Therefore, the 10% limit of profit or loss is P154,000 or (P1,540,000 x 10%). Segments A, B and D are
reportable under this test since each of their reported profits or loss is at least P154,000 or 10% of
P1,540,000.

Based on the tests performed, the reportable segments to be disclosed in ABC’s notes to financial
statements are segments A, B, C, and D.

2. Answer: LIMPID Co. shall disclose three reportable segments in its notes to financial statements,
namely, “A/B,” “E” and “C/D.”

Solution:
Under the “management approach,” segments A and B (aggregated as one segment) and segment E are
reportable operating segments because these segments are used by ABC Co. in its internal reporting.

Segments C and D are subjected to the quantitative thresholds as shown below:


Segments Revenues Revenue test Profit Profit test Assets Asset test
A 1,600 N/A 400 N/A 20,000 N/A
B 1,600 N/A 200 N/A 4,000 N/A
C 100 3% 20 3% 2,000 5%
D 300 8% 40 5% 4,000 9%
E 400 N/A 140 N/A 14,000 N/A
Totals 4,000 800 44,000

Segments C and D do not individually meet any of the quantitative thresholds. However, since the problem
states that segments C and D have similar economic characteristics and share a majority of the
aggregation criteria, they are aggregated and tested if their combined results qualify under the
quantitative thresholds.

The combined revenue of C and D of P400 (100 + 300) is equal to the revenue threshold of P400 (4,000 x
10%). Therefore, C and D shall be disclosed as one reportable segment.

ABC Co. shall disclose three reportable segments in its notes to financial statements, namely, “A/B,” “E”
and “C/D.”

3. Answer: The three reportable segments are segments A, B, and C. The other segments are combined
and disclosed as “all other segments.”

Solution:
Page |7

Under the revenue test, segments A and B are reportable because they each have total revenues
(external and internal) exceeding the threshold of P600,000 or (6,000,000 x 10%).

Under the profit or loss test, segments A and B are reportable because they each have profit exceeding
the threshold of P280,000 (2,800,000 x 10%).

Under the total assets test, segments A and B are reportable because they each have total assets
exceeding the threshold of P4,400,000 (44,000,000 x 10%).

However, the sum of the external revenues of segments A and B does not meet the 75% limit as shown
below:
External
Segments revenues
A 2,400,000
B 800,000
Total external revenues of A and B 3,200,000

Total entity's external revenues 4,600,000


Multiply by: 75%
Limit on external revenues of reportable segments 3,450,000

Since management believes that of the other segments (i.e., C, D, E, and F), information on segment C is
most relevant to users, segment C shall be disclosed as a reportable segment even if it does not meet any
of the quantitative thresholds in order for the “75% limit” to be met.

If segment C is included as reportable segment, the total external revenues of reportable segments A, B,
and C is P3,700,000 (2,400,000 + 800,000 + 500,000) which meets the “75% limit” of P1,725,000.

4. Answer: at least P460,000 or (10% x P4,600,000 total external revenues)


Page |1

Chapter 9
Interim Financial Reporting

1. Temporary decline in the fair value of an investment in equity securities.

2. Significant and permanent decline in the fair value of an investment in equity securities.

3. Casualty loss from typhoon.

4. Government grant received as aid for the loss incurred in item #2 above.

5. Depreciation.

6. Year-end bonuses of employees which they earn as they render service.

7. Results of discontinued operations.

8. Premium paid for a one-year insurance.

9. Regular repairs and maintenance costs.

10. Dividend income.

11. Effect of change in foreign exchange rates on foreign currency denominated liabilities.

12. Temporary decline in the value of inventories.

13. Property tax for the year.

14. Post-employment benefits.

15. Significant but temporary increase in the fair value of investment in equity securities measured
at fair value through other comprehensive income.

ANSWERS
1. IMMEDIATELY
2. IMMEDIATELY
3. IMMEDIATELY
4. IMMEDIATELY
5. SPREAD OUT
6. SPREAD OUT
7. IMMEDIATELY
8. SPREAD OUT
Page |2

9. IMMEDIATELY
10. IMMEDIATELY
11. IMMEDIATELY
12. IMMEDIATELY
13. SPREAD OUT
14. SPREAD OUT
15. IMMEDIATELY

1. QUIRK ACCIDENT Co. reports profit before tax of ₱200,000 in its 2nd quarter interim financial
statements before consideration for the following:
a. Inventory with a carrying amount ₱10,000 has a net realizable value of ₱12,000. It is expected
that the change in value will reverse in the 3rd quarter. There have been no write-downs of
inventory recognized in previous periods.
Page |3

b. An investment property measured under the cost model has a carrying amount of ₱150,000 but
its recoverable amount is ₱140,000.
c. An investment in FVPL measured at acquisition cost of ₱20,000 has a fair value of ₱38,000 as at
the end of 2nd the quarter. However, the increase in fair value is expected to be only temporary.
d. No depreciation is recognized during the 2nd quarter. The annual straight-line depreciation of
items of PPE is ₱60,000.
e. ABC Co. has a policy of providing 12 days paid vacation leaves for its employees. The vacation
leaves are vesting and accumulating. Total paid vacation leaves eligibility of employees for the
full year is ₱140,000. However, only ₱20,000 worth of paid vacation leaves have been availed of
during the quarter.
f. It was discovered that depreciation in the previous year was overstated by ₱7,000.

Requirement: Compute for the adjusted profit before tax.

2. FATUOUS SILLY Co. is preparing its interim financial statements for the period ended March
31, 20x1. The following relate to the transactions during the first quarter:
a. Total sales for the interim period was ₱2,000,000.
b. Cost of sales was ₱900,000.
c. FATUOUS is liable for 5% commission on its sales to its sales representatives and agents. No
commission has yet been paid as of March 31, 20x1.
d. The allowance for doubtful accounts has a balance of ₱10,000 as of January 1, 20x1. The
required balance as of March 31, 20x1 is ₱30,000. There were no write-offs or recoveries during
the period.
e. A building with historical cost of ₱2,400,000 is being depreciated over 5 years using straight
line method.
f. FATUOUS prepaid a one-year insurance on its assets for ₱80,000 on January 1, 20x1,.
g. Property taxes for 20x1 amounting to ₱52,000 was paid in January.
h. Advertising costs of ₱100,000 were incurred in February on promotional activities held on
Valentine’s Day.
i. Year-end staff bonuses are expected to be around ₱184,000. Employees become entitled to the
bonuses as they provide services to FATUOUS during the year.
j. FATUOUS’s president is entitled to a 10% bonus on profit before bonus and taxes.
k. Loss on sale of a used equipment on March 2, 20x1 was ₱60,000.
l. FATUOUS incurred ₱24,000 on unanticipated repairs on its factory equipment on March 16,
20x1.
m. Due to the unexpected breakdown of the factory equipment on March 16, 20x1, FATUOUS has
planned a major periodic overhaul of its other equipment to be held annually starting on
December 31, 20x1. The cost of the major planned periodic overhaul is estimated at ₱96,000.
n. FATUOUS leases one of its retail stores. Monthly rentals are ₱10,000, however, the lease
contracts provide for a contingent rent equal to 2% of the excess of sales over ₱1,800,000.
o. FATUOUS’s budget for 20x1 included charitable contributions of ₱58,000 and employee
training costs of ₱26,000. None of those costs were incurred as of March 31, 20x1.
p. p Other operating expenses incurred during the first quarter totaled ₱240,000.

Requirement: Compute for the profit or loss for the first quarter ended March 31, 20x1.
Page |4

3. IGNOMINY DISGRACE Co.’s profits before tax for the 1st and 2nd quarters of 20x1 were
₱1,760,000 and ₱1,840,000 before any necessary adjustments for the items listed below.
a. Total unfavorable manufacturing cost variances amounted to ₱48,000 in the 1st quarter.
IGNOMINY expects that the manufacturing cost variances will be absorbed by year-end.
There were no work-in-process inventories as of the end of the 1st and 2nd quarters.
b. Newspaper advertisement costs of ₱180,000 were paid on April 1, 20x1. The advertisement
shall appear in the weekly newspaper publications over the remaining months of the year.
c. IGNOMINY’s held for trading securities acquired on February 4, 20x1 for ₱400,000 had a fair
value of ₱200,000 on March 31, 20x1. IGNOMINY had expected that the fair value decline
was only temporary. In fact, on June 30, 20x1, the recovery exceeded the previous write-
down in investment by ₱40,000.
d. Research and development costs incurred during the 1st and 2nd quarters totaled ₱20,000 and
₱24,000, respectively. In July 20x1, technical feasibility has been established and, therefore,
development costs of ₱10,000 and ₱14,000 expensed in the 1st and 2nd quarters would have
qualified for capitalization.
e. On January 20x1, IGNOMINY recognized an account receivable denominated in US dollars
amounting to $2,000. The exchange rate on that date was ₱40:$1. On March 31, 20x1, the
exchange rate was ₱30:$1. IGNOMINY had expected that the change in the exchange rate
was only temporary. In fact, on June 30, 20x1, the exchange rate was ₱45:$1. The receivable is
collectible on September 2, 20x1.
f. A land with a carrying amount of ₱400,000 had a recoverable amount of ₱384,000 on March
31, 20x1.

Requirement: Compute for the adjusted profits before tax for the 1st and 2nd quarters.

4. Among the transactions of WRY TO TWIST Company for the first two quarters of 20x1 were the
following:
a. WRY recognized a ₱200,000 write-down in its inventory during the first quarter. WRY had
expected that the write-down will reverse in the second quarter, and in fact, in the second
quarter, the recovery exceeded the previous write-down by ₱40,000.
b. WRY provides warranty for its sales. In the first quarter, WRY estimated a 5% warranty
obligation on its first quarter sales of ₱2,000,000. In the second quarter, a change in
accounting estimate was made. It was estimated that the cost of warranty should be 10% of
total sales. The second quarter sales amounted to ₱2,400,000.
c. WRY has been estimating its bad debt expense as 2% of credit sales. However, in the second
quarter, a change was made to the percentage of ending receivable. Under this method, the
required balance of the allowance for doubtful accounts as of June 30, 20x1 is computed at
₱60,000. The allowance has a balance of ₱10,000 at the beginning of the year. Total write-offs
during the first six months of 20x1 amounted to ₱24,000; recoveries totaled ₱6,000. Credit
sales for the 1st and 2nd quarters amounted to ₱2,000,000 and ₱4,000,000, respectively.

Requirement: What are the effects of the transactions listed above on profit or loss before tax in the
first and second quarter interim financial statements of WRY?
Page |5

5. APPOSITE FITTING Co. expects to earn ₱200,000 pre-tax profit each quarter. APPOSITE has tax
rates of 20% on the first ₱400,000 of annual earnings and 30% on all additional earnings. Actual
earnings match expectations.

Requirement: Compute for (a) the weighted average annual income tax rate and (b) income tax expense
recognized in the quarterly interim financial statements.

“The heart of the discerning acquires knowledge, for the ears of the wise seek it
out.” (Proverbs 18:15)
- END -
Page |6

SOLUTIONS
1. Solution:
Unadjusted profit before tax 200,000
Write-down of inventory -
Impairment of asset (150,000 – 140,000) (10,000)
Unrealized gain (38,000 – 20,000) 18,000
Depreciation (60,000 x 1/4) (15,000)
Employee benefits (140,000 x 1/4) (35,000)
Adjusted profit before tax 158,000

2. Solution:
a. Sales 2,000,000
b. Cost of sales (900,000)
Gross income 1,100,000
c. Commission (5% x 2,000,000) (100,000)
d. Bad debts (30,000 - 10,000) (20,000)
e. Depreciation (2,400,000 ÷ 5) x 3/12 (120,000)
f. Insurance (80,000 x 3/12) (20,000)
g. Property tax (52,000 x 3/12) (13,000)
h. Advertising costs (100,000)
i. Staff bonuses (184,000 x 3/12) (46,000)
k. Loss on sale (60,000)
l. Repairs (24,000)
n. Rent (10,000 x 3) + [(2,000,000 – 1,800,000) x 2%] (34,000)
p. Other operating expenses (240,000)
Profit before bonus to key personnel 323,000
j. Bonus to key personnel (323,000 x 10%) (32,300)
Profit for the first quarter 290,700

3. Solution:
1st quarter 2nd quarter
Unadjusted profit before tax 1,760,000 1,840,000
Unfavorable variance (48,000) -
Newspaper advertisement cost (180 x 3/12) - (60,000)
Unrealized (loss) gain on investment (200,000) 240,000
Research and development expense (20,000) (24,000)
Foreign exchange (loss) gain (20,000) 30,000
Impairment loss (16,000) -
1,456,000 2,026,000

Unrealized loss and gain on the held for trading securities are computed as follows:
1st Qtr. 2nd Qtr.
Carrying amount before end of quarter adjustment 400,000 200,000
Fair value as of end of quarter 200,000 440,000 *
Unrealized gain (loss) for the quarter (200,000) 240,000
*Recovery exceeded previous write-down by P40,000.

Foreign exchange loss and gain on the foreign currency denominated receivable is computed as follows:
1st quarter 2nd quarter
Carrying amount before end of quarter adjustment 80,000 60,000
Page |7

Translated at current exchange rate as of end of qtr. 60,000 90,000


FOREX (loss) gain (20,000) 30,000

4. Solution:
First quarter Second quarter
a. Inventory write-down (200,000)
Reversal of write-down 200,000
b. Warranty expense (100,000) (340,000)
c. Bad debt expense (40,000) (28,000)
Net effect on profit or loss (340,000) (168,000)

a. The inventory write-down of P200,000 shall be recognized as expense in the first quarter. The
recovery of the write-down of P200,000 is a reduction to expense in the second quarter. Notice that
the reversal of write-down of inventory recognized is limited only to the amount of the previous write-
down.

b. Warranty expense in the second quarter is computed as follows:


Total warranty expense – 1st and 2nd quarters
[(2M + 2.4M) x 10%] 440,000
Warranty expense recognized in 1st quarter (2M x 5%) (100,000)
Warranty expense – 2nd quarter 340,000

c. Bad debt expense in the second quarter is computed as follows:


Allowance for doubtful accounts
10,000 1/1/20x1
Write-offs 24,000 6,000 Recoveries
40,000 Bad debt expense - 1st quarter (2M x 2%)
28,000 Bad debt expense – 2nd quarter (squeeze)
6/30/20x1 60,000

5. Solutions:
Requirement (a):
Pre-tax profit (first P400,000) 400,000
Multiply by: Tax rate applicable for the first P200,000 profit 20%
Income tax expense on the first P200,000 profit 80,000

Pre-tax profit (in excess of P400,000)


(200,000 per qtr. x 4 qtrs.) minus 400,000 500,000
Multiply by: Tax rate applicable for additional earnings 30%
Income tax expense on additional earnings 120,000

Total income tax expense for the year 200,000


Divide by: Total profit for the year (200,000 per qtr. x 4 qtrs.) 800,000
Weighted average annual income tax rate 25%
Page |8

Requirement (b):
Quarterly income tax expenses are computed as follows:
1st qtr. 2nd qtr. 3rd qtr. 4th qtr. Annual
Pretax profit 200,000 200,000 200,000 200,000 800,000
Ave. income tax rate 25% 25% 25% 25% 25%
Income tax expense 50,000 50,000 50,000 50,000 200,000
Page |1

Chapter 10
Cash to Accrual Basis of Accounting

1. An analysis of Thrift Corp.’s unadjusted prepaid expense account at December 31, 20x3,
revealed the following:
• An opening balance of ₱1,500 for Thrift’s comprehensive insurance policy. Thrift had paid an
annual premium of ₱3,000 on July 1, 20x2.
• A ₱3,200 annual insurance premium payment made July 1, 20x3.
• A ₱2,000 advance rental payment for a warehouse

Thrift leased for one year beginning January 1, 2004. In its December 31, 20x3 balance sheet, what
amount should Thrift report as prepaid expenses?
a. 5,200 b. 3,600 c. 2,000 d. 1,600

B
Solution:
Insurance paid on July 1, 20x3 3,200
Less: Expired portion (3,200 x 6/12) (1,600)
Prepaid - Dec. 31, 20x3 1,600
Advanced rental payment 2,000
Total prepayments - 12/31/x3 3,600

2. The balance in retained earnings at December 31, 2003 was ₱810,000 and at December 31, 2004
was ₱654,000. Net income for 2004 was ₱563,000. A stock dividend was declared and distributed
which increased common stock ₱225,000 and paid-in capital ₱125,000. A cash dividend was
declared and paid. The amount of the cash dividend was
a. ₱279,000. c. ₱494,000.
b. ₱369,000. d. ₱719,000.

₱810,000 + ₱563,000 – (₱225,000 + ₱125,000) – X = ₱654,000


X = ₱369,000.

3. On April 1, 2008, Ivy began operating a service proprietorship with an initial cash investment of
₱1,000. The proprietorship provided ₱3,200 of services in April and received full payment in
May. The proprietorship incurred expenses of ₱1,500 in April which were paid in June. During
May, Ivy drew ₱500 against her capital account.

What was the proprietorship's income for the two months ended May 31, 2008, under the following
methods of accounting?
Cash basis Accrual basis
a. 1,200 1,200
b. 1,700 1,700
c. 2,700 1,200
d. 3,200 1,700
Page |2

Solution:
Cash basis Accrual basis
Revenue 3,200 3,200
Expenses - (1,500)
Profit 3,200 1,700

4. Entity Co. uses the cash basis of accounting and reported income of ₱87,000 in 20x1. The
following items were considered in the computation of the cash basis net income.

Inventory, beginning 12,000


Inventory, ending 18,000
Receivables, beginning 40,000
Receivables, ending 38,000
Payables, beginning 19,000
Payables, ending 25,000

The accrual basis income is


a. 97,000 b. 73,000 c. 89,000 d. 85,000

D
Solution:
Accrual basis profit (squeeze) 85,000
Increase in inventory (6,000)
Decrease in receivables 2,000
Increase in payables 6,000
Cash basis profit 87,000

5. Information on an entity’s accounts is shown below:


Current tax payable, beg. 150,000
Current tax payable, end. 400,000
Increase in deferred tax
liability 60,000
Increase in deferred tax asset 20,000
Income tax paid 280,000

How much is the income tax expense for the period?


a. 530,000 b. 540,000 c. 570,000 d. 610,000
C
Solution:
Current tax payable
150,000 beg.
Income tax paid 280,000 530,000 Current tax expense

end. 400,000

Income tax expense 570,000


Page |3

Less: Increase in deferred tax liability (60,000)


Add: Increase in deferred tax asset 20,000
Current tax expense 530,000

“The fear of the LORD is the beginning of knowledge, but fools despise wisdom and
instruction.” (Proverbs 1:7)
- END -
Page |1

Chapter 11
PFRS for SMEs

: TRUE OR FALSE

1. An SME is not required to provide related party disclosures.


2. The accounting for stockholders’ equity does not apply to SMEs.
3. An SME is not required to present any interim financial reports.
4. An SME does not prepare a trial balance.
5. The basic accounting concepts of prudence or conservatism, stable monetary assumption, and
going concern do not apply to SMEs.
6. The FVOCI classification is not permitted under the PFRS for SMEs.
7. The accounting procedures followed by an SME is very different from the accounting
procedures applicable to large entities.
8. An entity cannot qualify as an SME if its accountant is a CPA.
9. The PFRS for SMEs does not include an option to designate financial assets at fair value through
profit or loss.
10. An SME does not recognize any share premium from issuances of share capital at above par
value.
11. An SME does not account for any events after the reporting period.
12. An SME does not capitalize any borrowing costs.
13. An SME tests for impairment annually any goodwill recognized from a business combination.
14. An SME measures non-monetary government grants either at fair value or at nominal cost.
15. An SME may recognize goodwill from an investment in associate, which shall be amortized over
its useful life.
16. An SME does not need to present a statement of cash flows.
17. An SME may disregard vesting conditions when accounting for share-based payments granted
to employees.
18. An SME measures biological assets at fair value less costs to sell only when fair value is readily
determinable without undue cost or effort.
19. An SME does not need to estimate the residual value of its depreciable assets when computing
for depreciation expense.
20. An SME does not need to account for its post-employment benefits costs.

“Gold there is, and rubies in abundance, but lips that speak knowledge are a rare
jewel.” (Proverbs 20:15)

- END -
Page |2

ANSWERS
1. FALSE 6. TRUE 11. FALSE 16. FALSE
2. FALSE 7. FALSE 12. TRUE 17. FALSE
3. TRUE 8. FALSE 13. FALSE 18. TRUE
4. FALSE 9. TRUE 14. FALSE 19. FALSE
5. FALSE 10. FALSE 15. TRUE 20. FALSE
Page |3

1. An SME shall measure a provision


a. at fair value
b. at cost
c. at the best estimate of the amount required to settle the obligation at the reporting date.
d. any of these

2. An SME shall account for compound financial instruments that it issues as follows
a. The SME shall account for compound instruments that it issues as either liability or equity,
but not both. Therefore, no allocation is necessary.
b. The SME shall allocate the proceeds received from the issuance, first to the equity
component, and the balance to the liability component.
c. The SME shall allocate the proceeds received from the issuance based on the relative fair
values of the components of the compound financial instrument.
d. The SME shall allocate the proceeds received from the issuance, first to the fair value of the
liability component without the conversion feature, and the remaining balance to the equity
component.

3. For an SME, revenue shall arise on all of the following transactions and events except
a. Sale of goods or rendering of services.
b. Construction contracts in which the entity is the contractor.
c. Use by others of entity assets yielding interest, royalties or dividends.
d. Revenue shall arise in any of these transactions.

4. SMEs account for government grants as follows (choose the incorrect statement):
a. A grant that does not impose specified future performance conditions on the recipient is
recognized in income when the grant proceeds are receivable.
b. A grant that imposes specified future performance conditions on the recipient is recognized
in income only when the performance conditions are met.
c. Grants received before the revenue recognition criteria are satisfied are recognized as a
liability.
d. Government grants are measured at the fair value of the asset received or receivable or at
nominal cost.

5. Borrowing costs that are directly attributable to the acquisition or construction of a qualifying
asset are accounted for by an SME as
a. Outright expenses
b. Capitalizable costs
c. a or b
d. a component of other comprehensive income

6. What is the effect on the financial statement elements if an SME receives goods or services in an
equity-settled share-based payment transaction?
a. Increase in assets and increase in income
b. Increase in assets and increase in liability
c. Increase in assets and increase in equity
Page |4

d. No effect on any of the financial statement elements

7. Which of the following indicators of impairment is considered an internal source of information?


a. Significant decline in market value than what is expected as a result of passage of time of
normal use.
b. Significant changes in technological, market, economic or legal environment in which the
entity operates or in the market to which an asset is dedicated.
c. Carrying amount of the net assets is more than its market capitalization.
d. Significant change with adverse effect to the entity has taken place or will take place, which
will affect expected use of asset, e.g., discontinuance, disposal, restructuring plans.

8. How do SMEs account for impairment losses on CGUs?


a. The impairment loss is charged first to goodwill and any excess is allocated to the other
assets of the CGU pro rata on the basis of the carrying amount of each asset in the CGU.
b. The impairment loss is allocated only to the other assets of the CGU pro rata on the basis of
the carrying amount of each asset in the CGU because an SME shall not recognize any
goodwill.
c. There is no accounting for impairment losses on CGUs because an SME needs only to
account for individual assets rather than groups of assets such as CGUs.
d. b and c.

9. For an SME, an impairment loss recognized for goodwill


a. shall not be reversed in a subsequent period.
b. shall be reversed in a subsequent period.
c. shall be reversed in a subsequent period, but limited only to the amount of loss previously
recognized.
d. shall not be reversed in a subsequent period in profit or loss but may be reversed through
other comprehensive income.

10. For an SME, a deductible temporary difference


a. gives rise to a deferred tax asset
b. gives rise to a deferred tax liability
c. is not recognized
d. is recognized directly in equity

11. If different tax rates apply to different levels of taxable profit, an SME shall measure its deferred
tax liability or asset using
a. the tax rates and laws that have been enacted or substantively enacted by the reporting date.
b. the tax rate(s) applicable to the period where the transactions that gave rise temporary
differences occurred.
c. the average enacted or substantively enacted rates.
d. any of these.

12. If the deferred tax asset is not wholly recoverable, the SME shall
a. recognize a valuation allowance reducing the carrying amount of the deferred tax asset.
Page |5

b. recognize a valuation allowance increasing the carrying amount of the deferred tax asset.
c. recognize a corresponding increase in deferred tax liability to decrease the net amount of
deferred taxes.
d. a or c.

13. According to the PFRS for SMEs, foreign activities can be conducted
a. by entering into foreign currency transactions
b. by having foreign operations
c. a or b
d. the PFRS for SMEs does not address this matter.

14. An entity’s functional currency is


a. the currency of the primary economic environment in which the entity operates.
b. the currency in which the entity uses in presenting its financial statements.
c. the currency required by regulatory agencies to be used in financial statements filed by the
entity.
d. all of these.

15. When an SME’s functional currency is the currency of a hyperinflationary economy,


a. all amounts in the financial statements shall be stated in terms of the measuring unit current
at the end of the reporting period.
b. some amounts in the financial statements shall be stated in terms of the measuring unit
current at the end of the reporting period.
c. all amounts for monetary assets and liabilities in the financial statements shall be restated
using the general price index, rather than the specific price index.
d. none of these. The PFRS for SMEs does not address this matter.

16. How does an SME account for events after the reporting period that provide evidence of
conditions that existed at the end of the reporting period?
a. As adjusting events after the end of the reporting period.
b. As disclosures only in the notes.
c. Either a or b as an accounting policy choice.
d. None of these. The PFRS for SMEs does not require SMEs to identify and account for events
after the reporting period.

17. If an SME declares dividends to holders of its equity instruments after the end of the reporting
period, the SME
a. shall not recognize those dividends as a liability at the end of the reporting period.
b. may recognize those dividends as a liability at the end of the reporting period.
c. may present the dividends declared as a segregated component of retained earnings at the
end of the reporting period.
d. a and b

18. Relationships between a parent and its subsidiaries


Page |6

a. shall be disclosed by an SME irrespective of whether there have been related party
transactions.
b. shall be disclosed by an SME only if there have been related party transactions.
c. is not disclosed by an SME because an SME cannot be a parent. If the SME is a subsidiary, it
is required to use the full PFRSs as these are the required standards for use by the SME’s
parent.
d. a or b as a matter of accounting policy choice.

19. The biological assets of an SME is measured using


a. The fair value model for those biological assets for which fair value is readily determinable
without undue cost or effort.
b. The cost model for those biological assets for which fair value is not readily determinable
without undue cost or effort.
c. a or b
d. The biological assets classification is not applicable to SMEs.

20. For an SME, changes in the fair value less costs to sell of biological assets are
a. recognized in profit or loss
b. recognized in other comprehensive income
c. not recognized
d. a or b

“The discerning heart seeks knowledge, but the mouth of a fool feeds on folly.”
(Proverbs 15:14)
- END -
Page |7

ANSWERS
1. C 11. C
2. D 12. A
3. D 13. C
4. D 14. A
5. A 15. A
6. C 16. A
7. D 17. D
8. A 18. A
9. A 19. C
10. A 20. A
1. The standard that addresses the accounting for revenues is
a. PFRS 16.
b. PFRS 18.
c. PFRS 5.
d. PFRS 15.

2. The objective of PAS 1 Presentation of Financial Statements is


a. to provide the basic principles in the presentation of general purpose financial statements to
improve comparability.
b. to provide the basic principles in the presentation of general and special purpose financial
statements to improve comparability.
c. to provide the basic principles in the presentation of general purpose financial statements to
improve consistency.
d. all of these

3. The heading of a financial statement most likely will not include


a. the name of the reporting entity.
b. the title of the financial statement.
c. the date of the financial statement.
d. the name(s) of the business owner(s).

4. According to PAS 1, an asset shall be classified as current when it satisfies any of the following
criteria, except
a. it is expected to be realized in, or is intended for sale or consumption in, the entity’s normal
operating cycle
b. it is held primarily for the purpose of being traded
c. it is expected to be realized within twelve months after the balance sheet date
d. it is cash or a cash equivalent that is restricted

5. A liability shall be classified as current when it satisfies any of the following criteria, except
a. it is expected to be settled in the entity’s normal operating cycle
b. it is held primarily for the purpose of being traded
c. it is due to be settled within twelve months after the balance sheet date
d. the entity has an unconditional right to defer settlement of the liability for at least twelve
months after the balance sheet date.

6. If an entity expects, and has the discretion, to refinance or roll over an obligation for at least
twelve months after the balance sheet date under an existing loan facility, it classifies the
obligation as non-current,
a. even if it would otherwise be due within a shorter period.
b. even if the original term was for a period longer than twelve months
c. even if an agreement to refinance, or to reschedule payments, on a long-term basis is
completed after the reporting period and before the financial statements are authorized for
issue
d. choices b and c
7. When an entity breaches an undertaking under a long-term loan agreement on or before the end
of the reporting period with the effect that the liability becomes payable on demand, (choose the
incorrect statement)
a. The liability is classified as current, even if the lender has agreed, after the balance sheet date
and before the authorization of the financial statements for issue, not to demand payment as
a consequence of the breach.
b. The liability is classified as current because, at the balance sheet date, the entity does not
have an unconditional right to defer its settlement for at least twelve months after that date.
c. The liability is classified as non-current, even if the lender has agreed, after the balance sheet
date and before the authorization of the financial statements for issue, not to demand
payment as a consequence of the breach.
d. The liability is normally classified as current; however, the liability is classified as non-
current if the lender agreed by the balance sheet date to provide a period of grace ending at
least twelve months after the balance sheet date, within which the entity can rectify the
breach and during that period the lender cannot demand immediate repayment.

8. Material Omissions or misstatements of items are material if they could, individually or


collectively; influence the economic decisions of users taken on the basis of the financial
statements. Materiality depends on
a. the peso amount and degree of financial consequence of the omission or misstatement
judged in the surrounding circumstances
b. the size and peso amount of the omission or misstatement judged in the surrounding
circumstances
c. the peso amount and nature of the omission but not the misstatement judged in the
surrounding circumstances
d. the size and nature of the omission or misstatement judged in the surrounding
circumstances

9. In the extremely rare circumstances in which management concludes that compliance with a
requirement in a Standard or an Interpretation would be so misleading that it would conflict
with the objective of financial statements set out in the Framework, the entity shall depart from
that requirement in the manner set under PAS 1. When an entity departs from a requirement of a
Standard or an Interpretation, it shall disclose: (choose the incorrect statement)
a. that management has concluded that the financial statements present fairly the entity’s
financial position, financial performance and cash flows;
b. that it has complied with applicable Standards and Interpretations, except that it has
departed from a particular requirement to achieve a fair presentation;
c. that it has complied with other applicable standards other than those issued by FRSC or
IASB and the description of those accounting standards which the entity has complied to.
d. the title of the Standard or Interpretation from which the entity has departed, the nature of
the departure, including the treatment that the Standard or Interpretation would require, the
reason why that treatment would be so misleading in the circumstances that it would
conflict with the objective of financial statements set out in the Framework, and the treatment
adopted; and
e. for each period presented, the financial impact of the departure on each item in the financial
statements that would have been reported in complying with the requirement.
10. Identify the incorrect statement.
a. When an entity has departed from a requirement of a Standard or an Interpretation in a prior
period, and that departure affects the amounts recognized in the financial statements for the
current period, it shall disclose the (a) title of the Standard or Interpretation from which the
entity has departed and the (b) impact of such departure.
b. In the extremely rare circumstances in which management concludes that compliance with a
requirement in a Standard or an Interpretation would be so misleading that it would conflict
with the objective of financial statements set out in the Framework, but the relevant regulatory
framework prohibits departure from the requirement, the entity shall, to the maximum extent
possible, reduce the perceived misleading aspects of compliance by disclosing:(a) the title of
the Standard or Interpretation in question and (b) for each period presented, the adjustments
to each item in the financial statements that management has concluded would be necessary
to achieve a fair presentation.
c. Financial statements shall be prepared on a going concern basis unless management either
intends to liquidate the entity or to cease trading, or has no realistic alternative but to do so.
d. PAS 1 requires an entity preparing financial statements, to make an assessment of the
entity’s ability to continue as a going concern. In assessing whether the going concern
assumption is appropriate, management takes into account all available information about
the future, which is at least, but is not limited to, five years from the balance sheet date.

11. Identify the incorrect statement.


a. The final stage in the process of aggregation and classification is the presentation of
condensed and classified data, which form line items on the face of the financial statements.
b. PAS 1 sometimes uses the term ‘disclosure’ in a broad sense, encompassing items presented
on the face of the balance sheet, statement of profit or loss and other comprehensive income,
statement of changes in equity and cash flow statement, as well as in the notes.
c. Applying the concept of materiality means that a specific disclosure requirement in a
Standard or an Interpretation need not be satisfied if the information is not material.
d. An entity shall prepare its financial statements, including cash flow information, using the
accrual basis of accounting.
e. PAS 1 requires an entity presenting its current year financial statements to also present its
financial statements for the previous year.

12. The ledger of SCHOLIAST COMMENTATOR Co. as of December 31, 20x1 includes the
following:
Assets
Cash 10,000
Trade accounts receivable (net of ₱10,000 credit balance in accounts) 40,000
Held for trading securities 80,000
Financial assets designated at FVPL 30,000
Investment in equity securities at FVOCI 70,000
Investment in bonds measured at amortized cost (due in 3 years) 60,000
Prepaid assets 10,000
Deferred tax asset (expected to reverse in 20x2) 12,000
Investment in Associate 36,000
Investment property 46,000
Sinking fund 38,000
Property, plant, and equipment 100,000
Goodwill 28,000
Totals 560,000

How much is the total current assets?


a. 220,000
b. 180,000
c. 340,000
d. 164,000

Solution:
Current assets
Cash 10,000
Trade accounts receivable (40,000 + 10,000) 50,000
Held for trading securities 80,000
Financial assets designated at FVPL 30,000
Prepaid assets 10,000
Total current assets 180,000

13. The ledger of PERNICIOUS DEADLY Co. as of December 31, 20x1 includes the following:
Liabilities
Bank overdraft 10,000
Trade accounts payable (net of ₱10,000 debit balance in accounts) 40,000
Notes payable (due in 20 semi-annual payments of ₱4,000) 80,000
Interest payable 30,000
Bonds payable (due on March 31, 20x2) 70,000
Discount on bonds payable (30,000)
Dividends payable 10,000
Share dividends payable 12,000
Deferred tax liability (expected to reverse in 20x2) 36,000
Income tax payable 44,000
Contingent liability 100,000
Reserve for contingencies 28,000
Totals 430,000

How much is the total current liabilities?


a. 192,000
b. 186,000
c. 212,000
d. 178,000

Solution:
Current liabilities
Bank overdraft 10,000
Trade accounts payable (P20,000 + P5,000) 50,000
Notes payable (P2,000 semi-annual instalment x 2) 8,000
Interest payable 30,000
Bonds payable (due on March 31, 20x2) 70,000
Discount on bonds payable (30,000)
Dividends payable 10,000
Income tax payable 44,000
Total current liabilities 192,000

14. The ledger of CALLOW IMMATURE Co. in 20x1 includes the following:
Share capital 200,000
Share premium 40,000
Retained earnings, appropriated 36,000
Retained earnings, unappropriated 84,000
Revaluation surplus 60,000
Remeasurements of the net defined benefit liability (asset) - gain 30,000
Cumulative net unrealized gain on fair value changes of investment
in FVOCI 46,000
Effective portion of losses on hedging instruments in a cash
flow hedge 20,000
Cumulative translation loss on foreign operation 10,000
Treasury shares, at cost 26,000

How much is the total shareholders’ equity?


a. 460,000
b. 440,000
c. 420,000
d. 390,000

Solution:
Share capital 200,000
Share premium 40,000
Retained earnings, appropriated 36,000
Retained earnings, unappropriated 84,000
Revaluation surplus 60,000
Remeasurements of the net defined benefit liability (asset)
- gain 30,000
Cumulative net unrealized gain on fair value
changes of investment in FVOCI 46,000
Effective portion of losses on hedging instruments in a
cash flow hedge (20,000)
Cumulative translation loss on foreign operation (10,000)
Treasury shares, at cost (26,000)
Total shareholders' equity 440,000

Use the following information for the next two questions:


15. GUILE DECEITFULNESS Co. was incorporated on January 1, 20x1. The following were the
transactions during the year:
- Total consideration from share issuances amounted to ₱2,000,000.
- A land and building were acquired through a lump sum payment of ₱400,000. A mortgage
amounting to ₱100,000 was assumed on the land and building.
- Total payments of ₱80,000 were made during the year on the mortgage assumed on the land and
building, The payments are inclusive of interest amounting to ₱10,000.
- Additional capital of ₱200,000 was obtained through bank loans. None of the bank loans were
paid during the year. Half of the bank loans required a secondary mortgage on the land and
building.
- There is no accrued interest as of year-end.
- Dividends declared during the year but remained unpaid amounted to ₱60,000.
- No other transactions during the year affected liabilities.
- Retained earnings as of December 31, 20x1 is ₱120,000.

16. How much is the profit for the year?


a. 120,000
b. 160,000
c. 180,000
d. 220,000

Retained earnings
- Jan. 1, 20x1
Dividends 60,000 180,000 Profit for the year (squeeze)
Dec. 31, 20x1 120,000

17. How much is the total assets as of December 31, 20x1?


a. 2,410,000
b. 2,520,000
c. 2,380,000
d. 2,420,000

Asset = Liabilities + Equity

Mortgage assumed on land and building 100,000


Principal payment on the mortgage (80K – 10K interest) ( 70,000)
Bank loans 200,000
Dividends payable 60,000
Liabilities, Dec. 31, 20x1 290,000

Share issuances 2,000,000


Retained earnings – Dec. 31, 20x1 120,000
Equity, Dec. 31, 20x1 2,120,000
Total assets, Dec. 31, 20x1 (liabilities + equity) 2,410,000

18. The ledger of DEROGATORY DEGRADING Co. in 20x1 includes the following:
Cash 200,000
Accounts receivable 400,000
Inventory 1,000,000
Accounts payable 300,000
Note payable 100,000
During the audit of DEROGATORY’s 20x1 financial statements, the following were noted by the
auditor:
- Cash sales in 20x2 amounting to ₱20,000 were inadvertently included as sales in 20x1.
DEROGATORY recognized gross profit of ₱6,000 on the sales.
- A collection of a ₱40,000 accounts receivable in 20x2 was recorded as collection in 20x1. A cash
discount of ₱2,000 was given to the customer.
- During January 20x2, a short-term bank loan of ₱50,000 obtained in 20x1 was paid together with
₱5,000 interest accruing in January 20x2. The payment transaction in 20x2 was inadvertently
included as 20x1 transaction.

How much is the adjusted working capital as of December 31, 20x1?


a. 1,651,000
b. 1,014,000
c. 1,450,000
d. 1,201,000

Solution:
The adjusted balance of cash is computed as follows:
Cash (unadjusted) 200,000
Cash sales in 20x2 recorded as 20x1 sale (20,000)
Collection of account in 20x2 recorded as 20x1 collection
(40,000 account less 2,000 cash discount) (38,000)
Loan payment in 20x2 recorded as 20x1 transaction 50,000
Interest payment in 20x2 recorded as 20x1 transaction 5,000
Adjusted cash balance, Dec. 31, 20x1 197,000

The adjusted balance of accounts receivable is computed as follows:


Accounts receivable (unadjusted) 400,000
Collection of account in 20x2 recorded as 20x1 collection 40,000
Adjusted accounts receivable balance, Dec. 31, 20x1 440,000

The adjusted balance of inventory is computed as follows:


Inventory (unadjusted) 1,000,000
Cost of cash sale in 20x2 recorded as 20x1 sale
(20,000 sale - 6,000 gross profit) 14,000
Adjusted inventory balance, Dec. 31, 20x1 1,014,000

Adjusted current assets, Dec. 31, 20x1: (197K + 440K + 1,014K) = 1,651,000

The adjusted current liabilities are computed as follows:


Accounts payable 300,000
Note payable 100,000
Loan payable 50,000
Adjusted current liabilities, Dec. 31, 20x1 450,000

Working capital, Dec. 31, 20x1 = Current assets – Current liabilities


Working capital, Dec. 31, 20x1 = (1,651,000 – 450,000) = 1,201,000

19. According to PAS 1 Presentation of Financial Statements, expenses are presented using
a. Nature of expense method
b. Function of expense method
c. a or b
d. Classified and Unclassified

Use the following information for the next two questions:


Anne Jeng Inc.’s accounts show the following balances:

Cost of goods sold ₱320,000


Insurance expense 75,000
Advertising expense 25,000
Freight-out 30,000
Loss on sale of equipment 7,000
Rent expense (one-half pertains sales department) 80,000
Salaries expense (1/4 pertains to non-sales personnel) 150,000
Sales commission expense 10,000
Bad debts expense 5,000
Interest expense 5,000

20. How much is the total distribution costs (selling expenses)?


a. 198,000
b. 210,500
c. 217,500
d. 221,500

Advertising expense 25,000


Freight-out 30,000
Rent expense (80K x 1/2) 40,000
Salaries expense (150K x 3/4) 112,500
Sales commission expense 10,000
Distribution costs 217,500

21. How much is the total administrative expenses?


a. 157,500
b. 156,500
c. 147,500
d. 175,500

Insurance expense 75,000


Rent expense (80K x 1/2) 40000
Salaries expense (150K x 1/4) 37500
Bad debts expense 5,000
Administrative expenses 157,500
22. Entity A has the following information:

Inventory, beg. 80,000


Inventory, end. 128,000
Purchases 320,000
Freight-in 16,000
Purchase returns 8,000
Purchase discounts 11,200

How much is Entity A’s cost of sales?


a. 286,800
b. 292,800
c. 288,600
d. 268,800

Inventory, beg. 80,000


Net purchases:
Purchases 320,000
Freight-in 16,000
Purchase returns (8,000)
Purchase discounts (11,200) 316,800
Total goods available for sale 396,800
Less: Inventory, end. (128,000)
Cost of goods sold 268,800

23. A correct dating of financial statements is


Statement of financial position Statement of comprehensive income
a. as of a point in time for a period of time
b. for a period of time as of a point in time
c. for a period of time for a period of time
d. time after time time and time again

24. Which of the following is considered revenue?


a. gain on sale of equipment
b. service fees
c. other income
d. other comprehensive income

25. Which of the following items is likely to be presented in the statement of comprehensive income
of a merchandising business but not of a service business?
a. Service fees c. Cost of sales
b. Salaries expense d. Income tax expense
26. A statement of comprehensive income that presents cost of sales separately from other expenses
is prepared under the
a. single-step method. c. multi-step method.
b. single-presentation. d. two-statement presentation.

27. In a two-statement presentation, information on profit or loss and other comprehensive income
is shown
a. in two separate statements, a statement of profit or loss and a statement showing other
comprehensive income.
b. in two separate statements, a statement of profit or loss and an income statement.
c. in two separate statements, a single-step statement and a multi-step statement.
d. in a single statement called “statement of comprehensive income.”

28. Expenses are presented in the statement of comprehensive income using


a. nature of expense method.
b. function of expense method.
c. single or two-statement method.
d. a or b

29. Under this presentation method, expenses are presented in the statement of comprehensive
income without distinctions as to their functions within the entity.
a. nature of expense method
b. function of expense method
c. single-statement presentation
d. two-statement presentation

30. Under this presentation, expenses are classified as either operating or non-operating item. At a
minimum, cost of sales is presented separately.
a. nature of expense method
b. function of expense method
c. single-statement presentation
d. two-statement presentation

31. In a statement of comprehensive income showing expenses according to their function, which of
the following is included in the line item “Distribution costs” or “Selling costs?”
a. Insurance expense c. Freight-in
b. Legal and accounting fees d. Advertising expense

32. In a statement of comprehensive income showing expenses according to their function, which of
the following is included in the line item “Administrative expenses?”
a. Salaries of sales personnel
b. Cost of sales
c. Freight-out
d. Legal and accounting fees

Use the following information for the next five questions:


The nominal accounts of Rommel SP Corp. on December 31, 20x1 have the following balances:
Accounts Dr. Cr.
Sales ₱739,000
Interest income 45,000
Gains 15,000
Inventory, beg. ₱65,000
Purchases 180,000
Freight-in 10,000
Purchase returns 5,000
Purchase discounts 9,000
Freight-out 30,000
Sales commission 45,000
Advertising expense 25,000
Salaries expense 240,000
Rent expense 30,000
Depreciation expense 50,000
Utilities expense 25,000
Supplies expense 15,000
Transportation and travel expense 15,000
Insurance expense 10,000
Taxes and licenses 60,000
Interest expense 5,000
Miscellaneous expense 3,000
Loss on the sale of equipment 5,000

Additional information:
a. Ending inventory is ₱90,000.
b. One-fourth of the salaries, rent, and depreciation expenses pertain to the non-sales department.
The sales department does not share in the other expenses.

33. How much is the net purchases?


a. ₱185,000 c. ₱194,000
b. ₱176,000 d. ₱192,000

Purchases 180,000
Freight-in 10,000
Purchase returns (5,000)
Purchase discounts (9,000)
Net purchases 176,000

34. How much is the “change in inventory” in 20x1?


a. ₱90,000 increase c. ₱25,000 decrease
b. ₱65,000 decrease d. ₱25,000 increase

Inventory, beg. 65,000


Inventory, end. 90,000
Change in inventory – increase (25,000)

35. How much is the cost of goods sold?


a. ₱151,000 c. ₱169,000
b. ₱95,000 d. ₱127,000

Net purchases 176,000


Less: Net increase in inventory (25,000)
Cost of sales 151,000

36. How much is the total selling expense?


a. ₱420,000 c. ₱180,000
b. ₱260,000 d. ₱340,000

Freight-out 30,000
Sales commission 45,000
Advertising expense 25,000
Salaries expense (240K x 3/4) 180,000
Rent expense (30K x 3/4) 22,500
Depreciation expense (50K x 3/4) 37,500
Selling expenses/Distribution costs 340,000

37. How much is the total general and administrative expense?


a. 280,000 c. 330,000
b. 320,000 d. 208,000

Salaries expense (240K x 1/4) 60,000


Rent expense (30,000 x 1/4) 7,500
Depreciation expense (50K x 1/4) 12,500
Utilities expense 25,000
Supplies expense 15,000
Transportation and travel expense 15,000
Insurance expense 10,000
Taxes and licenses 60,000
Miscellaneous expense 3,000
Administrative expenses 208,000

38. One of the conditions that must be satisfied in order to recognize revenue in a transaction
involving the rendering of services over a contractual period is that the stage of completion of
the transaction at the end of the reporting period can be measured reliably. Which of the
following methods for determining the stage of completion of a contract involving the rendering
of services are specifically referred to in PFRS 15 as being acceptable?
I. Costs incurred to date as a percentage of the estimated total costs of the transaction
II. Advances received to date as a percentage of the total amount receivable
III. Surveys of work performed
IV. Revenue to date divided by total contract revenue
a. I, III, IV b. I, III c. I, II, IV d. I, II, III

39. The Grand Company placed an order with The Little Company for new specialist machinery.
The order was non-cancellable once signed and Grand agreed to pay for the machinery at the
time the order was signed on 1 February 20X7. Little held the machinery to Grand's order from 1
June 20X7, the date on which it was completed. Grand commenced using the machinery on 1
August 20X7 when Little completed the installation process. The installation is not distinct. Little
had staff on standby to deal with any operating problems until the warranty period ended on 1
November 20X7. The warranty does not provide service in addition to assurance that the
machinery complies with agreed-upon specifications. Under PFRS15 Revenue, Little should
recognize the revenue from the sale of this specialist machinery on
a. 1 February 20X7 c. 1 August 20X7
b. 1 June 20X7 d. 1 November 20X7

40. Which is incorrect concerning recognition of revenue?


a. Revenue from rendering of services over an extended contractual period shall be recognized
by reference to the stage of completion of the transaction at balance sheet date.
b. Interest revenue shall be recognized on a time proportion basis that does not take into
account the effective yield on the asset.
c. Royalty revenue shall be recognized on an accrual basis in accordance with the substance of
the relevant agreement,
d. Dividend revenue shall be recognized when the stockholder’s right to receive payment is
established.

41. In a normal sale, generally the most uncertain factor in the revenue recognition process is
a. the seller's fulfillment of its responsibility in the transaction
b. the measurability of the resource or item received by the seller
c. the realizability of the resource or item received by the seller
d. the relevance of the resource or item received by the seller

42. Which of the following methods of service revenue recognition usually would be most
appropriate for a business engaged in packing, loading, transporting and delivering freight
(where each of the processes is an input to a combined output specified by the customer)?
a. Proportional performance method (i.e., over time as the entity progresses towards the
complete satisfaction of the performance obligation)
b. Completed performance method (i.e., at a point in time when the entity completes the output
specified in the contract)
c. Specific performance method (i.e., when the customer pays for the completion of a single
specific activity)
d. Collection method (i.e., when cash is collected)

43. An entity is a large manufacturer of machines. A major customer has placed an order for a
special machine for which it has given a deposit to the entity. The parties have agreed on a price
for the machine. As per the terms of the sale agreement, it is FOB (tree on board) contract and
the title passes to the buyer when goods are loaded into the ship at the port. When should the
revenue be recognized by the entity?
a. When the customer orders the machine.
b. When the deposit is received.
c. When the machine is loaded on the port.
d. When the machine has been received by the customer.

44. A company manufacturing and selling consumable products has come out with an offer to
refund the cost of purchase within one month of sale if the customer is not satisfied with the
product. When should the company recognize the revenue?
a. When goods are sold to the customers.
b. After one month of sale.
c. Only if goods are not returned by the customers after the period of one month.
d. At the time of sale along with an offset to revenue for the refund liability for the products
expected to be returned.

45. A computer chip manufacturing company sells its products to its distributors for onward sales
to the ultimate customers. Due to frequent fluctuations in the market prices for these goods, the
company has a “price protection” clause in the distributor agreement that entitles it to raise
additional billings in case of upward price movement, Another clause in the distributor’s
agreement is that the company can at any lime reduce its inventory by buying back goods at the
cost at which it sold the goods to the distributors. Distributors pay for the goods within 60 days
from the sale of goods to them. When should the company recognize revenue on sale of goods to
the distributors?
a. When the goods are sold to the distributors.
b. When the distributors pay to the company the cost of the goods.
c. When goods are sold to the distributors provided estimated additional revenue is also
booked under the “protection clause” based on past experience,
d. When the distributors sell goods to the ultimate customers and there is no uncertainty with
respect to the “price protection” clause or the buyback of goods.

46. An entity manufactures and sells standard machinery. One of the conditions in the sale contract
is that installation of machinery will be undertaken by the entity. During December of the
current year, the entity received a special onetime contract from a customer to manufacture,
install and maintain customized machinery. It is the first time the entity will be producing this
kind of machinery, and it is expecting numerous changes that would need to be made to the
machine after the installation is completed, which one period is described in the contract of sale
as the “maintenance period.” The maintenance services are an input to a combined output
specified in the contract. The total cost of making the changes during the maintenance period
cannot be reasonably estimated at the time of the installation. Costs incurred are not recoverable
if, during the maintenance period, the machinery is discovered as non-compliant with agreed-
upon specifications and the non-compliance is beyond remediation. The customer shall signify
its acceptance of the machinery at the end of the maintenance period. When should revenue
from the sale of the special machine most likely be recognized?
a. When the machinery is produced.
b. When the machinery is produced and delivered.
c. When the installation is complete
d. When the maintenance period as per the contract of sale expires.

47. Revenue is recognized at the time of sale under the:


a. cost recovery method (i.e., the outcome of a performance obligation cannot be reasonably
measured but the entity expects to recover the costs incurred in satisfying the performance
obligation)
b. collection method (i.e., when cash is collected)
c. percentage-of-completion method (i.e., the performance obligation is satisfied over time)
d. sales method when goods are sold on credit (i.e., the performance obligation is satisfied
when the goods are transferred to the customer).

The next three items are based on the following information:


Lake Corporation’s accounting records showed the following investments at January 1, 20x3:
Ordinary shares:
Kar Corp. (1,000 shares) 10,000
Aub Corp. (5,000 shares) 100,000

Real estate:
Parking lot (leased to Day Co.) 300,000

Other:
Trademark (at cost, less accumulated amortization) 25,000
Total investments 435,000

Lake owns 1% of Kar and 30% of Aub. The Day lease, which commenced on January 1, 20x1, is for
ten years, at an annual rental of ₱48,000. In addition, on January 1, 20x1, Day paid a nonrefundable
deposit of ₱50,000, as well as a security deposit of ₱8,000 to be refunded upon expiration of the lease.
The trademark was licensed to Barr Co. for royalties of 10% of sales of the trademarked items.
Royalties are payable semiannually on March 1 (for sales in July through December of the prior
year), and on September 1 (for sales in January through June of the same year).

During the year ended December 31, 20x3, Lake received cash dividends of ₱1,000 from Kar, and
₱15,000 from Aub, whose 20x3 net incomes were ₱75,000 and ₱150,000, respectively. Lake also
received ₱48,000 rent from Day in 20x3 and the following royalties from Barr:

March 1 September 1
20x2 3,000 5,000
20x3 4,000 7,000

Barr estimated that sales of the trademarked items would total ₱20,000 for the last half of 20x3.

48. In Lake’s 20x3 income statement, how much should be reported for dividend revenue?
a. 16,000
b. 2,400
c. 1,000
d. 150
C 1,000 – the dividend from the 1% investment. The dividend from the 30% investment is not dividend
income but rather a deduction to the carrying amount of the investment in associate (significant influence
is presumed to exist).

49. In Lake’s 20x3 income statement, how much should be reported for royalty revenue?
a. 14,000
b. 13,000
c. 11,000
d. 9,000
D
Solution:
Royalty revenue for Jan. to June, 20x3
(received on Sept. 20x3) 7,000
Royalty revenue for July to Dec., 20x3 (20,000 x 10%) 2,000
Total royalty revenue 9,000

50. In Lake’s 20x3 income statement, how much should be reported for rental revenue?
a. 43,000
b. 48,000
c. 53,000
d. 53,800

C
Solution:
Annual rental 48,000
Amortization of nonrefundable deposit (50K ÷ 10 yrs.) 5,000
Total rental revenue 53,000

Use the following information for the next two questions:


DECORTICATE PEEL, Inc. is committed to a plan to sell a manufacturing facility and has initiated
actions to locate a buyer. As of this date, the building has a carrying amount of ₱6,000,000, a fair
value of ₱5,000,000 and estimated costs to sell of ₱200,000. At the plan commitment date, there is a
backlog of uncompleted customer orders.

51. DECORTICATE, Inc. intends to sell the manufacturing facility with its operations. Any
uncompleted customer orders at the sale date will be transferred to the buyer. The transfer of
uncompleted customer orders at the sale date will not affect the timing of the transfer of the
facility. How should DECORTICATE Co. classify the manufacturing facility?
a. Included under property, plant and equipment at ₱6,000,000.
b. Included under property, plant and equipment at ₱4,800,000.
c. Classified as held for sale at ₱6,000,000
d. Classified as held for sale at ₱4,800,000

D (5,000,000 fair value – 200,000 costs to sell) = 4,800,000


52. DECORTICATE, Inc. intends to sell the manufacturing facility, but without its operations. The
entity does not intend to transfer the facility to a buyer until after it ceases all operations of the
facility and eliminates the backlog of uncompleted customer orders. How should
DECORTICATE Co. classify the manufacturing facility?
a. Included under property, plant and equipment at ₱6,000,000.
b. Included under property, plant and equipment at ₱4,800,000.
c. Classified as held for sale at ₱6,000,000
d. Classified as held for sale at ₱4,800,000

B – not available for immediate sale in its present condition; PPE at 4.8M (5M – 200K) because the
manufacturing facility is impaired.

53. An entity in the power generating industry is committed to a plan to sell a disposal group that
represents a significant portion of its regulated operations. The sale requires regulatory
approval, which could extend the period required to complete the sale beyond one year. Actions
necessary to obtain that approval cannot be initiated until after a buyer is known and a firm
purchase commitment is obtained. However, a firm purchase commitment is highly probable
within one year. The disposal group has a carrying amount of ₱10,000,000 and fair value less
costs to sell of ₱10,600,000. How should the entity classify the disposal group?
a. Held for sale, ₱10.6M c. Under previous classifications, ₱10M
b. Held for sale, ₱10M d. Under previous classifications, ₱10.6M

B – The exceptions to the “1-yr. requirement” are met.

Use the following information for the next two questions:


In 20x1, FORGETIVE CREATIVE Co. classified a property as held for sale. The carrying amount
prior to classification is ₱400,000 while fair value less cost to sell is ₱360,000. The property is being
sold at ₱360,000.

During 20x1, the market conditions that existed at the date the asset was classified initially as held
for sale deteriorate and, as a result, the asset is not sold by the end of that period. During that
period, FORGETIVE actively solicited but did not receive any reasonable offers to purchase the asset
and, in response, FORGETIVE reduced the price from ₱360,000 to ₱320,000. The fair value less costs
to sell on December 31, 20x1 is ₱340,000.

54. How should FORGETIVE Co. classify the property in its 20x1 annual financial statements?
a. Held for sale, ₱320,000 c. PPE, ₱340,000
b. Held for sale, ₱340,000 d. PPE, ₱400,000

B 340,000, the fair value less costs to sell, which is lower than the carrying amount of P360,000.

55. During 20x2, the market conditions deteriorate further, and the asset is not sold by December 31,
20x2. FORGETIVE Co. believes that the market conditions will improve and has not further
reduced the price of the asset. The fair value less costs to sell on December 31, 20x2 is ₱300,000. If
the property was not classified as held for sale in 20x1, its carrying amount by this time would
have been ₱350,000.
a. Held for sale, ₱300,000 c. PPE, ₱300,000
b. Held for sale, ₱320,000 d. PPE, ₱350,000

C – The asset is reclassified back to PPE at the lower of recoverable amount (i.e., 300,000) and the
carrying amount adjusted for depreciation not recognized during the asset was classified as held for sale
(i.e., 350,000).

56. WAYFARER TRAVELER Co. is preparing its December 31, 20x1, current year financial
statements. A land included in WAYFARER’s property, plant and equipment that did not
qualify as held for sale as of December 31, 20x1 was actually sold on January 5, 20x2. The
financial statements were authorized for issue on March 1, 20x2. On December 31, 20x1,
WAYFARER has total current assets of ₱9,000,000. Not included in this amount is the fair value
less costs to sell of the land amounting to ₱1,000,000. How much is the total current assets
current in WAYFARER’s December 31, 20x1 financial statements?
a. ₱8,000,000 c. ₱10,000,000
b. ₱9,000,000 d. ₱11,000,000

B – The event is disclosed only as a non-adjusting event after the reporting period.

57. On December 31, 20x1, STRIDENT HARSH-SOUNDING Co. classified its building with a
historical cost of ₱4,000,000 and accumulated depreciation of ₱2,400,000 as held for sale. All of
the criteria under PFRS 5 are complied with. On that date, the land has a fair value of ₱1,400,000
and cost to sell of ₱80,000. The entry on December 31, 20x1 includes
a. a debit to building for ₱1,320,000
b. a credit to accumulated depreciation for ₱2,400,000
c. a debit to impairment loss for ₱280,000
d. No reclassification entry will be made on December 31, 20x1

C
Jan. 1, Held for sale asset (1.4M – 80K) 1,320,000
20x1
Accumulated depreciation 2,400,000
Impairment loss 280,000
Building 4,000,000

58. On December 31, 20x1, OBSTINACY STUBBORNESS Co. classified its building with a carrying
amount of ₱1,600,000 and fair value less cost to sell of ₱1,320,000 as held for sale.

The building was not sold in 20x2. However, the exception to the one-year requirement was met. On
December 31, 20x2, the fair value less cost to sell of building is ₱1,240,000.

The building was not sold in 20x3. However, the exception to the one-year requirement was still
met. On December 31, 20x3, the fair value less cost to sell of building increased to ₱1,680,000. How
much is the gain on reversal of impairment to be recognized on December 31, 20x3?
a. 440,000
b. 360,000
c. 280,000
d. 0
B 360,000, limited to the total impairment losses recognized in previous years (1,240,000 - 1,600,000
original carrying amount)

Use the following information for the next four questions:


On December 31, 20x1, INSOUCIANT CAREFREE Co. plans to dispose of a group of its assets.
Information on these assets is shown below:
Carrying amount on Dec. Carrying amount as
31, 20x1 before remeasured immediately
classification as held for before classification as
sale held for sale
Inventory 9,600,000 8,800,000
Investment in FVOCI 7,200,000 6,000,000
Investment property (at cost model) 22,800,000 22,800,000
PPE (at cost model) 18,400,000 16,000,000
Goodwill 6,000,000 6,000,000
Total 64,000,000 59,600,000

INSOUCIANT Co. entity estimates that the fair value less costs to sell of the disposal group amounts
to ₱52,000,000.

59. How would the reduction in the value of the assets on classification as held for sale be treated in
the financial statements?
a. The entity recognizes a loss of ₱4.4M immediately before classification as held for sale and
then recognizes an impairment loss of ₱7.6M.
b. The entity recognizes an impairment loss of ₱12 million.
c. The entity recognizes an impairment loss of ₱7.6M.
d. The entity recognizes a loss of ₱12M immediately before classifying the disposal group as
held for sale.

A Step #1: 59.6M – 64M = 4.4M Impairment loss;


Step #2: 52M – 59.6M = 7.6 Additional impairment loss

60. How much is the carrying amount of the inventory after classification of the disposal group as
held for sale?
a. 8,800,000
b. 7,950,576
c. 7,899,324
d. 7,765,391

A 8,800,000 - Carrying amount as remeasured immediately before classification as held for sale. (See also
solutions below)

61. How much is the carrying amount of the Investment property (at cost model) after classification
of the disposal group as held for sale?
a. 22,800,000
b. 21,859,794
c. 21,786,665
d. 20,766,298

B (Refer to solutions below)

62. How much is the carrying amount of the PPE (at cost model) after classification of the disposal
group as held for sale?
a. 16,000,000
b. 15,780,740
c. 15,340,206
d. 15,211,612

Fair value less costs to sell 52,000,000


Carrying amount as remeasured immediately before
classification as held for sale 59,600,000
Additional impairment loss on initial classification under PFRS 5 (7,600,000)
Allocation to goodwill 6,000,000
Impairment loss to be allocated to the other assets 1,600,000

The excess is allocated to the other assets pro rata based on their carrying amounts as follows:
Allocation of
Assets Carrying amt. Fraction Impairment Loss
Inventory N/A N/A N/A
Investment in FVOCI N/A N/A N/A
IP – cost 22,800,000 22.8/38.8 (940,206)
PPE – cost model 16,000,000 16/38.8 (659,794)
38,800,000 (1,600,000)

The carrying amount after allocation of impairment loss is:


Inventory 8,800,000
Investment in FVOCI 6,000,000
Investment property (at cost model) (22.8M – 940,206) 21,859,794
PPE (at cost model) (16M – 659,794) 15,340,206
Goodwill -
Total 52,000,000

63. On December 31, 20x1, INGENIOUS NATURAL Co. classified its building with a carrying
amount of ₱1,600,000 and fair value less costs to sell of ₱1,320,000 as held for sale. Impairment
loss of ₱280,000 was recognized on that date. The building has a remaining useful life of 4 years
and it was depreciated using the straight-line method.

As of December 31, 20x2, the building was not yet sold and management decided not to sell the
building anymore. The fair value less cost to sell of the building on December 31, 20x2 is ₱1,240,000
while the value in use is ₱1,220,000.
How much is the carrying amount of the building upon reclassification back to property, plant and
equipment?
a. 1,220,000
b. 1,320,000
c. 1,240,000
d. 1,200,000

D
a. Carrying amount adjusted for depreciation not recognized (1.6M x ¾) = 1.2M;
b. Recoverable amount = 1.240M the higher of FVLCS and VIN
 Measurement = 1.2M - the lower of a and b above

64. On December 31, 20x1, INIMICAL UNFRIENDLY Co. entered into an agreement to sell a
component. On that date, INIMICAL estimated the gain from the disposal to be made in 20x2 at
₱2,000,000 and the operating losses prior to the date of sale to be ₱1,200,000. As a result of the
sale, the component’s operations and cash flows will be eliminated from the entity’s operations
and the entity will not have any significant continuing post-sale involvement in the component’s
operations. Accordingly, the component was classified as held for sale and discontinued
operations.

The component’s actual operating losses in 20x1 and 20x2 were ₱2,800,000 and ₱2,600,000,
respectively, and the actual gain on disposal of the component in 20x2 was ₱1,600,000. INIMICAL’s
income tax rate is 30%. Any income tax benefit is expected to be realizable. There were no other
temporary differences during the year.

What single, post-tax amounts should be reported for discontinued operations in INIMICAL’s
comparative 20x2 and 20x1 income statements, respectively?
a. (1,960,000), (700,000)
b. (560,000), (1,960,000)
c. (650,000), (1,950,000)
d. (700,000), (1,960,000)

D 20x2: (1,600,000 – 2,600,000) x 70% = (700,000)


20x1: (-2,800,000 x 70%) = (1,960,000)

65. On April 30, 20x1, ABROGATE ABOLISH Co. approved a plan to dispose of a component of its
operations. The disposal meets the requirements for classification as discontinued operations.

From January 1 to April 30, 20x1, the component earned operating profit of ₱400,000 and from May 1
to December 31, 20x1, the segment suffered operating losses of ₱200,000.

The net assets of the component has a carrying amount of ₱32,000,000 as of April 30, 20x1. The fair
value less costs to sell of the component is ₱26,000,000. Additional estimated disposal loss includes
severance pay of ₱220,000 and employee relocation costs of ₱100,000, both of which are directly
associated with the decision to dispose of the segment. ABROGATE’s income tax rate is 30%. Any
income tax benefit is expected to be realizable. There were no other temporary differences during
the year.
How much is the profit (loss) from discontinued operations to be reported in ABROGATE's
statement of profit or loss and other comprehensive income for the year ended December 31, 20x1?
a. 4,564,000
b. 4,060,000
c. 4,340,000
d. 4,284,000

D
Solution:
Operating profit – January 1 to April 30, 20x1 400,000
Operating loss – May 1 to December 31, 20x1 (200,000)
Impairment loss (32M – 26M) (6,000,000)
Severance pay (220,000)
Employee relocation costs (100,000)
Total (6,120,000)
Multiply by: 1 minus Tax rate 70%
Loss for the period from discontinued operations (4,284,000)

66. You are a CPA. Your client asked you for an advice regarding the items that are presented as
other comprehensive income. You will tell your client to refer to which of the following
standards?
a. PAS 1
b. PFRS 1
c. PFRS 15
d. PAS 8

67. Non-current assets held for sale and discontinued operations are accounted for under
a. PFRS 4.
b. PAS 41.
c. PFRS 5.
d. PFRS 8.

68. When measuring the fair value of an asset or a liability, an entity refers to
a. PFRS 13.
b. PAS 28.
c. PAS 1.
d. PAS 33.

69. This standard deals with the recognition and measurement of financial instruments.
a. PAS 1
b. PAS 7
c. PFRS 9
d. PFRS 5

70. As used in accounting parlance, PFRS stands for


a. Philippine Accounting Standards.
b. Philippine Financial Accounting Standards.
c. Philippine Financial Reporting Standards.
d. Palabok, Friedchicken, Rice and Sprite.
e. All of the above

“give thanks in all circumstances; for this is God’s will for you in Christ Jesus.” - (1
Thessalonians 5:18)

- END -
1. The objective of PAS 1 Presentation of Financial Statements is to prescribe the basis for presentation
of general purpose financial statements, to ensure
a. intra-comparability c. faithful representation
b. inter-comparability d. a and b

2. General purpose financial statements cater to what type of needs of users?


a. common needs c. a and b
b. specific needs d. loving and caring needs

3. The ledger of COPIOUS RICH Co. as of December 31, 20x1 includes the following:

15% Note payable 50,000


16% Bonds payable 100,000
18% Serial bonds 200,000
Interest payable -

Additional information:
COPIOUS Co.’s financial statements were authorized for issue on April 15, 20x2.
- The 15% note payable was issued on January 1, 20x1 and is due on January 1, 20x5. The note pays
annual interest every year-end. The agreement with the lender provides that COPIOUS Co. shall
maintain an average current ratio of 2:1. If at any time the current ratio falls below the agreement,
the note payable will become due on demand. As of the 3rd quarter in 20x1, COPIOUS’s average
current ratio is 0.50:1. Immediately, COPIOUS informed the lender of the breach of the agreement.
On December 31, 20x1, the lender gave COPIOUS a grace period ending on December 31, 20x2 to
rectify the deficiency in the current ratio. COPIOUS promised the creditor to liquidate some of its
long-term investments in 20x2 to increase its current ratio.
- The 16% bonds are 10-year bonds issued on December 31, 1992. The bonds pay annual interest
every year-end.
- The 18% serial bonds are issued at face amount and are due in semi-annual installments of ₱20,000
every April 1 and September 30. Interests on the bonds are also due semi-annually. The last
installment on the bonds is due on September 30, 20x7.

How much is the total current liabilities?


a. 9,000
b. 100,000
c. 109,000
d. 120,000

Solution:
16% Bonds payable 100,000
Interest payable on the serial bonds (200K x 18% x 3/12) 9,000
Current liabilities 109,000
4. Entity B has the following information:

Inventory, beg. 120,000


Inventory, end. 192,000
Purchases 480,000
Freight-in 24,000
Purchase returns 12,000
Purchase discounts 16,800

How much is Entity B’s the cost of sales?


a. 402,300
b. 416,300
c. 420,300
d. 422,300

Inventory, beg. 120,000


Net purchases:
Purchases 480,000
Freight-in 24,000
Purchase returns (12,000)
Purchase discounts (16,800) 475,200
Total goods available for sale 595,200
Less: Inventory, end. (192,000)
Cost of goods sold 403,200

Use the following information for the next four questions:


The nominal accounts of Hazel Lee Co. on December 31, 20x1 have the following balances:

Accounts Dr. Cr.


Sales ₱1,045,000
Interest income 80,000
Gains 30,000
Inventory, beg. ₱80,000
Purchases 300,000
Freight-in 30,000
Purchase returns 15,000
Purchase discounts 27,000
Freight-out 25,000
Sales commission 60,000
Advertising expense 35,000
Salaries expense 350,000
Rent expense 60,000
Depreciation expense 80,000
Utilities expense 40,000
Supplies expense 30,000
Transportation and travel expense 25,000
Insurance expense 10,000
Taxes and licenses 50,000
Interest expense 5,000
Miscellaneous expense 2,000
Loss on the sale of equipment 15,000

Additional information:
a. Ending inventory is ₱100,000.
b. Three-fourths of the salaries, rent, and depreciation expenses pertain to the sales department. The
sales department does not share in the other expenses.

5. In a statement of comprehensive income prepared using the single-step approach (nature of


expense method), how much is presented as ‘change in inventory’? (increase)/decrease
a. (288,000)
b. 288,000
c. (20,000)
d. 20,000

Single-step approach

Hazel Lee Co.


Statement of Comprehensive Income
For the period ended December 31, 20x1
INCOME
Sales ₱1,045,000
Interest income 80,000
Gains 30,000
TOTAL INCOME 1,155,000
EXPENSES
Net purchases (a) 288,000
Change in inventory (b) (20,000)
Freight-out 25,000
Sales commission 60,000
Advertising expense 35,000
Salaries expense 350,000
Rent expense 60,000
Depreciation expense 80,000
Utilities expense 40,000
Supplies expense 30,000
Transportation and travel expense 25,000
Insurance expense 10,000
Taxes and licenses 50,000
Interest expense 5,000
Miscellaneous expense 2,000
Loss on sale of equipment 15,000
TOTAL EXPENSES 1,055,000
PROFIT FOR THE YEAR 100,000
Other comprehensive income -
COMPREHENSIVE INCOME FOR THE YEAR ₱100,000

“Net purchases” is computed as follows:


(a)

Purchases ₱300,000
Freight-in 30,000
Purchase returns (15,000)
Purchase discounts (27,000)
Net Purchases ₱288,000

(b) “Change in inventory” is computed as follows:

Inventory, beg. ₱80,000


Inventory, end 100,000
Change in inventory – increase (₱20,000)

6. In a statement of comprehensive income prepared using the single-step approach (nature of


expense method), how much is presented as total expenses?
a. 1,055,000
b. 1,075,000
c. 787,000
d. 772,000

7. In a statement of comprehensive income prepared using the multi-step approach (function of


expense method), how much is presented as distribution costs?
a. 398,500
b. 487,500
c. 467,500
d. 512,500

Multi-step approach

Hazel Lee Co.


Statement of Comprehensive Income
For the period ended December 31, 20x1
Notes
Sales ₱1,045,000
Cost of sales 1 (268,000)
GROSS PROFIT 777,000
Other income 2 110,000
Distribution costs 3 (487,500)
Administrative expenses 4 (279,500)
Other expenses 5 (15,000)
Interest expense (5,000)
PROFIT FOR THE YEAR 100,000
Other comprehensive income -
COMPREHENSIVE INCOME FOR THE YR. ₱100,000

Note 1: Cost of sales


Inventory, beg. ₱80,000
Purchases 300,000
Freight-in 30,000
Purchase returns (15,000)
Purchase discounts (27,000)
Total goods available for sale 368,000
Inventory, end (100,000)
Cost of sales ₱268,000

Note 2: Other income


Interest income ₱80,000
Gains 30,000
Other income ₱110,000

Note 3: Distribution costs


Freight-out ₱25,000
Sales commission 60,000
Advertising expense 35,000
Salaries expense (350,000 x 3/4) 262,500
Rent expense (60,000 x 3/4) 45,000
Depreciation expense (80,000 x 3/4) 60,000
Distribution costs ₱487,500

Note 4: Administrative expenses


Salaries expense (350,000 x 1/4) ₱87,500
Rent expense (60,000 x 1/4) 15,000
Depreciation expense (80,000 x 1/4) 20,000
Utilities expense 40,000
Supplies expense 30,000
Transportation and travel expense 25,000
Insurance expense 10,000
Taxes and licenses 50,000
Miscellaneous expense 2,000
Administrative expenses ₱279,500

Note 5: Other expenses


Loss incurred on the sale of equipment – 15,000

8. In a statement of comprehensive income prepared using the multi-step approach (function of


expense method), how much is presented as administrative expenses?
a. 297,500
b. 302,500
c. 287,500
d. 279,500

Use the following information for the next two questions:


DEMOTIC POPULAR Co. acquires through foreclosure a property comprising land and buildings
that it intends to sell. The fair value of the land and buildings is ₱6,000,000 and costs to sell are
₱200,000. The related defaulted receivables have a carrying amount of ₱5,000,000.

9. The entity does not intend to transfer the property to a buyer until after it completes renovations
to increase the property’s sales value. How should DEMOTIC Co. classify the land and
buildings?
a. Included under property, plant and equipment at ₱5,000,000.
b. Included under investment property at ₱5,000,000.
c. Included under investment property at ₱5,800,000.
d. Classified as held for sale at ₱5,800,000

C (6M – 200K) = 5.8M investment property because the property is not available for sale in its present
condition

10. After the renovations are completed and the property is classified as held for sale but before a
firm purchase commitment is obtained, the entity becomes aware of environmental damage
requiring remediation. The entity still intends to sell the property. However, the entity does not
have the ability to transfer the property to a buyer until after the remediation is completed. The
costs of renovations made totaled ₱200,000. The estimated costs of remediation are ₱100,000.
How should DEMOTIC Co. classify the land and buildings?
a. Included under property, plant and equipment at ₱5,700,000.
b. Included under investment property at ₱6,000,000.
c. Included under investment property at ₱5,700,000.
d. Classified as held for sale at ₱5,700,000

B (5.8M (see no. 5) + 200K costs of renovations = 6M. The land and buildings are classified as investment
property and not as held for sale assets because they are not available for immediate sale in their
present condition.
11. An entity is committed to a plan to sell a manufacturing facility in its present condition and
classifies the facility as held for sale at that date. After a firm purchase commitment is obtained,
the buyer’s inspection of the property identifies environmental damage not previously known to
exist. The entity is required by the buyer to make good the damage, which will extend the period
required to complete the sale beyond one year. However, the entity has initiated actions to make
good the damage, and satisfactory rectification of the damage is highly probable. The
manufacturing facility has a carrying amount of ₱10,000,000 and fair value less costs to sell of
₱10,600,000. How should the entity classify the manufacturing facility?
a. Held for sale, ₱10.6M c. PPE, ₱10M
b. Held for sale, ₱10M d. PPE, ₱10.6M

B – The exception to the “1-yr. requirement” is met.

12. Under the indirect method, the cash flow from operating activities is determined by adjusting
the reported profit by (choose the incorrect statement)
a. adding back non-cash expenses
b. adding back decreases in operating assets
c. deducting decreases in operating liabilities
d. adding back increases in operating assets

13. Under the indirect method, the cash flow from operating activities is determined by adjusting
the reported profit by (choose the incorrect statement)
a. deducting non-cash income
b. deducting increases in operating assets
c. deducting decreases in nonoperating liabilities
d. deducting gains on sale of nonoperating assets

14. When preparing a statement of cash flows using the direct method, amortization of patent is
a. shown as an increase in cash flows from operating activities.
b. shown as a reduction in cash flows from operating activities.
c. included with supplemental disclosures of noncash transactions.
d. not reported in the statement of cash flows or related disclosures.

15. Which of the following statements regarding cash equivalents is correct?


a. A one-year Treasury note could not qualify as a cash equivalent.
b. All investments meeting the PFRS 9 Financial Instruments criteria must be reported as cash
equivalents.
c. The date a security is purchased determines its "original maturity" for cash equivalent
classification purposes.
d. Once established, management's policy for classifying items as cash equivalents cannot be
changed.

16. Using the indirect method, cash flows from operating activities would be increased by which of
the following?
a. Gain on sale of investments
b. Increase in prepaid expenses
c. Decrease in accounts payable
d. Decrease in accounts receivable

Use the following information for the next three questions:


The movements in the cash account of DEADLOCK STANDSTILL Co. during 20x2 are shown
below.
Cash
beg. 400
Sales 12,000 7,600 Purchases
Interest income 40 2,400 Operating expenses
Rent income 540 60 Interest expense
Dividend income 80 140 Income taxes
Sale of held for trading securities 1,600 200 Investment in FVOCI
Sale of old building 1,040 2,200 Purchase of equipment
Collection of non-trade note 120 260 Loan granted to employee
Proceeds from loan with a bank 3,200 480 Payment of loan borrowed
Issuance of shares 1,940 400 Reacquisition of shares
180 Dividends
7,040 end.

17. How much is the cash flows from operating activities?


a. 4,600
b. 4,840
c. 5,040
d. 4,060

D (Refer to solutions below)

18. How much is the cash flows from investing activities?


a. (1,500)
b. 1,500
c. 1,240
d. (1,240)

A (Refer to solutions below)

19. How much is the cash flows from financing activities?


a. 4,800
b. (4,800)
c. 4,240
d. 4,080

D Solutions:

DEADLOCK STANDSTILL Co.


Statement of cash flows
For the year ended December 31, 20x2

Cash flows from operating activities


Cash receipts from customers 12,000
Cash receipts for interest income 40
Cash receipts for rent income 540
Cash receipts for dividend income 80
Cash paid to suppliers (7,600)
Cash paid for operating expenses (2,400)
Cash generated from operations 2,660
Interest paid (60)
Income taxes paid (140)
Cash receipt from sale of held for trading securities 1,600
Net cash from operating activities 4,060

Cash flows from investing activities


Cash payment for acquisition of investment in FVOCI (200)
Cash receipt from sale of old building 1,040
Cash payment for acquisition of equipment (2,200)
Cash receipt from collection of loan granted 120
Cash payment for loan granted (260)
Net cash used in investing activities (1,500)

Cash flows from financing activities


Cash proceeds from loan borrowed 3,200
Cash payment for loan borrowed (480)
Cash proceeds from issuance of share capital 1,940
Cash payment for acquisition of treasury shares (400)
Cash payment for dividends (180)
Net cash from financing activities 4,080

Net increase in cash and cash equivalents 6,640


Cash and cash equivalents, beginning 400
Cash and cash equivalents, end 7,040

Use the following information for the next four questions:


BLITHE JOYFUL Co. had the following information during 20x2:
Accounts receivable, January 1, 20x2 2,400
Accounts receivable, December 31, 20x2 1,600
Sales on account and cash sales 32,000
Bad debts expense 800
Accounts payable, January 1, 20x2 1,400
Accounts payable, December 31, 20x2 800
Cost of sales 16,000
Increase in inventory 3,600

Operating expenses on accrual basis 4,880


Increase in accrued payables for operating expenses 1,640
Decrease in prepaid operating expenses 1,560
Property, plant, and equipment, January 1, 20x2 7,200
Property, plant, and equipment, December 31, 20x2 10,800

Additional information:
• There were no write-offs of accounts receivable during the year.
• Equipment with an accumulated depreciation of ₱800 was sold during the year for ₱480
resulting to a gain on sale of ₱60.

20. How much is the cash receipts from customers?


a. 38,200
b. 37,400
c. 35,400
d. 32,800

Solution:
Accounts receivable
Jan. 1, 20x2 2,400 - Write-offs
Collections of accounts receivables
Sales 32,000 32,800 (squeeze)
1,600 Dec. 31, 20x2

21. How much is the cash payments to suppliers?


a. 19,000
b. 20,200
c. 22,000
d. 23,400

Solution:
Inventory
Jan. 1, 20x2 -
Net purchases (squeeze) 19,600 16,000 Cost of sales
3,600 Dec. 31, 20x2

Accounts payable
1,400 Jan. 1, 20x2
Payments for purchases on
account (squeeze) 20,200 19,600 Net purchases (accrual)
Dec. 31, 20x2 800

22. How much is the cash payments for operating expenses?


a. 1,680
b. 4,800
c. 4,960
d. 8,080

Solution:
Prepaid expense / Accrued expense
Prepaid expense, beginning 1,560 - Accrued expense, beg.
Cash paid for operating expenses
(squeeze) 1,680 4,880 Operating expense (accrual basis)
Accrued expense, end 1,640 - Prepaid expense, end

23. How much is the cash payments for acquisition of property, plant, and equipment?
a. 3,600
b. 4,820
c. 4,080
d. 4,940

Solution:
The entry for the sale of equipment is re-constructed as follows:
20x2 Cash on hand (given) 480
Accumulated depreciation (given) 800
Equipment (squeeze) 1,220
Gain on sale (given) 60
Property, plant and equipment
Jan. 1, 20x2 7,200
Historical cost of equipment sold (see
Acquisition of PPE (squeeze) 4,820 1,220 journal entry above)
10,800 Dec. 31, 20x2

24. ABC Co. has the following information as of December 31, 20x1:
Jan. 1 Dec. 31
Accounts receivable 100,000 250,000
Allowance for bad debts 15,000 20,000
Net credit sales 850,000
Bad debt expense 60,000
Recoveries 20,000

How much is the total cash receipts from customers during the period?
a. 970,000
b. 879,000
c. 907,000
d. 897,000
C

Solution:
Accounts receivable
beg. 90,000
Net credit sales 1,200,000 73,000 Write-offs a
Collections from customers (including
Recoveries 10,000 907,000 recoveries) - (squeeze)
320,000 end.

The amount of write-offs is computed as follows:


a

Allowance for doubtful accounts


26,000 beg.
Write-offs (squeeze) 73,000 10,000 Recoveries
80,000 Bad debt expense
end. 43,000

25. BLUFF DECEIVE Co. has the following information as of December 31, 20x2:
Jan. 1 Dec. 31
Accounts receivable 16,000 20,000
Allowance for bad debts (400) (1,000)
Prepaid rent 3,840 3,200
Accounts payable 6,800 8,800

BLUFF reported profit of ₱8,800 for the year, after depreciation expense of ₱200, gain on sale of
equipment of ₱240, and restructuring and other provisions of ₱400. None of the provisions
recognized during the period affected cash.

How much is the cash flows from operating activities?


a. 4,800
b. 5,600
c. 8,800
d. 8,400

Solution:
Cash flows from operating activities
Profit 8,800
Adjustments for:
Depreciation expense 200
Gain on sale of building (240)
Restructuring and other provisions 400
9,160
Increase in accounts receivable, net
[(20,000 – 1,000) – (16,000 – 400)] (3,400)
Decrease in prepaid rent 640
Increase in accounts payable 2,000
Net cash from operating activities 8,400

Use the following information for the next two questions:


INORDINATE EXCESSIVE Co. had the following information for 20x2:
• Acquired 3-month treasury bills for ₱200,000.
• Acquired equipment with a purchase price of ₱4,000,000 by paying 20% in cash and issuing a
note payable for the balance. There were no payments made on the note during the year.
• Acquired land with fair value of ₱3,200,000 by issuing shares with aggregate par value of
₱2,400,000. The excess is credited to share premium.
• Extended a ₱1,600,000 loan to a director.
• Borrowed ₱1,280,000 from a bank. Used the cash proceeds as follows: ₱800,000 for additional
working capital and ₱480,000 to settle scrip dividends declared in 20x1.
• Settled an outstanding note payable by issuing shares with aggregate par value of ₱800,000.
Share premium resulted from the transaction amounted to ₱280,000.

26. How much is the net cash flows from (used in) investing activities?
a. (2,400,000)
b. 2,400,000
c. 800,000
d. (800,000)

Cash flows from investing activities


Cash payment for acquisition of equipment (4M x 20%) (800,000)
Cash proceeds of loan granted to officer (1,600,000)
Net cash used in investing activities (2,400,000)

27. How much is the net cash flows from (used in) financing activities?
a. (800,000)
b. 800,000
c. (2,400,000)
d. 2,400,000

Cash flows from financing activities


Cash receipt from borrowings 1,280,000
Cash payment for dividends (480,000)
Net cash used in financing activities 800,000

Use the following information for the next three questions:


Information on LA-DI-DA SHOWY Co.'s financial position and performance as of December 31,
20x2 and 20x1 are presented below.

LA-DI-DA SHOWY Company


Statement of financial position
As of December 31, 20x2
ASSETS 20x2 20x1
Current assets
Cash and cash equivalents 1,000,000 600,000
Held for trading securities 480,000 -
Accounts receivable – net 1,520,000 1,240,000
Rent receivable 100,000 40,000
Inventory 2,000,000 3,600,000
Prepaid insurance 200,000 160,000
Total current assets 5,300,000 5,640,000
Noncurrent assets
Investment in bonds 360,000 340,000
Buildings 10,000,000 4,000,000
Accumulated depreciation (800,000) (800,000)
Goodwill 360,000 400,000
Total noncurrent assets 9,920,000 3,940,000
TOTAL ASSETS 15,220,000 9,580,000
LIABILITIES AND EQUITY
Current liabilities
Accounts payable 480,000 320,000
Unearned rent 80,000 120,000
Insurance payable 240,000 180,000
Dividends payable 920,000 480,000
Income tax payable 60,000 140,000
Short-term loan payable - 200,000
Total current liabilities 1,780,000 1,440,000
Noncurrent liabilities
Bonds payable 4,000,000 4,000,000
Discount on bonds (380,000) (400,000)
Deferred tax liability 60,000 40,000
Total noncurrent liabilities 3,680,000 3,640,000
TOTAL LIABILITIES 5,460,000 5,080,000
Equity
Share capital 8,000,000 4,000,000
Retained earnings 1,760,000 500,000
TOTAL EQUITY 9,760,000 4,500,000
TOTAL LIABILITIES AND
EQUITY 15,220,000 9,580,000

LA-DI-DA SHOWY Company


Statement of profit or loss
For the year ended December 31, 20x2
Sales 20,000,000
Cost of sales (12,000,000)
Gross income 8,000,000
Rent income 1,800,000
Interest income 80,000
Insurance expense (400,000)
Bad debts expense (60,000)
Interest expense (400,000)
Loss on sale of building (160,000)
Unrealized gain on investment 80,000
Other expenses (4,800,000)
Profit before tax 4,140,000
Income tax expense (1,200,000)
Profit for the year 2,940,000

Additional information:
• During 20x2, LA-DI-DA purchased held for trading securities for ₱400,000. The fair value of the
shares on December 31, 20x2 is ₱480,000.
• The allowance for doubtful accounts has balances of ₱80,000 and ₱40,000 as of December 31, 20x2
and 20x1, respectively.
• During 20x2, LA-DI-DA sold an old building with historical cost of ₱3,200,000 for ₱1,040,000.
• LA-DI-DA inadvertently included depreciation expense in the “Other expenses” line item.
• There were no acquisitions or disposals of investment in bonds during the period.
• During 20x2, LA-DI-DA issued shares with an aggregate par value of ₱4,000,000 for ₱4,000,000
cash.

28. How much is the net cash flows from (used in) operating activities?
a. (6,000,000)
b. 6,000,000
c. 6,600,000
d. (7,600,000)

Solution:
Cash flows from operating activities
Profit for the year 2,940,000
Adjustments for:
Depreciation expense 2,000,000
Impairment loss on goodwill 40,000
Loss on sale of building 160,000
Unrealized gain on held for trading securities (80,000)
Amortization of discount on investment in bonds (20,000)
Amortization of discount on bonds payable 20,000
5,060,000
Increase in accounts receivable, net (280,000)
Increase in rent receivable (60,000)
Decrease in inventory 1,600,000
Increase in prepaid insurance (40,000)
Increase in accounts payable 160,000
Decrease in unearned rent (40,000)
Increase in insurance payable 60,000
Decrease in income tax payable (80,000)
Increase in deferred tax liability 20,000
6,400,000
Cash paid for the acquisition of held for trading securities (400,000)
Net cash from operating activities 6,000,000

29. How much is the net cash flows from (used in) investing activities?
a. (8,160,000)
b. 8,460,000
c. (9,200,000)
d. 8,160,000

Solution:
Cash flows from investing activities
Cash receipt from sale of old building 1,040,000
Cash payment for acquisition of building (9,200,000)
Net cash used in investing activities (8,160,000)

30. How much is the net cash flows from (used in) financing activities?
a. (2,560,000)
b. 2,560,000
c. (2,960,000)
d. 2,960,000

Solution:
Cash flows from financing activities
Cash proceeds from issuance of share capital 4,000,000
Cash payment for short-term loan (200,000)
Cash payment for dividends (1,240,000)
Net cash from financing activities 2,560,000

31. REMNANT REMAINDER Co.’s cash balances as of December 31, 20x2 and 20x1 were ₱7,040,000
and ₱400,000 respectively. REMNANT’s December 31, 20x2 statement of cash flows reported net
cash used in investing activities of ₱1,500,000 and net cash from financing activities of
₱4,080,000.

How much is the net cash flows from (used in) operating activities?
a. (4,060,000)
b. 4,060,000
c. 4,600,000
d. (4,600,000)

Solution:
Net cash from operating activities 4,060,000 squeeze
Net cash from investing activities (1,500,000)
Net cash used in financing activities 4,080,000
Net increase in cash during the period 6,640,000
Cash, beginning balance 400,000
Cash, ending balance 7,040,000 start

32. During 20x1, ALBEIT ALTHOUGH Company decided to change from the Average cost formula
for inventory valuation to the FIFO cost formula. Inventory balances under each method were as
follows:
Average FIFO
January 1 4,000,000 4,800,000
December 31 8,000,000 8,400,000

Income tax rate is 30%. What is the net cumulative effect of the accounting change in ALBEIT’s
opening retained earnings balance?
a. 400,000 increase c. 280,000 increase
b. 560,000 decrease d. 560,000 increase

D (4,800,000 – 4,000,000) x 70% = 560,000 increase

33. On January 1, 20x1, PRISTINE UNCORRUPTED Co. acquired an equipment for ₱4,000,000. The
equipment will be depreciated using the straight-line method over 20 years. The estimated
residual value is ₱400,000.

In 20x6, following a reassessment of the realization of the expected economic benefits from the
equipment, PRISTINE Co. changed its depreciation method to sum-of-the-years digits (SYD). The
remaining useful life of the asset is estimated to be 4 years and the residual value is changed to
₱200,000. How much is the depreciation expense in 20x6?
a. 1,160,000 b. 1,140,000 c. 1,233,560 d. 1,110,669

A [(4M – 400K) x 15/20 + 400K] – 200K x 4/10 = 1,160,000

Use the following information for the next two questions:


On January 10, 20x2, prior to the authorization of LIBERTINE IMMORAL Co.’s December 31, 20x1
financial statements for issue, the accountant of LIBERTINE Co. received a bill for an advertisement
made in the month of December 20x1 amounting to ₱1,600,000. This expense was not accrued as of
December 31, 20x1.

34. The correcting entry, if the books are still open, includes
a. a debit to advertising expense for ₱1,600,000
b. a credit to advertising income for ₱1,600,000
c. a debit to retained earnings for ₱1,600,000
d. a credit to retained earnings for ₱1,600,000

A (Dr.) Advertising expense 1.6M; (Cr.) Advertising payable 1.6M


35. The correcting entry, if the books are already closed, includes
a. a debit to advertising expense for ₱1,600,000
b. a credit to advertising income for ₱1,600,000
c. a debit to retained earnings for ₱1,600,000
d. a credit to retained earnings for ₱1,600,000

C (Dr.) Retained earnings 1.6M; (Cr.) Advertising payable 1.6M

36. On January 15, 20x3 while finalizing its 20x2 financial statements, DIAPHANOUS
TRANSPARENT Co. discovered that depreciation expense recognized in 20x1 is overstated by
₱1,600,000. Ignoring income tax, the entry to correct the prior period error includes
a. a debit to depreciation expense for ₱1,600,000
b. a debit to retained earnings for ₱1,600,000
c. a credit to depreciation expense for ₱1,600,000
d. a debit to accumulated depreciation for ₱1,600,000

D (Dr.) Accum. Dep’n. 1.6M; (Cr.) Retained earnings 1.6M

Use the following information for the next four questions:


GULOSITY GREEDINESS Co. reported profits of ₱4,000,000 and ₱8,000,000 in 20x1 and 20x2,
respectively. In 20x3, the following prior period errors were discovered:
• The inventory on December 31, 20x1 was understated by ₱200,000.
• An equipment with an acquisition cost of ₱1,200,000 was erroneously charged as expense in
20x1. The equipment has an estimated useful life of 5 years with no residual value. GULOSITY
Co. provides full year depreciation in the year of acquisition.

The unadjusted balances of retained earnings are ₱8,800,000 and ₱16,800,000 as of December 31, 20x1
and 20x2, respectively.

37. How much is the correct profit in 20x1?


a. 7,560,000
b. 5,610,000
c. 4,760,000
d. 5,160,000

D (Refer to solutions below)

38. How much is the correct profit in 20x2?


a. 7,560,000
b. 5,160,000
c. 5,720,000
d. 5,610,000
A

20x1 20x2
Unadjusted profits 4,000,000 8,000,000
Corrections - (over) understatement:
(a) Understatement of Dec. 31, 20x1 inventory 200,000 (200,000)
(b.1) Capitalizable costs charged as expense 1,200,000 -
(b.2) Depreciation expense not recognized (240,000) (240,000)
Net adjustment to profit 1,160,000 (440,000)
Correct profits 5,160,000 7,560,000

39. How much is the correct retained earnings in 20x1?


a. 9,960,000 b. 17,520,000 c. 9,860,000 d. 18,420,000

A (Refer to solutions below)

40. How much is the correct retained earnings in 20x2?


a. 9,960,000 b. 17,520,000 c. 9,860,000 d. 18,420,000

20x1 20x2
Unadjusted retained earnings 8,800,000 16,800,000
Net effect of errors on retained earnings:
20x1: 1,160,000* 1,160,000
20x2: (440,000) + 1,160,000* 720,000
Adjusted retained earnings 9,960,000 17,520,000
*Amounts represent the net effect of errors in profits (refer to previous solution).

Use the following information for the next four questions:


HELICAL SPIRAL Co. reported profits of ₱1,600,000 and ₱2,400,000 in 20x1 and 20x2, respectively.
In 20x3, the following prior period errors were discovered:
• Prepaid supplies in 20x1 were overstated by ₱80,000.
• Accrued salaries payable in 20x1 were understated by ₱160,000.
• Repairs and maintenance expenses in 20x1 amounting to ₱400,000 were erroneously capitalized
and being depreciated over a period of 4 years.

The unadjusted balances of retained earnings are ₱6,400,000 and ₱8,800,000 as of December 31, 20x1
and 20x2, respectively.

41. How much is the correct profit in 20x1?


a. 1,006,000
b. 1,610,000
c. 1,720,000
d. 1,060,000

D (Refer to solutions below)

42. How much is the correct profit in 20x2?


a. 2,704,000
b. 2,160,000
c. 2,740,000
d. 2,610,000

Solutions:
20x1 20x2
Unadjusted profits 1,600,000 2,400,000
Corrections - (over) understatement:
(a) Overstatement of 20x1 prepaid assets (80,000) 80,000
(b) Understatement of 20x1 accrued salaries (160,000) 160,000
(c.1) Expenses erroneously capitalized (400,000) -
(c.2) Depreciation recognized on repair costs
(400,000 ÷ 4) 100,000 100,000
Net adjustment to profit (540,000) 340,000
Correct profits 1,060,000 2,740,000

43. How much is the correct retained earnings in 20x1?


a. 5,806,000
b. 5,520,000
c. 5,860,000
d. 5,420,000

C (Refer to solutions below)

44. How much is the correct retained earnings in 20x2?


a. 8,960,000
b. 8,600,000
c. 8,860,000
d. 8,420,000

B (Refer to solutions below)

Solutions:
20x1 20x2
Unadjusted retained earnings 6,400,000 8,800,000
Net effect of errors on retained earnings:
20x1: (540,000)* (540,000)
20x2: 340,000 + (540,000)* (200,000)
Adjusted retained earnings 5,860,000 8,600,000
*Amounts represent the net effect of errors in profits (refer to previous solution).

Use the following information for the next fifteen questions:


THRALL SLAVE Company made the following errors:
a. December 31, 20x1 inventory was understated by ₱100,000.
b. December 31, 20x2 inventory was overstated by ₱160,000.
c. Purchases on account in 20x1 were understated by ₱400,000 (not included in physical count).
d. Advances to suppliers in 20x2 totaling ₱520,000 were inappropriately charged as purchases.
e. December 31, 20x1 prepaid insurance was overstated by ₱20,000.
f. December 31, 20x1 unearned rent income was overstated by ₱104,000.
g. December 31, 20x2 interest receivable was understated by ₱68,000.
h. December 31, 20x2 accrued salaries payable was understated by ₱120,000.
i. Advances from customers in 20x2 totaling ₱240,000 were inappropriately recognized as sales but
the goods were delivered in 20x3.
j. Depreciation expense in 20x1 was overstated by ₱28,800
k. In 20x2, the acquisition cost of a delivery truck amounting to ₱360,000 was inappropriately
charged as expense. The delivery truck has a useful life of five years. THRALL’s policy is to
provide a full year’s straight line depreciation in the year of acquisition and none in the year of
disposal.
l. A fully depreciated equipment with no residual value was sold in 20x3 for ₱200,000 but the sale
was recorded in the following year.

Profits before correction of errors were ₱492,000, ₱624,000, and ₱840,000 in 20x1, 20x2, and 20x3,
respectively.

Retained earnings before correction of errors were ₱4,492,000, ₱5,116,000 and ₱5,956,000 in 20x1,
20x2, and 20x3, respectively.

45. What is the net effect of the errors on the 20x1 profit? (over) understatement
a. (187,200)
b. 187,200
c. (164,200)
d. 164,200

A (Refer to solutions below)

46. What is the net effect of the errors on the 20x2 profit? (over) understatement
a. (572,000)
b. 572,000
c. 563,400
d. (563,400)

B (Refer to solutions below)


47. What is the net effect of the errors on the 20x3 profit? (over) understatement
a. (78,000)
b. 78,000
c. (60,000)
d. 60,000

D (Refer to solutions below)

48. How much is the correct profit (loss) in 20x1?


a. (348,000)
b. 348,000
c. 324,800
d. 304,800

D (Refer to solutions below)

49. How much is the correct profit (loss) in 20x2?


a. 1,196,000
b. 1,296,000
c. 1,684,800
d. 1,286,000

A (Refer to solutions below)

50. How much is the correct profit (loss) in 20x3?


a. 900,000
b. 926,000
c. 968,400
d. 986,000

A (Refer to solutions below)

Solutions:
20x1 20x2 20x3
Unadjusted profits 492,000 624,000 840,000
Corrections - (over)/understatement
a. Understatement of 20x1 inventory 100,000 (100,000) -
b. Overstatement of 20x2 inventory (160,000) 160,000
c. Understatement of 20x1 purchases (400,000) 400,000 -
d. Overstatement of 20x2 purchases 520,000 (520,000)
e. Overstatement of 20x1 prepaid
insurance (20,000) 20,000 -
f. Overstatement of 20x1 unearned
rent 104,000 (104,000) -
g. Understatement of 20x2 interest 68,000 (68,000)
income
h. Understatement of 20x2 accrued
salaries (120,000) 120,000
i. Overstatement of 20x2 advances/
sales (240,000) 240,000
j. Overstatement of 20x1 depreciation
expense 28,800 - -
k. Acquisition cost of delivery truck in
20x2 360,000
k.1 Depreciation on delivery truck (72,000) (72,000)
l. Gain on sale in 20x3 not recorded* 200,000
Net correction on profits
(over) / under (187,200) 572,000 60,000
Corrected profits 304,800 1,196,000 900,000

*Since the equipment sold is fully depreciated and it has no residual value, the proceeds represents the gain on sale.

51. What is the net effect of the errors on the 20x1 retained earnings? (over) understatement
a. (182,700)
b. 182,700
c. (165,200)
d. (187,200)

D (Refer to solutions below)

52. What is the net effect of the errors on the 20x2 retained earnings? (over) understatement
a. 348,800
b. (348,800)
c. (384,800)
d. 384,800

D (Refer to solutions below)

53. What is the net effect of the errors on the 20x3 retained earnings? (over) understatement
a. 444,800
b. (444,800)
c. 524,800
d. (524,800)

A (Refer to solutions below)

54. How much is the correct retained earnings in 20x1?


a. 4,304,800
b. 4,404,800
c. 4,524,400
d. 4,340,800
A (Refer to solutions below)

55. How much is the correct retained earnings in 20x2?


a. 5,500,800
b. 5,756,800
c. 5,246,400
d. 5,340,400

A (Refer to solutions below)

56. How much is the correct retained earnings in 20x3?


a. 6,340,800
b. 6,400,800
c. 6,479,800
d. 7,004,400

Solutions:
20x1 20x2 20x3
Unadjusted retained earnings 4,492,000 5,116,000 5,956,000
Net effect of errors on profits in:
20x1 (187,200) (187,200) (187,200)
20x2 572,000 572,000
20x3 60,000
Net effect of errors on retained earnings (over) /
under (187,200) 384,800 444,800
Adjusted retained earnings 4,304,800 5,500,800 6,400,800

57. What is the net effect of the errors on the 20x1 working capital? (over) understatement
a. (216,000)
b. 216,000
c. 80,000
d. (80,000)

58. What is the net effect of the errors on the 20x2 working capital? (over) understatement
a. 228,000
b. (228,000)
c. (68,000)
d. 68,000

D
59. What is the net effect of the errors on the 20x3 working capital? (over) understatement
a. No effect
b. 132,000
c. 200,000
d. (200,000)

Solutions:
20x1 20x2 20x3
Effect of errors on working capital (over)/under
a. Understatement of 20x1 inventory 100,000
b. Overstatement of 20x2 inventory (160,000)
c. Understatement of 20x1 accounts
payable a (400,000)
d. Understatement of 20x2 advances to
suppliers b 520,000
e. Overstatement of 20x1 prepaid
insurance (20,000)
f. Overstatement of 20x1 unearned rent 104,000
g. Understatement of 20x2 interest
receivable 68,000
h. Understatement of 20x2 accrued salaries (120,000)
i. Understatement of 20x2 advances to
customers c (240,000)
l. Understatement of cash due to the sale of
equipment not recorded in 20x3 200,000

Net effect of errors on working capital (over)/under (216,000) 68,000 200,000

a
If purchases on account is understated, accounts payable is also understated. Understatement in
current liabilities overstates working capital.
b
Advances to suppliers are normally classified as current receivables. Understatement in current assets
understates working capital.
c
Advances from customers are normally classified as current liabilities. Understatement in current
liabilities overstates working capital.

60. TRIBULATION GREAT DISTRESS Co.’s current reporting period ends on December 31, 20x1.
The following transactions occurred after the end of reporting period:
• On January 5, 20x2, TRIBULATION declared ₱8,000,000 dividends.
• On January 15, 20x2, TRIBULATION issued 1,000 shares with par value per share of ₱400 for
₱2,400 per share.
• On January 20, 20x2, TRIBULATION installed an oil rig. Current legislation requires that the oil
rig be uninstalled at the end of its useful life and the site where it was installed be restored.
TRIBULATION estimates the present value of the decommissioning and restoration cost at
₱4,000,000.
• On February 1, 20x2, a building with a carrying amount as of December 31, 20x1 of ₱2,000,000
was totally razed by fire.
• On February 10, 20x2, TRIBULATION received notice of a litigation in relation to an accident
that happened on December 31, 20x1. TRIBULATION estimates a probable loss of ₱800,000.
• On March 5, 20x2, TRIBULATION purchased a subsidiary for ₱40,000,000 in a business
combination accounted for using the acquisition method. Goodwill of ₱10,000,000 was
recognized on the business combination.

The financial statements were authorized for issue on March 1, 20x2.

What is the total amount of the adjusting events?


a. 6,800,000
b. 800,000
c. 4,800,000
d. 30,000,000

B 800,000 probable loss on litigation

61. UNCORK RELEASE Co.’s current reporting period ends on December 31, 20x1. The following
transactions occurred after the end of reporting period:
• On January 20, 20x2, a pending litigation was resolved requiring a settlement amount of
₱400,000. The 20x1 year-end financial statements included a provision for loss on litigation of
₱480,000.
• Inventories costing ₱4,000,000 were recognized at their net realizable value of ₱3,600,000 in the
20x1 year-end financial statements. During January 20x2, the inventories were sold for
₱3,520,000. Actual selling costs amounted to ₱120,000.
• The year-end accounts receivable include a ₱400,000 receivable from RELINQUISH, Inc. No
allowance for doubtful accounts was recognized on this receivable as of December 31, 20x1. On
February 3, 20x2, RELINQUISH filed for bankruptcy. It was estimated that the receivable will
not be collected.
• The fair value of financial assets measured at fair value through profit or loss significantly
declined to ₱320,000 on February 28, 20x2. The financial assets are recognized in the 20x1 year-
end financial statements at ₱1,200,000 which is their fair value as of December 31, 20x1.
• On March 5, 20x2, a case was resolved requiring a settlement amount of ₱800,000. The 20x1 year-
end financial statements included a provision for loss on litigation of ₱600,000.

UNCORK Co.’s profit for the year ended December 31, 20x1 before consideration of the above
transactions is ₱8,800,000. The financial statements were authorized for issue on March 1, 20x2.

How much is the adjusted profit?


a. 8,820,000
b. 9,020,000
c. 10,820,000
d. 8,280,000

Solution:
Unadjusted profit, December 31, 20x1 8,800,000
(a) Reduction in provision for loss on pending litigation 80,000
(480K – 400K)
(b) Reduction in NRV of inventories [3.6M - (3.52M –120K)] (200,000)
(c) Impairment loss on receivables (400,000)
Adjusted profit, December 31, 20x1 8,280,000

Use the following information for the next two questions:


The following relates to the transactions of GRIMACE FROWN Co. during 20x1:
Directors' and officers' remuneration 8,000,000
Post-employment benefits of officers 800,000
Fringe benefits in the form of housing assistance to
directors and officers 20,000,000
Share options granted to officers 1,200,000
Officers' expenses on travels, representation and
entertainment subject to liquidation and
reimbursement 400,000
Loans to directors and officers 12,000,000
Sales to related entities 40,000,000

62. How much is the amount of related party disclosures on GRIMACE’s separate financial
statements?
a. 30,000,000
b. 52,000,000
c. 82,000,000
d. 42,000,000

C Key management personnel compensation (8M + 800K + 20M + 1.2M) + Related party transactions
(12M + 40M) = 82M

63. How much is the amount of related party disclosures on GRIMACE’s consolidated financial
statements?
a. 12,000,000
b. 30,000,000
c. 82,000,000
d. 42,000,000

D Key management personnel compensation (8M + 800K + 20M + 1.2M) + Related party transactions
(12M) = 42M

64. DEMENTED INSANE Co. is preparing its year-end financial statements and has identified the
following operating segments:
Segments Revenues Profit (loss) Assets
A 4,000,000 800,000 56,000,000
B 4,800,000 560,000 72,000,000
C 1,080,000 (280,000) 48,000,000
D 960,000 (2,800,000) 4,000,000
E 1,160,000 200,000 5,600,000
Totals 12,000,000 (1,520,000) 185,600,000

What are the reportable segments?


a. A, B and D
b. A, B, C and D
c. A and B
d. A, B, C, D and E

 Revenue test: Threshold = 1,200,000 (12,000,000 x 10%). Reportable segments are A and B.

 Profit or loss test: Total profits (800,000 + 560,000 + 200,000 = 1,560,000); Total losses (280,000
+ 2,800,000 = 3,080,000).

Threshold = 308,000 (3,080,000 (higher) x 10%). Reportable segments are A, B and D.


 Asset test: Threshold = 18,560,000 (185,600,000 x 10%). Reportable segments are A, B and C.

 The reportable segments are A, B, C and D.

65. EMBOSOM CHERISH Co. engages in five diversified operations namely, operations A, B, C, D,
and E. Information on these segments are shown below:
Segments Revenues Profit (loss) Assets
A 3,200 800 40,000
B 3,200 400 8,000
C 200 40 4,000
D 600 80 8,000
E 800 280 24,000
Totals 8,000 1,600 84,000

Additional information:
a. For internal reporting purposes, segments A and B are considered as one operating segment.
b. Segment E is considered as an operating segment for internal decision making purposes.
c. Segments C and D have similar economic characteristics and share a majority of the aggregation
criteria.

What are the reportable segments?


a. A, B, C, D and E
b. A, B and E
c. A and B as one segment and E
d. A and B as one segment, E, and C and D as one segment

D
 Management approach: Reportable segments are A and B aggregated as a single reportable
segment and E.

 Quantitative tests: C and D aggregated as a single reportable segment.

66. SORDID DIRTY Co. is preparing its year-end financial statements and has identified the
following operating segments:
External Inter-segment Total
Segments revenues revenues revenues Profit Assets
A 4,800,000 2,400,000 7,200,000 2,800,000 48,000,000
B 1,600,000 400,000 2,000,000 1,600,000 28,000,000
C 1,000,000 - 1,000,000 400,000 4,000,000
D 800,000 - 800,000 320,000 3,200,000
E 600,000 - 600,000 280,000 2,800,000
F 400,000 - 400,000 200,000 2,000,000
Totals 9,200,000 2,800,000 12,000,000 5,600,000 88,000,000

Management believes that between segments C, D, E and F, segment C is most relevant to external
users of financial statements.

What are the reportable segments?


a. A and B
b. A, B, C and D
c. A, B and C
d. A, B, C, D, E and F

 Quantitative tests: A and B. However, their total external revenues is less than the 75% limit.
Therefore, C is included as reportable in order to meet the 75% limit, even if segment C does not
qualify in any of the quantitative tests.

67. RUSTIC RURAL Co. has the following information on its operating segments.

External Inter-segment Total


Segments revenues revenues revenues Profit Assets
A 4,800,000 2,400,000 7,200,000 2,800,000 48,000,000
B 1,600,000 400,000 2,000,000 1,600,000 28,000,000
C 1,000,000 - 1,000,000 400,000 4,000,000
D 800,000 - 800,000 320,000 3,200,000
E 600,000 - 600,000 280,000 2,800,000
F 400,000 - 400,000 200,000 2,000,000
Totals 9,200,000 2,800,000 12,000,000 5,600,000 88,000,000

RUSTIC Co. shall provide disclosure for major customers if revenues from transactions with a single
external customer amount to how much?
a. 920,000
b. 280,000
c. 1,200,000
d. 560,000

A (9,200,000 x 10%) = 920,000

68. You are the accountant of Entity X. The board of directors asked you for an advice because they
feel like the company’s financial statements do not properly reflect the company’s financial
position. The board noted out that the company’s properties (i.e., land) are absurdly stated at
their historical cost. The properties were acquired 50 years ago and the market prices of the
properties have more than tripled since then. In providing your professional advice, you will
most certainly quote the provisions of which of the following standards?
a. PAS 7
b. PAS 1
c. PAS 16
d. PAS 8

69. PFRS 8 relates to which of the following?


a. Disclosure of operating segments
b. Disclosure of related party relationships and transactions
c. Disclosure of events after the reporting period
d. Interim financial reporting

70. "Aanhin mo pa ang damo kung patay na ang kabayo.”


a. Relevance
b. Timeliness
c. Biological asset - Horse
d. PFRH – Philippine Financial Reporting Horse
e. b and c

B – choice (c) is wrong. A dead animal cannot qualify as a biological asset. Stocks of grass used for
feeding horses are considered “Supplies” rather than biological assets. “Bio” means life.

“I press on toward the goal to win the prize for which God has called me
heavenward in Christ Jesus.” – (Philippians 3:14)

- END -
1. The ledger of LEEWAY TOLERANCE Co. as of December 31, 20x1 includes the following:
Assets
Petty cash fund 14,000
Cash in bank 40,000
Accounts receivable (including ₱30,000 pledged accounts) 70,000
Accounts receivable – assigned 50,000
Equity in assigned receivables 20,000
Notes receivable (including ₱40,000 notes receivable discounted) 90,000
Notes receivable discounted 40,000
Advances to subsidiary 64,000
Held for trading securities 40,000
Inventory 124,000
Deferred charges 36,000
Cash surrender value 12,000
Bond sinking fund 200,000
Total assets 800,000

Liabilities
Accounts payable 80,000
Estimated warranty liability 28,000
Loans payable related to assigned receivables (due in 12 months) 30,000
Accrued expenses 26,000
Bonds payable (due on December 31, 20x2) 200,000
Premium on bonds payable 16,000
Total liabilities 380,000

Additional information:
- Petty cash fund includes IOU’s from employees amounting to ₱4,000. The remaining balance of
₱10,000 represents bills and coins.
- The cash in bank balance represents the balance per bank statement. As of December 31, 20x1,
deposits in transit amounted to ₱20,000 while outstanding checks amounted to ₱3,000. Included
in the bank statement as of December 31, 20x1 is an NSF check amounting to ₱16,000.
- Accounts receivable (unassigned) includes uncollectible past due accounts of ₱8,000 which need
to be written-off.
- Also included in accounts receivable (unassigned) is a ₱10,000 receivable from a customer which
was given a special credit term. Under the special credit term, the customer shall pay the ₱10,000
receivable in equal quarterly installments of ₱1,250. The last payment is due on December 31,
20x3.
- The held for trading securities include the reacquisition cost of LEEWAY Co.’s shares amounting
to ₱8,000.
- Inventory includes ₱60,000 goods in transit purchased FOB Destination but excludes ₱24,000
goods in transit purchased FOB Shipping point.

How much is the working capital?


a. 204,000
b. 224,000
c. 246,000
d. 254,000

A
Solution:
Current assets
Petty cash fund (P14,000 – P4,000 IOU's) 10,000
Cash in bank (40,000 + 20,000 DIT - 3,000 OC) 57,000
Advances to employees (representing the IOU's) 4,000
Accounts receivable* 57,000
Accounts receivable – assigned 50,000
Notes receivable 90,000
Notes receivable discounted (40,000)
Held for trading securities (P40,000 – P8,000 Treasury shares) 32,000
Inventory (P112,000 – P60,000 FOB Dest. + P24,000 FOB SP) 88,000
Bond sinking fund 200,000
Total current assets 548,000

Current liabilities
Accounts payable (80,000 - 60,000 FOB Dest. + 24,000 FOB SP) 44,000
Estimated warranty liability 28,000
Loans payable related to assigned receivables (due in 12 mos.) 30,000
Accrued expenses 26,000
Bonds payable (due on December 31, 20x2) 200,000
Premium on bonds payable 16,000
Total current liabilities 344,000

Working capital 204,000

*The adjusted accounts receivable is computed as follows:


Accounts receivable 70,000
Uncollectible accounts written-off ( 8,000)
Accounts with special credit term – noncurrent portion
(P1,250 quarterly installment x 4 installments in 20x3) ( 5,000)
Adjusted accounts receivable (unassigned) 57,000

2. The following statements relate to PAS1 Presentation of Financial Statements. Choose the correct
statement.
a. Many entities also present, outside the financial statements, reports and statements such as
environmental reports and value added statements, particularly in industries in which
environmental factors are significant and when employees are regarded as an important user
group. Reports and statements presented outside financial statements should be accounted
for using applicable PFRSs.
b. Applying a requirement is impracticable when the entity cannot apply it after making every
reasonable effort to do so.
c. An entity whose financial statements do not comply with PFRSs shall make an explicit and
unreserved statement of such noncompliance in the notes. If the entity’s financial statements
do comply with PFRSs, there is no need to make an explicit and unreserved statement of
such compliance in the notes.
d. Financial statements shall not be described as complying with PFRSs unless they comply
with most of the requirements of PFRSs.

3. Which of the following financial statements would not be dated as covering a certain reporting
period?
a. Statement of financial position
b. Statement of profit or loss and other comprehensive income
c. Statement of cash flows
d. Statement of changes in equity

Use the following information for the next two questions:


The records of HACK TO CHOP Co. on December 31, 20x1 showed the following information:
Sales 2,000,000
Sales discounts 20,000
Cost of sales 800,000
Distribution costs 96,000
Administrative costs 240,000
Casualty loss on typhoon 40,000
Dividends received from investments in FVPL 24,000
Dividends received from investment in associate 48,000
Share in the profit of an associate 72,000
Dividends declared and paid 28,000
Interest expense 44,000
Unrealized gain on investments in FVPL 30,000
Unrealized gain on investments in FVOCI 38,000
Income tax expense 300,000
Loss on revaluation 26,000
Remeasurements of the net defined benefit liability (asset) - gain 22,000
Correction of understatement in depreciation in prior year 32,000
Translation adjustment of foreign operation – loss 8,000

4. How much is the other comprehensive income?


a. 42,000
b. 36,000
c. 34,000
d. 26,000

5. How much is the total comprehensive income?


a. 612,000
b. 627,000
c. 516,000
d. 584,000
Solution:
HACK TO CHOP Co.
Statement of profit or loss and other comprehensive income
For the year ended December 31, 20x1

Sales 2,000,000
Sales discounts (20,000)
Net sales 1,980,000
Cost of sales (800,000)
Gross profit 1,180,000
Distribution costs (96,000)
Administrative costs (240,000)
Dividends received from investments in FVPL 24,000
Share in the profit of an associate 72,000
Unrealized gain on investments in FVPL 30,000
Casualty loss on typhoon (40,000)
Interest expense (44,000)
Profit before tax 886,000
Income tax expense (300,000)
Profit for the year 586,000
Other comprehensive income:
Items that will not be reclassified subsequently to profit or loss:
Loss on revaluation (26,000)
Unrealized gain on investments in FVOCI 38,000
Remeasurements of defined benefit pension plans 22,000
34,000
Items that may be reclassified subsequently to profit or loss:
Loss on translation of foreign operation (8,000)
Other comprehensive income for the year 26,000
TOTAL COMPREHENSIVE INCOME FOR THE YEAR 612,000

6. Comprehensive income (or total comprehensive income) includes


a. Profit or loss
b. Other comprehensive income
c. Transactions with owners
d. a and b
e. All of these

7. What is the purpose of reporting comprehensive income?


a. To report changes in equity due to transactions with owners.
b. To report a measure of overall performance of an entity.
c. To replace profit with a better measure.
d. To combine income from continuing operations with income from discontinued operations
and extraordinary items.

8. PFRS 15 applies to
a. contracts with customers.
b. contracts with sellers.
c. all contracts entered into by an entity in the ordinary course of its business.
d. a and b

9. ABC Co., a dealer of medical machines, enters into the following contracts:
I. ABC Co. transfers a machine to X Hospital at contract inception but ABC Co. retains legal
title until the full payment of the consideration.
II. ABC Co. transfers a machine to Y Medical Clinic at contract inception. The consideration is
due after two years. At contract inception, Y is undergoing financial difficulties. This raises
significant doubt in Y’s ability and intention of paying the consideration. ABC Co. cannot
reliably estimate the outcome of the contract.
III. ABC Co. transfers a machine to Z Co. under a lease contract. The contractual period is 5
years, which is equal to the machine’s estimated useful life. At the end of the contract, Z Co.
is given the option of purchasing the machine. ABC’s past experience shows that almost all
customers avail of the purchase option.

Identify the contracts to which PFRS 15 may be applied.


a. Contract 1 c. Contracts 1 and 3
b. Contract 2 d. None of these

10. The consideration received from a contract with a customer that does not meet the criteria under
‘Step 1’ of PFRS 15 is
a. recognized as liability.
b. recorded through memo entry only.
c. disclosed only.
d. b and c

11. A good or service that is not distinct (choose the incorrect statement)
a. shall be combined with the other promises in the contract.
b. may be treated, together with other promises in the contract, as a single performance
obligation.
c. may be identified as a part of a bundle of goods or services or a part of a series of goods or
services to be transferred to the customer.
d. shall be ignored. The entity allocates the transaction price only to the other promises in the
contract that are distinct.

12. According to PFRS 15, revenue from a performance obligation that is not satisfied over time is
recognized
a. over time as the entity progresses towards the complete satisfaction of the obligation.
b. at a point in time when the performance obligation is satisfied.
c. when the contract ceases to be enforceable.
d. a or b

13. Arrange the following steps of revenue recognition in accordance with PFRS 15.
I. Identify the performance obligations in the contract
II. Recognize revenue when (or as) the entity satisfies a performance obligation
III. Determine the transaction price
IV. Identify the contract with the customer
V. Allocate the transaction price to the performance obligations in the contract
a. IV, I, V, III, II c. IV, I, III, V, II
b. III, IV, I, V, II d. IV, III, I, V, II

14. Which of the following must be met before a contract with a customer is accounted for under
PFRS 15?
a. The collection of the consideration must be certain.
b. The contract must be in writing so that there will be no doubt in the customer’s ability and
intention to pay the consideration.
c. The promised goods or services must have already been transferred to the customer.
d. Both contracting parties must acknowledge, whether explicitly or implicitly, the rights and
obligations created under the contract.

15. Which of the following may be treated as a performance obligation to be accounted for
separately?
I. A promise to transfer a distinct good or service
II. A promise to transfer a distinct bundle of goods or services
III. A promise to transfer a series of distinct goods or services that are substantially the same and
have the same pattern of transfer to the customer
IV. A promise that is implied by the entity’s customary business practices which, at contract
inception, creates a valid expectation on the part of the customer that the entity will satisfy
the promise
a. I only c. I, II and III
b. I and II d. all of these

16. A good or service is distinct if:


I. The customer can benefit from the good or service either on its own or together with other
resources that are readily available to the customer.
II. The promise to transfer the good or service is separately identifiable from other promises in
the contract.
a. I only c. I and II
b. II only d. none of these

17. Revenue is recognized when (or as) the entity satisfies a performance obligation. According to
PFRS 15, revenue is measured at
a. the fair value of the consideration received or receivable.
b. the transaction price.
c. the stand-alone selling price of the good or services transferred.
d. the amount of the transaction price allocated to the performance obligation satisfied.

18. According to PFRS 15, the transaction price is allocated to each performance obligation
identified in a contract based on the
a. relative stand-alone prices of the distinct goods or services promised to be transferred.
b. contractual agreement with the customer.
c. expected costs of satisfying the performance obligations.
d. a or b
19. According to PFRS 15, revenue from a performance obligation that is satisfied over time is
recognized
a. over time as the entity progresses towards the complete satisfaction of the obligation.
b. at a point in time when the performance obligation is satisfied.
c. when the contract ceases to be enforceable.
d. a or b

20. ABC Co. enters into a contract with XYZ, Inc. to deliver 2 apples, 3 mangoes and 5 potatoes for a
total consideration of ₱100. In accounting for the contract, which of the following is probably not
true?
a. ABC Co. identifies three performance obligations in the contract.
b. ABC Co. allocates the ₱100 transaction price over the promises to deliver the apples,
mangoes and potatoes on the basis of relative stand-alone selling prices of those goods.
c. The allocation of the transaction price may result to the identification of a discount.
d. No revenue is recognized until all of the 2 apples, 3 mangoes and 5 potatoes are delivered
even though the 2 apples were delivered first before the mangoes and potatoes.

21. Non-current assets are presented as current assets in the statement of financial position
a. only when they are expected to be sold within 12 months from the end of reporting period.
b. only if they are actually sold after the reporting period but before the date of authorization of
the financial statements for issue.
c. only when they qualify as held for sale assets under PFRS 5.
d. never presented as current items.

22. The qualification of an asset to be classified as held for sale after the reporting period but before
the financial statements are authorized for issue
a. is a non-adjusting event after the reporting period.
b. is an adjusting event after the reporting period.
c. is an extraordinary item.
d. a or b

23. A noncurrent asset classified as held for sale in accordance with PFRS 5 has not been sold after a
year. The asset shall continue to be presented as held for sale under PFRS 5 if
a. the delay is due to events beyond the entity’s control.
b. the entity remains committed to its plan to sell the asset.
c. the noncurrent asset is actually sold after the reporting period but before the financial
statements were authorized for issue.
d. a and b

24. According to PFRS 5, gain on impairment reversal on an asset held for sale is
a. recognized for the fair value change during the period.
b. recognized in other comprehensive income.
c. recognized only to the extent of cumulative impairment losses previously recognized.
d. not recognized.

25. The results of discontinued operations are presented separately in the statement of profit or loss
and other comprehensive income
a. as a single amount gross of tax.
b. as a single amount net of tax.
c. as part of the regular line items.
d. a or b

26. According to PFRS 5, held for sale classification is permitted when


a. the noncurrent asset or disposal group is available for immediate sale in its present
condition.
b. the sale is highly probable.
c. a and b
d. the sale actually occurred after the reporting period but before the financial statements were
authorized for issue.

27. According to PFRS 5, assets held for sale are measured at


a. fair value. c. carrying amount.
b. fair value less costs to sell. d. lower of b and c

28. According to PFRS 5, a disposal group may qualify as discontinued operation if


a. it is a component of an entity.
b. it meets the held for sale classification criteria.
c. a and b
d. none of these

29. The results of a discontinued operations are presented in the statement of profit or loss
a. before the profit or loss from continuing operations but after the profit for the year.
b. after the profit or loss from continuing operations but before the profit for the year.
c. separately from the profit or loss from continuing operations and it does not affect the profit
for the year.
d. as an adjustment to the beginning balance of the retained earnings.

30. Which of the following is included in profit from continuing operations?


a. extraordinary items c. other comprehensive income
b. discontinued operations d. income tax expense

31. Entity A’s total shareholders’ equity on January 1, 20x1 was ₱180,000. The following were the
transactions during the year:
• Entity A issued additional share capital amounting to ₱360,000.
• Total income earned amounted to ₱1,000,000.
• Total expenses incurred amounted to ₱560,000.
• Entity A declared dividends of ₱140,000.

How much is the total shareholders’ equity on December 31, 20x1?


a. 840,000
b. 700,000
c. 640,000
d. 540,000
A 180K + 360K + 1M – 560K – 140K = 840K

32. Entity A reported profit of ₱340,000 for the year ended December 31, 20x1. Depreciation expense
for the year was ₱100,000. The following are the changes in the operating assets and liabilities of
Entity A during 20x1:

20x1 20x0
Accounts receivable 560,000 300,000
Accounts payable 240,000 120,000

How much is the net cash from operating activities?


a. 820,000
b. 580,000
c. 300,000
d. 100,000

C 340K profit + 100K depreciation – 260K inc. in A/R + 120K inc. in A/P = 300K

Use the following information for the next two questions:


The following were the cash transactions of Entity A during the period:

Cash receipts from sale of goods 650,000


Cash paid for purchases of inventory 340,000
Cash receipts on loans taken from a bank 200,000
Cash paid for interest expense 20,000
Cash payment for the acquisition of property, plant and equipment 180,000

33. How much is the net cash from (used in) operating activities?
a. 155,000
b. (155,000)
c. 290,000
d. (290,000)

C 650K sale – 340K purchases – 20K interest = 290K

34. How much is the net cash from (used in) investing activities?
a. 180,000
b. (180,000)
c. 20,000
d. 0

B acquisition of PPE

Use the following information for the next three questions:


The comparative statement of financial position and statement of comprehensive income of Entity A
on December 31, 20x1 are shown below:
Entity A
Statement of Financial Position
As of December 31, 20x1

ASSETS 20x1 20x0


Cash and cash equivalents 440,000 200,000
Trade and other receivables 130,000 120,000
Inventory 120,000 480,000
Prepaid assets 40,000 160,000
Total current assets 730,000 960,000

Property, plant & equipment 760,000 440,000


Total noncurrent assets 760,000 440,000

TOTAL ASSETS 1,490,000 1,400,000

LIABILITIES
Trade and other payables 620,000 560,000

EQUITY
Owner’s capital 870,000 840,000

TOTAL LIABILITIES & EQUITY 1,490,000 1,400,000

Entity A
Statement of Comprehensive Income
For the year ended December 31, 20x1

Sales 1,000,000
Cost of sales (600,000)
GROSS PROFIT 400,000
Rent income 150,000
Depreciation expense (240,000)
Insurance expense (120,000)
Bad debts expense (30,000)
Loss on sale of equipment (40,000)
PROFIT FOR THE YEAR 120,000
Other comprehensive income -
COMPREHENSIVE INCOME FOR THE YR. 120,000
Additional information:
• Equipment with carrying amount of ₱240,000 was sold for ₱200,000 resulting to a loss on sale of
₱40,000.
• Acquisition of equipment for cash amounted to ₱800,000.
• Owner drawings totalled ₱90,000.

35. How much is the cash flows from (used in) operating activities?
a. 930,000
b. (930,000)
c. 400,000
d. (400,000)

36. How much is the cash flows from (used in) investing activities?
a. 600,000
b. (600,000)
c. 400,000
d. (400,000)

37. How much is the cash flows from (used in) financing activities?
a. 440,000
b. (440,000)
c. 90,000
d. (90,000)

Solution:
Entity A
Statement of cash flows
For the year ended December 31, 20x1

Cash flows from operating activities


Profit before tax 120,000
Depreciation expense 240,000
Loss on sale of equipment 40,000
Increase in trade and other receivable, net (10,000)
Decrease in inventory 360,000
Decrease in prepaid insurance 120,000
Increase in trade and other payables 60,000
Net cash from operating activities 930,000

Cash flows from investing activities


Cash receipt from sale of equipment 200,000
Cash payment for acquisition of equipment (800,000)
Net cash used in investing activities (600,000)

Cash flows from financing activities


Owner's drawings (90,000)
Net cash from financing activities (90,000)

Net increase in cash and cash equivalents 240,000


Cash and cash equivalents, beginning 200,000
Cash and cash equivalents, end 440,000

38. Year-end net assets would be overstated and current expenses would be understated as a result
of failure to record which of the following adjusting entries?
a. Expiration of prepaid insurance
b. Depreciation of fixed assets
c. Accrued wages payable
d. All of these

39. Dane Co. received merchandise on consignment. As of March 31, Dane had recorded the
transaction as a purchase and included the goods in the physical count of ending inventory.
Dane uses the periodic inventory system. None of the consigned goods have been sold during
the period. The effect of this on its financial statements for March 31 would be
a. no effect.
b. profit is correct but current assets and current liabilities are overstated.
c. profit, current assets and current liabilities are overstated.
d. profit and current liabilities are overstated.

40. If the cost of ordinary repairs is capitalized as an addition to the building account during the
current year,
a. net income for the current year will be understated.
b. stockholders' equity at the end of the current year will be understated.
c. total assets at the end of the current year will not be affected.
d. total liabilities at the end of the current year will not be affected.

41. ABC Co. completes the draft of its December 31, 20x1 year-end financial statements on January
31, 20x2. On February 5, 20x2, the board of directors reviews the financial statements and
authorizes them for issue. The entity announces its profit and selected other financial
information on February 23, 20x2. The financial statements are made available to shareholders
and others on March 1, 20x2. The shareholders approve the financial statements at their annual
meeting on March 18, 20x2 and the approved financial statements are then filed with a
regulatory body on April 1, 20x2. Events after the reporting period are those occurring
a. from December 31, 20x1 to February 5, 20x2.
b. from January 1, 20x2 to February 5, 20x2.
c. from January 1, 20x2 to February 23, 20x2.
d. from January 1, 20x2 to March 18, 20x2.

42. These are events that are indicative of conditions that arose after the reporting period.
a. Events after the reporting period c. Adjusting events
b. Non-adjusting events d. all of these

43. Entity A recognized a provision for a pending litigation amounting to ₱50,000 on December 31,
20x1 (end of current reporting period). This amount is reflected in Entity A’s reported profit of
₱600,000 for the year 20x1. Shortly after December 31, 20x1, but before the financial statements
were authorized for issue, the litigation is settled for ₱40,000. The correct profit in 20x1 is
a. 600,000 . c. 640,000.
b. 610,000 . d. 590,000

44. Which of the following is an example of an adjusting event?


a. Major business combination after the reporting period.
b. A building is totally razed by fire after the reporting period.
c. Sale of inventories after the reporting period that gives evidence to their net realizable value
at the end of reporting period.
d. Issuance of shares of stocks after the reporting period.

45. Which of the following is an example of a non-adjusting event?


a. Bankruptcy of a customer after the reporting period that indicates that the carrying amount
of a trade receivable at the end of reporting period is impaired.
b. Evidence indicating that an asset is impaired as at the end of the reporting period.
c. Legal proceedings after the reporting period for an incident that occurred before the end of
the reporting period.
d. Significant decline in foreign exchange rates after the reporting period resulting to massive
losses on recognized foreign currency denominated financial instruments.

46. According to PAS 8, in the absence of a PFRS that specifically deals with a transaction,
management shall
a. refer to the concepts under the Conceptual Framework.
b. adopt the provisions of the US GAAP.
c. use its judgment in developing and applying an accounting policy that results in information
that is relevant and reliable.
d. consider the applicability of relevant accounting literature.

47. According to PAS 8, a change in accounting policy is accounted for


a. using a transitional provision, if any.
b. retrospectively.
c. prospectively, if retrospective application is impracticable.
d. a, b or c, whichever is most appropriate

48. This refers to applying a new accounting policy to transactions, other events and conditions as if
that policy had always been applied.
a. Retrospective application c. Prospective application
b. Retrospective restatement d. Impracticable application

49. According to PAS 8, a change in accounting estimate is accounted for


a. using a transitional provision, if any.
b. retrospectively.
c. prospectively.
d. a, b or c, whichever is most appropriate

50. Entity A changes its inventory cost formula from FIFO to weighted average. How should Entity
A account for this change?
a. by retrospective restatement, as a change in accounting policy
b. by prospective application, as a change in accounting estimate
c. by retrospective application, as a change in accounting policy
d. as a correction of prior period error

51. According to PAS 24, related party disclosures are necessary


a. because related party transactions may have resulted to assets and liabilities that were
recognized in the financial statements of the reporting entity.
b. to notify users of financial statements of the fact that related party transactions may not have
been made on arm’s length basis.
c. to indicate the possibility that an entity’s financial position and performance might have
been affected by the existence of such relationship.
d. in order to eliminate or minimize the effects of related party transactions on the financial
statements of the reporting entity.

52. What is overriding consideration when determining the existence of a related party relationship?
a. The ability of one party to affect decisions of another party regarding relevant activities
through the existence of control, joint control or significant influence.
b. The presence of relationship either by consanguinity or affinity.
c. The presence of a significant interest by one party over the other.
d. The presence of significant business transactions and economic dependence between the
parties.

53. Mr. Y and Ms. Z share joint control over Ventures, Inc. Which of the following are related
parties?
a. Mr. Y and Ms. Z c. Ventures, Inc. and PAS 24
b. Ventures, Inc. and Mr. Y d. none of these

54. Entity A is the parent company of Entity B. Which of the following is required to be disclosed in
the group’s (Entity A and B’s) consolidated financial statements?
a. the related party relationship between Entity A and Entity B
b. the related party transactions during the period
c. the outstanding balances in (c)
d. all of these

55. Catalyst Co. is engaged in business process outsourcing. Catalyst subcategorizes its main
services into four: Information Technology, After-sales Support, Accounting, and Offsite Data
Management. Catalyst operates in five major geographical areas: Southeast Asia, North America,
South America, Australia and Europe. Internal reports are based on these five geographical
areas. What is the most appropriate basis of segment reporting for Catalyst?
a. On the basis of the main services provided.
b. On the basis of the geographical areas of operations.
c. On the basis of the domicile country of Catalyst and the rest of the world.
d. Any of these.

56. Segment A qualifies under the 10% test of total revenues but not on the profit or loss and total
assets tests. Segment A
a. is not a reportable segment.
b. is nonetheless included in the “all others” segment.
c. may be reported as a separate segment.
d. all of these

57. Information on an entity’s operating segments is shown below:

Operating segments Total revenue Profit Identifiable assets


A 1,000,000 200,000 4,000,000
B 500,000 120,000 1,000,000
C 300,000 30,000 800,000
D 500,000 50,000 1,700,000
E 200,000 60,000 800,000
F 900,000 400,000 1,000,000
Totals 3,400,000 860,000 9,300,000

The reportable segments are


a. A, B and F c. A, B, C, D and F
b. A, B, D and F d. All segments

58. Entity A wants to publish quarterly interim financial reports. Which of the following standards
may Entity A apply in preparing and presenting its interim financial reports?
a. PAS 1 c. PFRS 1
b. PAS 34 d. a or b

59. If an entity does not prepare interim financial reports,


a. its annual financial statements would not conform to the PFRSs.
b. its annual financial statements should not be described to have been prepared in accordance
with PFRSs
c. the conformance of its annual financial statements with the PFRSs is not affected.
d. a and b

60. Which of the following is correct regarding the provisions of PAS 34?
a. All entities should publish quarterly interim reports.
b. All publicly-listed entities should publish quarterly interim reports.
c. All publicly-listed entities should publish semi-annual interim reports.
d. PAS 34 does not require any entity to publish interim reports, and how often.

61. Interim financial reports prepared in accordance with PAS 34 shall, at a minimum, include
a. semi-annual interim financial statements.
b. complete set of financial statements.
c. condensed set of financial statements.
d. a statement of financial position and an income statement.

62. Entity A publishes quarterly interim financial reports. Entity A’s annual depreciation for items
of PPE is ₱120,000. At the end of the first quarter, Entity A’s inventories have a cost of ₱600,000
and a net realizable value of ₱510,000. Entity A expects that the total employee bonuses (13th
month pay) that will be paid at year-end will amount to ₱60,000. How much is the total amount
of expense to be recognized from the items described above in Entity A’s first quarter statement
of profit or loss?
a. 120,000 c. 30,000
b. 135,000 d. 270,000

63. Under the cash basis of accounting, revenues are recorded


a. when they are earned and realized.
b. when they are earned and realizable.
c. when they are earned.
d. when they are collected.

64. White Co. wants to convert its 2003 financial statements from the accrual basis of accounting to
the cash basis. Both supplies inventory and office salaries payable increased between January 1,
2003 and December 31, 2003. To obtain the 2003 cash basis net income, how should these
increases be added to or deducted from accrual-basis net income?
Supplies inventory Office salaries payable
a. Deducted Deducted
b. Deducted Added
c. Added Deducted
d. Added Added

65. Insurance payments P150,000


Prepaid insurance, Jan. 1 65,000
Prepaid insurance, Dec. 31 85,000
Accrued insurance payable decreased by 35,000

How much is the insurance expense under accrual basis accounting?


a. 205,000
b. 65,000
c. 130,000
d. 95,000

D Solution:
Prepaid, beg. 65,000 35,000 Accrued payable, beg.
Payments 150,000 95,000 Insurance expense (squeeze)
Accrued payable, end. - 85,000 Prepaid, end.

66. Unearned rent, Jan. 1 P170,000


Unearned rent, Dec. 31 85,000
Accrued rent income, Jan. 1 180,000
Accrued rent income, Dec. 31 200,000
Rental payments received 560,000

How much is the Rent income under the accrual basis accounting?
a. 455,000
b. 625,000
c. 665,000
d. 645,000

C
Solution:
Accrued, beg. 180,000 170,000 Unearned, beg.
Rent income (squeeze) 665,000 560,000 Payments received
Unearned, end. 85,000 200,000 Accrued, end.

67. Payments made for income taxes P760,000


Income tax payable increased by 200,000
Deferred tax liability, Jan. 1 360,000
Deferred tax liability, Dec. 31 470,000
Deferred tax asset, Jan. 1 85,000
Deferred tax asset, Dec. 31 65,000

Income tax expense under accrual basis accounting is


a. 1,090,000
b. 960,000
c. 850,000
d. 830,000

A
Solution:
Income tax payable
- beg.
Tax payments 760,000 960,000 Current tax expense (squeeze)
end. 200,000

Income tax expense (squeeze) 1,090,000


Less: Increase in DTL (110,000)
Less: Decrease in DTA (20,000)
Current tax expense 960,000

68. All of the following may not qualify as “small and medium-sized entity” (SME) except
a. banks c. investment house
b. insurance company d. cooperative

69. Which of the following most likely would not qualify as a “small and medium-sized entity”
(SME)?
a. A cooperative with total assets of ₱3M and liabilities of ₱2M.
b. A real estate company with total assets of ₱350M and liabilities of ₱250M.
c. A finance corporation with total assets of ₱2M and liabilities of ₱1M.
d. All of these entities qualify as SMEs.

70. Generally, non-financial liabilities of SMEs are measured at


a. the present value of future cash flows on the obligation
b. the best estimate of the amount that would be required to settle the obligation at the
reporting date
c. the mid-point value of the obligation
d. fair value

71. Which of the following is incorrect regarding the application and compliance with the PFRS for
SMEs?
a. The application of the PFRS for SMEs, with additional disclosure when necessary, is
presumed to result in financial statements that achieve a fair presentation of the financial
position, financial performance and cash flows of SMEs.
b. The application of the PFRS for SME by an entity with public accountability does not result
in a fair presentation even when a local legislation permits entities with public accountability
to use the PFRS for SMEs.
c. An entity whose financial statements comply with the PFRS for SMEs shall make an explicit
and unreserved statement of such compliance in the notes and on the face of each
component of a complete set of financial statements as provided under the PFRS for SME.
d. Financial statements shall not be described as complying with the PFRS for SMEs unless they
comply with all the requirements of the PFRS for SMEs.

72. According to the PFRS for SMEs, in assessing whether the going concern assumption is
appropriate, management takes into account all available information about the future, which is
at least, but is not limited to,
a. 12 months from the reporting date.
b. two years from the reporting date.
c. 3 months from the reporting date
d. it depends on professional judgment

73. Under the PFRS for SMEs, investments in equity instruments that are not publicly traded and do
not give the entity significant influence, control, or joint control over the investee, shall be
measured at
a. cost less impairment
b. amortized cost
c. fair value unless fair value cannot be measured reliably, in which case , at cost less
impairment
d. fair value with changes in fair value recognized in other comprehensive income
74. An SME shall measure its investment in associate using
a. Fair value model
b. Cost model
c. Equity method
d. any of these

75. Under the PFRS for SMEs, relationships between a parent and its subsidiaries shall be disclosed
a. only when there have been related party transactions.
b. irrespective of whether there have been related party transactions.
c. even when control is lost
d. any of these

76. The ceiling of the threshold for total assets of an SME qualifier is
a. 400M b. 3M c. 350M d. 250M

77. (Use the PFRS for SMEs) On 15 March 20X1 the entity authorized for issue its annual financial
statements for the year ended 31 December 20X0. On 10 March 20X1 the entity’s factory and
several items of equipment were damaged in an earthquake. The event (quake damage):
a. is an adjusting event after the end of the 31 December 20X0 reporting period.
b. is a non-adjusting event after the end of the 31 December 20X0 reporting period.
c. is neither an adjusting event after the end of the 31 December 20X0 reporting period nor a
non-adjusting event after the end of the 31 December 20X0 reporting period.
d. None of these

78. (Use the PFRS for SMEs) Which of the following is a non-adjusting event after the end of the
reporting period that an entity should disclose in its financial statements for 20X5? In each case,
the financial statements for 20X5 have not yet been authorized for issue.
a. An entity has a portfolio of shares with quoted market prices. These are measured at fair
value through profit or loss in accordance with Section 11 of the PFRS for SMEs. After the
end of the reporting period, there was a substantial decline in the stock market. The fair
value of the entity’s portfolio of shares declined significantly.
b. At 31 December 20X5 one individual owned 100 per cent of the entity’s outstanding shares.
In February 20X6 that individual sold 80 per cent of her holding to another party.
c. All of the above.
d. None of these

79. (Use the PFRS for SMEs) The goods or services received or acquired in a share-based payment
transaction are recognized as
a. assets
b. expenses
c. income
d. a or b

80. (Use the PFRS for SMEs) Notes to the financial statements:
a. contain only information required to be disclosed by the PFRS for SMEs that was not
presented in the statement of financial position, statement of comprehensive income,
statement of changes in equity or cash flow statement.
b. contain information required by Section 8 Notes to the Financial Statements without
reference to the other sections of the PFRS for SMEs.
c. contain the information required to be disclosed by the PFRS for SMEs that was not
presented in the statement of financial position, statement of comprehensive income,
statement of changes in equity or statement of cash flows and additional information
relevant to an understanding of the financial statements.
d. None, an SME is not required to present notes.

81. (Use the PFRS for SMEs) The cross-reference between each line item in the financial statements
and any related information disclosed in the notes to the financial statements:
a. is voluntary.
b. is mandatory.
c. depends on the industry.
d. any of these

82. (Use the PFRS for SMEs) The presentation of the notes to the financial statements in a systematic
manner:
a. is voluntary.
b. is mandatory.
c. is mandatory, as far as is practicable
d. any of these

83. (Use the PFRS for SMEs) An entity normally presents the notes in the following order:
a. First, a statement that the financial statements have been prepared in compliance with the
PFRS for SMEs. Second, a summary of significant accounting policies applied. Third,
supporting information for items presented in the financial statements, in the sequence in
which each statement and each line item is presented. Last, any other disclosures.
b. First, supporting information for items presented in the financial statements, in the sequence
in which each statement and each line item is presented. Second, a statement that the
financial statements have been prepared in compliance with the PFRS for SMEs. Third, a
summary of significant accounting policies applied. Last, any other disclosures.
c. First, supporting information for items presented in the financial statements, in the sequence
in which each statement and each line item is presented. Second, a summary of significant
accounting policies applied. Third, a statement that the financial statements have been
prepared in compliance with the PFRS for SMEs. Last, any other disclosures.

84. (Use the PFRS for SMEs) An entity shall disclose in the summary of significant accounting
policies:
a. the measurement basis (or bases) used in preparing the financial statements.
b. all the measurement bases specified in the PFRS for SMEs irrespective of whether they were
used by the entity in preparing its financial statements.
c. the measurement basis (or bases) used in preparing the financial statements and the
accounting policies used that are relevant to an understanding of the financial statements.
d. all of the measurement bases and the accounting policy choices available to the entity (i.e.,
specified in the PFRS for SMEs) irrespective of whether they were used by the entity in
preparing its financial statements.
85. (Use the PFRS for SMEs) Disclosure of information about key sources of estimation uncertainty:
a. is voluntary.
b. is mandatory.
c. is not required.
d. a and c

86. (Use the PFRS for SMEs) Disclosure of information about judgments, apart from those involving
estimations, that management has made in the process of applying the entity’s accounting
policies and that have the most significant effect on the amounts recognized in the financial
statements:
a. is voluntary.
b. is mandatory.
c. is not required.
d. a and c

87. (Use the PFRS for SMEs) On 1 January 20X1 an entity acquired goods for sale in the ordinary
course of business for ₱100,000, including ₱5,000 refundable purchase taxes. The supplier
usually sells goods on 30 days’ interest-free credit. However, as a special promotion, the
purchase agreement for these goods provided for payment to be made in full on 31 December
20X1. In acquiring the goods transport charges of ₱2,000 were incurred: these were due on 1
January 20X1. An appropriate discount rate is 10 per cent per year. The entity shall measure the
cost of inventories at:
a. ₱102,000 b. ₱97,000 c. ₱88,364 d. ₱107,000

C
Solution:
₱95,000 (₱100,000 excluding refundable tax of ₱5,000) multiplied by PV of ₱1 @10%, n=1 (or simply
divide by 1.1) plus ₱2,000 transport costs = ₱88,364.

88. (Use the PFRS for SMEs) On 31 December 20X1 entity A acquired 30 per cent of the ordinary
shares that carry voting rights of entity B for ₱100,000. Entity A incurred transaction costs of
₱1,000 in acquiring these shares. Entity A has significant influence over entity B. Entity A uses
the cost model to account for its investments in associates. In January 20X2 entity B declared and
paid a dividend of ₱20,000 out of profits earned in 20X1. No further dividends were paid in
20X2, 20X3 or 20X4. A published price quotation does not exist for entity B. At 31 December
20X1, 20X2 and 20X3, in accordance with Section 27 Impairment of Assets, management assessed
the fair values of its investment in entity B as ₱102,000, ₱110,000 and ₱90,000 respectively. Costs
to sell are estimated at ₱4,000 throughout. Entity A measures its investment in entity B on 31
December 20X1, 20X2 and 20X3 respectively at:
a. ₱100,000, ₱100,000, ₱100,000.
b. ₱95,000, ₱95,000, ₱86,000.
c. ₱98,000, ₱106,000, ₱86,000.
d. ₱98,000, ₱101,000, ₱86,000.
e. ₱102,000, ₱110,000, ₱90,000.

D
Solution:
20x1: Fair value less cost to sell (102K – 4K) = ₱98,000 lower than cost of ₱101K (cost of 100K +
transaction cost of 1K).
20x2: Cost of ₱101,000 = previous carrying amount of 98K + 3K reversal of impairment loss.
20x3: Fair value less cost to sell (90K – 4K) = ₱86,000 lower than previous carrying amount of ₱101K.

89. (Use the PFRS for SMEs) The facts are the same as in the immediately preceding question.
However, in this example, a published price quotation exists for entity B. Entity A measures its
investment in entity B on 31 December 20X1, 20X2 and 20X3 respectively at:
a. ₱100,000, ₱100,000, ₱100,000.
b. ₱95,000, ₱95,000, ₱86,000.
c. ₱98,000, ₱106,000, ₱86,000.
d. ₱98,000, ₱101,000, ₱86,000.
e. ₱102,000, ₱110,000, ₱90,000.

E
Solution:
₱102,000, ₱110,000, and ₱90,000 published price quotations without deduction for costs to sell.

90. (Use the PFRS for SMEs) On 1 January 20X1 an entity acquired a building for ₱95,000, including
₱5,000 non-refundable purchase taxes. The purchase agreement provided for payment to be
made in full on 31 December 20X1. Legal fees of ₱2,000 were incurred in acquiring the building
and paid on 1 January 20X1. The building is held to earn lease rentals and for capital
appreciation. An appropriate discount rate is 10 per cent per year. The entity shall measure the
initial cost of the building at:
a. ₱88,364
b. ₱97,000
c. ₱102,000
d. ₱107,000

A
Solution:
₱95,000 including non-refundable tax multiplied by PV of ₱1 @10%, n=1 (or simply divide by 1.1) plus
₱2,000 legal fees = ₱88,364.

91. (Use the PFRS for SMEs) A manufacturer gives warranties at the time of sale to purchasers of its
product. Under the terms of the contract for sale the manufacturer undertakes to make good, by
repair or replacement, manufacturing defects that become apparent within one year from the
date of sale. On the basis of experience, it is probable (i.e., more likely than not) that there will be
some claims under the warranties. Sales of ₱10 million were made evenly throughout 20X1. At
31 December 20X1 the expenditures for warranty repairs and replacements for the product sold
in 20X1 are expected to be made 50 per cent in 20X1 and 50 per cent in 20X2. Assume for
simplicity that all the 20X2 outflows of economic benefits related to the warranty repairs and
replacements take place on 30 June 20X2. Experience indicates that 95 per cent of products sold
require no warranty repairs; 3 per cent of products sold require minor repairs costing 10 per cent
of the sale price; and 2 per cent of products sold require major repairs or replacement costing 90
per cent of sale price. The entity has no reason to believe future warranty claims will be different
from its experience. At 31 December 20X1 the appropriate discount factor for cash flows
expected to occur on 30 June 20X2 is 0.95238. Furthermore, an appropriate risk adjustment factor
to reflect the uncertainties in the cash flow estimates is an increment of 6 per cent to the
probability-weighted expected cash flows. At 31 December 20X1 the entity recognizes a
warranty provision measured at:
a. ₱0.
b. ₱210,000.
c. ₱222,600.
d. ₱113,300.
e. ₱106,000.

E
Solution:
10M x 3% x 10% 30,000
10M x 2% x 90% 180,000
Total 210,000
Multiply by: Discount rate (given) 0.95238
Total 200,000
Multiply by: Risk adjustment (100% + 6%) 106%
Total 212,000
Multiply by: Amount to be settled in 20x2 50%
Warranty provision – Dec. 31, 20x1 106,000

92. (Use the PFRS for SMEs) An entity is the defendant in a patent infringement lawsuit. The entity’s
lawyers believe there is a 30 per cent chance that the court will dismiss the case and the entity
will incur no outflow of economic benefits. However, if the court rules in favor of the claimant,
the lawyers believe that there is a 20 per cent chance that the entity will be required to pay
damages of ₱200,000 (the amount sought by the claimant) and an 80 per cent chance that the
entity will be required to pay damages of ₱100,000 (the amount that was recently awarded by
the same judge in a similar case). Other outcomes are unlikely. The court is expected to rule in
late December 20X2. There is no indication that the claimant will settle out of court. A 7 per cent
risk adjustment factor to the probability-weighted expected cash flows is considered appropriate
to reflect the uncertainties in the cash flow estimates. An appropriate discount rate is 10 per cent
per year. At 31 December 20X1 the entity recognizes a provision for the lawsuit measured at:
a. ₱0.
b. ₱100,000.
c. ₱89,880.
d. ₱81,709.

D
Solution:
200K x 20% 40,000
100K x 80% 80,000
Total 120,000
Multiply by: PV of ₱1 @10%, n=1 0.90909
Total 109,090
Multiply by: Risk adjustment (100% + 7%) 107%
Total 116,727
Multiply by: Probability of settlement (100% - 30%) 70%
Provision for lawsuit – Dec. 31, 20x1 81,709

93. (Use the PFRS for SMEs) An entity operates in a jurisdiction where income taxes are payable at a
lower rate on undistributed profits (20 per cent) with an additional amount (10 per cent) being
payable when profits are distributed (i.e., the tax rate on distributed profits is 30 per cent). On 31
December 20X1 the entity expects to propose dividends in March 20X2 of approximately ₱20,000
for the year ended 20X1. The financial statements will be authorized for issue in April 20X2.
Taxable profit for 20X1 is ₱100,000. The entity has temporary differences that are expected to
increase taxable profit in the future for the year 20X1 of ₱30,000. The entity was formed on 1
January 20X1. On 31 December 20X1 the entity should recognize the following:
a. A current tax liability (and expense) of ₱20,000 and a deferred tax liability (and expense) of
₱6,000.
b. A current tax liability (and expense) of ₱20,000 and a deferred tax liability (and expense) of
₱9,000.
c. A current tax liability (and expense) of ₱22,000 and a deferred tax liability (and expense) of
₱6,000.
d. A current tax liability (and expense) of ₱25,000 and a deferred tax liability (and expense) of
₱7,500.
e. A current tax liability (and expense) of ₱30,000 and a deferred tax liability (and expense) of
₱9,000.

A
Solution:
Current tax expense = Taxable profit of ₱100,000 x Tax rate on taxable profit of 20% = ₱20,000
Deferred tax liability = Future increase in taxable profit ₱30,000 x Tax rate on taxable profit of 20% =
₱6,000

94. The disclosure of related party relationships is addressed by this standard.


a. PAS 1
b. PAS 8
c. PAS 10
d. PAS 24

95. The preparation of a statement of cash flows is addressed by this standard.


a. PAS 1
b. PAS 7
c. PFRS 6
d. a and b

96. Inventories are accounted for under


a. PAS 1
b. PAS 2
c. PFRS 5
d. PAS 24

97. Events after the reporting period are accounted for under
a. PAS 1
b. PAS 10
c. PFRS 1
d. PAS 24

98. The presentation of financial statements is addressed by this standard.


a. PAS 1
b. PAS 8
c. PFRS 3
d. PAS 28

99. PAS 34 relates to


a. the accounting for inventories.
b. the identification and disclosure of related party relationships.
c. interim financial reporting.
d. the presentation of financial instruments.

100. Which of the following is not one of the current PFRSs?


a. PAS 3
b. PAS 7
c. PAS 8
d. PFRS 5

“It does not matter how slowly you go as long as you do not stop.” - Confucius

- END -

You might also like